Sunteți pe pagina 1din 239

G.R. No.

L-41919-24 May 30, 1980


QUIRICO P. UNGAB, petitioner,
vs.
HON. VICENTE N. CUSI, JR., in his capacity as Judge of the Court of First Instance,
Branch 1, 16TH Judicial District, Davao City, THE COMMISSIONER OF INTERNAL
REVENUE, and JESUS N. ACEBES, in his capacity as State Prosecutor, respondents.

CONCEPCION JR., J:
Petition for certiorari and prohibition with preliminary injunction and restraining order to
annul and set aside the informations filed in Criminal Case Nos. 1960, 1961, 1962, 1963,
1964, and 1965 of the Court of First Instance of Davao, all entitled: "People of the
Philippines, plaintiff, versus Quirico Ungab, accused;" and to restrain the respondent Judge
from further proceeding with the hearing and trial of the said cases.
It is not disputed that sometime in July, 1974, BIR Examiner Ben Garcia examined the
income tax returns filed by the herein petitioner, Quirico P. Ungab, for the calendar year
ending December 31, 1973. In the course of his examination, he discovered that the
petitioner failed to report his income derived from sales of banana saplings. As a result, the
BIR District Revenue Officer at Davao City sent a "Notice of Taxpayer" to the petitioner
informing him that there is due from him (petitioner) the amount of P104,980.81,
representing income, business tax and forest charges for the year 1973 and inviting
petitioner to an informal conference where the petitioner, duly assisted by counsel, may
present his objections to the findings of the BIR Examiner. 1 Upon receipt of the notice, the
petitioner wrote the BIR District Revenue Officer protesting the assessment, claiming that
he was only a dealer or agent on commission basis in the banana sapling business and that
his income, as reported in his income tax returns for the said year, was accurately stated.
BIR Examiner Ben Garcia, however, was fully convinced that the petitioner had filed a
fraudulent income tax return so that he submitted a "Fraud Referral Report," to the Tax
Fraud Unit of the Bureau of Internal Revenue. After examining the records of the case, the
Special Investigation Division of the Bureau of Internal Revenue found sufficient proof that
the herein petitioner is guilty of tax evasion for the taxable year 1973 and recommended
his prosecution: t.hqw
(1) For having filed a false or fraudulent income tax return for 1973 with intent
to evade his just taxes due the government under Section 45 in relation to
Section 72 of the National Internal Revenue Code;
(2) For failure to pay a fixed annual tax of P50.00 a year in 1973 and 1974, or
a total of unpaid fixed taxes of P100.00 plus penalties of 175.00 or a total of
P175.00, in accordance with Section 183 of the National Internal Revenue
Code;
(3) For failure to pay the 7% percentage tax, as a producer of banana poles or
saplings, on the total sales of P129,580.35 to the Davao Fruit Corporation,

depriving thereby the government of its due revenue in the amount of


P15,872.59, inclusive of surcharge. 2
In a second indorsement to the Chief of the Prosecution Division, dated December 12,
1974, the Commissioner of Internal Revenue approved the prosecution of the petitioner.

Thereafter, State Prosecutor Jesus Acebes who had been designated to assist all Provincial
and City Fiscals throughout the Philippines in the investigation and prosecution, if the
evidence warrants, of all violations of the National Internal Revenue Code, as amended,
and other related laws, in Administrative Order No. 116 dated December 5, 1974, and to
whom the case was assigned, conducted a preliminary investigation of the case, and
finding probable cause, filed six (6) informations against the petitioner with the Court of
First Instance of Davao City, to wit: t.hqw
(1) Criminal Case No. 1960 Violation of Sec. 45, in relation to Sec. 72 of the
National Internal-Revenue Code, for filing a fraudulent income tax return for
the calendar year ending December 31, 1973; 4
(2) Criminal Case No. 1961 Violation of Sec. 182 (a), in relation to Secs. 178,
186, and 208 of the National Internal Revenue Code, for engaging in business
as producer of saplings, from January, 1973 to December, 1973, without first
paying the annual fixed or privilege tax thereof; 5
(3) Criminal Case No. 1962 Violation of Sec. 183 (a), in relation to Secs. 186
and 209 of the National Internal Revenue Code, for failure to render a true and
complete return on the gross quarterly sales, receipts and earnings in his
business as producer of banana saplings and to pay the percentage tax due
thereon, for the quarter ending December 31, 1973; 6
(4) Criminal Case No. 1963 Violation of Sec. 183 (a), in relation to Secs. 186
and 209 of the National Internal Revenue Code, for failure to render a true and
complete return on the gross quarterly sales receipts and earnings in his
business as producer of saplings, and to pay the percentage tax due thereon,
for the quarter ending on March 31, 1973; 7
(5) Criminal Case No. 1964 Violation of Sec. 183 (a), in relation to Secs. 186
and 209 of the National Internal Revenue Code, for failure to render a true and
complete return on the gross quarterly sales, receipts and earnings in his
business as producer of banana saplings for the quarter ending on June 30,
1973, and to pay the percentage tax due thereon; 8
(6) Criminal Case No. 1965 Violation of Sec. 183 (a), in relation to Secs. 186
and 209 of the National Internal Revenue Code, for failure to render a true and
complete return on the gross quarterly sales, receipts and earnings as
producer of banana saplings, for the quarter ending on September 30, 1973,
and to pay the percentage tax due thereon. 9
On September 16, 1975, the petitioner filed a motion to quash the informations upon the
grounds that: (1) the informations are null and void for want of authority on the part of the

State Prosecutor to initiate and prosecute the said cases; and (2) the trial court has no
jurisdiction to take cognizance of the above-entitled cases in view of his pending protest
against the assessment made by the BIR Examiner. 10 However, the trial court denied the
motion on October 22, 1975. 11 Whereupon, the petitioner filed the instant recourse. As
prayed for, a temporary restraining order was issued by the Court, ordering the respondent
Judge from further proceeding with the trial and hearing of Criminal Case Nos. 1960, 1961,
1962, 1963, 1964, and 1965 of the Court of First Instance of Davao, all entitled: "People of
the Philippines, plaintiff, versus Quirico Ungab, accused."
The petitioner seeks the annulment of the informations filed against him on the ground that
the respondent State Prosecutor is allegedly without authority to do so. The petitioner
argues that while the respondent State Prosecutor may initiate the investigation of and
prosecute crimes and violations of penal laws when duly authorized, certain requisites,
enumerated by this Court in its decision in the case of Estrella vs. Orendain, 12 should be
observed before such authority may be exercised; otherwise, the provisions of the Charter
of Davao City on the functions and powers of the City Fiscal will be meaningless because
according to said charter he has charge of the prosecution of all crimes committed within
his jurisdiction; and since "appropriate circumstances are not extant to warrant the
intervention of the State Prosecution to initiate the investigation, sign the informations and
prosecute these cases, said informations are null and void." The ruling adverted to by the
petitioner reads, as follows: t.hqw
In view of all the foregoing considerations, it is the ruling of this Court that
under Sections 1679 and 1686 of the Revised Administrative Code, in any
instance where a provincial or city fiscal fails, refuses or is unable, for any
reason, to investigate or prosecute a case and, in the opinion of the Secretary
of Justice it is advisable in the public interest to take a different course of
action, the Secretary of Justice may either appoint as acting provincial or city
fiscal to handle the investigation or prosecution exclusively and only of such
case, any practicing attorney or some competent officer of the Department of
Justice or office of any city or provincial fiscal, with complete authority to act
therein in all respects as if he were the provincial or city fiscal himself, or
appoint any lawyer in the government service, temporarily to assist such city
of provincial fiscal in the discharge of his duties, with the same complete
authority to act independently of and for such city or provincial fiscal provided
that no such appointment may be made without first hearing the fiscal
concerned and never after the corresponding information has already been
filed with the court by the corresponding city or provincial fiscal without the
conformity of the latter, except when it can be patently shown to the court
having cognizance of the case that said fiscal is intent on prejudicing the
interests of justice. The same sphere of authority is true with the prosecutor
directed and authorized under Section 3 of Republic Act 3783, as amended
and/or inserted by Republic Act 5184. The observation in Salcedo vs. Liwag,
supra, regarding the nature of the power of the Secretary of Justice over
fiscals as being purely over administrative matters only was not really
necessary, as indicated in the above relation of the facts and discussion of the
legal issues of said case, for the resolution thereof. In any event, to any extent
that the opinion therein may be inconsistent herewith the same is hereby
modified.

The contention is without merit. Contrary to the petitioner's claim, the rule therein
established had not been violated. The respondent State Prosecutor, although believing
that he can proceed independently of the City Fiscal in the investigation and prosecution of
these cases, first sought permission from the City Fiscal of Davao City before he started the
preliminary investigation of these cases, and the City Fiscal, after being shown
Administrative Order No. 116, dated December 5, 1974, designating the said State
Prosecutor to assist all Provincial and City fiscals throughout the Philippines in the
investigation and prosecution of all violations of the National Internal Revenue Code, as
amended, and other related laws, graciously allowed the respondent State Prosecutor to
conduct the investigation of said cases, and in fact, said investigation was conducted in the
office of the City Fiscal. 13
The petitioner also claims that the filing of the informations was precipitate and premature
since the Commissioner of Internal Revenue has not yet resolved his protests against the
assessment of the Revenue District Officer; and that he was denied recourse to the Court of
Tax Appeals.
The contention is without merit. What is involved here is not the collection of taxes where
the assessment of the Commissioner of Internal Revenue may be reviewed by the Court of
Tax Appeals, but a criminal prosecution for violations of the National Internal Revenue Code
which is within the cognizance of courts of first instance. While there can be no civil action
to enforce collection before the assessment procedures provided in the Code have been
followed, there is no requirement for the precise computation and assessment of the tax
before there can be a criminal prosecution under the Code. t.hqw
The contention is made, and is here rejected, that an assessment of the
deficiency tax due is necessary before the taxpayer can be prosecuted
criminally for the charges preferred. The crime is complete when the violator
has, as in this case, knowingly and willfully filed fraudulent returns with intent
to evade and defeat a part or all of the tax. 14
An assessment of a deficiency is not necessary to a criminal prosecution for
willful attempt to defeat and evade the income tax. A crime is complete when
the violator has knowingly and willfuly filed a fraudulent return with intent to
evade and defeat the tax. The perpetration of the crime is grounded upon
knowledge on the part of the taxpayer that he has made an inaccurate return,
and the government's failure to discover the error and promptly to assess has
no connections with the commission of the crime. 15
Besides, it has been ruled that a petition for reconsideration of an assessment may affect
the suspension of the prescriptive period for the collection of taxes, but not the prescriptive
period of a criminal action for violation of law. 16 Obviously, the protest of the petitioner
against the assessment of the District Revenue Officer cannot stop his prosecution for
violation of the National Internal Revenue Code. Accordingly, the respondent Judge did not
abuse his discretion in denying the motion to quash filed by the petitioner.
WHEREFORE, the petition should be, as it is hereby dismissed. The temporary restraining
order heretofore issued is hereby set aside. With costs against the petitioner.

SO ORDERED.

G.R. No. 177982

October 17, 2008

FITNESS BY DESIGN, INC., petitioner,


vs.
COMMISSIONER ON INTERNAL REVENUE, respondent.
DECISION
CARPIO MORALES, J.:
On March 17, 2004, the Commissioner on Internal Revenue (respondent) assessed
Fitness by Design, Inc. (petitioner) for deficiency income taxes for the tax year 1995
in the total amount of P10,647,529.69.1 Petitioner protested the assessment on the
ground that it was issued beyond the three-year prescriptive period under Section
203 of the Tax Code.2 Additionally, petitioner claimed that since it was incorporated
only on May 30, 1995, there was no basis to assume that it had already earned
income for the tax year 1995.3
On February 1, 2005, respondent issued a warrant of distraint and/or levy against
petitioner,4 drawing petitioner to file on March 1, 2005 a Petition for Review (with
Motion to Suspend Collection of Income Tax, Value Added Tax, Documentary Stamp
Tax and Surcharges and Interests subject of this Petition) 5 before the Court of Tax
Appeals (CTA) before which it reiterated its defense of prescription. The petition was
docketed as CTA Case No. 7160.
In his Answer,6 respondent alleged:
The right of the respondent to assess petitioner for deficiency income tax, VAT and
Documentary Stamp Tax for the year 1995 has not prescribed pursuant to Section
222(a) of the 1997 Tax Code.Petitioners 1995 Income Tax Return (ITR) filed on April
11, 1996 was false and fraudulent for its deliberate failure to declare its true sales.
Petitioner declared in its 1995 Income Tax Return that it was on its pre-operation
stage and has not declared its income. Investigation by the revenue officers of the
respondent, however, disclosed that it has been operating/doing business and
had sales operations for the year 1995 in the total amount of P7,156,336.08 which it
failed to report in its 1995 ITR. Thus, for the year 1995, petitioner filed a fraudulent
annual income return with intent to evade tax. Likewise, petitioner failed to file
Value-Added Tax (VAT) Return and reported the amount of P7,156,336.08 as its gross
sales for the year 1995. Hence, for failure to file a VAT return and for filing a
fraudulent income tax return for the year 1995, the corresponding taxes
may be assessed at any time within ten (10) years after the discovery of
such omission or fraud pursuant to Section 222(a) of the 1997 Tax Code.

The subject deficiency tax assessments have already become final, executory and
demandable forfailure of the petitioner to file a protest within the reglementary
period provided for by law. The "alleged protest" allegedly filed on June 25, 2004 at
the Legal Division, Revenue Region No. 8, Makati City is nowhere to be found in the
BIR Records nor reflected in the Record Book of the Legal Division as normally done
by our receiving clerk when she receive[s] any document. The respondent, therefore,
has legal basis to collect the tax liability either by distraint and levy or civil
action.7 (Emphasis and underscoring supplied)
The aforecited Section 222(a)8 of the 1997 Tax Code provides:
In the case of a false or fraudulent return with intent to evade tax or of failure to file
a return, the tax may be assessed, or a proceeding in court for the collection of such
tax may be filed without assessment, at any time within ten (10) years after the
discovery of the falsity, fraud, or omission: Provided, That in a fraud assessment
which has become final and executory, the fact of fraud shall be judicially taken
cognizance of in the civil or criminal action for the collection thereof. (Underscoring
supplied)
The Bureau of Internal Revenue (BIR) in fact filed on March 10, 2005 a criminal
complaint before the Department of Justice against the officers and accountant of
petitioner for violation of the provisions of "The National Internal Revenue Code of
1977, as amended,9 covering the taxable year 1995." The criminal complaint was
docketed as I.S. No. 2005-203.
On motion of petitioner in CTA Case No. 7160, a preliminary hearing on the issue of
prescription10 was conducted during which petitioners former bookkeeper attested
that a former colleague certified public accountant Leonardo Sablan (Sablan)
illegally took custody of petitioners accounting records, invoices, and official
receipts and turned them over to the BIR. 11
On petitioners request, a subpoena ad testificandum was issued to Sablan for the
hearing before the CTA scheduled on September 4, 2006 but he failed to appear. 12
Petitioner thus requested for the issuance of another subpoena ad testificandum to
Sablan for the hearing scheduled on October 23, 2006, 13 and of subpoena duces
tecum to the chief of the National Investigation Division of the BIR for the production
of the Affidavit of the Informer bearing on the assessment in question. 14 Petitioners
requests were granted.15
During the scheduled hearing of the case on October 23, 2006, on respondents
counsels manifestation that he was not furnished a copy of petitioners motion for
the issuance of subpoenaes, the CTA ordered petitioner to file a motion for the
issuance of subpoenas and to furnish respondents counsel a copy
thereof.16 Petitioner complied with the CTA order.17

In a related move, petitioner submitted written interrogatories addressed to Sablan


and to Henry Sarmiento and Marinella German, revenue officers of the National
Investigation Division of the BIR.18
By Resolution19 of January 15, 2007, the CTA denied petitioners Motion for
Issuance of Subpoenas and disallowed the submission by petitioner of written
interrogatories to Sablan, who is not a party to the case, and the revenue officers, 20 it
finding that the testimony, documents, and admissions sought are not
relevant.21 Besides, the CTA found that to require Sablan to testify would violate
Section 2 of Republic Act No. 2338, as implemented by Section 12 of Finance
Department Order No. 46-66, proscribing the revelation of identities of informers of
violations of internal revenue laws, except when the information is proven to be
malicious or false.22
In any event, the CTA held that there was no need to issue a subpoena duces
tecum to obtain the Affidavit of the Informer as the same formed part of the BIR
records of the case, the production of which had been ordered by it. 23
Petitioners Motion for Reconsideration24 of the CTA Resolution of January 15, 2007
was denied,25 hence, the present Petition for Certiorari 26 which imputes grave abuse
of discretion to the CTA
I.
x x x in holding that the legality of the mode of acquiring the documents which are
the bases of the above discussed deficiency tax assessments, the subject matter of
the Petition for Review now pending in the Honorable Second Division, is not material
and relevant to the issue of prescription.
II.
x x x in holding that Mr. Leonardo Sablans testimony, if allowed, would violate RA
2338 which prohibits the BIR to reveal the identity of the informer since 1) the
purpose of the subpoena is to elicit from him the whereabouts of the original
accounting records, documents and receipts owned by the Petitioner and not to
discover if he is the informer since the identity of the informer is not relevant to the
issues raised; 2) RA 2338 cannot legally justify violation of the Petitioners property
rights by a person, whether he is an informer or not, since such RA cannot allow such
invasion of property rights otherwise RA 2338 would run counter to the constitutional
mandate that "no person shall be deprive[d] of life, liberty or property without due
process of law."
III.
x x x in holding that the issuance of subpoena ad testificandum would constitute a
violation of the prohibition to reveal the identity of the informer because compliance

with such prohibition has been rendered moot and academic by the voluntary
admissions of the Respondent himself.
IV.
x x x in holding that the constitutional right of an accused to examine the witness
against him does not exist in this case. The Petitioners liability for tax deficiency
assessment which is the main issue in the Petition for Review is currently pending at
the Honorable Second Division. Therefore, it is a prejudicial question raised in the
criminal case filed by the herein Respondent against the officers of the Petitioner
with the Department of Justice.
V.
x x x in dismissing the request for subpoena ad testificandum because the
Opposition thereto submitted by the Respondent was not promptly filed as provided
by the Rules of Court thus, it is respectfully submitted that, Respondent has waived
his right to object thereto.
VI.
x x x when the Honorable Court of Tax Appeals ruled that the purpose of the
Petitioner in requesting for written interrogatories is to annoy, embarrass, or oppress
the witness because such ruling has no factual basis since Respondent never alleged
nor proved that the witnesses to whom the interrogatories are addressed will be
annoyed, embarrassed or oppressed; besides the only obvious purpose of the
Petitioner is to know the whereabouts of accounting records and documents which
are in the possession of the witnesses to whom the interrogatories are directed and
to ultimately get possession thereof. Granting without admitting that there is
annoyance, embarrassment or oppression; the same is not unreasonable.
VII.
x x x when it failed to rule that the BIR officers and employees are not covered by
the prohibition under RA 2338 and do not have the authority to withhold from the
taxpayer documents owned by such taxpayer.
VIII.
x x x when it required the "clear and unequivocal proof" of relevance of the
documents as a condition precedent for the issuance of subpoena duces tecum.
IX.
x x x when it quashed the subpoena duces tecum as the Honorable Court had issued
an outstanding order to the Respondent to certify and forward to the CTA all the

records of the case because up to the date of this Petition the BIR records have not
been submitted yet to the CTA.27
Grave abuse of discretion implies such capricious and whimsical exercise of
judgment as equivalent to lack of jurisdiction or, in other words, when the power is
exercised in an arbitrary or despotic manner by reason of passion or personal
hostility, and it must be so patent and gross as to amount to an evasion of positive
duty or a virtual refusal of duty enjoined or to act at all in contemplation of law. 28
The Court finds that the issuance by the CTA of the questioned resolutions was not
tainted by arbitrariness.
The fact that Sablan was not a party to the case aside, the testimonies, documents,
and admissions sought by petitioner are not indeed relevant to the issue before the
CTA. For in requesting the issuance of the subpoenas and the submission of written
interrogatories, petitioner sought to establish that its accounting records and related
documents, invoices, and receipts which were the bases of the assessment against it
were illegally obtained. The only issues, however, which surfaced during the
preliminary hearing before the CTA, were whether respondents issuance of
assessment against petitioner had prescribed and whether petitioners tax return
was false or fraudulent.
Besides, as the CTA held, the subpoenas and answers to the written interrogatories
would violate Section 2 of Republic Act No. 2338 as implemented by Section 12 of
Finance Department Order No. 46-66.
Petitioner claims, however, that it only intended to elicit information on the
whereabouts of the documents it needs in order to refute the assessment, and not to
disclose the identity of the informer.29 Petitioners position does not persuade. The
interrogatories addressed to Sablan and the revenue officers show that they were
intended to confirm petitioners belief that Sablan was the informer. Thus the
questions for Sablan read:
1. Under what circumstances do you know petitioner corporation? Please state in
what capacity, the date or period you obtained said knowledge.
2. Do you know a Ms. Elnora Carpio, who from 1995 to the early part of 1996 was the
book keeper of petitioner? Please state how you came to know of Ms. Carpio.
3. At the time that Ms. Carpio was book keeper of petitioner did she consult you or
show any accounting documents and records of petitioner?
4. What documents, if any, did you obtain from petitioner?

5. Were these documents that you obtained from petitioner submitted to the Bureau
of Internal Revenue (BIR)? Please describe said documents and under what
circumstances the same were submitted.
6. Was the consent of the petitioner, its officers or employees obtained when the
documents that you obtained were submitted to the BIR? Please state when and
from whom the consent was obtained.
7. Did you execute an affidavit as an informer in the assessment which was issued
by the BIR against petitioner for the tax year 1995 and other years? 30 (Underscoring
supplied)
while the questions for the revenue officers read:
1. Where did you obtain the documents, particularly the invoices and official
receipts, which [were] used by your office as evidence and as basis of the
assessment for deficiency income tax and value added tax for the tax year 1995
issued against petitioner?
2. Do you know Mr. Leonardo Sablan? Please state under what circumstance you
came to know Mr. Sablan?31 (Underscoring supplied)
Petitioner impugns the manner in which the documents in question reached the BIR,
Sablan having allegedly submitted them to the BIR without its (petitioners) consent.
Petitioners lack of consent does not, however, imply that the BIR obtained them
illegally or that the information received is false or malicious. Nor does the lack of
consent preclude the BIR from assessing deficiency taxes on petitioner based on the
documents. Thus Section 5 of the Tax Code provides:
In ascertaining the correctness of any return, or in making a return when none has
been made, or in determining the liability of any person for any internal revenue tax,
or in collecting any such liability, or in evaluating tax compliance, the Commissioner
is authorized:
(A) To examine any book, paper, record or other data which may be relevant or
material to such query;
(B) To obtain on a regular basis from any person other than the person whose
internal revenue tax liability is subject to audit or investigation, or from any
office or officer of the national and local governments, government agencies and
instrumentalities, including the Bangko Sentral ng Pilipinasand government-owned
and controlled corporations, any information such as, but not limited to, costs and
volume of production, receipts or sales and gross incomes of taxpayers, and the
names, addresses, and financial statements of corporations, mutual fund companies,
insurance companies, regional operating headquarters of multinational companies,

joint accounts, associations, joint ventures or consortia and registered partnerships


and their members;
(C) To summon the person liable for tax or required to file a return, or any officer or
employee of such person, or any person having possession, custody, or care of
the books of accounts and other accounting records containing entries
relating to the business of the person liable for tax, or any other person , to
appear before the Commissioner or his duly authorized representatives at a time and
place specified in the summons and to produce such books, papers, records, or other
data, and to give testimony;
(D) To take such testimony of the person concerned, under oath, as may be relevant
or material to such inquiry; and
(E) To cause revenue officers and employees to make a canvass from time to time of
any revenue district or region and inquire after and concerning all persons therein
who may be liable to pay any internal revenue tax, and all persons owning or having
the care, management or possession of any object with respect to which a tax is
imposed.
x x x x (Emphasis and underscoring supplied)
The law thus allows the BIR access to all relevant or material records and data in the
person of the taxpayer,32 and the BIR can accept documents which cannot be
admitted in a judicial proceeding where the Rules of Court are strictly observed. 33 To
require the consent of the taxpayer would defeat the intent of the law to help the BIR
assess and collect the correct amount of taxes.
Petitioners invocation of the rights of an accused in a criminal prosecution to cross
examine the witness against him and to have compulsory process issued to secure
the attendance of witnesses and the production of other evidence in his behalf does
not lie. CTA Case No. 7160 is not a criminal prosecution, and even granting that it is
related to I.S. No. 2005-203, the respondents in the latter proceeding are the officers
and accountant of petitioner-corporation, not petitioner. From the complaint and
supporting affidavits in I.S. No. 2005-203, Sablan does not even appear to be a
witness against the respondents therein.34
AT ALL EVENTS, issuance of subpoena duces tecum for the production of the
documents requested by the petitioner which documents petitioner claims to be
crucial to its defense35 is unnecessary in view of the CTA order for respondent to
certify and forward to it all the records of the case. 36 If the order has not been
complied with, the CTA can enforce it by citing respondent for indirect contempt. 37
WHEREFORE, in light of the foregoing disquisition, the petition is DISMISSED.
Costs against petitioner.

SO ORDERED.
G.R. No. 81446 August 18, 1988
BONIFACIA SY PO, petitioner,
vs.
HONORABLE COURT OF TAX APPEALS AND HONORABLE COMMISSIONER OF
INTERNAL REVENUE,respondents.
Basilio E. Duaban for petitioner.

SARMIENTO, J.:
This is an appeal from the decision 1 of the respondent Court of Tax Appeals, dated
September 30,1987, which affirmed an earlier decision of the correspondent
Commissioner of Internal Revenue in assessment letters dated August 16, 1972 and
September 26, 1972, which ordered the payment by the petitioner of deficiency
income tax for 1966 to 1970 in the amount of P7,154,685.16 and deficiency specific
tax for January 2, 1964 to January 19, 1972, in the amount of P5,595,003.68.
We adopt the respondent court's finding of facts, to wit:
Petitioner is the widow of the late Mr. Po Bien Sing who died on
September 7, 1980. In the taxable years 1964 to 1972, the deceased Po
Bien Sing was the sole proprietor of Silver Cup Wine Factory (Silver Cup
for brevity), Talisay, Cebu. He was engaged in the business of
manufacture and sale of compounded liquors, using alcohol and other
ingredients as raw materials.
On the basis of a denunciation against Silver Cup allegedly "for tax
evasion amounting to millions of pesos" the then Secretary of Finance
Cesar Virata directed the Finance-BIR--NBI team constituted under
Finance Department Order No. 13-70 dated February 19, 1971 (Exh- 3,
pp. 532-553, Folder II, BIR rec.) to conduct the corresponding
investigation in a memorandum dated April 2, 1971 (p. 528, Folder II,
BIR rec.). Accordingly, a letter and a subpoena duces tecum dated April
13,1971 and May 3,1971, respectively, were issued against Silver Cup
requesting production of the accounting records and other related
documents for the examination of the team. (Exh. 11, pp. 525-526,
Folder II, BIR rec.). Mr. Po Bien Sing did not produce his books of
accounts as requested (Affidavit dated December 24, 1971 of Mr.
Generoso. Quinain of the team, p. 525, Folder H, BIR rec.). This
prompted the team with the assistance of the PC Company, Cebu City,
to enter the factory bodega of Silver Cup and seized different brands,

consisting of 1,555 cases of alcohol products. (Exh. 22, Memorandum


Report of the Team dated June 5, 1971, pp. 491-492, Folder II, BIR rec.).
The inventory lists of the seized alcohol products are contained in
Volumes I, II, III, IV and V (Exhibits 14, 15, 16, 17, and 18, respectively,
BIR rec.). On the basis of the team's report of investigation, the
respondent Commissioner of Internal Revenue assessed Mr. Po Bien
Sing deficiency income tax for 1966 to 1970 in the amount of
P7,154,685.16 (Exh. 6 pp. 17-19, Folder I, BIR rec.) and for deficiency
specific tax for January 2,1964 to January 19, 1972 in the amount of
P5,595,003.68 (Exh. 8, p. 107, Folder I, BIR rec.).
Petitioner protested the deficiency assessments through letters dated
October 9 and October 30, 1972 (Exhs. 7 and 9, pp. 27-28; pp. 152-159,
respectively, BIR rec.), which protests were referred for reinvestigation.
The corresponding report dated August 13, 1981 (Exh. 1 0, pp. 355,
Folder I, BIR rec.) recommended the reiteration of the assessments in
view of the taxpayer's persistent failure to present the books of
accounts for examination (Exh. 8, p. 107, Folder I, BIR rec.), compelling
respondent to issue warrants of distraint and levy on September 10,
1981 (Exh. 11, p. 361, Folder I, BIR rec.).
The warrants were admittedly received by petitioner on October 14,
1981 (Par. IX, Petition; admitted par. 2, Answer), which petitioner
deemed respondent's decision denying her protest on the subject
assessments. Hence, petitioner's appeal on October 29,1981. 2
The petitioner assigns the following errors:
I
RESPONDENT INTENTIONALLY ERRED IN HOLDING THAT PETITIONER HAS NOT
PRESENTED ANY EVIDENCE OF RELEVANCE AND COMPETENCE REQUIRED TO BASH
THE TROUBLING DISCREPANCIES AND SQUARE THE ISSUE OF ILLEGALITY POSITED
ON THE SUBJECT ASSESSMENTS.
II
RESPONDENT COURT OF TAX APPEALS PALPABLY ERRED IN DECIDING THE CASE IN A
WAY CONTRARY TO THE DOCTRINES ALREADY LAID DOWN BY THIS COURT.
III
RESPONDENT COURT OF TAX APPEALS GRAVELY ERRED IN FINDING PO BEEN SING TO
HAVE INCURRED THE ALLEGED DEFICIENCY TAXES IN QUESTION. 3
We affirm.

Settled is the rule that the factual findings of the Court of Tax Appeals are binding
upon this Honorable Court and can only be disturbed on appeal if not supported by
substantial evidence. 4
The assignments of errors boils down to a single issue previously raised before the
respondent Court, i.e., whether or not the assessments have valid and legal bases.
The applicable legal provision is Section 16(b) of the National Internal Revenue Code
of 1977 as amended. It reads:
Sec. 16. Power of the Commissioner of Internal Revenue to make
assessments.
xxx xxx xxx
(b) Failure to submit required returns, statements, reports and other
documents. - When a report required by law as a basis for the
assessment of an national internal revenue tax shall not be forthcoming
within the time fixed by law or regulation or when there is reason to
believe that any such report is false, incomplete, or erroneous, the
Commissioner of Internal Revenue shall assess the proper tax on the
best evidence obtainable.
In case a person fails to file a required return or other document at the
time prescribed by law, or willfully or otherwise, files a false or
fraudulent return or other documents, the Commissioner shall make or
amend the return from his own knowledge and from such information as
he can obtain through testimony or otherwise, which shall be prima
facie correct and sufficient for all legal purposes.
The law is specific and clear. The rule on the "best evidence obtainable" applies
when a tax report required by law for the purpose of assessment is not available or
when the tax report is incomplete or fraudulent.
In the instant case, the persistent failure of the late Po Bien Sing and the herein
petitioner to present their books of accounts for examination for the taxable years
involved left the Commissioner of Internal Revenue no other legal option except to
resort to the power conferred upon him under Section 16 of the Tax Code.
The tax figures arrived at by the Commissioner of Internal Revenue are by no means
arbitrary. We reproduce the respondent court's findings, to wit:
As thus shown, on the basis of the quantity of bottles of wines seized
during the raid and the sworn statements of former employees Messrs.
Nelson S. Po and Alfonso Po taken on May 26, and 27,1971,
respectively, by the investigating team in Cebu City (Exhs. 4 and 5, pp.

514-517, pp. 511-513, Folder 11, BIR rec.), it was ascertained that the
Silver Cup for the years 1964 to 1970, inclusive, utilized and consumed
in the manufacture of compounded liquours and other products 20,105
drums of alcohol as raw materials 81,288,787 proof liters of alcohol. As
determined, the total specific tax liability of the taxpayer for 1964 to
1971 amounted to P5,593,003.68 (Exh. E, petition, p. 10, CTA rec.)
Likewise, the team found due from Silver Cup deficiency income taxes
for the years 1966 to 1970 inclusive in the aggregate sum of
P7,154,685.16, as follows:
1966 P207,636.24
1967 645,335.04
1968 1,683,588.48
1969 1,589,622.48
1970 3,028,502.92
Total amount due.
and collectible P7,154,685.16

The 50% surcharge has been imposed, pursuant to Section 72 * of the Tax Code and
tax 1/2% monthly interest has likewise been imposed pursuant to the provision of
Section 51(d) ** of the Tax Code (Exh. O, petition). 5
The petitioner assails these assessments as wrong.
In the case of Collector of Internal Revenue vs. Reyes,

we ruled:

Where the taxpayer is appealing to the tax court on the ground that the
Collector's assessment is erroneous, it is incumbent upon him to prove
there what is the correct and just liability by a full and fair disclosure of
all pertinent data in his possession. Otherwise, if the taxpayer confines
himself to proving that the tax assessment is wrong, the tax court
proceedings would settle nothing, and the way would be left open for
subsequent assessments and appeals in interminable succession.
Tax assessments by tax examiners are presumed correct and made in good faith.
The taxpayer has the duty to prove otherwise. 7 In the absence of proof of any
irregularities in the performance of duties, an assessment duly made by a Bureau of

Internal Revenue examiner and approved by his superior officers will not be
disturbed. 8 All presumptions are in favor of the correctness of tax assessments.

On the whole, we find that the fraudulent acts detailed in the decision under review
had not been satisfactorily rebutted by the petitioner. There are indeed clear
indications on the part of the taxpayer to deprive the Government of the taxes due.
The Assistant Factory Superintendent of Silver Cup, Nelson Po gave the following
testimony:
Annexes "A", "A-1 " to "A-17" show that from January to December
1970, Silver Cup had used in production 189 drums of untaxed distilled
alcohol and 3,722 drums of untaxed distilled alcohol. Can you tell us
how could this be possible with the presence of a revenue inspector in
the premises of Silver Cup during working hours?
Actually, the revenue inspector or storekeeper comes
around once a week on the average. Sometimes, when the
storekeeper is around in the morning and Po Bein Sing
wants to operate with untaxed alcohol as raw materials, Po
Bien Sing tells the storekeeper to go home because the
factory is not going to operate for the day. After the
storekeeper leaves, the illegal operation then begins.
Untaxed alcohol is brought in from Cebu Alcohol Plant into
the compound of Silver Cup sometimes at about 6:00 A.M.
or at 12:00 noon or in the evening or even at mid-night
when the storekeeper is not around. When the storekeeper
comes, he sees nothing because untaxed alcohol is
brought directly to, and stored at, a secret tunnel within
the bodega itself inside the compound of Silver Cup.
In the same vein, the factory personnel manager testified that false
entries were entered in the official register book: thus,
A As factory personnel manager and all-around handy
man of Po Bien Sing, owner of Silver Cup, these labels
were entrusted to me to make the false entries in the
official register book of Silver Cup, which I did under the
direction of Po Bien Sing. (Sworn statement, p. 512, Folder
II, BIR rec.) 10 (Emphasis ours)
The existence of fraud as found by the respondents can not be lightly set aside
absent substantial evidence presented by the petitioner to counteract such finding.
The findings of fact of the respondent Court of Tax Appeals are entitled to the highest
respect. 11 We do not find anything in the questioned decision that should disturb this
long-established doctrine.

WHEREFORE, the Petition is DENIED. The Decision of the respondent Court of Tax
Appeals is hereby AFFIRMED. Costs against the petitioner.
SO ORDERED.
G.R. No. L-13656

January 31, 1962

COLLECTOR OF INTERNAL REVENUE, (now Commissioner), petitioner,


vs.
ALBERTO D. BENIPAYO, respondent.
Office of the Solicitor General for petitioner.
Carlos J. Antiporda for respondent.
DIZON, J.:
This is an appeal taken by the Collector of Internal Revenue from the decision of the
Court of Tax Appeals dated January 23, 1948, reversing the one rendered by the
former, thereby relieving respondent Alberto D. Benipayo from the payment of the
deficiency amusement tax assessed against him in the total amount of P12,093.45.
Respondent is the owner and operator of the Lucena Theater located in the
municipality of Lucena, Quezon. On October 3, 1953 Internal Revenue Agent Romeo
de Guia investigated respondent's amusement tax liability in connection with the
operation of said theater during the period from August, 1952 to September, 1953.
On October 15, 1953 De Guia submitted his report to the Provincial Revenue Agent
to the effect that respondent had disproportionately issued tax-free 20-centavo
children's tickets. His finding was that during the years 1949 to 1951 the average
ratio of adults and children patronizing the Lucena Theater was 3 to 1, i.e., for every
three adults entering the theater, one child was also admitted, while during the
period in question, the proportion is reversed - three children to one adult. From this
he concluded that respondent must have fraudulently sold two tax-free 20-centavo
tickets, in order to avoid payment of the amusement tax prescribed in Section 260 of
the National Internal Revenue Code. Based on the average ratio between adult and
children attendance in the past years, Examiner de Guia recommended a deficiency
amusement tax assessment against respondent in the sum of P11,193.45, inclusive
of 25% surcharge, plus a suggested compromise penalty of P900.00 for violation of
section 260 of the National Internal Revenue Code, or a total sum of P12,093.45
covering the period from August, 1952 to September, 1953 inclusive. On July 14,
1954, petitioner issued a deficiency amusement tax assessment against respondent,
demanding from the latter the payment of the total sum of P12,152.93 within thirty
days from receipt thereof. On August 16, 1954, respondent filed the corresponding
protest with the Conference Staff of the Bureau of Internal Revenue. After due
hearing, the Conference Staff submitted to petitioner Collector of Internal Revenue
its finding to the effect that the "meager reports of these fieldmen (Examiner de
Guia and the Provincial Revenue Agent of Quezon) are mere presumptions and

conclusions, devoid of findings of the fact of the alleged fraudulent practices of the
herein taxpayer". In view thereof, and as recommended by the Conference Staff,
petitioner referred the case back to the Provincial Revenue Agent of Quezon for
further investigation. The report submitted by Provincial Revenue Officer H.I.
Bernardo after this last investigation partly reads as follows:.
The returns from July 1 to July 11, showed that 31.43% of the entire audience
of 12,754 consisted of adults, the remaining 68.57% of children. During this
said period due, perhaps, to the absence of agents in the premises, subject
taxpayer was able to manipulate the issuance of tickets in the way and
manner alleged in Asst. De Guia's indorsement report mentioned above. But
during the period from July 14 to July 24, 1955, when agents of this Office
supervised in the sales of admission tickets the sales for adults soared
upwards to 76% while that for children dropped correspondingly to 24%.
It is opined without fear of contradiction that the ratio of three (3) adults to
every one (1) child in the audience or a proportion of 75:25 as reckoned in
Asst. De Guia's indorsement report to this Office's new findings of a proportion
of 76:24, represents and conveys the true picture of the situation under the
law of averages, provided that the film being shown is not a children's show.
There is no hard and fast rule in this regard, but this findings would seem to
admit no contradiction.
Please note that the new findings of this Office is not a direct proof of what
has transpired during the period investigated by Asst. De Guia and now
pending before the Conference Staff", . . (Exh. 3, BIR Record, p. 137-138).
After considering said report, the Conference Staff of the Bureau of Internal Revenue
recommended to the Collector of Internal Revenue the issuance of the deficiency
amusement tax assessment in question.
The only issue in this appeal is whether or not there is sufficient evidence in the
record showing that respondent, during the period under review, sold and issued to
his adult customers two tax-free 20-centavo children's tickets, instead of one 40centavo ticket for each adult customer; to cheat or defraud the Government. On this
question the Court of Tax Appeals said the following in the appealed decision:.
To our mind, the appealed decision has no factual basis and must be reversed.
An assessment fixes and determines the tax liability of a taxpayer. As soon as
it is served, an obligation arises on the part of the taxpayer concerned to pay
the amount assessed and demanded. Hence, assessments should not be
based on mere presumptions no matter how reasonable or logical said
presumptions may be. Assuming arguendothat the average ratio of adults and
children patronizing the Lucena Theater from 1949 to 1951 was 3 to 1, the
same does not give rise to the inference that the same conditions existed
during the years in question (1952 and 1953). The fact that almost the same

ratio existed during the month of July, 1955 does not provide a sufficient
inference on the conditions in 1952 and 1953. . .
In order to stand the test of judicial scrutiny, the assessment must be based
on actual facts. The presumption of correctness of assessment being a mere
presumption cannot be made to rest on another presumption that the
circumstances in 1952 and 1953 are presumed to be the same as those
existing in 1949 to 1951 and July 1955. In the case under consideration there
are no substantial facts to support the assessment in question. ...
A review of the records has not disclosed anything sufficient to justify a reversal of
the above finding made by the Court of Tax Appeals. It should be borne in mind that
to sustain the deficiency tax assessed against respondent would amount, in effect, to
a finding that he had, for a considerable period of time, cheated and defrauded the
government by selling to each adult patron two children's tax-free tickets instead of
one ticket subject to the amusement tax provided for in Section 260 of the National
Internal Revenue Code. Fraud is a serious charge and, to be sustained, it must be
supported by clear and convincing proof which, in the present case, is lacking.
The claim that respondent admitted having resorted to the anomalous practice
already mentioned is not entirely correct. What respondent appears to have
admitted was that during a certain limited period he had adopted a sort of rebate
system applicable to cases where adults and children came in groups and were al
anomalous practice already mentioned is not entirely correct. What respondent
appears to have admitted was that during a certain limited period he had adopted a
sort of rebate ystem applicable to cases where adults and children came in group
and were all charged 20 centavo admission tickets. This practice was, however,
discontinued when he was informed by the Bureau of Internal Revenue that it was
not in accordance with law.
WHEREFORE, the appealed judgment is hereby affirmed with costs.
G.R. No. L-36181 October 23, 1982
MERALCO SECURITIES CORPORATION (now FIRST PHILIPPINE HOLDINGS
CORPORATION), petitioner,
vs.
HON. VICTORINO SAVELLANO and ASUNCION BARON VDA. DE MANIAGO, et
al., as heirs of the late Juan G. Maniago, respondents.
G.R. No. L-36748 October 23, 1982
COMMISSIONER OF INTERNAL REVENUE, petitioner,
vs.
HON. VICTORINO SAVELLANO and ASUNCION BARON VDA. DE MANIAGO, et
al., as heirs of the late Juan G. Maniago, respondents.

G.R. No. L-36181


San Juan, Africa, Gonzales & San Agustin for petitioner.
Ramon A. Gonzales for respondents.

TEEHANKEE, J.:
These are original actions for certiorari to set aside and annul the writ of mandamus
issued by Judge Victorino A. Savellano of the Court of First Instance of Manila in Civil
Case No. 80830 ordering petitioner Meralco Securities Corporation (now First
Philippine Holdings Corporation) to pay, and petitioner Commissioner of Internal
Revenue to collect from the former, the amount of P51,840,612.00, by way of
alleged deficiency corporate income tax, plus interests and surcharges due thereon
and to pay private respondents 25% of the total amount collectible as informer's
reward.
On May 22, 1967, the late Juan G. Maniago (substituted in these proceedings by his
wife and children) submitted to petitioner Commissioner of Internal Revenue
confidential denunciation against the Meralco Securities Corporation for tax evasion
for having paid income tax only on 25 % of the dividends it received from the Manila
Electric Co. for the years 1962-1966, thereby allegedly shortchanging the
government of income tax due from 75% of the said dividends.
Petitioner Commissioner of Internal Revenue caused the investigation of the
denunciation after which he found and held that no deficiency corporate income tax
was due from the Meralco Securities Corporation on the dividends it received from
the Manila Electric Co., since under the law then prevailing (section 24[a] of the
National Internal Revenue Code) "in the case of dividends received by a domestic or
foreign resident corporation liable to (corporate income) tax under this
Chapter . . . .only twenty-five per centum thereof shall be returnable for the purposes
of the tax imposed under this section." The Commissioner accordingly rejected
Maniago's contention that the Meralco from whom the dividends were received is
"not a domestic corporation liable to tax under this Chapter." In a letter dated April 5,
1968, the Commissioner informed Maniago of his findings and ruling and therefore
denied Maniago's claim for informer's reward on a non-existent deficiency. This
action of the Commissioner was sustained by the Secretary of Finance in a 4th
Indorsement dated May 11, 1971.
On August 28, 1970, Maniago filed a petition for mandamus, and subsequently an
amended petition for mandamus, in the Court of First Instance of Manila, docketed
therein as Civil Case No. 80830, against the Commissioner of Internal Revenue and
the Meralco Securities Corporation to compel the Commissioner to impose the

alleged deficiency tax assessment on the Meralco Securities Corporation and to


award to him the corresponding informer's reward under the provisions of R.A. 2338.
On October 28, 1978, the Commissioner filed a motion to dismiss, arguing that since
in matters of issuance and non-issuance of assessments, he is clothed under the
National Internal Revenue Code and existing rules and regulations with discretionary
power in evaluating the facts of a case and since mandamus win not lie to compel
the performance of a discretionary power, he cannot be compelled to impose the
alleged tax deficiency assessment against the Meralco Securities Corporation. He
further argued that mandamus may not lie against him for that would be tantamount
to a usurpation of executive powers, since the Office of the Commissioner of Internal
Revenue is undeniably under the control of the executive department.
On the other hand, the Meralco Securities Corporation filed its answer, dated January
15, 1971, interposing as special and/or affirmative defenses that the petition states
no cause of action, that the action is premature, that mandamus win not lie to
compel the Commissioner of Internal Revenue to make an assessment and/or effect
the collection of taxes upon a taxpayer, that since no taxes have actually been
recovered and/or collected, Maniago has no right to recover the reward prayed for,
that the action of petitioner had already prescribed and that respondent court has no
jurisdiction over the subject matter as set forth in the petition, the same being
cognizable only by the Court of Tax Appeals.
On January 10, 1973, the respondent judge rendered a decision granting the writ
prayed for and ordering the Commissioner of Internal Revenue to assess and collect
from the Meralco Securities Corporation the sum of P51,840,612.00 as deficiency
corporate income tax for the period 1962 to 1969 plus interests and surcharges due
thereon and to pay 25% thereof to Maniago as informer's reward.
All parties filed motions for reconsideration of the decision but the same were denied
by respondent judge in his order dated April 6, 1973, with respondent judge denying
respondents' claim for attorneys fees and for execution of the decision pending
appeal.
Hence, the Commissioner filed a separate petition with this Court, docketed as G.R.
No. L-36748 praying that the decision of respondent judge dated January 10, 1973
and his order dated April 6, 1973 be reconsidered for respondent judge has no
jurisdiction over the subject matter of the case and that the issuance or nonissuance of a deficiency assessment is a prerogative of the Commissioner of Internal
Revenue not reviewable by mandamus.
The Meralco Securities Corporation (now First Philippine Holdings Corporation)
likewise appealed the same decision of respondent judge in G.R. No. L-36181 and in
the Court's resolution dated June 13, 1973, the two cases were ordered consolidated.
We grant the petitions.

Respondent judge has no jurisdiction to take cognizance of the case because the
subject matter thereof clearly falls within the scope of cases now exclusively within
the jurisdiction of the Court of Tax Appeals. Section 7 of Republic Act No. 1125,
enacted June 16, 1954, granted to the Court of Tax Appeals exclusive appellate
jurisdiction to review by appeal, among others, decisions of the Commissioner of
Internal Revenue in cases involving disputed assessments, refunds of internal
revenue taxes, fees or other charges, penalties imposed in relation thereto, or other
matters arising under the National Internal Revenue Code or other law or part of law
administered by the Bureau of Internal Revenue. The law transferred to the Court of
Tax Appeals jurisdiction over all cases involving said assessments previously
cognizable by courts of first instance, and even those already pending in said
courts. 1The question of whether or not to impose a deficiency tax assessment on
Meralco Securities Corporation undoubtedly comes within the purview of the words
"disputed assessments" or of "other matters arising under the National Internal
Revenue Code . . . .In the case of Blaquera vs. Rodriguez, et al, 2 this Court ruled that
"the determination of the correctness or incorrectness of a tax assessment to
which the taxpayer is not agreeable, falls within the jurisdiction of the Court of Tax
Appeals and not of the Court of First Instance, for under the provisions of Section 7 of
Republic Act No. 1125, the Court of Tax Appeals has exclusive appellate jurisdiction
to review, on appeal, any decision of the Collector of Internal Revenue in cases
involving disputed assessments and other matters arising under the National Internal
Revenue Code or other law or part of law administered by the Bureau of Internal
Revenue."
Thus, even assuming arguendo that the right granted the taxpayers affected to
question and appeal disputed assessments, under section 7 of Republic Act No.
1125, may be availed of by strangers or informers like the late Maniago, the most
that he could have done was to appeal to the Court of Tax Appeals the ruling of
petitioner Commissioner of Internal Revenue within thirty (30) days from receipt
thereof pursuant to section 11 of Republic Act No. 1125. 3 He failed to take such an
appeal to the tax court. The ruling is clearly final and no longer subject to review by
the courts. 4
It is furthermore a well-recognized rule that mandamus only lies to enforce the
performance of a ministerial act or duty 5 and not to control the performance of a
discretionary power. 6 Purely administrative and discretionary functions may not be
interfered with by the courts. 7 Discretion, as thus intended, means the power or
right conferred upon the office by law of acting officially under certain circumstances
according to the dictates of his own judgment and conscience and not controlled by
the judgment or conscience of others. 8 mandamus may not be resorted to so as to
interfere with the manner in which the discretion shall be exercised or to influence or
coerce a particular determination. 9
In an analogous case, where a petitioner sought to compel the Rehabilitation Finance
Corporation to accept payment of the balance of his indebtedness with his backpay
certificates, the Court ruled that "mandamus does not compel the Rehabilitation

Finance Corporation to accept backpay certificates in payment of outstanding loans.


Although there is no provision expressly authorizing such acceptance, nor is there
one prohibiting it, yet the duty imposed by the Backpay Law upon said corporation
as to the acceptance or discount of backpay certificates is neither clear nor
ministerial, but discretionary merely, and such special civil action does not issue to
control the exercise of discretion of a public officer." 10 Likewise, we have held that
courts have no power to order the Commissioner of Customs to confiscate goods
imported in violation of the Import Control Law, R.A. 426, as said forfeiture is subject
to the discretion of the said official, 11 nor may courts control the determination of
whether or not an applicant for a visa has a non-immigrant status or whether his
entry into this country would be contrary to public safety for it is not a simple
ministerial function but an exercise of discretion. 12
Moreover, since the office of the Commissioner of Internal Revenue is charged with
the administration of revenue laws, which is the primary responsibility of the
executive branch of the government, mandamus may not he against the
Commissioner to compel him to impose a tax assessment not found by him to be
due or proper for that would be tantamount to a usurpation of executive functions.
As we held in the case of Commissioner of Immigration vs. Arca13 anent this
principle, "the administration of immigration laws is the primary responsibility of the
executive branch of the government. Extensions of stay of aliens are discretionary on
the part of immigration authorities, and neither a petition for mandamus nor one for
certiorari can compel the Commissioner of Immigration to extend the stay of an alien
whose period to stay has expired.
Such discretionary power vested in the proper executive official, in the absence of
arbitrariness or grave abuse so as to go beyond the statutory authority, is not
subject to the contrary judgment or control of others. " "Discretion," when applied to
public functionaries, means a power or right conferred upon them by law of acting
officially, under certain circumstances, uncontrolled by the judgment or consciences
of others. A purely ministerial act or duty in contradiction to a discretional act is one
which an officer or tribunal performs in a given state of facts, in a prescribed
manner, in obedience to the mandate of a legal authority, without regard to or the
exercise of his own judgment upon the propriety or impropriety of the act done. If
the law imposes a duty upon a public officer and gives him the right to decide how or
when the duty shall be performed, such duty is discretionary and not ministerial. The
duty is ministerial only when the discharge of the same requires neither the exercise
of official discretion or judgment." 14
Thus, after the Commissioner who is specifically charged by law with the task of
enforcing and implementing the tax laws and the collection of taxes had after a
mature and thorough study rendered his decision or ruling that no tax is due or
collectible, and his decision is sustained by the Secretary, now Minister of Finance
(whose act is that of the President unless reprobated), such decision or ruling is a
valid exercise of discretion in the performance of official duty and cannot be
controlled much less reversed by mandamus. A contrary view, whereby any stranger

or informer would be allowed to usurp and control the official functions of the
Commissioner of Internal Revenue would create disorder and confusion, if not chaos
and total disruption of the operations of the government.
Considering then that respondent judge may not order by mandamus the
Commissioner to issue the assessment against Meralco Securities Corporation when
no such assessment has been found to be due, no deficiency taxes may therefore be
assessed and collected against the said corporation. Since no taxes are to be
collected, no informer's reward is due to private respondents as the informer's heirs.
Informer's reward is contingent upon the payment and collection of unpaid or
deficiency taxes. An informer is entitled by way of reward only to a percentage of the
taxes actually assessed and collected. Since no assessment, much less any
collection, has been made in the instant case, respondent judge's writ for the
Commissioner to pay respondents 25% informer's reward is gross error and without
factual nor legal basis.
WHEREFORE, the petitions are hereby granted and the questioned decision of
respondent judge dated January 10, 1973 and order dated April 6, 1973 are hereby
reversed and set aside. With costs against private respondents.
G.R. No. 173176

August 26, 2008

JUDY ANNE L. SANTOS, petitioner,


vs.
PEOPLE OF THE PHILIPPINES and BUREAU OF INTERNAL
REVENUE, respondents.
DECISION
CHICO-NAZARIO, J.:
Before this Court is a Petition for Review on Certiorari1 under Rule 45 of the Revised
Rules of Court filed by petitioner Judy Anne L. Santos (Santos) seeking the reversal
and setting aside of the Resolution,2 dated 19 June 2006, of the Court of Tax Appeals
(CTA) en banc in C.T.A. EB. CRIM. No. 001 which denied petitioners Motion for
Extension of Time to File Petition for Review. Petitioner intended to file the Petition for
Review with the CTA en banc to appeal the Resolutions dated 23 February 2006 3 and
11 May 20064 of the CTA First Division in C.T.A. Crim. Case No. 0-012 denying,
respectively, her Motion to Quash the Information filed against her for violation of
Section 255, in relation to Sections 254 and 248(B) of the National Internal Revenue
Code (NIRC), as amended; and her Motion for Reconsideration.
There is no controversy as to the facts that gave rise to the present Petition.
On 19 May 2005, then Bureau of Internal Revenue (BIR) Commissioner Guillermo L.
Parayno, Jr. wrote to the Department of Justice (DOJ) Secretary Raul M. Gonzales a

letter5 regarding the possible filing of criminal charges against petitioner. BIR
Commissioner Parayno began his letter with the following statement:
I have the honor to refer to you for preliminary investigation and filing of an
information in court if evidence so warrants, the herein attached Joint Affidavit
of RODERICK C. ABAD, STIMSON P. CUREG, VILMA V. CARONAN,
RHODORA L. DELOS REYES under Group SupervisorTEODORA V. PURINO,
of the National Investigation Division, BIR National Office Building, BIR Road,
Diliman, Quezon City, recommending the criminal prosecution of MS. JUDY
ANNE LUMAGUI SANTOS for substantial underdeclaration of income,
which constitutes asprima facie evidence of false or fraudulent
return under Section 248(B) of the NIRC and punishable under Sections 254
and 255 of the Tax Code.
In said letter, BIR Commissioner Parayno summarized the findings of the
investigating BIR officers that petitioner, in her Annual Income Tax Return for taxable
year 2002 filed with the BIR, declared an income of P8,033,332.70 derived from her
talent fees solely from ABS-CBN; initial documents gathered from the BIR offices and
those given by petitioners accountant and third parties, however, confirmed that
petitioner received in 2002 income in the amount of at least P14,796,234.70, not
only from ABS-CBN, but also from other sources, such as movies and product
endorsements; the estimated tax liability arising from petitioners underdeclaration
amounted to P1,718,925.52, including incremental penalties; the non-declaration by
petitioner of an amount equivalent to at least 84.18% of the income declared in her
return was considered a substantial underdeclaration of income, which
constituted prima facie evidence of false or fraudulent return under Section
248(B)6 of the NIRC, as amended; and petitioners failure to account as part of her
income the professional fees she received from sources other than ABS-CBN and her
underdeclaration of the income she received from ABS-CBN amounted to manifest
violations of Sections 2547 and 255,8 as well as Section 248(B) of the NIRC, as
amended.
After an exchange of affidavits and other pleadings by the parties, Prosecution
Attorney Olivia Laroza-Torrevillas issued a Resolution 9 dated 21 October 2005 finding
probable cause and recommending the filing of a criminal information against
petitioner for violation of Section 255 in relation to Sections 254 and 248(B) of the
NIRC, as amended. The said Resolution was approved by Chief State Prosecutor
Jovencito R. Zuno.
Pursuant to the 21 October 2005 DOJ Resolution, an Information 10 for violation of
Section 255 in relation to Sections 254 and 248(B) of the NIRC, as amended, was
filed with the CTA on 3 November 2005 and docketed as C.T.A. Crim. Case No. 0-012.
However, the CTA First Division, after noting several discrepancies in the Information
filed, required the State Prosecutor to clarify and explain the same, and to submit the
original copies of the parties affidavits, memoranda, and all other evidence on
record.11

Consequently, Prosecution Attorney Torrevillas, on behalf of respondent People,


submitted on 1 December 2005 a Compliance with Ex Parte Motion to Admit
Attached Information.12 Prosecution Attorney Torrevillas moved that the documents
submitted be admitted as part of the record of the case and the first Information be
substituted by the attached second Information. The second Information 13 addressed
the discrepancies noted by the CTA in the first Information, by now reading thus:
The undersigned Prosecution Attorney of the Department of Justice hereby
accuses JUDY ANNE SANTOS y Lumagui of the offense of violation of
Section 255, of Republic Act No. 8424, otherwise known as the "Tax Reform
Act of 1997," as amended, committed as follows:
"That on or about the 15th day of April, 2003, at Quezon City,
Philippines, and within the jurisdiction of this Honorable Court, the
above-named accused did then and there, willfully, unlawfully, and
feloniously file a false and fraudulent income tax return for taxable year
2002 by indicating therein a gross income of P8,033,332.70 when in
truth and in fact her correct income for taxable year 2002
is P16,396,234.70 or a gross underdeclaration/difference of P8,362,902
resulting to an income tax deficiency ofP1,395,116.24 excluding
interest and penalties thereon of P1,319,500.94 or a total income tax
deficiency of P2,714,617.18 to the damage and prejudice of the
government of the same amount.["]
In a Resolution14 dated 8 December 2005, the CTA First Division granted the
Peoples Ex ParteMotion and admitted the second Information.
The CTA First Division then issued on 9 December 2005 a warrant for the arrest of
petitioner.15 The tax court lifted and recalled the warrant of arrest on 21 December
2005 after petitioner voluntarily appeared and submitted herself to its jurisdiction
and filed the required bail bond in the amount ofP20,000.00.16
On 10 January 2006, petitioner filed with the CTA First Division a Motion to
Quash17 the Information filed in C.T.A. Crim. Case No. 0-012 on the following grounds:
1. The facts alleged in the INFORMATION do not constitute an offense;
2. The officer who filed the information had no authority to do so;
3. The Honorable Court of Tax Appeals has no jurisdiction over the subject
matter of the case; and
4. The information is void ab initio, being violative of due process, and the
equal protection of the laws.

In a Resolution18 dated 23 February 2006, the CTA First Division denied petitioners
Motion to Quash and accordingly scheduled her arraignment on 2 March 2006 at
9:00 a.m. Petitioner filed a Motion for Reconsideration and/or
Reinvestigation,19 which was again denied by the CTA First Division in a
Resolution20 dated 11 May 2006.
Petitioner received a copy of the 11 May 2006 Resolution of the CTA First Division on
17 May 2006. On 1 June 2006, petitioner filed with the CTA en banc a Motion for
Extension of Time to File Petition for Review, docketed as C.T.A. EB. CRIM. No. 001.
She filed her Petition for Review with the CTA en banc on 16 June 2006. However, in
its Resolution21 dated 19 June 2006, the CTA en banc denied petitioners Motion for
Extension of Time to File Petition for Review, ratiocinating that:
In the case before Us, the petitioner is asking for an extension of time to file
her Petition for Review to appeal the denial of her motion to quash in C.T.A.
Crim. Case No. 0-012. As stated above, a resolution denying a motion to quash
is not a proper subject of an appeal to the CourtEn Banc under Section 11 of
R.A. No. 9282 because a ruling denying a motion to quash is only an
interlocutory order, as such, it cannot be made the subject of an appeal
pursuant to said law and the Rules of Court. Section 1 of Rule 41 of the Rules
of Court provides that "no appeal may be taken from an interlocutory order"
and Section 1 (i) of Rule 50 provides for the dismissal of an appeal on the
ground that "the order or judgment appealed from is not appealable". Time
and again, the Supreme Court had ruled that the remedy of the accused in
case of denial of a motion to quash is for the accused to enter a plea, go to
trial and after an adverse decision is rendered, to appeal therefrom in the
manner authorized by law.
Since a denial of a Motion to Quash is not appealable, granting petitioners
Motion for Extension of Time to File Petition for Review will only be an exercise
in futility considering that the dismissal of the Petition for Review that will be
filed by way of appeal is mandated both by law and jurisprudence. 22
Ultimately, the CTA en banc decreed:
WHEREFORE, premises considered, petitioners Motion for Extension of Time
to File Petition for Review filed on June 1, 2006 is hereby DENIED for lack of
merit.23
Now comes petitioner before this Court raising the sole issue of:
WHETHER A RESOLUTION OF A CTA DIVISION DENYING A MOTION TO QUASH
IS A PROPER SUBJECT OF AN APPEAL TO THE CTA EN BANC UNDER SECTION
11 OF REPUBLIC ACT NO. 9282, AMENDING SECTION 18 OF REPUBLIC ACT NO.
1125.24

Section 18 of Republic Act No. 1125,25 as amended by Republic Act No.


9282,26 provides:
SEC. 18. Appeal to the Court of Tax Appeals En Banc. No civil proceedings
involving matters arising under the National Internal Revenue Code, the Tariff
and Customs Code or the Local Government Code shall be maintained, except
as herein provided, until and unless an appeal has been previously filed with
the CTA and disposed of in accordance with the provisions of this Act.
A party adversely affected by a resolution of a Division of the CTA on a motion
for reconsideration or new trial, may file a petition for review with the CTA en
banc.
Petitioners primary argument is that a resolution of a CTA Division denying a motion
to quash is a proper subject of an appeal to the CTA en banc under Section 18 of
Republic Act No. 1125, as amended, because the law does not say that only a
resolution that constitutes a final disposition of a case may be appealed to the
CTA en banc. If the interpretation of the law by the CTA en bancprevails, a procedural
void is created leaving the parties, such as petitioner, without any remedy involving
erroneous resolutions of a CTA Division.
The Court finds no merit in the petitioners assertion.
The petition for review under Section 18 of Republic Act No. 1125, as
amended, may be new to the CTA, but it is actually a mode of appeal long
available in courts of general jurisdiction.
Petitioner is invoking a very narrow and literal reading of Section 18 of Republic Act
No. 1125, as amended.
Indeed, the filing of a petition for review with the CTA en banc from a decision,
resolution, or order of a CTA Division is a remedy newly made available in
proceedings before the CTA, necessarily adopted to conform to and address the
changes in the CTA.
There was no need for such rule under Republic Act No. 1125, prior to its
amendment, since the CTA then was composed only of one Presiding Judge and two
Associate Judges.27 Any two Judges constituted a quorum and the concurrence of two
Judges was necessary to promulgate any decision thereof. 28
The amendments introduced by Republic Act No. 9282 to Republic Act No. 1125
elevated the rank of the CTA to a collegiate court, with the same rank as the Court of
Appeals, and increased the number of its members to one Presiding Justice and five
Associate Justices.29 The CTA is now allowed to siten banc or in two Divisions with
each Division consisting of three Justices. Four Justices shall constitute a quorum for
sessions en banc, and the affirmative votes of four members of the Court en

banc are necessary for the rendition of a decision or resolution; while two Justices
shall constitute a quorum for sessions of a Division and the affirmative votes of two
members of the Division shall be necessary for the rendition of a decision or
resolution.30
In A.M. No. 05-11-07-CTA, the Revised CTA Rules, this Court delineated the
jurisdiction of the CTAen banc31 and in Divisions.32 Section 2, Rule 4 of the Revised
CTA Rules recognizes the exclusive appellate jurisdiction of the CTA en banc to
review by appeal the following decisions, resolutions, or orders of the CTA Division:
SEC. 2. Cases within the jurisdiction of the Court en banc. The Court en
banc shall exercise exclusive appellate jurisdiction to review by appeal the
following:
(a) Decisions or resolutions on motions for reconsideration or new trial of the
Court in Divisions in the exercise of its exclusive appellate jurisdiction over:
(1) Cases arising from administrative agencies Bureau of Internal
Revenue, Bureau of Customs, Department of Finance, Department of
Trade and Industry, Department of Agriculture;
(2) Local tax cases decided by the Regional Trial Courts in the exercise
of their original jurisdiction; and
(3) Tax collection cases decided by the Regional Trial Courts in the
exercise of their original jurisdiction involving final and executory
assessments for taxes, fees, charges and penalties, where the principal
amount of taxes and penalties claimed is less than one million pesos;
xxxx
(f) Decisions, resolutions or orders on motions for reconsideration or new trial
of the Court in Division in the exercise of its exclusive original jurisdiction over
cases involving criminal offenses arising from violations of the National
Internal Revenue Code or the Tariff and Customs Code and other laws
administered by the Bureau of Internal Revenue or Bureau of Customs.
(g) Decisions, resolutions or order on motions for reconsideration or new trial
of the Court in Division in the exercise of its exclusive appellate jurisdiction
over criminal offenses mentioned in the preceding subparagraph; x x x.
Although the filing of a petition for review with the CTA en banc from a decision,
resolution, or order of the CTA Division, was newly made available to the CTA, such
mode of appeal has long been available in Philippine courts of general jurisdiction.
Hence, the Revised CTA Rules no longer elaborated on it but merely referred to
existing rules of procedure on petitions for review and appeals, to wit:

RULE 7
PROCEDURE IN THE COURT OF TAX APPEALS
SEC. 1. Applicability of the Rules of the Court of Appeals. The procedure in
the Court en bancor in Divisions in original and in appealed cases shall be the
same as those in petitions for review and appeals before the Court of
Appeals pursuant to the applicable provisions of Rules 42, 43, 44 and
46 of the Rules of Court, except as otherwise provided for in these Rules.
RULE 8
PROCEDURE IN CIVIL CASES
xxxx
SEC. 4. Where to appeal; mode of appeal.
xxxx
(b) An appeal from a decision or resolution of the Court in Division on a motion
for reconsideration or new trial shall be taken to the Court by petition for
review as provided in Rule 43 of the Rules of Court. The Court en
banc shall act on the appeal.
xxxx
RULE 9
PROCEDURE IN CRIMINAL CASES
SEC. 1. Review of cases in the Court. The review of criminal cases in the
Court en banc or in Division shall be governed by the applicable provisions of
Rule 124 of the Rules of Court.
xxxx
SEC. 9. Appeal; period to appeal.
xxxx
(b) An appeal to the Court en banc in criminal cases decided by the Court in
Division shall be taken by filing a petition for review as provided in Rule 43 of
the Rules of Court within fifteen days from receipt of a copy of the decision
or resolution appealed from. The Court may, for good cause, extend the time
for filing of the petition for review for an additional period not exceeding
fifteen days. (Emphasis ours.)

Given the foregoing, the petition for review to be filed with the CTA en banc as the
mode for appealing a decision, resolution, or order of the CTA Division, under Section
18 of Republic Act No. 1125, as amended, is not a totally new remedy, unique to the
CTA, with a special application or use therein. To the contrary, the CTA merely adopts
the procedure for petitions for review and appeals long established and practiced in
other Philippine courts. Accordingly, doctrines, principles, rules, and precedents laid
down in jurisprudence by this Court as regards petitions for review and appeals in
courts of general jurisdiction should likewise bind the CTA, and it cannot depart
therefrom.
General rule: The denial of a motion to quash is an interlocutory order
which is not the proper subject of an appeal or a petition for certiorari.
According to Section 1, Rule 41 of the Revised Rules of Court, governing appeals
from the Regional Trial Courts (RTCs) to the Court of Appeals, an appeal may be
taken only from a judgment or final order that completely disposes of the case or of
a matter therein when declared by the Rules to be appealable. Said provision, thus,
explicitly states that no appeal may be taken from an interlocutory order. 33
The Court distinguishes final judgments and orders from interlocutory orders in this
wise:
Section 2, Rule 41 of the Revised Rules of Court provides that "(o)nly final
judgments or orders shall be subject to appeal." Interlocutory or incidental
judgments or orders do not stay the progress of an action nor are they subject
of appeal "until final judgment or order is rendered for one party or the other."
The test to determine whether an order or judgment is interlocutory or final is
this: "Does it leave something to be done in the trial court with respect to the
merits of the case? If it does, it is interlocutory; if it does not, it is final". A
court order is final in character if it puts an end to the particular matter
resolved or settles definitely the matter therein disposed of, such that no
further questions can come before the court except the execution of the order.
The term "final" judgment or order signifies a judgment or an order which
disposes of the cause as to all the parties, reserving no further questions or
directions for future determination. The order or judgment may validly refer to
the entire controversy or to some definite and separate branch thereof. "In the
absence of a statutory definition, a final judgment, order or decree has been
held to be x x x one that finally disposes of, adjudicates, or determines the
rights, or some right or rights of the parties, either on the entire controversy
or on some definite and separate branch thereof, and which concludes them
until it is reversed or set aside." The central point to consider is, therefore, the
effects of the order on the rights of the parties. A court order, on the other
hand, is merely interlocutory in character if it is provisional and leaves
substantial proceeding to be had in connection with its subject. The word
"interlocutory" refers to "something intervening between the commencement

and the end of a suit which decides some point or matter but is not a final
decision of the whole controversy."34
In other words, after a final order or judgment, the court should have nothing more
to do in respect of the relative rights of the parties to the case. Conversely, "an order
that does not finally dispose of the case and does not end the Court's task of
adjudicating the parties' contentions in determining their rights and liabilities as
regards each other, but obviously indicates that other things remain to be done by
the Court, is interlocutory."35
The rationale for barring the appeal of an interlocutory order was extensively
discussed in Matute v. Court of Appeals,36 thus:
It is settled that an "interlocutory order or decree made in the progress of a
case is always under the control of the court until the final decision of the suit,
and may be modified or rescinded upon sufficient grounds shown at any time
before final judgment . . ." Of similar import is the ruling of this Court declaring
that "it is rudimentary that such (interlocutory) orders are subject to change in
the discretion of the court." Moreover, one of the inherent powers of the court
is "To amend and control its process and orders so as to make them
conformable to law and justice. In the language of Chief Justice Moran,
paraphrasing the ruling in Veluz vs. Justice of the Peace of Sariaya, "since
judges are human, susceptible to mistakes, and are bound to administer
justice in accordance with law, they are given the inherent power of amending
their orders or judgments so as to make them conformable to law and justice,
and they can do so before they lose their jurisdiction of the case, that is before
the time to appeal has expired and no appeal has been perfected." And in the
abovecited Veluz case, this Court held that "If the trial court should discover or
be convinced that it had committed an error in its judgment, or had done an
injustice, before the same has become final, it may, upon its own motion or
upon a motion of the parties, correct such error in order to do justice between
the parties. . . . It would seem to be the very height of absurdity to prohibit a
trial judge from correcting an error, mistake, or injustice which is called to his
attention before he has lost control of his judgment." Corollarily, it has also
been held "that a judge of first instance is not legally prevented from revoking
the interlocutory order of another judge in the very litigation subsequently
assigned to him for judicial action."
Another recognized reason of the law in permitting appeal only from a final order or
judgment, and not from an interlocutory or incidental one, is to avoid multiplicity of
appeals in a single action, which must necessarily suspend the hearing and decision
on the merits of the case during the pendency of the appeal. If such appeal were
allowed, the trial on the merits of the case would necessarily be delayed for a
considerable length of time, and compel the adverse party to incur unnecessary
expenses, for one of the parties may interpose as many appeals as incidental

questions may be raised by him, and interlocutory orders rendered or issued by the
lower court.37
There is no dispute that a court order denying a motion to quash is interlocutory. The
denial of the motion to quash means that the criminal information remains pending
with the court, which must proceed with the trial to determine whether the accused
is guilty of the crime charged therein. Equally settled is the rule that an order
denying a motion to quash, being interlocutory, is not immediately appealable, 38 nor
can it be the subject of a petition for certiorari. Such order may only be reviewed in
the ordinary course of law by an appeal from the judgment after trial. 39
The Court cannot agree in petitioners contention that there would exist a procedural
void following the denial of her Motion to Quash by the CTA First Division in its
Resolutions dated 23 February 2006 and 11 May 2006, leaving her helpless. The
remedy of an accused from the denial of his or her motion to quash has already been
clearly laid down as follows:
An order denying a Motion to Acquit (like an order denying a motion to quash)
is interlocutory and not a final order. It is, therefore, not appealable. Neither
can it be the subject of a petition for certiorari. Such order of denial may only
be reviewed, in the ordinary course of law, by an appeal from the judgment,
after trial. As stated in Collins vs. Wolfe, and reiterated in Mill vs. Yatco, the
accused, after the denial of his motion to quash, should have proceeded with
the trial of the case in the court below, and if final judgment is rendered
against him, he could then appeal, and, upon such appeal, present the
questions which he sought to be decided by the appellate court in a petition
for certiorari.
In Acharon vs. Purisima, the procedure was well defined, thus:
"Moreover, when the motion to quash filed by Acharon to nullify the
criminal cases filed against him was denied by the Municipal Court of
General Santos his remedy was not to file a petition for certiorari but to
go to trial without prejudice on his part to reiterate the special defenses
he had invoked in his motion and, if, after trial on the merits, an
adverse decision is rendered, to appeal therefrom in the manner
authorized by law. This is the procedure that he should have followed as
authorized by law and precedents. Instead, he took the usual step of
filing a writ of certiorari before the Court of First Instance which in our
opinion is unwarranted it being contrary to the usual course of law." 40
Hence, the CTA en banc herein did not err in denying petitioners Motion for
Extension of Time to File Petition for Review, when such Petition for Review is the
wrong remedy to assail an interlocutory order denying her Motion to Quash.

While the general rule proscribes the appeal of an interlocutory order, there are also
recognized exceptions to the same. The general rule is not absolute. Where special
circumstances clearly demonstrate the inadequacy of an appeal, then the special
civil action of certiorari or prohibition may exceptionally be allowed.41 This Court
recognizes that under certain situations, recourse to extraordinary legal remedies,
such as a petition for certiorari, is considered proper to question the denial of a
motion to quash (or any other interlocutory order) in the interest of a "more
enlightened and substantial justice";42 or to promote public welfare and public
policy;43 or when the cases "have attracted nationwide attention, making it essential
to proceed with dispatch in the consideration thereof"; 44 or when the order was
rendered with grave abuse of discretion. 45 Certiorari is an appropriate remedy to
assail an interlocutory order (1) when the tribunal issued such order without or in
excess of jurisdiction or with grave abuse of discretion; and (2) when the assailed
interlocutory order is patently erroneous, and the remedy of appeal would not afford
adequate and expeditious relief.46
Recourse to a petition for certiorari to assail an interlocutory order is now expressly
recognized in the ultimate paragraph of Section 1, Rule 41 of the Revised Rules of
Court on the subject of appeal, which states:
In all the above instances where the judgment or final order is not appealable,
the aggrieved party may file an appropriate special civil action under Rule 65.
As to whether the CTA en banc, under its expanded jurisdiction in Republic Act No.
9282, has been granted jurisdiction over special civil actions for certiorari is not
raised as an issue in the Petition at bar, thus, precluding the Court from making a
definitive pronouncement thereon. However, even if such an issue is answered in the
negative, it would not substantially affect the ruling of this Court herein, for a party
whose motion to quash had been denied may still seek recourse, under exceptional
and meritorious circumstances, via a special civil action for certiorari with this Court,
refuting petitioners assertion of a procedural void.
The CTA First Division did not commit grave abuse of discretion in denying
petitioners Motion to Quash.
Assuming that the CTA en banc, as an exception to the general rule, allowed and
treated petitioners Petition for Review in C.T.A. EB. CRIM. No. 001 as a special civil
action for certiorari, 47 it would still be dismissible for lack of merit.
An act of a court or tribunal may only be considered as committed in grave abuse of
discretion when the same was performed in a capricious or whimsical exercise of
judgment, which is equivalent to lack of jurisdiction. The abuse of discretion must be
so patent and gross as to amount to an evasion of positive duty or to a virtual refusal
to perform a duty enjoined by law or to act at all in contemplation of law, as where
the power is exercised in an arbitrary and despotic manner by reason of passion or
personal hostility. In this connection, it is only upon showing that the court acted

without or in excess of jurisdiction or with grave abuse of discretion that an


interlocutory order such as that involved in this case may be impugned. Be that as it
may, it must be emphasized that this practice is applied only under certain
exceptional circumstances to prevent unnecessary delay in the administration of
justice and so as not to unduly burden the courts. 48
Certiorari is not available to correct errors of procedure or mistakes in the judges
findings and conclusions of law and fact. It is only in the presence of extraordinary
circumstances evincing a patent disregard of justice and fair play where resort to a
petition for certiorari is proper. A party must not be allowed to delay litigation by the
sheer expediency of filing a petition for certiorari under Rule 65 of the Revised Rules
of Court based on scant allegations of grave abuse. 49
A writ of certiorari is not intended to correct every controversial interlocutory ruling:
it is resorted to only to correct a grave abuse of discretion or a whimsical exercise of
judgment equivalent to lack of jurisdiction. Its function is limited to keeping an
inferior court within its jurisdiction and to relieve persons from arbitrary acts acts
which courts or judges have no power or authority in law to perform. It is not
designed to correct erroneous findings and conclusions made by the courts. 50
The Petition for Review which petitioner intended to file before the CTA en
banc relied on two grounds: (1) the lack of authority of Prosecuting Attorney
Torrevillas to file the Information; and (2) the filing of the said Information in violation
of petitioners constitutional rights to due process and equal protection of the laws.
Anent the first ground, petitioner argues that the Information was filed without the
approval of the BIR Commissioner in violation of Section 220 of NIRC, as amended,
which provides:
SEC. 220. Form and Mode of Proceeding in Actions Arising under this Code. Civil and criminal actions and proceedings instituted in behalf of the
Government under the authority of this Code or other law enforced by the
Bureau of Internal Revenue shall be brought in the name of the Government of
the Philippines and shall be conducted by legal officers of the Bureau of
Internal Revenue but no civil or criminal action for the recovery of taxes or the
enforcement of any fine, penalty or forfeiture under this Code shall be filed in
court without the approval of the Commissioner.
Petitioners argument must fail in light of BIR Commissioner Paraynos letter dated
19 May 2005 to DOJ Secretary Gonzales referring "for preliminary investigation and
filing of an information in courtif evidence so warrants," the findings of the BIR
officers recommending the criminal prosecution of petitioner. In said letter, BIR
Commissioner Parayno already gave his prior approval to the filing of an information
in court should the DOJ, based on the evidence submitted, find probable cause
against petitioner during the preliminary investigation. Section 220 of the NIRC, as
amended, simply requires that the BIR Commissioner approve the institution of civil

or criminal action against a tax law violator, but it does not describe in what form
such approval must be given. In this case, BIR Commissioner Paraynos letter of 19
May 2005 already states his express approval of the filing of an information against
petitioner and his signature need not appear on the Resolution of the State
Prosecutor or the Information itself.
Still on the purported lack of authority of Prosecution Attorney Torrevillas to file the
Information, petitioner asserts that it is the City Prosecutor under the Quezon City
Charter, who has the authority to investigate and prosecute offenses allegedly
committed within the jurisdiction of Quezon City, such as petitioners case.
The Court is not persuaded. Under Republic Act No. 537, the Revised Charter of
Quezon City, the City Prosecutor shall have the following duties relating to the
investigation and prosecution of criminal offenses:
SEC. 28. The City Attorney - His assistants - His duties.
xxxx
(g) He shall also have charge of the prosecution of all crimes, misdemeanors,
and violations of city ordinances, in the Court of First Instance and the
municipal courts of the city, and shall discharge all the duties in respect to the
criminal prosecutions enjoined by law upon provincial fiscals.
(h) He shall cause to be investigated all charges of crimes, misdemeanors, and
violations of ordinances and have the necessary information or complaints
prepared or made against the persons accused. He or any of his assistants
may conduct such investigations by taking oral evidence of reputable
witnesses, and for this purpose may issue subpoena, summon witnesses to
appear and testify under oath before him, and the attendance or evidence of
an absent or recalcitrant witness may be enforced by application to the
municipal court or the Court of First Instance. No witness summoned to testify
under this section shall be under obligation to give any testimony which tend
to incriminate himself.
Evident from the foregoing is that the City Prosecutor has the power to investigate
crimes, misdemeanors, and violations of ordinances committed within the territorial
jurisdiction of the city, and which can be prosecuted before the trial courts of the
said city. The charge against petitioner, however, is already within the exclusive
original jurisdiction of the CTA,51 as the Information states that her gross
underdeclaration resulted in an income tax deficiency of P1,395,116.24, excluding
interest and penalties. The City Prosecutor does not have the authority to appear
before the CTA, which is now of the same rank as the Court of Appeals.
In contrast, the DOJ is the principal law agency of the Philippine government which
shall be both its legal counsel and prosecution arm. 52 It has the power to investigate

the commission of crimes, prosecute offenders and administer the probation and
correction system.53 Under the DOJ is the Office of the State Prosecutor whose
functions are described as follows:
Sec. 8. Office of the Chief State Prosecutor. - The Office of the Chief State
Prosecutor shall have the following functions:
(1) Assist the Secretary in the performance of powers and functions of the
Department relative to its role as the prosecution arm of the government;
(2) Implement the provisions of laws, executive orders and rules, and carry out
the policies, plans, programs and projects of the Department relative to the
investigation and prosecution of criminal cases;
(3) Assist the Secretary in exercising supervision and control over the National
Prosecution Service as constituted under P.D. No. 1275 and/or otherwise
hereinafter provided; and
(4) Perform such other functions as may be provided by law or assigned by the
Secretary.54
As explained by CTA First Division in its Resolution dated 11 May 2006:
[T]he power or authority of the Chief State Prosecutor Jovencito Zuo, Jr. and
his deputies in the Department of Justice to prosecute cases is national in
scope; and the Special Prosecutors authority to sign and file informations in
court proceeds from the exercise of said persons authority to conduct
preliminary investigations.55
Moreover, there is nothing in the Revised Quezon City Charter which would suggest
that the power of the City Prosecutor to investigate and prosecute crimes,
misdemeanors, and violations of ordinances committed within the territorial
jurisdiction of the city is to the exclusion of the State Prosecutors. In fact, the Office
of the State Prosecutor exercises control and supervision over City Prosecutors under
Executive Order No. 292, otherwise known as the Administrative Code of 1987.
As regards petitioners second ground in her intended Petition for Review with the
CTA en banc, she asserts that she has been denied due process and equal protection
of the laws when similar charges for violation of the NIRC, as amended, against
Regina Encarnacion A. Velasquez (Velasquez) were dismissed by the DOJ in its
Resolution dated 10 August 2005 in I.S. No. 2005-330 for the reason that Velasquezs
tax liability was not yet fully determined when the charges were filed.
The Court is unconvinced.

First, a motion to quash should be based on a defect in the information which is


evident on its face.56The same cannot be said herein. The Information against
petitioner appears valid on its face; and that it was filed in violation of her
constitutional rights to due process and equal protection of the laws is not evident on
the face thereof. As pointed out by the CTA First Division in its 11 May 2006
Resolution, the more appropriate recourse petitioner should have taken, given the
dismissal of similar charges against Velasquez, was to appeal the Resolution dated
21 October 2005 of the Office of the State Prosecutor recommending the filing of an
information against her with the DOJ Secretary. 57
Second, petitioner cannot claim denial of due process when she was given the
opportunity to file her affidavits and other pleadings and submit evidence before the
DOJ during the preliminary investigation of her case and before the Information was
filed against her. Due process is merely an opportunity to be heard. In addition,
preliminary investigation conducted by the DOJ is merely inquisitorial. It is not a trial
of the case on the merits. Its sole purpose is to determine whether a crime has been
committed and whether the respondent therein is probably guilty of the crime. It is
not the occasion for the full and exhaustive display of the parties evidence. Hence, if
the investigating prosecutor is already satisfied that he can reasonably determine
the existence of probable cause based on the parties evidence thus presented, he
may terminate the proceedings and resolve the case. 58
Third, petitioner cannot likewise aver that she has been denied equal protection of
the laws.
The equal protection clause exists to prevent undue favor or privilege. It is intended
to eliminate discrimination and oppression based on inequality. Recognizing the
existence of real differences among men, the equal protection clause does not
demand absolute equality. It merely requires that all persons shall be treated alike,
under like circumstances and conditions, both as to the privileges conferred and
liabilities enforced.59
Petitioner was not able to duly establish to the satisfaction of this Court that she and
Velasquez were indeed similarly situated, i.e., that they committed identical acts for
which they were charged with the violation of the same provisions of the NIRC; and
that they presented similar arguments and evidence in their defense - yet, they were
treated differently.
Furthermore, that the Prosecution Attorney dismissed what were supposedly similar
charges against Velasquez did not compel Prosecution Attorney Torrevillas to rule the
same way on the charges against petitioner. In People v. Dela Piedra,60 this Court
explained that:
The prosecution of one guilty person while others equally guilty are not
prosecuted, however, is not, by itself, a denial of the equal protection of the
laws. Where the official action purports to be in conformity to the statutory

classification, an erroneous or mistaken performance of the statutory duty,


although a violation of the statute, is not without more a denial of the equal
protection of the laws. The unlawful administration by officers of a statute fair
on its face, resulting in its unequal application to those who are entitled to be
treated alike, is not a denial of equal protection unless there is shown to be
present in it an element of intentional or purposeful discrimination. This may
appear on the face of the action taken with respect to a particular class or
person, or it may only be shown by extrinsic evidence showing a
discriminatory design over another not to be inferred from the action
itself. But a discriminatory purpose is not presumed, there must be a
showing of "clear and intentional discrimination." Appellant has failed to
show that, in charging appellant in court, that there was a "clear and
intentional discrimination" on the part of the prosecuting officials.
The discretion of who to prosecute depends on the prosecutions sound
assessment whether the evidence before it can justify a reasonable belief that
a person has committed an offense.The presumption is that the
prosecuting officers regularly performed their duties, and this
presumption can be overcome only by proof to the contrary, not by
mere speculation. Indeed, appellant has not presented any evidence to
overcome this presumption. The mere allegation that appellant, a Cebuana,
was charged with the commission of a crime, while a Zamboanguea, the
guilty party in appellants eyes, was not, is insufficient to support a conclusion
that the prosecution officers denied appellant equal protection of the laws.
There is also common sense practicality in sustaining appellants prosecution.
While all persons accused of crime are to be treated on a basis of
equality before the law, it does not follow that they are to be
protected in the commission of crime. It would be unconscionable, for
instance, to excuse a defendant guilty of murder because others have
murdered with impunity. The remedy for unequal enforcement of the law
in such instances does not lie in the exoneration of the guilty at the
expense of society x x x. Protection of the law will be extended to all
persons equally in the pursuit of their lawful occupations, but no person has
the right to demand protection of the law in the commission of a crime.
Likewise, [i]f the failure of prosecutors to enforce the criminal laws as to some
persons should be converted into a defense for others charged with crime, the
result would be that the trial of the district attorney for nonfeasance would
become an issue in the trial of many persons charged with heinous crimes and
the enforcement of law would suffer a complete breakdown. (Emphasis ours.)
In the case at bar, no evidence of a clear and intentional discrimination against
petitioner was shown, whether by Prosecution Attorney Torrevillas in recommending
the filing of Information against petitioner or by the CTA First Division in denying

petitioners Motion to Quash. The only basis for petitioners claim of denial of equal
protection of the laws was the dismissal of the charges against Velasquez while
those against her were not.
And lastly, the Resolutions of the CTA First Division dated 23 February 2006 and 11
May 2006 directly addressed the arguments raised by petitioner in her Motion to
Quash and Motion for Reconsideration, respectively, and explained the reasons for
the denial of both Motions. There is nothing to sustain a finding that these
Resolutions were rendered capriciously, whimsically, or arbitrarily, as to constitute
grave abuse of discretion amounting to lack or excess of jurisdiction.
In sum, the CTA en banc did not err in denying petitioners Motion for Extension of
Time to File Petition for Review. Petitioner cannot file a Petition for Review with the
CTA en banc to appeal the Resolution of the CTA First Division denying her Motion to
Quash. The Resolution is interlocutory and, thus, unappealable. Even if her Petition
for Review is to be treated as a petition for certiorari, it is dismissible for lack of
merit.
WHEREFORE, premises considered, the instant Petition for Review is
hereby DENIED. Costs against petitioner.
SO ORDERED.
[G.R. No. 130430. December 13, 1999]
REPUBLIC OF THE PHILIPPINES, represented by the Commissioner of the
Bureau of Internal Revenue (BIR), petitioner, vs. SALUD V.
HIZON, respondent.
DECISION
MENDOZA, J.:
This is a petition for review of the decision [1] of the Regional Trial Court, Branch
44, San Fernando, Pampanga, dismissing the suit filed by the Bureau of Internal
Revenue for collection of tax.
The facts are as follows:
On July 18, 1986, the BIR issued to respondent Salud V. Hizon a deficiency
income tax assessment of P1,113,359.68 covering the fiscal year 19811982. Respondent not having contested the assessment, petitioner, on January 12,
1989, served warrants of distraint and levy to collect the tax deficiency. However,
for reasons not known, it did not proceed to dispose of the attached properties.

More than three years later, or on November 3, 1992, respondent wrote the BIR
requesting a reconsideration of her tax deficiency assessment. The BIR, in a letter
dated August 11, 1994, denied the request. On January 1, 1997, it filed a case with
the Regional Trial Court, Branch 44, San Fernando, Pampanga to collect the tax
deficiency. The complaint was signed by Norberto Salud, Chief of the Legal Division,
BIR Region 4, and verified by Amancio Saga, the Bureaus Regional Director in
Pampanga.
Respondent moved to dismiss the case on two grounds: (1) that the complaint
was not filed upon authority of the BIR Commissioner as required by 221 [2] of the
National Internal Revenue Code, and (2) that the action had already
prescribed. Over petitioners objection, the trial court, on August 28, 1997, granted
the motion and dismissed the complaint. Hence, this petition. Petitioner raises the
following issues:[3]
I. WHETHER OR NOT THE INSTITUTION OF THE CIVIL CASE FOR COLLECTION OF
TAXES WAS WITHOUT THE APPROVAL OF THE COMMISSIONER IN VIOLATION OF
SECTION 221 OF THE NATIONAL INTERNAL REVENUE CODE.
II. WHETHER OR NOT THE ACTION FOR COLLECTION OF TAXES FILED AGAINST
RESPONDENT HAD ALREADY BEEN BARRED BY THE STATUTE OF LIMITATIONS.
First. In sustaining respondents contention that petitioners complaint was filed
without the authority of the BIR Commissioner, the trial court stated: [4]
There is no question that the National Internal Revenue Code explicitly provides that
in the matter of filing cases in Court, civil or criminal, for the collection of taxes, etc.,
the approval of the commissioner must first be secured. . . . [A]n action will not
prosper in the absence of the commissioners approval. Thus, in the instant case,
the absence of the approval of the commissioner in the institution of the action is
fatal to the cause of the plaintiff . . . .
The trial court arrived at this conclusion because the complaint filed by the BIR was
not signed by then Commissioner Liwayway Chato.
Sec. 221 of the NIRC provides:
Form and mode of proceeding in actions arising under this Code. Civil and criminal
actions and proceedings instituted in behalf of the Government under the authority
of this Code or other law enforced by the Bureau of Internal Revenue shall be
brought in the name of the Government of the Philippines and shall be conducted by
the provincial or city fiscal, or the Solicitor General, or by the legal officers of the
Bureau of Internal Revenue deputized by the Secretary of Justice, but no civil and
criminal actions for the recovery of taxes or the enforcement of any fine, penalty or
forfeiture under this Code shall be begun without the approval of the Commissioner.
(Emphasis supplied)

To implement this provision Revenue Administrative Order No. 5-83 of the BIR
provides in pertinent portions:
The following civil and criminal cases are to be handled by Special Attorneys and
Special Counsels assigned in the Legal Branches of Revenue Regions:
....
II. Civil Cases
1. Complaints for collection on cases falling within the jurisdiction of the Region . . . .
In all the abovementioned cases, the Regional Director is authorized to sign all
pleadings filed in connection therewith which, otherwise, requires the signature of
the Commissioner.
....
Revenue Administrative Order No. 10-95 specifically authorizes the Litigation and
Prosecution Section of the Legal Division of regional district offices to institute the
necessary civil and criminal actions for tax collection. As the complaint filed in this
case was signed by the BIRs Chief of Legal Division for Region 4 and verified by the
Regional Director, there was, therefore, compliance with the law.
However, the lower court refused to recognize RAO No. 10-95 and, by
implication, RAO No. 5-83. It held:
[M]emorand[a], circulars and orders emanating from bureaus and agencies whether
in the purely public or quasi-public corporations are mere guidelines for the internal
functioning of the said offices. They are not laws which courts can take judicial
notice of. As such, they have no binding effect upon the courts for such
memorand[a] and circulars are not the official acts of the legislative, executive and
judicial departments of the Philippines . . . . [5]
This is erroneous. The rule is that as long as administrative issuances relate
solely to carrying into effect the provisions of the law, they are valid and have the
force of law.[6] The governing statutory provision in this case is 4(d) of the NIRC
which provides:
Specific provisions to be contained in regulations. - The regulations of the Bureau of
Internal Revenue shall, among other things, contain provisions specifying,
prescribing, or defining:
....

(d) The conditions to be observed by revenue officers, provincial fiscals and other
officials respecting the institution and conduct of legal actions and proceedings.
RAO Nos. 5-83 and 10-95 are in harmony with this statutory mandate.
As amended by R.A. No. 8424, the NIRC is now
the present Code authorizes the BIR Commissioner
him under the pertinent provisions of the Code to
rank equivalent to a division chief or higher, except

even more categorical. Sec. 7 of


to delegate the powers vested in
any subordinate official with the
the following:

(a) The power to recommend the promulgation of rules and regulations by the
Secretary of Finance;
(b) The power to issue rulings of first impression or to reverse, revoke or modify any
existing ruling of the Bureau;
(c) The power to compromise or abate under 204(A) and (B) of this Code, any tax
deficiency: Provided, however, that assessments issued by the Regional Offices
involving basic deficiency taxes of five hundred thousand pesos (P500,000.00) or
less, and minor criminal violations as may be determined by rules and regulations to
be promulgated by the Secretary of Finance, upon the recommendation of the
Commissioner, discovered by regional and district officials, may be compromised by
a regional evaluation board which shall be composed of the Regional Director as
Chairman, the Assistant Regional Director, heads of the Legal, Assessment and
Collection Divisions and the Revenue District Officer having jurisdiction over the
taxpayer, as members; and
(d) The power to assign or reassign internal revenue officers to establishments where
articles subject to excise tax are produced or kept.
None of the exceptions relates to the Commissioners power to approve the filing of
tax collection cases.
Second. With regard to the issue that the case filed by petitioner for the
collection of respondents tax deficiency is barred by prescription, 223(c) of the
NIRC provides:
Any internal revenue tax which has been assessed within the period of limitation
above-prescribed may be collected by distraint or levy or by a proceeding in court
within three years[7]following the assessment of the tax.
The running of the three-year prescriptive period is suspended [8]
for the period during which the Commissioner is prohibited from making the
assessment or beginning distraint or levy or a proceeding in court and for sixty
days thereafter; when the taxpayer requests for a reinvestigation which is

granted by the Commissioner; when the taxpayer cannot be located in the


address given by him in the return filed upon which the tax is being assessed or
collected; provided, that, if the taxpayer informs the Commissioner of any
change in address, the running of the statute of limitations will not be
suspended; when the warrant of distraint or levy is duly served upon the
taxpayer, his authorized representative or a member of his household with
sufficient discretion, and no property could be located; and when the taxpayer is
out of the Philippines.
Petitioner argues that, in accordance with this provision, respondents request for
reinvestigation of her tax deficiency assessment on November 3, 1992 effectively
suspended the running of the period of prescription such that the government could
still file a case for tax collection.[9]
The contention has no merit. Sec. 229[10] of the Code mandates that a request
for reconsideration must be made within 30 days from the taxpayers receipt of the
tax deficiency assessment, otherwise the assessment becomes final, unappealable
and, therefore, demandable.[11] The notice of assessment for respondents tax
deficiency was issued by petitioner on July 18, 1986. On the other hand, respondent
made her request for reconsideration thereof only on November 3, 1992, without
stating when she received the notice of tax assessment. She explained that she was
constrained to ask for a reconsideration in order to avoid the harassment of BIR
collectors.[12] In all likelihood, she must have been referring to the distraint and levy
of her properties by petitioners agents which took place on January 12, 1989. Even
assuming that she first learned of the deficiency assessment on this date, her
request for reconsideration was nonetheless filed late since she made it more than
30 days thereafter. Hence, her request for reconsideration did not suspend the
running of the prescriptive period provided under 223(c). Although the
Commissioner acted on her request by eventually denying it on August 11, 1994,
this is of no moment and does not detract from the fact that the assessment had
long become demandable.
Nonetheless, it is contended that the running of the prescriptive period under
223(c) was suspended when the BIR timely served the warrants of distraint and levy
on respondent on January 12, 1989.[13]Petitioner cites for this purpose our ruling
in Advertising Associates Inc. v. Court of Appeals.[14] Because of the suspension, it is
argued that the BIR could still avail of the other remedy under 223(c) of filing a case
in court for collection of the tax deficiency, as the BIR in fact did on January 1, 1997.
Petitioners reliance on the Courts ruling in Advertising Associates Inc. v. Court
of Appeals is misplaced. What the Court stated in that case and, indeed, in the
earlier case of Palanca v. Commissioner of Internal Revenue,[15] is that the timely
service of a warrant of distraint or levy suspends the running of the period to collect
the tax deficiency in the sense that the disposition of the attached properties might
well take time to accomplish, extending even after the lapse of the statutory period
for collection. In those cases, the BIR did not file any collection case but merely

relied on the summary remedy of distraint and levy to collect the tax deficiency. The
importance of this fact was not lost on the Court. Thus, in Advertising Associates, it
was held:[16] It should be noted that the Commissioner did not institute any judicial
proceeding to collect the tax. He relied on the warrants of distraint and levy to
interrupt the running of the statute of limitations.
Moreover, if, as petitioner in effect says, the prescriptive period was suspended
twice, i.e., when the warrants of distraint and levy were served on respondent on
January 12, 1989 and then when respondent made her request for reinvestigation of
the tax deficiency assessment on November 3, 1992, the three-year prescriptive
period must have commenced running again sometime after the service of the
warrants of distraint and levy. Petitioner, however, does not state when or why this
took place and, indeed, there appears to be no reason for such. It is noteworthy that
petitioner raised this point before the lower court apparently as an alternative
theory, which, however, is untenable.
For the foregoing reasons, we hold that petitioners contention that the action in
this case had not prescribed when filed has no merit. Our holding, however, is
without prejudice to the disposition of the properties covered by the warrants of
distraint and levy which petitioner served on respondent, as such would be a mere
continuation of the summary remedy it had timely begun. Although considerable
time has passed since then, as held in Advertising Associates Inc. v. Court of
Appeals[17] and Palanca v. Commissioner of Internal Revenue,[18] the enforcement of
tax collection through summary proceedings may be carried out beyond the
statutory period considering that such remedy was seasonably availed of.
WHEREFORE, the petition is DENIED.
G.R. No. 173594

February 6, 2008

SILKAIR (SINGAPORE) PTE, LTD., petitioner,


vs.
COMMISSIONER OF INTERNAL REVENUE, respondent.
DECISION
CARPIO MORALES, J.:
Petitioner, Silkair (Singapore) Pte. Ltd. (Silkair), a corporation organized under the
laws of Singapore which has a Philippine representative office, is an online
international air carrier operating the Singapore-Cebu-Davao-Singapore, SingaporeDavao-Cebu-Singapore, and Singapore-Cebu-Singapore routes.
On December 19, 2001, Silkair filed with the Bureau of Internal Revenue (BIR) a
written application for the refund ofP4,567,450.79 excise taxes it claimed to have
paid on its purchases of jet fuel from Petron Corporation from January to June 2000. 1

As the BIR had not yet acted on the application as of December 26, 2001, Silkair filed
a Petition for Review2 before the CTA following Commissioner of Internal Revenue v.
Victorias Milling Co., Inc., et al.3
Opposing the petition, respondent Commissioner on Internal Revenue (CIR) alleged
in his Answer that, among other things,
Petitioner failed to prove that the sale of the petroleum products was directly
made from a domestic oil company to the international carrier. The excise tax
on petroleum products is the direct liability of the manufacturer/producer,
and when added to the cost of the goods sold to the buyer, it is no longer a
tax butpart of the price which the buyer has to pay to obtain the
article.4 (Emphasis and underscoring supplied)
By Decision of May 27, 2005, the Second Division of the CTA denied Silkairs petition
on the ground that as the excise tax was imposed on Petron Corporation as the
manufacturer of petroleum products, any claim for refund should be filed by the
latter; and where the burden of tax is shifted to the purchaser, the amount passed
on to it is no longer a tax but becomes an added cost of the goods purchased. Thus
the CTA discoursed:
The liability for excise tax on petroleum products that are being removed from
its refinery is imposed on the manufacturer/producer (Section 130 of the NIRC
of 1997). x x x
xxxx
While it is true that in the case of excise tax imposed on petroleum products,
the seller thereof may shift the tax burden to the buyer, the latter is the
proper party to claim for the refund in the case of exemption from excise
tax. Since the excise tax was imposed upon Petron Corporation as the
manufacturer of petroleum products, pursuant to Section 130(A)(2), and
that the corresponding excise taxes were indeed, paid by it, . . . any claim for
refund of the subject excise taxes should be filed by Petron
Corporation as the taxpayer contemplated under the law. Petitioner cannot
be considered as the taxpayer because it merely shouldered the burden of the
excise tax and not the excise tax itself.
Therefore, the right to claim for the refund of excise taxes paid on petroleum
products lies with Petron Corporation who paid and remitted the excise tax to
the BIR. Respondent, on the other hand, may only claim from Petron
Corporation the reimbursement of the tax burden shifted to the former by the
latter. The excise tax partaking the nature of an indirect tax, is clearly the
liability of the manufacturer or seller who has the option whether or not to
shift the burden of the tax to the purchaser. Where the burden of the tax is
shifted to the [purchaser], the amount passed on to it is no longer a

tax but becomes an added cost on the goods purchased which


constitutes a part of the purchase price. The incidence of taxation or the
person statutorily liable to pay the tax falls on Petron Corporation though the
impact of taxation or the burden of taxation falls on another person, which in
this case is petitioner Silkair.5 (Italics in the original; emphasis and
underscoring supplied)
Silkair filed a Motion for Reconsideration6 during the pendency of which or on
September 12, 2005 the Bengzon Law Firm entered its appearance as
counsel,7 without Silkairs then-counsel of record (Jimenez Gonzales Liwanag Bello
Valdez Caluya & Fernandez or "JGLaw") having withdrawn as such.
By Resolution8 of September 22, 2005, the CTA Second Division denied Silkairs
motion for reconsideration. A copy of the Resolution was furnished Silkairs counsel
JGLaw which received it on October 3, 2005.9
On October 13, 2005, JGLaw, with the conformity of Silkair, filed its Notice of
Withdrawal of Appearance.10 On even date, Silkair, through the Bengzon Law Firm,
filed a Manifestation/Motion11 stating:
Petitioner was formerly represented xxx by JIMENEZ GONZALES LIWANAG
BELLO VALDEZ CALUYA & FERNANDEZ (JGLaw).
1. On 24 August 2005, petitioner served notice to JGLaw of its decision
to cease all legal representation handled by the latter on behalf of the
petitioner. Petitioner also requested JGLaw to make arrangements for
the transfer of all files relating to its legal representation on behalf of
petitioner to the undersigned counsel. x x x
2. The undersigned counsel was engaged to act as counsel for the
petitioner in the above-entitled case; and thus, filed its entry of
appearance on 12 September 2005. x x x
3. The undersigned counsel, through petitioner, has received
information that the Honorable Court promulgated a Resolution on
petitioners Motion for Reconsideration. To date, the undersigned
counsel has yet to receive an official copy of the above-mentioned
Resolution. In light of the foregoing,undersigned counsel hereby
respectfully requests for an official copy of the Honorable Courts
Resolution on petitioners Motion for Reconsideration x x
x.12 (Underscoring supplied)
On October 14, 2005, the Bengzon Law Firm received its requested copy of the
September 22, 200513 CTA Second Division Resolution. Thirty-seven days later or on
October 28, 2005, Silkair, through said counsel, filed a Motion for Extension of Time

to File Petition for Review14 before the CTA En Banc which gave it until November 14,
2005 to file a petition for review.
On November 11, 2005, Silkair filed another Motion for Extension of Time. 15 On even
date, the Bengzon Law Firm informed the CTA of its withdrawal of appearance as
counsel for Silkair with the information, that Silkair would continue to be represented
by Atty. Teodoro A. Pastrana, who used to be with the firm but who had become a
partner of the Pastrana and Fallar Law Offices. 16
The CTA En Banc granted Silkairs second Motion for Extension of Time, giving Silkair
until November 24, 2005 to file its petition for review. On November 17, 2005, Silkair
filed its Petition for Review17 before the CTA En Banc.
By Resolution of May 19,2006, the CTA En Banc dismissed 18 Silkairs petition for
review for having been filed out of time in this wise:
A petitioner is given a period of fifteen (15) days from notice of award,
judgment, final order or resolution, or denial of motion for new trial or
reconsideration to appeal to the proper forum, in this case, the CTA En Banc.
This is clear from both Section 11 and Section 9 of Republic Act No.
9282 x x x.
xxxx
The petitioner, through its counsel of record Jimenez, Gonzalez, L[iwanag],
Bello, Valdez, Caluya & Fernandez Law Offices, received the Resolution dated
September 22, 2005 on October 3, 2005. At that time, the petitioner had two
counsels of record, namely, Jimenez, Gonzales, L[iwanag], Bello, Valdez,
Caluya & Fernandez Law Offices and The Bengzon Law Firm which filed its
Entry of Appearance on September 12, 2005. However, as of said date, Atty.
Mary Jane B. Austria-Delgado of Jimenez, Gonzales, L[iwanag], Bello, Valdez,
Caluya & Fernandez Law Offices was still the counsel of record considering
that the Notice of Withdrawal of Appearance signed by Atty. Mary Jane B.
Austria-Delgado was filed only on October 13, 2005 or ten (10) days after
receipt of the September 22, 2005 Resolution of the Courts Second Division.
This notwithstanding, Section 2 of Rule 13 of the Rules of Court provides
that if any party has appeared by counsel, service upon him shall be made
upon his counsel or one of them, unless service upon the party himself is
ordered by the Court. Where a party is represented by more than one counsel
of record, "notice to any one of the several counsel on record is equivalent to
notice to all the counsel (Damasco vs. Arrieta, et. al., 7 SCRA 224)."
Considering that petitioner, through its counsel of record, had received the
September 22, 2005 Resolution as early as October 3, 2005, it had only until
October 18, 2005 within which to file its Petition for Review. Petitioner only
managed to file the Petition for Review with the Court En Banc on November

17, 2005 or [after] thirty (30) days had lapsed from the final date of October
18, 2005 to appeal.
The argument that it requested Motions for Extension of Time on October 28,
2005 or ten (10) days from the appeal period and the second Motion for
Extension of Time to file its Petition for Review on November 11, 2005 and its
allowance by the CTA En Banc notwithstanding, the questioned Decision is no
longer appealable for failure to timely file the necessary Petition for
Review.19 (Emphasis in the original)
In a Separate Concurring Opinion,20 CTA Associate Justice Juanito C. Castaeda, Jr.
posited that Silkair is not the proper party to claim the tax refund.
Silkair filed a Motion for Reconsideration21 which the CTA En Banc denied.22 Hence,
the present Petition for Review23 which raises the following issues:
I. WHETHER OR NOT THE PETITION FOR REVIEW FILED WITH THE HONORABLE
COURT OF TAX APPEALS EN BANC WAS TIMELY FILED.
II. APPEAL BEING AN ESSENTIAL PART OF OUR JUDICIAL SYSTEM, WHETHER OR
NOT PETITIONER SHOULD BE DEPRIVED OF ITS RIGHT TO APPEAL ON THE
BASIS OF TECHNICALITY.
III. ASSUMING THE HONORABLE SUPREME COURT WOULD HOLD THAT THE
FILING OF THE PETITITON FOR REVIEW WITH THE HONORABLE COURT OF TAX
APPEALS EN BANC WAS TIMELY, WHETHER OR NOT THE PETITIONER IS THE
PROPER PARTY TO CLAIM FOR REFUND OR TAX CREDIT.24 (Underscoring
supplied)
Silkair posits that "the instant case does not involve a situation where the petitioner
was represented by two (2) counsels on record, such that notice to the former
counsel would be held binding on the petitioner, as in the case ofDamasco v. Arrieta,
etc., et al.25 x x x heavily relied upon by the respondent"; 26 and that "the case
of Dolores De Mesa Abad v. Court of Appeals27 has more appropriate application to
the present case."28
In Dolores De Mesa Abad, the trial court issued an order of November 19, 1974
granting the therein private respondents Motion for Annulment of documents and
titles. The order was received by the therein petitioners counsel of record, Atty.
Escolastico R. Viola, on November 22, 1974 prior to which or on July 17, 1974, Atty.
Vicente Millora of the Millora, Tobias and Calimlim Law Office had filed an
"Appearance and Manifestation." Atty. Millora received a copy of the trial courts
order on December 9, 1974. On January 4, 1975, the therein petitioners, through
Atty. Ernesto D. Tobias also of the Millora, Tobias and Calimlim Law Office, filed their
Notice of Appeal and Cash Appeal Bond as well as a Motion for Extension of the
period to file a Record on Appeal. They filed the Record on Appeal on January 24,

1975. The trial court dismissed the appeal for having been filed out of time, which
was upheld by the Court of Appeals on the ground that the period within which to
appeal should be counted from November 22, 1974, the date Atty. Viola received a
copy of the November 19, 1974 order. The appellate court held that Atty. Viola was
still the counsel of record, he not having yet withdrawn his appearance as counsel
for the therein petitioners. On petition for certiorari, 29 this Court held
x x x [R]espondent Court reckoned the period of appeal from the time
petitioners original counsel, Atty. Escolastico R. Viola, received the Order
granting the Motion for Annulment of documents and titles on November 22,
1974. But as petitioners stress, Atty. Vicente Millora of the Millora, Tobias and
Calimlim Law Office had filed an "Appearance and Manifestation" on July 16,
1974. Where there may have been no specific withdrawal by Atty. Escolastico
R. Viola, for which he should be admonished, by the appearance of a new
counsel, it can be said that Atty. Viola had ceased as counsel for petitioners. In
fact, Orders subsequent to the aforesaid date were already sent by the trial
Court to the Millora, Tobias and Calimlim Law Office and not to Atty. Viola.
Under the circumstances, December 9, 1974 is the controlling date of receipt
by petitioners counsel and from which the period of appeal from the Order of
November 19, 1974 should be reckoned. That being the case, petitioners x x
x appeal filed on January 4, 1975 was timely filed. 30 (Underscoring supplied)
The facts of Dolores De Mesa Abad are not on all fours with those of the present
case. In any event, more recent jurisprudence holds that in case of failure to comply
with the procedure established by Section 26, Rule 138 31 of the Rules of Court re the
withdrawal of a lawyer as a counsel in a case, the attorney of record is regarded as
the counsel who should be served with copies of the judgments, orders and
pleadings.32 Thus, where no notice of withdrawal or substitution of counsel has been
shown, notice to counsel of record is, for all purposes, notice to the client. 33 The
court cannot be expected to itself ascertain whether the counsel of record has been
changed.34
In the case at bar, JGLaw filed its Notice of Withdrawal of Appearance on October 13,
200535 after the Bengzon Law Firm had entered its appearance. While Silkair claims it
dismissed JGLaw as its counsel as early as August 24, 2005, the same was
communicated to the CTA only on October 13, 2005. 36 Thus, JGLaw was still Silkairs
counsel of record as of October 3, 2005 when a copy of the September 22, 2005
resolution of the CTA Second Division was served on it. The service upon JGLaw on
October 3, 2005 of the September 22, 2005 resolution of CTA Second Division was,
therefore, for all legal intents and purposes, service to Silkair, and the CTA correctly
reckoned the period of appeal from such date.
TECHNICALITY ASIDE, on the merits, the petition just the same fails.

Silkair bases its claim for refund or tax credit on Section 135 (b) of the NIRC of 1997
which reads
Sec. 135. Petroleum Products sold to International Carriers and
Exempt Entities of Agencies. Petroleum products sold to the following are
exempt from excise tax:
xxxx
(b) Exempt entities or agencies covered by tax treaties, conventions, and
other international agreements for their use and consumption: Provided,
however, That the country of said foreign international carrier or exempt
entities or agencies exempts from similar taxes petroleum products sold to
Philippine carriers, entities or agencies; x x x
x x x x,
and Article 4(2) of the Air Transport Agreement between the Government of the
Republic of the Philippines and the Government of the Republic of Singapore (Air
Transport Agreement between RP and Singapore) which reads
Fuel, lubricants, spare parts, regular equipment and aircraft stores introduced
into, or taken on board aircraft in the territory of one Contracting party by, or
on behalf of, a designated airline of the other Contracting Party and intended
solely for use in the operation of the agreed services shall, with the exception
of charges corresponding to the service performed, be exempt from the same
customs duties, inspection fees and other duties or taxes imposed in the
territories of the first Contracting Party , even when these supplies are to be
used on the parts of the journey performed over the territory of the
Contracting Party in which they are introduced into or taken on board. The
materials referred to above may be required to be kept under customs
supervision and control.
The proper party to question, or seek a refund of, an indirect tax is the statutory
taxpayer, the person on whom the tax is imposed by law and who paid the same
even if he shifts the burden thereof to another. 37 Section 130 (A) (2) of the NIRC
provides that "[u]nless otherwise specifically allowed, the return shall be filed and
the excise tax paid by the manufacturer or producer before removal of domestic
products from place of production." Thus, Petron Corporation, not Silkair, is the
statutory taxpayer which is entitled to claim a refund based on Section 135 of the
NIRC of 1997 and Article 4(2) of the Air Transport Agreement between RP and
Singapore.
Even if Petron Corporation passed on to Silkair the burden of the tax, the additional
amount billed to Silkair for jet fuel is not a tax but part of the price which Silkair had
to pay as a purchaser.38

Silkair nevertheless argues that it is exempt from indirect taxes because the Air
Transport Agreement between RP and Singapore grants exemption "from the same
customs duties, inspection fees and other duties or taxes imposed in the territory of
the first Contracting Party."39 It invokes Maceda v. Macaraig, Jr.40 which upheld the
claim for tax credit or refund by the National Power Corporation (NPC) on the ground
that the NPC is exempt even from the payment of indirect taxes.
Silkairss argument does not persuade. In Commissioner of Internal Revenue v.
Philippine Long Distance Telephone Company,41 this Court clarified the ruling
in Maceda v. Macaraig, Jr., viz:
It may be so that in Maceda vs. Macaraig, Jr., the Court held that an exemption
from "all taxes" granted to the National Power Corporation (NPC) under its
charter includes both direct and indirect taxes. But far from providing PLDT
comfort, Maceda in fact supports the case of herein petitioner, the correct
lesson of Macedabeing that an exemption from "all taxes" excludes indirect
taxes, unless the exempting statute, like NPCs charter, is so couched as to
include indirect tax from the exemption. Wrote the Court:
x x x However, the amendment under Republic Act No. 6395
enumerated the details covered by the exemption. Subsequently, P.D.
380, made even more specific the details of the exemption of NPC to
cover, among others, both direct and indirect taxes on all petroleum
products used in its operation. Presidential Decree No. 938 [NPCs
amended charter] amended the tax exemption by simplifying the same
law in general terms. It succinctly exempts NPC from "all forms of taxes,
duties[,] fees"
The use of the phrase "all forms" of taxes demonstrates the intention of
the law to give NPC all the tax exemptions it has been enjoying before
xxxx
It is evident from the provisions of P.D. No. 938 that its purpose is to
maintain the tax exemption of NPC from all forms of taxes including
indirect taxes as provided under R.A. No. 6395 and P.D. 380 if it is to
attain its goals. (Italics in the original; emphasis supplied) 42
The exemption granted under Section 135 (b) of the NIRC of 1997 and Article 4(2) of
the Air Transport Agreement between RP and Singapore cannot, without a clear
showing of legislative intent, be construed as including indirect taxes. Statutes
granting tax exemptions must be construed in strictissimi juris against the taxpayer
and liberally in favor of the taxing authority, 43 and if an exemption is found to exist,
it must not be enlarged by construction. 44
WHEREFORE, the petition is DENIED.

Costs against petitioner.


SO ORDERED.
SILKAIR (SINGAPORE) PTE.
LTD.,
Petitioner,

- versus -

G.R. Nos. 171383 & 172379


Present:
CARPIO, J.,
Acting Chairperson,*
AUSTRIA-MARTINEZ,**
CORONA,
CARPIO MORALES,*** and
LEONARDO-DE CASTRO, JJ.

COMMISSIONER OF INTERNAL
Promulgated:
REVENUE,
Respondent.
November 14, 2008
x--------------------------------------------------x
DECISION

CARPIO, J.:
The Case
G.R. No. 171383

Silkair (Singapore) Pte. Ltd. (petitioner) filed this Petition for Review [1] to
reverse the Court of Tax Appeals Decision [2] dated 20 October 2005 in C.T.A. Case
No. 6217 as well as the Resolution dated 3 February 2006 denying the Motion for
Reconsideration. In the assailed decision, the Court of Tax Appeals En Banc denied
petitioners claim for refund or issuance of a tax credit certificate of P4,239,374.81,
representing excise taxes paid on petitioners purchase of aviation jet fuel from
Petron Corporation (Petron) for the period from 1 January 1999 to 30 June 1999.

G.R. No. 172379

Petitioner filed this Petition for Review[3] to reverse the Court of Tax Appeals
Decision[4] dated 5 January 2006 in C.T.A. Case No. 6308 as well as the Resolution
dated 18 April 2006 denying the Motion for Reconsideration. In the assailed decision,
the Court of Tax Appeals En Banc denied petitioners claim for refund or issuance of a
tax credit certificate of P4,831,224.70, representing excise taxes paid on petitioners
purchase of aviation jet fuel from Petron for the period from 1 July 1999 to 31
December 1999.
On 2 August 2006, this Court issued a resolution to consolidate both cases
since they involve the same parties and the same issue, whether petitioner is
entitled to a refund of the excise taxes paid on its purchases of aviation jet fuel from
Petron.

The Facts

Petitioner is a foreign corporation organized under the laws of Singapore with a


Philippine representative office in Cebu City. It is engaged in business as an on-line
international carrier, operating the Singapore-Cebu-Singapore, Singapore-DavaoCebu-Singapore, and Singapore-Cebu-Davao-Singapore routes. [5]

From 1 January 1999 to 31 December 1999, petitioner purchased aviation jet


fuel from Petron for use on petitioners international flights. [6] Based on the Aviation
Delivery Receipts and Invoices presented, P3.67 per liter as excise (specific) tax was
added to the amount paid by petitioner on its purchases of aviation jet fuel.
[7]

Petitioner, through its sister company Singapore Airlines Ltd., paid P4,239,374.81

from 1 January 1999 to 30 June 1999[8] and P4,831,224.70 from 1 July 1999 to 31
December 1999,[9] as excise taxes for its purchases of the aviation jet fuel from
Petron. Petitioner, contending that it is exempt from the payment of excise taxes,
filed a formal claim for refund with the Commissioner of Internal Revenue
(respondent).
Petitioner claims that it is exempt from the payment of excise tax under the
1997 National Internal Revenue Code (NIRC), specifically Section 135, and under

Article 4 of the Air Transport Agreement between the Governments of the Republic of
the Philippines and the Republic of Singapore (Air Agreement). [10]
Section 135 of the NIRC provides:
SEC. 135. Petroleum Products Sold to International Carriers and
Exempt Entities or Agencies. - Petroleum products sold to the following are
exempt from excise tax:
(a) International carriers of Philippine or foreign registry on their use or
consumption outside the Philippines: Provided, That the petroleum products
sold to these international carriers shall be stored in a bonded storage tank
and may be disposed of only in accordance with the rules and regulations to
be prescribed by the Secretary of Finance, upon recommendation of the
Commissioner;
(b) Exempt entities or agencies covered by tax treaties, conventions
and other international agreements for their use or consumption: Provided,
however, That the country of said foreign international carrier or exempt
entities or agencies exempts from similar taxes petroleum products sold to
Philippine carriers, entities or agencies; and
(c)
taxes.[11]

Entities which are by law exempt from direct and indirect

Article 4 of the Air Agreement provides:


Art. 4
xxx
2. Fuel, lubricants, spare parts, regular equipment and aircraft stores
introduced into, or taken on board aircraft in the territory of one Contracting
Party by, or on behalf of, a designated airline of the other Contracting Party
and intended solely for use in the operation of the agreed services shall, with
the exception of charges corresponding to the services performed, be exempt
from the same custom duties, inspection fees and other duties or taxes
imposed in the territory of the first Contracting Party, even when these
supplies are to be used on the parts of the journey performed over the
territory of the Contracting Party in which they are introduced into or taken on
board. The materials referred to above may be required to be kept under
customs supervision and control.[12]

Petitioner contends that in reality, it paid the excise taxes due on the
transactions and Petron merely remitted the payment to the Bureau of Internal
Revenue (BIR). Petitioner argues that to adhere to the view that Petron is the legal
claimant of the refund will make petitioners right to recover the erroneously paid

taxes dependent solely on Petrons action over which petitioner has no control. If
Petron fails to act or acts belatedly, petitioners claim will be barred, depriving
petitioner of its private property. [13]
Petitioner also maintains that to hold that only Petron can legally claim the
refund will negate the tax exemption expressly granted to petitioner under the NIRC
and the Air Agreement.[14] Petitioner argues that a tax exemption is a personal
privilege of the grantee, which is petitioner in this case. Petitioner further argues
that a tax exemption granted to the buyer cannot be availed of by the seller; hence,
in the present case, Petron as seller cannot legally claim the refund. On the other
hand, if only the entity that paid the tax Petron in this case can claim the refund,
then petitioner as the grantee of the tax exemption cannot enjoy its tax exemption.
In short, neither petitioner nor Petron can claim the refund, rendering the tax
exemption useless. Petitioner submits that this is contrary to the language and intent
of the NIRC and the Air Agreement.[15]
Petitioner also cites this Courts Resolution in Maceda v. Macaraig, Jr.,
[16]

quoting the opinion of the Secretary of Justice which states, thus:


The view which refuses to accord the exemption because the tax is first
paid by the seller disregards realities and gives more importance to form than
substance. Equity and law always exalt substance over form. [17]

Petitioner believes that its tax exemption under Section 135 of the NIRC also
includes its entitlement to a refund from the BIR in any case of erroneous payment of
excise tax.[18]
Respondent claims that as explained in Philippine Acetylene Co., Inc. v.
Commissioner of Internal Revenue,[19] the nature of an indirect tax allows the tax to
be passed on to the purchaser as part of the commoditys purchase price. However,
an indirect tax remains a tax on the seller. Hence, if the buyer happens to be tax
exempt, the seller is nonetheless liable for the payment of the tax as the same is a
tax not on the buyer but on the seller. [20]

Respondent insists that in indirect taxation, the manufacturer or seller has the
option to shift the burden of the tax to the purchaser. If and when shifted, the
amount added by the manufacturer or seller becomes part of the purchase price of
the goods. Thus, the purchaser does not really pay the tax but only the price of the
commodity and the liability for the payment of the indirect tax remains with the
manufacturer or seller.[21] Since the liability for the excise tax payment is imposed by
law on Petron as the manufacturer of the petroleum products, any claim for refund
should only be made by Petron as the statutory taxpayer. [22]
The Ruling of the Court of Tax Appeals

G.R. No. 171383


On 20 October 2005, the Court of Tax Appeals En Banc (CTA) ruled that the
excise tax imposed on the removal of petroleum products by the oil companies is an
indirect tax.[23] Although the burden to pay an indirect tax can be passed on to the
purchaser of the goods, the liability to pay the indirect tax remains with the
manufacturer or seller.[24]When the manufacturer or seller decides to shift the burden
of the indirect tax to the purchaser, the tax becomes a part of the price; therefore,
the purchaser does not really pay the tax per se but only the price of the commodity.
[25]

The CTA pointed out that Section 130(A)(2) [26] of the NIRC provides that the
liability for the payment of excise taxes is imposed upon the manufacturer or
producer of the petroleum products. Under the law, the manufacturer or producer is
the taxpayer. The CTA stated that it is only the taxpayer that may ask for a refund in
case of erroneous payment of taxes. Citing Cebu Portland Cement Co. v. Collector of
Internal Revenue,[27] the CTA ruled that the producer of the goods is the one entitled
to claim for a refund of indirect taxes. [28] The CTA held that since the liability for the
excise taxes was placed on Petron as the manufacturer of the petroleum products
and it was shown in the Excise Tax Returns [29] that the excise taxes were paid by
Petron, any claim for refund of the excise taxes should only be made by Petron as
the taxpayer. This is in consonance with the rule on strictissimi juris with respect to
tax exemptions. Petitioner cannot be considered the taxpayer because what was

transferred to petitioner was only the burden and not the liability to pay the excise
tax on petroleum products.[30]
The CTA also considered the Aviation Fuel Supply Agreement between
petitioner and Petron, which states:
Buyer shall pay any taxes, fees or other charges imposed by any
national, local or airport authority on the delivery, sale, inspection, storage
and use of fuel, except for taxes on Sellers income and taxes on raw material.
To the extent allowed, Seller shall show these taxes, fees and other charges as
separate items on the invoice for the account of the Buyer. [31]

However, the CTA held that even with this provision, the liability for the excise
tax remained with Petron as manufacturer or producer of the aviation jet fuel. The
shifting of the burden of the excise tax to petitioner did not transform petitioner into
a taxpayer. Hence, Petron is the proper party that can claim for refund of any
erroneous excise tax payments.[32]
G.R. No. 172379
The CTA En Banc held that excise taxes on domestic products are paid by the
manufacturer or producer before removal of the products from the place of
production. The payment of an excise tax, being an indirect tax, can be shifted to the
purchaser of goods but the statutory liability for such payment is still with the seller
or manufacturer.[33] The CTA cited Maceda v. Macaraig, Jr.:[34]
It may be useful to make a distinction, for the purpose of this
disposition, between a direct tax and an indirect tax. A direct tax is a tax for
which a taxpayer is directly liable on the transaction or business it is engaged
in. Examples are custom duties and ad valorem taxes paid by the oil
companies to the Bureau of Customs for their importation of crude oil, and the
specific and ad valorem taxes they pay to the Bureau of Internal Revenue
after converting the crude oil into petroleum products.
On the other hand, indirect taxes are taxes primarily paid by persons
who can shift the burden upon someone else. For example, the excise tax
and ad valorem taxes that the oil companies pay to the Bureau of Internal
Revenue upon removal of petroleum products from its refinery can be shifted
to its buyer, like the NPC, by adding them to the cash and/or selling
price.[35]

The CTA further cited Philippine Acetylene Co., Inc. v. Commissioner of Internal
Revenue[36] and Contex Corporation v. Hon. Commissioner of Internal Revenue [37] and
concluded that the tax sought to be refunded is an excise tax on petroleum products,
partaking of the nature of an indirect tax.[38]
The CTA further ruled that while it is cognizant of the exempt status of
petitioner under the NIRC and the Air Agreement, it is also aware that the right to
claim for refund of taxes erroneously paid lies with the person statutorily liable to
pay the tax in accordance with Section 204 of the NIRC.[39] The CTA also suggested
that petitioner should invoke its tax exemption to Petron before buying the
petroleum products.[40] The CTA concluded that the right to claim for the refund of
the excise taxes paid on the petroleum products lies with Petron which paid and
remitted the excise taxes to the BIR.
The Issue
Petitioner submits this sole issue for our consideration: whether petitioner is
the proper party to claim a refund for the excise taxes paid. [41]
The Ruling of the Court
The issue presented is not novel. In a similar case involving the same parties,
this Court has categorically ruled that the proper party to question, or seek a refund
of an indirect tax is the statutory taxpayer, the person on whom the tax is imposed
by law and who paid the same even if he shifts the burden thereof to
another.[42] The Court added that even if Petron Corporation passed on to Silkair
the burden of the tax, the additional amount billed to Silkair for jet fuel is not a tax
but part of the price which Silkair had to pay as a purchaser. [43]
An excise tax is an indirect tax where the tax burden
can be shifted to the consumer but the tax liability remains with the
manufacturer or producer.

Section 129 of the NIRC provides that excise taxes refer to taxes imposed on
specified goods manufactured or produced in the Philippines for domestic sale or
consumption or for any other disposition and to things imported. The excise taxes
are collected from manufacturers or producers before removal of the domestic
products from the place of production. Although excise taxes can be considered as
taxes on production, they are really taxes on property as they are imposed on certain
specified goods.[44]
Section 148(g) of the NIRC provides that there shall be collected on aviation jet
fuel an excise tax of P3.67 per liter of volume capacity. Since the tax imposed is
based on volume capacity, the tax is referred to as specific tax. [45] However, excise
tax, whether classified as specific or ad valorem tax, is basically an indirect tax
imposed on the consumption of a specified list of goods or products. The tax is
directly levied on the manufacturer upon removal of the taxable goods from the
place of production but in reality, the tax is passed on to the end consumer as part of
the selling price of the goods sold.[46]
In Commissioner of Internal Revenue v. Philippine Long Distance Company,
[47]

the Court explained the difference between a direct tax and an indirect tax:
Based on the possibility of shifting the incidence of taxation, or as to
who shall bear the burden of taxation, taxes may be classified into either
direct tax or indirect tax.
In context, direct taxes are those that are exacted from the very
person who, it is intended or desired, should pay them; they are impositions
for which a taxpayer is directly liable on the transaction or business he is
engaged in.
On the other hand, indirect taxes are those that are demanded,
in the first instance, from, or are paid by, one person in the
expectation and intention that he can shift the burden to someone
else. Stated elsewise, indirect taxes are taxes wherein the liability for
the payment of the tax falls on one person but the burden thereof
can be shifted or passed on to another person, such as when the tax
is imposed upon goods before reaching the consumer who ultimately
pays for it. When the seller passes on the tax to his buyer, he, in
effect, shifts the tax burden, not the liability to pay it, to the
purchaser as part of the price of goods sold or services
rendered. (Emphasis supplied)

In Maceda v. Macaraig, Jr., the Court specifically mentioned excise tax as an


example of an indirect tax where the tax burden can be shifted to the buyer:
On the other hand, indirect taxes are taxes primarily paid by persons
who can shift the burden upon someone else. For example, the excise and ad
valorem taxes that the oil companies pay to the Bureau of Internal Revenue
upon removal of petroleum products from its refinery can be shifted to its
buyer, like the NPC, by adding them to the cash and/or selling price. [48]

When Petron removes its petroleum products from its refinery in Limay,
Bataan,[49] it

pays

the

excise

tax

due

on

the

petroleum

products

thus

removed. Petron, as manufacturer or producer, is the person liable for the payment
of the excise tax as shown in the Excise Tax Returns filed with the BIR. Stated
otherwise, Petron is the taxpayer that is primarily, directly and legally liable for the
payment of the excise taxes. However, since an excise tax is an indirect tax, Petron
can transfer to its customers the amount of the excise tax paid by treating it as part
of the cost of the goods and tacking it on to the selling price.
As correctly observed by the CTA, this Court held in Philippine Acetylene Co.,
Inc. v. Commissioner of Internal Revenue:
It may indeed be that the economic burden of the tax finally falls on the
purchaser; when it does the tax becomes part of the price which the purchaser
must pay.[50]

Even if the consumers or purchasers ultimately pay for the tax, they are not
considered the taxpayers. The fact that Petron, on whom the excise tax is imposed,
can shift the tax burden to its purchasers does not make the latter the taxpayers and
the former the withholding agent.
Petitioner, as the purchaser and end-consumer, ultimately bears the tax
burden, but this does not transform petitioners status into a statutory taxpayer.
In the refund of indirect taxes, the statutory taxpayer
is the proper party who can claim the refund.

Section 204(c) of the NIRC provides:


Sec. 204. Authority of the Commissioner to Compromise, Abate, and
Refund or Credit Taxes. The Commissioner may xxx
(c) Credit or refund taxes erroneously or illegally received or penalties
imposed without authority, refund the value of internal revenue stamps when
they are returned in good condition by the purchaser, and, in his discretion,
redeem or change unused stamps that have been rendered unfit for use and
refund their value upon proof of destruction. No credit or refund of taxes
or penalties shall be allowed unless the taxpayer files in writing with
the Commissioner a claim for credit or refund within two (2) years after
the payment of the tax or penalty: Provided, however, That a return filed
showing an overpayment shall be considered as a written claim for credit or
refund. (Emphasis and underscoring supplied)

The person entitled to claim a tax refund is the statutory taxpayer. Section
22(N)

of

the

NIRC

defines

taxpayer

as

any

person

subject

to

tax.

In Commissioner of Internal Revenue v. Procter and Gamble Phil. Mfg. Corp., the
Court ruled that:
A person liable for tax has been held to be a person subject to tax
and properly considered a taxpayer. The terms liable for tax and subject
to tax both connote a legal obligation or duty to pay a tax. [51]

The excise tax is due from the manufacturers of the petroleum products and is
paid upon removal of the products from their refineries. Even before the aviation jet
fuel is purchased from Petron, the excise tax is already paid by Petron. Petron,
being the manufacturer, is the person subject to tax. In this case, Petron, which
paid the excise tax upon removal of the products from its Bataan refinery, is the
person liable for tax. Petitioner is neither a person liable for tax nor a person
subject to tax. There is also no legal duty on the part of petitioner to pay the excise
tax; hence, petitioner cannot be considered the taxpayer.
Even if the tax is shifted by Petron to its customers and even if the tax is billed
as a separate item in the aviation delivery receipts and invoices issued to its
customers, Petron remains the taxpayer because the excise tax is imposed directly

on Petron as the manufacturer. Hence, Petron, as the statutory taxpayer, is the


proper party that can claim the refund of the excise taxes paid to the BIR.
The General Terms & Conditions for Aviation Fuel Supply (Supply Contract)
signed between petitioner (buyer) and Petron (seller) provide:
11.3 If Buyer is entitled to purchase any Fuel sold pursuant to the
Agreement free of any taxes, duties or charges, Buyer shall timely deliver
to Seller a valid exemption certificate for such purchase.[52] (Emphasis
supplied)

This provision instructs petitioner to timely submit a valid exemption certificate


to Petron in order that Petron will not pass on the excise tax to petitioner.

As

correctly suggested by the CTA, petitioner should invoke its tax exemption to Petron
before buying the aviation jet fuel. Petron, however, remains the statutory taxpayer
on those excise taxes.
Revenue Regulations No. 3-2008 (RR 3-2008) provides that subject to the
subsequent filing of a claim for excise tax credit/refund or product replenishment, all
manufacturers of articles subject to excise tax under Title VI of the NIRC of 1997, as
amended, shall pay the excise tax that is otherwise due on every removal thereof
from the place of production that is intended for exportation or sale/delivery to
international carriers or to tax-exempt entities/agencies. [53] The Department of
Finance and the BIR recognize the tax exemption granted to international carriers
but they consistently adhere to the view that manufacturers of articles subject to
excise tax are the statutory taxpayers that are liable to pay the tax, thus, the proper
party to claim any tax refunds.
WHEREFORE, we DENY the petition. We AFFIRM the assailed Decisions dated
20 October 2005 and 5 January 2006 and the Resolutions dated 3 February 2006 and
18 April 2006 of the Court of Tax Appeals in C.T.A. Case Nos. 6217 and 6308,
respectively.

SO ORDERED.

G.R. No. L-39910 September 26, 1988


CECILIA TEODORO DAYRIT, TORIBIA TEODORO CASTANEDA, PRUDENCIO J.
TEODORO, FRANCISCO J. TEODORO, AND JOSEFINA TEODORO
TIONGSON, petitioners,
vs.
THE HONORABLE FERNANDO A. CRUZ, Presiding Judge, Branch XII, Court of
First Instance of Rizal, and MISAEL P. VERA, in his capacity as the
Commissioner of Internal Revenue, respondents.
Atienza, Tabora, Del Rosario & Castillo Law Offices and Tanada, Sanchez, Tanada &
Tanada Law Offices for petitioners.

GANCAYCO, J.:
The application of tax amnesty to the estate of the Teodoros is the issue in this case.
Petitioners are the legitimate children and heirs of the deceased spouses Marta J.
Teodoro who died intestate on July 1, 1965 and Don Toribio Teodoro who died testate
on August 30, 1965. Thereafter, the heirs of the deceased filed separate estate and
inheritance tax returns for the estates of the late spouses with the Bureau of Internal
Revenue. *
In the meantime, testate and intestate proceedings for the settlement of the
decedents' estates were filed 1 by Cecilia Teodoro-Dayrit, one of the petitioners
herein, in the then Court of First Instance of Caloocan City, ** Branch XII docketed as
Special Proceedings No. C-113. 2 On August 14, 1968, said petitioner was appointed
administratrix of the estate of Dona Marta and letters testamentary was issued in
her favor as executrix of the estate of Don Toribio.
On August 9,1972, the respondent Commissioner of Internal Revenue issued the
following deficiency estate and inheritance tax assessments: 3

Estate Tax & penalties

Estate of Doa

Estate of Don

Marta

Toribio ***

P1,662,072.34

P1,542,293.01

Inheritance Tax & interests

1,747,790.94

1,518,458.72

The aforementioned notice of deficiency assessments was received by petitioner


Dayrit on August 14, 1972. In a letter dated October 7, 1972, **** petitioners
through counsel, asked for a reconsideration of the said assessments alleging that
the same are contrary to law and not supported by sufficient evidence. 4 In the same
letter, petitioners requested a period of thirty (30) days within which to submit their
position paper in support of their claim.
Meanwhile, on October 16, 1972, Presidential Decree (P.D) No. 23, entitled
"Proclaiming Tax Amnesty Subject to Certain Conditions," was issued by then
President Ferdinand E. Marcos, quoted hereunder as follows:
xxx xxx xxx
1. In all cases of voluntary disclosure of previously untaxed income
realized here or abroad by any taxpayer, natural or juridical, the
collection of the income tax and penalties incident to nonpayment, as
well as all criminal and civil liabilities under the National Internal
Revenue Code, the Revised Penal Code, the Anti-Graft and Corrupt
Practices Act or any other law applicable thereto, is hereby condoned
and, in lieu thereof, a tax of TEN PERCENTUM (10%) on such previously
untaxed income is hereby imposed, subject to the following conditions:
(a) Such previously untaxed income must have been
earned or realized prior to 1972;
(b) The taxpayer must file a notice and return with the
Commissioner of Internal Revenue on or before March 31,
1972 showing such previously untaxed income; ...
2. The tax imposed under Paragraph 1 hereof, shall be paid within the
following period:
(a) If the amount does not exceed P10,000.00 the tax must
be paid at the time of the filing of notice and return but not
later than March 31, 1973;
(b) If the amount exceeds P10,000.00 the tax maybe paid
in two (2) installments, the first installment to be paid
upon the filing of the notice and return but not later than
March 31, 1973; and the second installment within three
(3) months from the date of the filing of the return but not
later than June 30, 1973. ....

On November 24, 1972, P.D. No. 67, was issued amending paragraphs 1 and 3 of P.D.
No. 23, to read as follows:
xxx xxx xxx
1. In all cases of voluntary disclosure of previously untaxed income
and/or wealth such as earnings, receipts, gifts bequests or any other
acquisitions from any source whatsoever which are taxable under the
National Internal Revenue Code, as amended realized here or abroad by
any taxpayer, natural or juridical; the collection of all internal revenue
taxes including the increments or penalties on account of non-payment
as well as all civil, criminal or administrative liabilities arising from or
incident to such disclosures under the National Internal Revenue Code,
the Revised Penal Code, the Anti-Graft and Corrupt Practices Act, the
Revised Administrative Code, the Civil Service laws and regulations,
laws and regulations on Immigration and Deportation, or any other
applicable law or proclamation, are hereby condoned and, in lieu
thereof, a tax of ten per centum (10%) on such previously untaxed
income or wealth is hereby imposed, subject to the following conditions:
a. Such previously untaxed income and/or wealth must
have been earned or realized prior to 1972;
b. The taxpayer must file a return with the Commissioner
of Internal Revenue on or before March 31, 1973, showing
such previously untaxed income and/or wealth; ....
In a tax return dated March 31, 1973, petitioner Cecilia Teodoro-Dayrit declared an
additional amount of P3,655,595.78 as part of the estates of the Teodoro spouses, for
additional valuation over and above the amount declared in the previous return for
estates and inheritance taxes of the said late spouses. 5 The Bureau of Internal
Revenue issued tax payment acceptance order Nos.1127185-86 and
1533011. 6 Pursuant to the aforesaid tax acceptance orders, the estates and heirs of
the deceased spouses Teodoro paid the amounts of P5,000.00, P30,046.68 and
P250,000.00 per official receipts Nos. 73201, 774037 and 964467 dated April 2,
1973, July 17, 1973 and October 31, 1973, respectively, 7 amounting to a total of
P285,046.68.
On March 14, 1974, respondent Commissioner of Internal Revenue filed a motion for
Allowance of Claim against the estates of spouses Teodoro and for an order of
payment of taxes in S.P. No. C-113 with the then Court of First Instance of Rizal,
Branch XII, praying that petitioner Dayrit be ordered to pay the Bureau of Internal
Revenue the sum of P6,470,396.81 plus surcharges and interest 8 Petitioners filed
two (2) separate oppositions alleging that the estate and inheritance taxes sought to
be collected have already been settled in accordance with the provisions of P.D. No.
23, as amended by P.D. No. 67 and that at any rate, the assessments have not
become final and executory. 9 In reply thereto, respondent Commissioner alleged that
petitioners could not avail of the tax amnesty in view of the existence of a prior
assessment. 10 Petitioners insisted that the tax amnesty could still be availed of
invoking Section 4, BIR Revenue Regulation No. 8-72. 11

On July 10, 1974, respondent Judge issued an order approving the claim of
respondent Commissioner and directing the payment of the estate and inheritance
taxes. 12 Dissastisfied with the decision, petitioners filed a motion for
reconsideration 13 but it was denied 14 in an order dated September 30, 1974.*****
Hence, the present petition.
Petitioners contend that respondent Judge acted without jurisdiction or in excess of
jurisdiction or with grave abuse of discretion amounting to lack of jurisdiction in
granting the respondent Commissioner's claim for estate and inheritance taxes
against the estates of the Teodoro spouses on the ground that due to the pendency
of their motion for reconsideration of the deficiency assessments issued by the
Commissioner, said tax assessments are not yet final and executory. Petitioners
stressed that the absence of a decision on the disputed assessments was a bar
against collection of taxes. Finally, petitioners insist that their act of filing an estate
and inheritance tax return of a previously untaxed wealth of the estates entitles said
estates to tax amnesty under P.D. No. 23, as amended by P.D. 67 and hence, it is an
error to grant respondent Commissioner's claim for collection of estate and
inheritance taxes.
On the other hand, respondent Commissioner contends that petitioners cannot avail
of the tax amnesty in view of the prior existing assessments issued against the
estates of the deceased spouses before the promulgation of P.D. No. 23. In support
thereof, respondent cited Section 4 of Revenue Regulation No. 15-72, amending
Section 4 of Regulation No. 8-12. Respondent Commissioner contends further that
neither may petitioners' act of filing a return of a previously untaxed income or
wealth in the amount of P3,655,595.98 entitled the estates to tax amnesty where
petitioners failed to pay the 10% tax in full within the time frame required under P.D.
No. 23, and that to allow petitioners to avail of the tax amnesty will render nugatory
the provisions of P.D. No. 68. Moreover, said respondent argues that certiorari is not
the proper remedy in that respondent Judge committed no grave abuse of discretion
in allowing the claim for collection of taxes and that if at all, it was merely an error of
judgment which can be corrected only on appeal, and in which case the
reglementary period for the same has already prescribed.
The main issue in this petition is whether an estate may avail of tax amnesty under
Presidential Decree No. 23 where there is already an existing assessment made prior
to the issuance of the said decree on the basis of the submitted estate and
inheritance tax returns by merely filing separate estate tax returns of an undeclared
and untaxed income over and above the original amount of the estate declared.
Anent petitioners' claim that the tax assessments against the estates of the Teodoro
spouses are not yet final, the court finds the claim untenable. In petitioners' motion
for reconsideration of the aforementioned assessments, petitioners requested then
Commissioner Misael P. Vera for a period of thirty (30) days from October 7, 1972
within which to submit a position paper that would embody their grounds for
reconsideration. However, no position paper was ever filed. 15 Such failure to file a
position paper may be construed as abandonment of the petitioners' request for
reconsideration. The court notes that it took the respondent Commissioner a period
of more than one (1) year and five (5) months, from October 7, 1972 to March 14,

1974, before finally instituting the action for collection. Under the circumstances of
the case, the act of the Commissioner in filing an action for allowance of the claim
for estate and inheritance taxes, may be considered as an outright denial of
petitioners' request for reconsideration.
From the date of receipt of the copy of the Commissioner's letter for collection of
estate and inheritance taxes against the estates of the late Teodoro spouses,
petitioners must contest or dispute the same and, upon a denial thereof, the
petitioners have a period of thirty (30) days within which to appeal the case to the
Court of Tax Appeals.16 This they failed to avail of .
Tax assessments made by tax examiners are presumed correct and made in good
faith. A taxpayer has to prove otherwise. 17 Failure of the petitioners to appeal to the
Court of Tax Appeals in due time made the assessments in question, final, executory
and demandable. 18
The petitioners' allegation that the Court of First Instance (CFI) lacks jurisdiction over
the subject of the case is likewise untenable. The assessments having become final
and executory, the CFI properly acquired jurisdiction. 19Neither is there merit in
petitioners' claim that the exclusive jurisdiction of the Court of Tax Appeals (CTA)
applies in the case. The aforesaid exclusive jurisdiction of the CTA arises only in
cases of disputed tax assessments. 20 As noted earlier, petitioners' letter dated
October 7, 1972 asking for reconsideration of the questioned assessments cannot be
considered as one disputing the assessments because petitioners failed to
substantiate their claim that the deficiency assessments are contrary to law.
Petitioners asked for a period of thirty (30) days within which to submit their position
paper but they failed to submit the same nonetheless. Hence, petitioners' letter for a
reconsideration of the assessments is nothing but a mere scrap of paper.
Petitioners' contention that the absence of a decision on their request for
reconsideration of the assessments is a bar to granting the claim for collection is
likewise without merit. In Republic vs. Lim Tian Teng Son & Co., Inc., 21 this Court had
occasion to rule that a decision on a request for reinvestigation is not a condition
precedent to the filing of an action for collection of taxes already assessed. This
Court ruled that "nowhere in the Tax Code is the Collector of Internal Revenue
required to rule first on a taxpayer's request for reconsideration before he can go to
court for the purpose of collecting the tax assessed. On the contrary, Section 305 of
the same Code withheld from all courts, except the Court of Tax Appeals under
Republic Act No. 1125, 22 the authority to restrain the collection of any national
internal revenue tax, fee or charge, thereby indicating the legislative policy to allow
the Collector of Internal Revenue much latitude on the speedy and prompt collection
of taxes."
Petitioners argue, however, that the Commissioner of Internal Revenue must first
rule on the taxpayer's protest against tax assessment so as not to deprive the
taxpayer of the remedy of appeal and that it is only from the receipt of the decision
that the right to appeal to the Court of Tax Appeals should run, citing for the
purpose San Juan vs. Velasquez 23 as well as Commissioner of Internal Revenue vs.
Gonzales. 24

The aforementioned cases are both not in point. In San Juan, the taxpayer
concerned, through his accountant, disputed the assessments of income tax and
deficiency income tax by adducing the reasons and explanations why said
assessments of income tax were not due and owing from the taxpayer. Thus, it was
therein ruled that having disputed the assessments at the opportune time, the
Commissioner of Internal Revenue cannot ignore the disputed assessments by
immediate immediately bringing an action to collect. By the same token
in Commissioner of Internal Revenue vs. Gonzales, the assessments of estate and
inheritance taxes were disputed by the taxpayer by invoking prescription as a
defense claiming that the assessments were made after the lapse of more than five
(5) years.
Payment of taxes being admittedly a burden, taxpayers should not be left without
any recourse when they feel aggrieved due to the erroneous and burdensome
assessments made by a Bureau of Internal Revenue agent or by the Commissioner.
Said right is vested upon adversely affected taxpayers under Republic Act No. 1125.
It cannot be rendered nugatory through the Commissioner's act of immediately filing
an action for collection without ruling beforehand on the disputed
assessments. 25 However, the remedy of an aggrieved taxpayer is not without any
limitation. A taxpayer's right to contest assessments, particularly the right to appeal
to the Court of Tax Appeals, may be waived or lost as in this case. 26
The requirement for the Commissioner to rule on disputed assessments before
bringing an action for collection is applicable only in cases where the assessment
was actually disputed, adducing reasons in support thereto. In the present case
where the petitioners did not actually contest the assessments by stating the basis
thereof, the respondent Commissioner need not rule on their request.
Taxes are the lifeblood of the nation through which the government agencies
continue to operate and with which the State effects its functions for the welfare of
its constituents. We cannot tolerate taxpayers hampering expedient collection of
taxes by their failure to act within a reasonable period. No government could exist if
all litigants were permitted to delay the collection of its taxes. 27 Thus, this Court
ruled earlier that a suit for the collection of internal revenue taxes, as in this case,
where the assessment has already become final and executory, the action to collect
is akin to an action to enforce the judgment. No inquiry can be made therein as to
the merits of the original case or the justness of the judgment relied upon. 28
In view of the foregoing discussions, petitioners' allegation of grave abuse of
discretion on the part of the respondent judge must perforce fall. Considering further
that the court a quo properly acquired jurisdiction over the subject matter of the
case, petitioners should have appealed the case. The order of the court a quo dated
September 30,1974, was received by the petitioners on October 16, 1974.
Petitioners should have appealed within a period of fifteen (15) days from receipt
thereof but they failed to do so. ****** As petitioners failed to file a timely appeal
from the order of the trial court, they can no longer avail of the remedy of a special
civil action for certiorari in lieu of appeal. There is no error of jurisdiction committed
by the trial court. 29

On the other hand with respect the petitioners' plea that the estate is at any rate
entitled to tax amnesty, a reading of P.D. No. 23 30 reveals that in order to avail of
tax amnesty, it is required, among others, that there should be a voluntary
disclosure of a previously untaxed income. This was the pronouncement of this Court
in Nepomuceno vs. Montecillo 31 with respect to P.D. 370 32 which was decreed as a
complement of P.D. Nos. 23 and 157. In addition thereto, said income must have
been earned or realized prior to 1972 and the tax return must be filed on or before
March 31, 1973. Considering that P.D. No. 23 was issued on October 16, 1972, the
court rules that the said decree embraces only those income declared in pursuance
thereof within the taxable year 1972. The time frame cannot be stretched to include
declarations made prior to the issuance of the said decree or those made outside of
the time frame as envisioned in the said decree. Thus, the estates of the Teodoro
spouses which have been declared separately sometime in the 1960's are clearly
outside the coverage of the tax amnesty provision.
Petitioners argue, however, that even if a notice of deficiency assessment had
already been issued, the estates may still avail of tax amnesty if the basis of such
deficiency assessment is either the failure to file a return or the omission of items of
taxable income for a return already filed or the under declaration of said return,
citing P.D. No. 67 and Section 4 of BIR Revenue Regulation No. 8-72.
There is no merit in this contention. Even if P.D. No. 67, as an amendment to P.D. 23,
enlarges the coverage of tax amnesty to include wealth such as earnings, receipts,
gifts, bequests or any other acquisitions from any source whatsoever, said decree
reiterates the need of voluntary disclosure on the part of the taxpayer filing the
return in order to avail of the tax amnesty. The only noticeable departure from P.D.
No. 23 is the extension of the date for the filing of the return from March 31, 1972 to
March 31, 1973. Thus, this Court finds that the same policy observed in the issuance
of P.D. No. 23, governs P.D. No. 67. In addition thereto, it gives the tax evaders who
failed to avail of the provisions of P.D. No. 23 a chance to reform themselves. An
examination of both decrees does not show that taxpayers availing of the tax
amnesty in accordance with P.D. No. 67, are entitled to blanket coverage of
declarations made prior to the issuance of said decrees.
Petitioners argue that the estates of their parents declared for estate tax valuation
sometime in the 1960's can avail of the tax amnesty when petitioners declared an
additional amount of the estates over and above that which was previously declared.
A reading of P.D. No. 67 reveals that tax amnesty is extendible only to those
declarations made pursuant to said decree. Thus, if at all, it is only the estates in the
amount of P3,655,595.78 declared pursuant to P.D. No. 67 that is covered, upon
payment of 10% of the said amount within the period prescribed under P.D. No. 23,
which was up to June 30, 1973. Considering that there has been partial compliance
with the said requirement by the payment of P285,046.68, petitioner may claim the
benefit of amnesty for said declared amount upon payment of the balance of 10%
thereof required to be paid.
WHEREFORE, with the above modification of the questioned order of July 10, 1974,
said order is hereby affirmed in all other respect. No pronouncement as to costs.
SO ORDERED.

[G.R. No. 120880. June 5, 1997]


FERDINAND R. MARCOS II, petitioner, vs. COURT OF APPEALS,
COMMISSIONER OF THE BUREAU OF INTERNAL REVENUE
HERMINIA D. DE GUZMAN, respondents.

THE
and

DECISION
TORRES, JR., J.:
In this Petition for Review on Certiorari, Government action is once again assailed
as precipitate and unfair, suffering the basic and oftly implored requisites of due
process of law. Specifically, the petition assails the Decision [1] of the Court of Appeals
dated November 29, 1994 in CA-G.R. SP No. 31363, where the said court held:
"In view of all the foregoing, we rule that the deficiency income tax assessments and
estate tax assessment, are already final and (u)nappealable -and- the subsequent
levy of real properties is a tax remedy resorted to by the government, sanctioned by
Section 213 and 218 of the National Internal Revenue Code. This summary tax
remedy is distinct and separate from the other tax remedies (such as Judicial Civil
actions and Criminal actions), and is not affected or precluded by the pendency of
any other tax remedies instituted by the government.
WHEREFORE, premises considered, judgment is hereby rendered DISMISSING the
petition for certiorari with prayer for Restraining Order and Injunction.
No pronouncements as to costs.
SO ORDERED."
More than seven years since the demise of the late Ferdinand E. Marcos, the
former President of the Republic of the Philippines, the matter of the settlement
of his estate, and its dues to the government in estate taxes, are still unresolved,
the latter issue being now before this Court for resolution. Specifically, petitioner
Ferdinand R. Marcos II, the eldest son of the decedent, questions the actuations of
the respondent Commissioner of Internal Revenue in assessing, and collecting
through the summary remedy of Levy on Real Properties, estate and income tax
delinquencies upon the estate and properties of his father, despite the pendency of
the proceedings on probate of the will of the late president, which is docketed as Sp.
Proc. No. 10279 in the Regional Trial Court of Pasig, Branch 156.
Petitioner had filed with the respondent Court of Appeals a Petition
for Certiorari and Prohibition with an application for writ of preliminary injunction
and/or temporary restraining order on June 28, 1993, seeking to -

I. Annul and set aside the Notices of Levy on real property dated February 22, 1993
and May 20, 1993, issued by respondent Commissioner of Internal Revenue;
II. Annul and set aside the Notices of Sale dated May 26, 1993;
III. Enjoin the Head Revenue Executive Assistant Director II (Collection Service), from
proceeding with the Auction of the real properties covered by Notices of Sale.
After the parties had pleaded their case, the Court of Appeals rendered its
Decision[2] on November 29, 1994, ruling that the deficiency assessments for estate
and income tax made upon the petitioner and the estate of the deceased President
Marcos have already become final and unappealable, and may thus be enforced by
the summary remedy of levying upon the properties of the late President, as was
done by the respondent Commissioner of Internal Revenue.
"WHEREFORE, premises considered judgment is hereby rendered DISMISSING the
petition for Certiorari with prayer for Restraining Order and Injunction.
No pronouncements as to cost.
SO ORDERED."
Unperturbed, petitioner is now before us assailing the validity of the appellate
court's decision, assigning the following as errors:
A. RESPONDENT COURT MANIFESTLY ERRED IN RULING THAT THE SUMMARY TAX
REMEDIES RESORTED TO BY THE GOVERNMENT ARE NOT AFFECTED AND
PRECLUDED BY THE PENDENCY OF THE SPECIAL PROCEEDING FOR THE ALLOWANCE
OF THE LATE PRESIDENT'S ALLEGED WILL. TO THE CONTRARY, THIS PROBATE
PROCEEDING PRECISELY PLACED ALL PROPERTIES WHICH FORM PART OF THE LATE
PRESIDENT'S ESTATE IN CUSTODIA LEGIS OF THE PROBATE COURT TO THE
EXCLUSION OF ALL OTHER COURTS AND ADMINISTRATIVE AGENCIES.
B. RESPONDENT COURT ARBITRARILY ERRED IN SWEEPINGLY DECIDING THAT
SINCE THE TAX ASSESSMENTS OF PETITIONER AND HIS PARENTS HAD ALREADY
BECOME FINAL AND UNAPPEALABLE, THERE WAS NO NEED TO GO INTO THE MERITS
OF THE GROUNDS CITED IN THE PETITION. INDEPENDENT OF WHETHER THE TAX
ASSESSMENTS HAD ALREADY BECOME FINAL, HOWEVER, PETITIONER HAS THE
RIGHT TO QUESTION THE UNLAWFUL MANNER AND METHOD IN WHICH TAX
COLLECTION IS SOUGHT TO BE ENFORCED BY RESPONDENTS COMMISSIONER AND
DE GUZMAN. THUS, RESPONDENT COURT SHOULD HAVE FAVORABLY CONSIDERED
THE MERITS OF THE FOLLOWING GROUNDS IN THE PETITION:
(1) The Notices of Levy on Real Property were issued beyond the period provided
in the Revenue Memorandum Circular No. 38-68.

(2) [a] The numerous pending court cases questioning the late President's
ownership or interests in several properties (both personal and real) make the
total value of his estate, and the consequent estate tax due, incapable of exact
pecuniary determination at this time. Thus, respondents assessment of the
estate tax and their issuance of the Notices of Levy and Sale are premature,
confiscatory and oppressive.
[b] Petitioner, as one of the late President's compulsory heirs, was never notified,
much less served with copies of the Notices of Levy, contrary to the mandate of
Section 213 of the NIRC. As such, petitioner was never given an opportunity to
contest the Notices in violation of his right to due process of law.
C. ON ACCOUNT OF THE CLEAR MERIT OF THE PETITION, RESPONDENT COURT
MANIFESTLY ERRED IN RULING THAT IT HAD NO POWER TO GRANT INJUNCTIVE
RELIEF TO PETITIONER. SECTION 219 OF THE NIRC NOTWITHSTANDING, COURTS
POSSESS THE POWER TO ISSUE A WRIT OF PRELIMINARY INJUNCTION TO RESTRAIN
RESPONDENTS COMMISSIONER'S AND DE GUZMAN'S ARBITRARY METHOD OF
COLLECTING THE ALLEGED DEFICIENCY ESTATE AND INCOME TAXES BY MEANS OF
LEVY.
The facts as found by the appellate court are undisputed, and are hereby
adopted:
"On September 29, 1989, former President Ferdinand Marcos died in Honolulu,
Hawaii, USA.
On June 27, 1990, a Special Tax Audit Team was created to conduct investigations
and examinations of the tax liabilities and obligations of the late president, as well as
that of his family, associates and "cronies". Said audit team concluded its
investigation with a Memorandum dated July 26, 1991. The investigation disclosed
that the Marcoses failed to file a written notice of the death of the decedent, an
estate tax returns [sic], as well as several income tax returns covering the years
1982 to 1986, -all in violation of the National Internal Revenue Code (NIRC).
Subsequently, criminal charges were filed against Mrs. Imelda R. Marcos before the
Regional Trial of Quezon City for violations of Sections 82, 83 and 84 (has penalized
under Sections 253 and 254 in relation to Section 252- a & b) of the National Internal
Revenue Code (NIRC).
The Commissioner of Internal Revenue thereby caused the preparation and filing of
the Estate Tax Return for the estate of the late president, the Income Tax Returns of
the Spouses Marcos for the years 1985 to 1986, and the Income Tax Returns of
petitioner Ferdinand 'Bongbong' Marcos II for the years 1982 to 1985.
On July 26, 1991, the BIR issued the following: (1) Deficiency estate tax assessment
no. FAC-2-89-91-002464 (against the estate of the late president Ferdinand Marcos in

the amount of P23,293,607,638.00 Pesos); (2) Deficiency income tax assessment no.
FAC-1-85-91-002452 and Deficiency income tax assessment no. FAC-1-86-91-002451
(against the Spouses Ferdinand and Imelda Marcos in the amounts of P149,551.70
and P184,009,737.40 representing deficiency income tax for the years 1985 and
1986); (3) Deficiency income tax assessment nos. FAC-1-82-91-002460 to FAC-1-8591-002463 (against petitioner Ferdinand 'Bongbong' Marcos II in the amounts of
P258.70 pesos; P9,386.40 Pesos; P4,388.30 Pesos; and P6,376.60 Pesos representing
his deficiency income taxes for the years 1982 to 1985).
The Commissioner of Internal Revenue avers that copies of the deficiency estate and
income tax assessments were all personally and constructively served on August 26,
1991 and September 12, 1991 upon Mrs. Imelda Marcos (through her caretaker Mr.
Martinez) at her last known address at No. 204 Ortega St., San Juan, M.M. (Annexes
'D' and 'E' of the Petition). Likewise, copies of the deficiency tax assessments issued
against petitioner Ferdinand 'Bongbong' Marcos II were also personally and
constructively served upon him (through his caretaker) on September 12, 1991, at
his last known address at Don Mariano Marcos St. corner P. Guevarra St., San Juan,
M.M. (Annexes 'J' and 'J-1' of the Petition). Thereafter, Formal Assessment notices
were served on October 20, 1992, upon Mrs. Marcos c/o petitioner, at his office,
House of Representatives, Batasan Pambansa, Quezon City. Moreover, a notice to
Taxpayer inviting Mrs. Marcos (or her duly authorized representative or counsel), to a
conference, was furnished the counsel of Mrs. Marcos, Dean Antonio Coronel - but to
no avail.
The deficiency tax assessments were not protested administratively, by Mrs. Marcos
and the other heirs of the late president, within 30 days from service of said
assessments.
On February 22, 1993, the BIR Commissioner issued twenty-two notices of levy on
real property against certain parcels of land owned by the Marcoses - to satisfy the
alleged estate tax and deficiency income taxes of Spouses Marcos.
On May 20, 1993, four more Notices of Levy on real property were issued for the
purpose of satisfying the deficiency income taxes.
On May 26, 1993, additional four (4) notices of Levy on real property were again
issued. The foregoing tax remedies were resorted to pursuant to Sections 205 and
213 of the National Internal Revenue Code (NIRC).
In response to a letter dated March 12, 1993 sent by Atty. Loreto Ata (counsel of
herein petitioner) calling the attention of the BIR and requesting that they be duly
notified of any action taken by the BIR affecting the interest of their client Ferdinand
'Bongbong Marcos II, as well as the interest of the late president - copies of the
aforesaid notices were served on April 7, 1993 and on June 10, 1993, upon Mrs.
Imelda Marcos, the petitioner, and their counsel of record, 'De Borja, Medialdea, Ata,
Bello, Guevarra and Serapio Law Office'.

Notices of sale at public auction were posted on May 26, 1993, at the lobby of the
City Hall of Tacloban City. The public auction for the sale of the eleven (11) parcels
of land took place on July 5, 1993. There being no bidder, the lots were declared
forfeited in favor of the government.
On June 25, 1993, petitioner Ferdinand 'Bongbong' Marcos II filed the instant petition
for certiorari and prohibition under Rule 65 of the Rules of Court, with prayer for
temporary restraining order and/or writ of preliminary injunction."
It has been repeatedly observed, and not without merit, that the enforcement of
tax laws and the collection of taxes, is of paramount importance for the sustenance
of government. Taxes are the lifeblood of the government and should be collected
without unnecessary hindrance. However, such collection should be made in
accordance with law as any arbitrariness will negate the very reason for government
itself. It is therefore necessary to reconcile the apparently conflicting interests of the
authorities and the taxpayers so that the real purpose of taxation, which is the
promotion of the common good, may be achieved." [3]
Whether or not the proper avenues of assessment and collection of the said tax
obligations were taken by the respondent Bureau is now the subject of the Court's
inquiry.
Petitioner posits that notices of levy, notices of sale, and subsequent sale of
properties of the late President Marcos effected by the BIR are null and void for
disregarding the established procedure for the enforcement of taxes due upon the
estate of the deceased. The case of Domingo vs. Garlitos[4] is specifically cited to
bolster the argument that "the ordinary procedure by which to settle claims of
indebtedness against the estate of a deceased, person, as in an inheritance (estate)
tax, is for the claimant to present a claim before the probate court so that said court
may order the administrator to pay the amount therefor." This remedy is allegedly,
exclusive, and cannot be effected through any other means.
Petitioner goes further, submitting that the probate court is not precluded from
denying a request by the government for the immediate payment of taxes, and
should order the payment of the same only within the period fixed by the probate
court for the payment of all the debts of the decedent. In this regard, petitioner cites
the case of Collector of Internal Revenue vs. The Administratrix of the Estate of
Echarri (67 Phil 502), where it was held that:
"The case of Pineda vs. Court of First Instance of Tayabas and Collector of Internal
Revenue (52 Phil 803), relied upon by the petitioner-appellant is good authority on
the proposition that the court having control over the administration proceedings has
jurisdiction to entertain the claim presented by the government for taxes due and to
order the administrator to pay the tax should it find that the assessment was proper,
and that the tax was legal, due and collectible. And the rule laid down in that case
must be understood in relation to the case of Collector of Customs vs. Haygood,

supra., as to the procedure to be followed in a given case by the government to


effectuate the collection of the tax. Categorically stated, where during the pendency
of judicial administration over the estate of a deceased person a claim for taxes is
presented by the government, the court has the authority to order payment by the
administrator; but, in the same way that it has authority to order payment or
satisfaction, it also has the negative authority to deny the same. While there are
cases where courts are required to perform certain duties mandatory and ministerial
in character, the function of the court in a case of the present character is not one of
them; and here, the court cannot be an organism endowed with latitude of
judgment in one direction, and converted into a mere mechanical contrivance in
another direction."
On the other hand, it is argued by the BIR, that the state's authority to collect
internal revenue taxes is paramount. Thus, the pendency of probate proceedings
over the estate of the deceased does not preclude the assessment and collection,
through summary remedies, of estate taxes over the same. According to the
respondent, claims for payment of estate and income taxes due and assessed after
the death of the decedent need not be presented in the form of a claim against the
estate. These can and should be paid immediately. The probate court is not the
government agency to decide whether an estate is liable for payment of estate of
income taxes. Well-settled is the rule that the probate court is a court with special
and limited jurisdiction.
Concededly, the authority of the Regional Trial Court, sitting, albeit with limited
jurisdiction, as a probate court over estate of deceased individual, is not a trifling
thing. The court's jurisdiction, once invoked, and made effective, cannot be treated
with indifference nor should it be ignored with impunity by the very parties invoking
its authority.
In testament to this, it has been held that it is within the jurisdiction of the
probate court to approve the sale of properties of a deceased person by his
prospective heirs before final adjudication; [5] to determine who are the heirs of the
decedent;[6] the recognition of a natural child; [7] the status of a woman claiming to be
the legal wife of the decedent; [8] the legality of disinheritance of an heir by the
testator;[9] and to pass upon the validity of a waiver of hereditary rights. [10]
The pivotal question the court is tasked to resolve refers to the authority of the
Bureau of Internal Revenue to collect by the summary remedy of levying upon, and
sale of real properties of the decedent, estate tax deficiencies, without the cognition
and authority of the court sitting in probate over the supposed will of the deceased.
The nature of the process of estate tax collection has been described as follows:
"Strictly speaking, the assessment of an inheritance tax does not directly involve the
administration of a decedent's estate, although it may be viewed as an incident to
the complete settlement of an estate, and, under some statutes, it is made the duty

of the probate court to make the amount of the inheritance tax a part of the final
decree of distribution of the estate. It is not against the property of decedent, nor is
it a claim against the estate as such, but it is against the interest or property right
which the heir, legatee, devisee, etc., has in the property formerly held by
decedent. Further, under some statutes, it has been held that it is not a suit or
controversy between the parties, nor is it an adversary proceeding between the
state and the person who owes the tax on the inheritance. However, under other
statutes it has been held that the hearing and determination of the cash value of the
assets and the determination of the tax are adversary proceedings. The proceeding
has been held to be necessarily a proceeding in rem. [11]
In the Philippine experience, the enforcement and collection of estate tax, is
executive in character, as the legislature has seen it fit to ascribe this task to the
Bureau of Internal Revenue. Section 3 of the National Internal Revenue Code attests
to this:
"Sec. 3. Powers and duties of the Bureau.-The powers and duties of the Bureau of
Internal Revenue shall comprehend the assessment and collection of all national
internal revenue taxes, fees, and charges, and the enforcement of all forfeitures,
penalties, and fines connected therewith, including the execution of judgments in all
cases decided in its favor by the Court of Tax Appeals and the ordinary courts. Said
Bureau shall also give effect to and administer the supervisory and police
power conferred to it by this Code or other laws."
Thus, it was in Vera vs. Fernandez [12] that the court recognized the liberal
treatment of claims for taxes charged against the estate of the decedent. Such
taxes, we said, were exempted from the application of the statute of non-claims, and
this is justified by the necessity of government funding, immortalized in the maxim
that taxes are the lifeblood of the government. Vectigalia nervi sunt rei publicae taxes are the sinews of the state.
"Taxes assessed against the estate of a deceased person, after administration is
opened, need not be submitted to the committee on claims in the ordinary course of
administration. In the exercise of its control over the administrator, the court may
direct the payment of such taxes upon motion showing that the taxes have been
assessed against the estate."
Such liberal treatment of internal revenue taxes in the probate proceedings
extends so far, even to allowing the enforcement of tax obligations against the heirs
of the decedent, even after distribution of the estate's properties.
"Claims for taxes, whether assessed before or after the death of the deceased, can
be collected from the heirs even after the distribution of the properties of the
decedent. They are exempted from the application of the statute of non-claims. The
heirs shall be liable therefor, in proportion to their share in the inheritance." [13]

"Thus, the Government has two ways of collecting the taxes in question. One, by
going after all the heirs and collecting from each one of them the amount of the tax
proportionate to the inheritance received. Another remedy, pursuant to the lien
created by Section 315 of the Tax Code upon all property and rights to property
belong to the taxpayer for unpaid income tax, is by subjecting said property of the
estate which is in the hands of an heir or transferee to the payment of the tax due
the estate. (Commissioner of Internal Revenue vs. Pineda, 21 SCRA 105, September
15, 1967.)
From the foregoing, it is discernible that the approval of the court, sitting in
probate, or as a settlement tribunal over the deceased is not a mandatory
requirement in the collection of estate taxes. It cannot therefore be argued that the
Tax Bureau erred in proceeding with the levying and sale of the properties allegedly
owned by the late President, on the ground that it was required to seek first the
probate court's sanction. There is nothing in the Tax Code, and in the pertinent
remedial laws that implies the necessity of the probate or estate settlement court's
approval of the state's claim for estate taxes, before the same can be enforced and
collected.
On the contrary, under Section 87 of the NIRC, it is the probate or settlement
court which is bidden not to authorize the executor or judicial administrator of the
decedent's estate to deliver any distributive share to any party interested in the
estate, unless it is shown a Certification by the Commissioner of Internal Revenue
that the estate taxes have been paid. This provision disproves the petitioner's
contention that it is the probate court which approves the assessment and collection
of the estate tax.
If there is any issue as to the validity of the BIR's decision to assess the estate
taxes, this should have been pursued through the proper administrative and judicial
avenues provided for by law.
Section 229 of the NIRC tells us how:
"Sec. 229. Protesting of assessment.-When the Commissioner of Internal Revenue or
his duly authorized representative finds that proper taxes should be assessed, he
shall first notify the taxpayer of his findings. Within a period to be prescribed by
implementing regulations, the taxpayer shall be required to respond to said
notice. If the taxpayer fails to respond, the Commissioner shall issue an assessment
based on his findings.
Such assessment may be protested administratively by filing a request for
reconsideration or reinvestigation in such form and manner as may be prescribed by
implementing regulations within (30) days from receipt of the assessment;
otherwise, the assessment shall become final and unappealable.

If the protest is denied in whole or in part, the individual, association or corporation


adversely affected by the decision on the protest may appeal to the Court of Tax
Appeals within thirty (30) days from receipt of said decision; otherwise, the decision
shall become final, executory and demandable. (As inserted by P.D. 1773)"
Apart from failing to file the required estate tax return within the time required
for the filing of the same, petitioner, and the other heirs never questioned the
assessments served upon them, allowing the same to lapse into finality, and
prompting the BIR to collect the said taxes by levying upon the properties left by
President Marcos.
Petitioner submits, however, that "while the assessment of taxes may have been
validly undertaken by the Government, collection thereof may have been done in
violation of the law. Thus, the manner and method in which the latter is enforced
may be questioned separately, and irrespective of the finality of the former, because
the Government does not have the unbridled discretion to enforce collection without
regard to the clear provision of law." [14]
Petitioner specifically points out that applying Memorandum Circular No. 38-68,
implementing Sections 318 and 324 of the old tax code (Republic Act 5203), the
BIR's Notices of Levy on the Marcos properties, were issued beyond the allowed
period, and are therefore null and void:
"...the Notices of Levy on Real Property (Annexes 0 to NN of Annex C of this Petition)
in satisfaction of said assessments were still issued by respondents well beyond the
period mandated in Revenue Memorandum Circular No. 38-68. These Notices of
Levy were issued only on 22 February 1993 and 20 May 1993 when at least
seventeen (17) months had already lapsed from the last service of tax assessment
on 12 September 1991. As no notices of distraint of personal property were first
issued by respondents, the latter should have complied with Revenue Memorandum
Circular No. 38-68 and issued these Notices of Levy not earlier than three (3) months
nor later than six (6) months from 12 September 1991. In accordance with the
Circular, respondents only had until 12 March 1992 (the last day of the sixth month)
within which to issue these Notices of Levy. The Notices of Levy, having been issued
beyond the period allowed by law, are thus void and of no effect." [15]
We hold otherwise. The Notices of Levy upon real property were issued within
the prescriptive period and in accordance with the provisions of the present Tax
Code. The deficiency tax assessment, having already become final, executory, and
demandable, the same can now be collected through the summary remedy of
distraint or levy pursuant to Section 205 of the NIRC.
The applicable provision in regard to the prescriptive period for the assessment
and collection of tax deficiency in this instance is Article 223 of the NIRC, which
pertinently provides:

"Sec. 223. Exceptions as to a period of limitation of assessment and collection of


taxes.- (a) In the case of a false or fraudulent return with intent to evade tax or of a
failure to file a return, the tax may be assessed, or a proceeding in court for the
collection of such tax may be begun without assessment, at any time within ten (10)
years after the discovery of the falsity, fraud, or omission: Provided, That, in a fraud
assessment which has become final and executory, the fact of fraud shall be
judicially taken cognizance of in the civil or criminal action for the collection thereof.
xxx
(c) Any internal revenue tax which has been assessed within the period of limitation
above prescribed, may be collected by distraint or levy or by a proceeding in court
within three years following the assessment of the tax.
xxx
The omission to file an estate tax return, and the subsequent failure to contest or
appeal the assessment made by the BIR is fatal to the petitioner's cause, as under
the above-cited provision, in case of failure to file a return, the tax may be assessed
at any time within ten years after the omission, and any tax so assessed may be
collected by levy upon real property within three years following the assessment of
the tax. Since the estate tax assessment had become final and unappealable by the
petitioner's default as regards protesting the validity of the said assessment, there is
now no reason why the BIR cannot continue with the collection of the said tax. Any
objection against the assessment should have been pursued following the avenue
paved in Section 229 of the NIRC on protests on assessments of internal revenue
taxes.
Petitioner further argues that "the numerous pending court cases questioning the
late president's ownership or interests in several properties (both real and personal)
make the total value of his estate, and the consequent estate tax due, incapable of
exact pecuniary determination at this time. Thus, respondents' assessment of the
estate tax and their issuance of the Notices of Levy and sale are premature and
oppressive." He points out the pendency of Sandiganbayan Civil Case Nos. 00010034 and 0141, which were filed by the government to question the ownership and
interests of the late President in real and personal properties located within and
outside the Philippines. Petitioner, however, omits to allege whether the properties
levied upon by the BIR in the collection of estate taxes upon the decedent's estate
were among those involved in the said cases pending in the Sandiganbayan. Indeed,
the court is at a loss as to how these cases are relevant to the matter at issue. The
mere fact that the decedent has pending cases involving ill-gotten wealth does not
affect the enforcement of tax assessments over the properties indubitably included
in his estate.
Petitioner also expresses his reservation as to the propriety of the BIR's total
assessment of P23,292,607,638.00, stating that this amount deviates from the

findings of the Department of Justice's Panel of Prosecutors as per its resolution of 20


September 1991. Allegedly, this is clear evidence of the uncertainty on the part of
the Government as to the total value of the estate of the late President.
This is, to our mind, the petitioner's last ditch effort to assail the assessment of
estate tax which had already become final and unappealable.
It is not the Department of Justice which is the government agency tasked to
determine the amount of taxes due upon the subject estate, but the Bureau of
Internal Revenue[16] whose determinations and assessments are presumed correct
and made in good faith.[17] The taxpayer has the duty of proving otherwise. In the
absence of proof of any irregularities in the performance of official duties, an
assessment will not be disturbed. Even an assessment based on estimates is prima
facie valid and lawful where it does not appear to have been arrived at arbitrarily or
capriciously. The burden of proof is upon the complaining party to show clearly that
the assessment is erroneous. Failure to present proof of error in the assessment will
justify the judicial affirmance of said assessment. [18] In this instance, petitioner has
not pointed out one single provision in the Memorandum of the Special Audit Team
which gave rise to the questioned assessment, which bears a trace of falsity. Indeed,
the petitioner's attack on the assessment bears mainly on the alleged improbable
and unconscionable amount of the taxes charged. But mere rhetoric cannot supply
the basis for the charge of impropriety of the assessments made.
Moreover, these objections to the assessments should have been raised,
considering the ample remedies afforded the taxpayer by the Tax Code, with the
Bureau of Internal Revenue and the Court of Tax Appeals, as described earlier, and
cannot be raised now via Petition for Certiorari, under the pretext of grave abuse of
discretion. The course of action taken by the petitioner reflects his disregard or even
repugnance of the established institutions for governance in the scheme of a wellordered society. The subject tax assessments having become final, executory and
enforceable, the same can no longer be contested by means of a disguised
protest. In the main, Certiorari may not be used as a substitute for a lost appeal or
remedy.[19] This judicial policy becomes more pronounced in view of the absence of
sufficient attack against the actuations of government.
On the matter of sufficiency of service of Notices of Assessment to the petitioner,
we find the respondent appellate court's pronouncements sound and resilient to
petitioner's attacks.
"Anent grounds 3(b) and (B) - both alleging/claiming lack of notice - We find, after
considering the facts and circumstances, as well as evidences, that there was
sufficient, constructive and/or actual notice of assessments, levy and sale, sent to
herein petitioner Ferdinand "Bongbong" Marcos as well as to his mother Mrs. Imelda
Marcos.

Even if we are to rule out the notices of assessments personally given to the
caretaker of Mrs. Marcos at the latter's last known address, on August 26, 1991 and
September 12, 1991, as well as the notices of assessment personally given to the
caretaker of petitioner also at his last known address on September 12, 1991 - the
subsequent notices given thereafter could no longer be ignored as they were sent at
a time when petitioner was already here in the Philippines, and at a place where said
notices would surely be called to petitioner's attention, and received by responsible
persons of sufficient age and discretion.
Thus, on October 20, 1992, formal assessment notices were served upon Mrs.
Marcos c/o the petitioner, at his office, House of Representatives, Batasan Pambansa,
Q.C. (Annexes "A", "A-1", "A-2", "A-3"; pp. 207-210, Comment/Memorandum of
OSG). Moreover, a notice to taxpayer dated October 8, 1992 inviting Mrs. Marcos to
a conference relative to her tax liabilities, was furnished the counsel of Mrs. Marcos Dean Antonio Coronel (Annex "B", p. 211, ibid). Thereafter, copies of Notices were
also served upon Mrs. Imelda Marcos, the petitioner and their counsel "De Borja,
Medialdea, Ata, Bello, Guevarra and Serapio Law Office", on April 7, 1993 and June
10, 1993. Despite all of these Notices, petitioner never lifted a finger to protest the
assessments, (upon which the Levy and sale of properties were based), nor appealed
the same to the Court of Tax Appeals.
There being sufficient service of Notices to herein petitioner (and his mother) and it
appearing that petitioner continuously ignored said Notices despite several
opportunities given him to file a protest and to thereafter appeal to the Court of Tax
Appeals, - the tax assessments subject of this case, upon which the levy and sale of
properties were based, could no longer be contested (directly or indirectly) via this
instant petition for certiorari."[20]
Petitioner argues that all the questioned Notices of Levy, however, must be
nullified for having been issued without validly serving copies thereof to the
petitioner. As a mandatory heir of the decedent, petitioner avers that he has an
interest in the subject estate, and notices of levy upon its properties should have
been served upon him.
We do not agree. In the case of notices of levy issued to satisfy the delinquent
estate tax, the delinquent taxpayer is the Estate of the decedent, and not
necessarily, and exclusively, the petitioner as heir of the deceased. In the same
vein, in the matter of income tax delinquency of the late president and his spouse,
petitioner is not the taxpayer liable. Thus, it follows that service of notices of levy in
satisfaction of these tax delinquencies upon the petitioner is not required by law, as
under Section 213 of the NIRC, which pertinently states:
"xxx
...Levy shall be effected by writing upon said certificate a description of the property
upon which levy is made. At the same time, written notice of the levy shall be

mailed to or served upon the Register of Deeds of the province or city where the
property is located and upon the delinquent taxpayer, or if he be absent from the
Philippines, to his agent or the manager of the business in respect to which the
liability arose, or if there be none, to the occupant of the property in question.
xxx"
The foregoing notwithstanding, the record shows that notices of warrants of
distraint and levy of sale were furnished the counsel of petitioner on April 7, 1993,
and June 10, 1993, and the petitioner himself on April 12, 1993 at his office at the
Batasang Pambansa.[21] We cannot therefore, countenance petitioner's insistence
that he was denied due process. Where there was an opportunity to raise objections
to government action, and such opportunity was disregarded, for no justifiable
reason, the party claiming oppression then becomes the oppressor of the orderly
functions of government. He who comes to court must come with clean
hands. Otherwise, he not only taints his name, but ridicules the very structure of
established authority.
IN VIEW WHEREOF, the Court RESOLVED to DENY the present petition. The
Decision of the Court of Appeals dated November 29, 1994 is hereby AFFIRMED in all
respects.
SO ORDERED.
GR. No. L-37061 September 5, 1984
MAMBULAO LUMBER COMPANY, petitioner,
vs.
REPUBLIC OF THE PHILIPPINES, respondent.

CUEVAS, J.:
Petitioner in this appeal by certiorari, seeks the reversal of the decision of the
defunct Court of Appeals which affirmed the judgment of the then Court of First
Instance of Manila ordering petitioner to pay respondent the amount of P15,739.80
representing its tax liability not secured by any bond, with legal interest thereon
from August 25, 1961 until fully paid.
Sometime in 1957 Agent Nestor Banzuela of the Bureau of Internal Revenue,
Regional District No. 6, Bicol Region, Naga City, conducted an examination of the
books of accounts of herein petitioner Mambulao number Company for the purpose
of determining said taxpayer's forest charges and percentage tax liabilities.

On July 31, 1957, Agent Banzuela submitted his report wherein it was stated among
others that
xxx xxx xxx
xxx xxx xxx
xxx xxx xxx
It can be stated in this connection that sometime in the early part of
1949, the personnel of the local office of the Bureau of Forestry in Daet,
Camarines Norte, manifested under the name of the subject taxpayer
2,052.48 cubic meters of timber, with the corresponding forest charges
in the total amount of P15,443.65 including surcharges. The Bureau of
Forestry then demanded for the payment of said forest charges on
January 15, 1949. However, the subject taxpayer, for one reason or the
other, contested this assessment until this case reached the hands of
the Secretary of Agriculture and Natural Resources, the undersigned
cannot therefore include in his assessment this amount in question,
hence, due course is given, recommending that this bureau take proper
action regarding this case.
Consequently, on August 29, 1958, the Acting Commissioner of Internal Revenue
addressed a letter to petitioner, the pertinent portion of which readsMambulao Lumber Company
R-406 Samanillo Building
Escolta, Manila
Gentlemen:
xxx xxx xxx
It was also ascertained that in 1949 you manifested 2,052.48 cubic
meters of timber, the forest charges and surcharges of which in the
total amount of P15,443.55 was demanded of you by the Bureau of
Forestry on January 15, 1949. ...
In view thereof there is due from you the amount of P33,595.26 as
deficiency sales tax, forest charges and surcharges, committed as
follows:
Sales Tax x x x
Forest Charges

Forest charges and surcharges for the year 1949 appealed to the
Secretary of Agriculture and Natural Resources P15,443.55
xxx xxx xxx
Total amount due & payable P33,595.26
Demand is hereby made upon you to pay the aforesaid amount of P
33,595.26 to the City Treasurer of Manila or this office within ten (10)
days from receipt hereof so that this case may be closed.
xxx xxx xxx
Sgd.M
elencio
Domin
go
Acting
Commi
ssioner
of
Interna
l
Reven
ue
The aforesaid letter was acknowledged to have been received by petitioner on
September 19, 1958. 3 On October 18, 1958, petitioner requested for a
reinvestigation of its tax liability. Subsequently, in a letter dated July 8, 1959,
respondent Commissioner of Internal Revenue give petitioner a period of twenty (20)
days from receipt thereof to submit the results of its verification of payments with a
warning that failure to comply therewith would be construed as an abandonment of
the request for reinvestigation.
For failure of petitioner to comply with the above letter-request and/or to pay its tax
liability despite demands for the payment thereof, respondent Commissioner of
Internal Revenue filed. a complaint for collection in the Court of First Instance of
Manila on August 25, 1961. 4
After trial, judgment was rendered by the trial court, the dispositive portion of which
reads
WHEREFORE, judgment is rendered
(a) Ordering both defendants, jointly and severally, to pay plaintiff the
amount of P1,219.95 plus legal interest thereon from August 25, 1961,

the date of the filing of the original complaint until fully paid, or in case
of failure to Pay the said amount, ordering the forfeiture of GISCOR
Bond No. 35 to the amount of P1,219.95; and
(b) Ordering defendant Mambulao Lumber Company to pay the plaintiff
the amount of P15,739.80 representing its tax liability not secured by
any bond, with legal interest thereon from August 25, 1961, until paid.
With costs against defendants.
From the aforesaid decision, petitioner appealed to the Court of Appeals 5 that
portion of the trial court's decision ordering it to pay the amount of P15,443.55
representing forest charges and surcharges due for the year 1949.
As herein earlier stated, the then Court of Appeals affirmed the decision of the trial
court. Petitioner filed a motion for reconsideration which was denied by the said
court in its Resolution dated June 7, 1973. Hence, the instant appeal, petitioner
presenting the lone issue of whether or not the right of plaintiff (respondent herein)
to file a judicial action for the collection of the amount of P15,443.55 as forest
charges and surcharges due from the petitioner Mambulao Lumber Company for the
year 1949 has already prescribed.
Relying on the provisions of Section 332 of the National Internal Revenue Code which
readsSection 332. Exemptions as to period of limitation of assessment and
collection of taxes
xxx xxx xxx
(c) Where the assessment of any internal revenue tax has been made
within the period of limitation above prescribed such tax may be
collected by distraint or levy or by a proceeding in court, but only if
begun (1) within five years after the assessment of the tax, or (2) prior
to the expiration of any period for collection agreed upon in writing by
the Collector of Internal Revenue and the taxpayer before the expiration
of such five-year period. The period so agreed upon may be extended
by subsequent agreements in writing made before the expiration of the
period previously agreed upon.
petitioner argues that counting from January 15, 1949 when the Bureau of Forestry in
Daet, Camarines Norte made an assessment and demand for payment of the
amount of P15,443.55 as forest charges and surcharges for the year 1949, up to the
filing of the complaint for collection before the lower court on August 25, 196 1,
more than five (5) years had already elapsed, hence, the action had clearly
prescribed.

Petitioner's aforesaid argument lacks merit. As correctly observed by the trial court
and the Court of Appeals in the appealed decision, the letter of demand of the Acting
Commissioner of Internal Revenue dated August 29, 1958 was the basis of
respondent's complaint filed in this case and not the demand letter of the Bureau of
Forestry dated January 15, 1949. This must be so because forest charges are internal
revenue taxes 6 and the sole power and duty to collect the same is lodged with the
Bureau of Internal Revenue 7 and not with the Bureau of Forestry. The computation
and/or assessment of forest charges made by the Bureau of Forestry may or may not
be adopted by the Commissioner of Internal Revenue and such computation made
by the Bureau of Forestry is not appealable to the Court of Tax Appeals. 8Therefore,
for the purpose of computing the five-year period within which to file a complaint for
collection, the demand or even the assessment made by the Bureau of Forestry is
immaterial.
In the case at bar, the commencement of the five-year period should be counted
from August 29, 1958, the date of the letter of demand of the Acting Commissioner
of Internal Revenue 9 to petitioner Mambulao Lumber Company. It is this demand or
assessment that is appealable to the Court of Tax Appeals. The complaint for
collection was filed in the Court of First Instance of Manila on August 25, 1961, very
much within the five-year period prescribed by Section 332 (c) of the Tax Code.
Consequently, the right of the Commissioner of Internal Revenue to collect the forest
charges and surcharges in the amount of P15,443.55 has not prescribed.
Furthermore, it is not disputed that on October 18, 1958, petitioner requested for a
reinvestigation of its tax liability. In reply thereto, respondent in a letter dated July 8,
1959, gave petitioner a period of twenty (20) days from receipt thereof to submit the
results of its verification of payments and failure to comply therewith would be
construed as abandonment of the request for reinvestigation. Petitioner failed to
comply with this requirement. Neither did it appeal to the Court of Tax Appeals within
thirty (30) days from receipt of the letter dated July 8, 1959, as prescribed under
Section 11 of Republic Act No. 1125, thus making the assessment final and
executory.
Taxpayer's failure to appeal to the Court of Tax Appeals in due time
made the assessment in question final, executory and demandable. And
when the action was instituted on September 2, 1958 to enforce the
deficiency assessment in question, it was already barred from disputing
the correctness of the assessment or invoking any defense that would
reopen the question of its tax liability. Otherwise, the period of thirty
days for appeal to the Court of Tax Appeals would make little sense.
In a proceeding like this the taxpayer's defenses are similar to those of
the defendant in a case for the enforcement of a judgment by judicial
action under Section 6 of Rule 39 of the Rules of Court. No inquiry can
be made therein as to the merits of the original case or the justness of
the judgment relied upon, other than by evidence of want of

jurisdiction, of collusion between the parties, or of fraud in the party


offering the record with respect to the proceedings. As held by this
Court in Insular Government vs. Nico the taxpayer may raise only the
questions whether or not the Collector of Internal Revenue had
jurisdiction to do the particular act, and whether any fraud was
committed in the doing of the act. In that case, Doroteo Nico was fined
by the Collector of Internal Revenue for violation of sub-paragraphs (d),
(e) and (g) of Section 28 as well as Sections 36, 101 and 107 of Act
1189. Under Section 54 of the same Act, the taxpayer was given the
right to appeal from the decision of the Collector of Internal Revenue to
the Court of First Instance within a period of ten days from notice of
imposition of the fine. Nico did not appeal, neither did he pay the fine.
Pursuant to Section 33 of the Act, the Collector of Internal Revenue filed
an action in the Court of First Instance to enforce his decision and
collect the fine. The decision of the Collector of Internal Revenue having
become final, this Court, on appeal, allowed no further inquiry into the
merits of the same. 10
In a suit for collection of internal revenue taxes, as in this case, where the
assessment has already become final and executory, the action to collect is akin to
an action to enforce a judgment. No inquiry can be made therein as to the merits of
the original case or the justness of the judgment relied upon. Petitioner is thus
already precluded from raising the defense of prescription.
Where the taxpayer did not contest the deficiency income tax assessed
against him, the same became final and properly collectible by means
of an ordinary court action. The taxpayer cannot dispute an assessment
which is being enforced by judicial action, He should have disputed it
before it was brought to court. 11
WHEREFORE, the decision appealed from is hereby AFFIRMED and the petition
DISMISSED. No costs.
SO ORDERED.
G.R. No. L-21731

March 31, 1966

REPUBLIC OF THE PHILIPPINES, plaintiff-appellant,


vs.
LIM TIAN TENG SONS and CO., INC., defendant-appellant.
Office of the Solicitor General for the plaintiff-appellant.
P. B. Uy Calderon for the defendant-appellee.
BENGZON, J.P., J.:

Lim Tian Teng Sons & Co., Inc., a domestic corporation with principal office in Cebu
City, engaged in 1951 and 1952, among others, in the exportation of copra. The
copra was weighed before shipment in the port of departure and upon arrival in the
port of destination. The weight before shipment was called copra outturn. To allow
for lose in weight due to shrinkage, said exporter collected only 95% of the amount
appearing in the letter of credit covering every copra outturn. The 5% balance
remained outstanding until final liquidation and adjustment.
On March 30, 1953 Lim Tian Teng Sons & Co., Inc. filed its income tax return for 1952
based on accrued income and expenses. Its return showed a loss of P56,109.98. It
took up as part of the beginning inventory for 1952 the copra outturn shipped in
1951 in the sum of P95,500.00 already partially collected, as part of its outstanding
stock as of December 31, 1951.
In the audit and examination of taxpayer's 1952 income tax return, the Collector of
Internal Revenue eliminated the P95,500.00 outturn from the beginning inventory for
1952 and considered it as accrued income for 1951. This increased taxpayer's 1952
net income by P95,500.00 which, considering disallowances in the sum of P9,980.85,
raised the taxpayer's net taxable income for 1952 to P50,370.87. Accordingly, in a
letter dated January 16, 1957 (Exhibit C), received by Lim Tian Teng Sons & Co., Inc.
on January 30, 1957, the Collector of Internal Revenue assessed a deficiency income
tax of P10,074.00 and 50% surcharge thereon amounting to P5,037.00 and
demanded payment thereof not later than February 15, 1957.
On January 31, 1957 Lim Tian Teng Sons & Co., Inc. requested reinvestigation of its
1952 income tax liability. The Collector of Internal Revenue did not reply; instead, he
referred the case to the Solicitor General for collection by judicial action.
On September 20, 1957 the Solicitor General demanded from Lim Tian Teng Sons &
Co., Inc. the payment of P15,111.50 within five days, stating that otherwise judicial
action would be instituted without further notice. In a letter dated October 5, 1957,
received by the Collector of Internal Revenue on October 7, 1957, Lim Tian Teng Sons
& Co., Inc. reiterated its request for reinvestigation. It also wrote the Solicitor General
on October 8, 1957 requesting that it be allowed to present its explanation together
with supporting papers relative to its income tax liability. The Solicitor General
transmitted the letter to the Collection of Internal Revenue. Thereupon, the Deputy
Collector of Internal Revenue, by his letter dated October 16, 1957, informed the
taxpayer that its request for reinvestigation would be granted provided it executed
within ten days a waiver of the statute of limitations as required in General Circular
V-258 dated August 20, 1957. In his letter dated December 10, 1957, the Deputy
Collector of Internal Revenue extended the period within which to execute and file
with him the waiver of the statute of limitations to December 31, 1957, but advised
that if no waiver is forthcoming on or before said date, judicial action for collection
would be instituted without further notice. Receipt of this letter is denied by
appellant company.

As Lim Tian Teng Sons & Co., Inc. failed to file a waiver of the statute of limitations,
the Collector of Internal Revenue instituted eight months after, specifically on
September 2, 1958, an action in the Court of First Instance of Cebu for the collection
of deficiency income tax.
After hearing the parties, the court below rendered the following judgment.
IN VIEW OF THE FOREGOING, judgment is hereby rendered, declaring the
assessment (Exh. D, D-1) of income tax in the sum of P15,111.00 due from the
defendant to the plaintiff for the year 1952 valid, final and executory;
condemning the defendant to pay the same to the plaintiff with interest at one
(1) per centum monthly from October 28, 1957 until fully paid.
With costs against the defendant.
IT IS SO ORDERED.
Not satisfied with the decision, the Collector of Internal Revenue moved for its
reconsideration on the ground that it did not include the 5% surcharge for late
payment of tax. The motion was denied for the reason that the taxpayer has already
been ordered to pay a surcharge of 50%.
Both parties appealed, raising only questions of law.
Plaintiff cites as errors the non-imposition of the 5% surcharge for the late payment
of tax and the computation of delinquency interest from October 8, 1957.
Defendant, on the other hand, assails the jurisdiction of the lower court, its finding
that the assessment in question has become final and executory, the correctness of
the assessment and the imposition of the 50% surcharge.1wph1.t
We will discuss first the taxpayer's appeal. It maintains that the lower court has no
jurisdiction to entertain this case on the ground that the Collector of Internal
Revenue has not yet issued his final decision on its requests for reinvestigation. The
taxpayer's stand is that final decision of the Collector of Internal Revenue on the
disputed assessment is a condition precedent to the filing of an action in the Court of
First Instance for the collection of a tax. This argument has no merit. The Collector of
Internal Revenue is authorized to collect delinquent internal revenue taxes either by
distraint and levy or by judicial action or both simultaneously. 1 The only requisite
before he can collect the tax is that he must first assess the same within the time
fixed by law.2 And in the case of a false or fraudulent return with intent to evade the
tax or of a failure to file a return, a proceeding in court for the collection of such tax
may be begun without assessment.3
Nowhere in the Tax Code is the Collector of Internal Revenue required to rule first on
a taxpayer's request for reinvestigation before he can go to court for the purpose of

collecting the tax assessed. On the contrary, Section 305 of the same Code
withholds from all courts, except the Court of Tax Appeals under Section 11 of
Republic Act 1125, the authority to restrain the collection of any national internalrevenue tax, fee or charge, thereby indicating the legislative policy to allow the
Collector of Internal Revenue much latitude in the speedy and prompt collection of
taxes. The reason is obvious. It is upon taxation that the government chiefly relies to
obtain the means the carry on its operations, and it is of the utmost importance that
the modes adopted to enforce collection of taxes levied should be summary and
interfered with as little as possible. No government could exist if all litigants were
permitted to delay the collection of its taxes. 4
Moreover, before the creation of the Court of Tax Appeals the remedy of a taxpayer
who desired to contest an assessment issued, by the Collector of Internal Revenue
was to pay the tax and bring an action in the ordinary courts for its recovery
pursuant to Section 306 of the Code.5 Collection or payment of the tax was not
made, to, wait until after the Collector of Internal Revenue has resolved all issues
raised by the taxpayer against an assessment. Republic Act 1125 creating the Court
of Appeals allows the taxpayer to dispute the correctness legality of an assessment
both in the purely administrative level and in said court, but it does not stop the
Collector of Internal Revenue from collecting the tax through any of the means
provided for in Section 316 of the Tax Code, except when enjoined by said Court of
Tax Appeals. Section 11 of Republic Act 1125 states in part:
No appeal taken to the Court of Tax Appeals from the decision of the Collector
of Internal Revenue ... shall suspend the payment, levy, distraint, and/or sale
of any property of the taxpayer for the satisfaction of his tax liability as
provided by existing law: Provided, however, That when in the opinion of the
Court the collection by the Bureau of Internal Revenue or the Commissioner of
Customs may jeopardize the interest of the Government and/or the taxpayer
the Court at any stage of the proceeding may suspend the said collection and
require the taxpayer either to deposit the amount claimed or to file a surety
bond for not more than double the amount with the Court.
We will now resolve the issue of whether or not the court a quo erred in considering
as final and executory the assessment contained in the letter of the Collector of
Internal Revenue dated January 16, 1957. As stated, defendant received said
assessment on January 30, 1957 and on the following day requested reinvestigation
of its tax liability. The Collector of Internal Revenue however did not reply to the
request for reinvestigation. Instead, he referred the case to the Solicitor General for
collection of the tax. The lower court interpreted this action of the Collector of
Internal Revenue as a denial of defendant's request for reinvestigation.
Said court, to our mind, committed no error. For what is more indicative of the
Collector's decision against reinvestigation than his insistence to collect the tax? This
decision was communicated to defendant in a letter dated September 20, 1957 of
the office of the Solicitor General which must have been received by defendant not

later than October 8, 1957 for on said date it acknowledged receipt thereof. It had
thirty days from October 8, 1957 within which to appeal to the Court of Tax Appeals
pursuant to Section 11 of Republic Act 1125. 6 Instead of appealing to the Tax Court,
however, the defendant herein in a letter dated October 8, 1957 reiterated its
request for reinvestigation.
On October 15, 1957 the Collector of Internal Revenue wrote defendant that its
"request for a reinvestigation will be granted only upon compliance with General
Circular No. V-258 dated August 20, 1957, which requires as a prerequisite to the
grant of a reinvestigation the execution of a waiver of the statute of limitations". In a
subsequent letter, he extended the period within which to submit the aforesaid
waiver to December 31, 1957.
In effect, the Collector of Internal Revenue placed in the hands of the defendant the
holding of a reinvestigation. However, no such reinvestigation was made inasmuch
as taxpayer failed to submit a written waiver of the statute of limitations on or before
December 31, 1957. Such omission automatically brought about the denial of the
request for reinvestigation.
Taxpayer however questions the legality of requiring waiver of the statute of
limitations before the grant of reinvestigation as provided for in General Circular No.
V-258. This question was not raised in the Bureau of Internal Revenue. Suffice it to
say in this connection that General Circular No. V-258 was promulgated pursuant to
Section 338 of the Tax Code. The authority thereunder of the Secretary of Finance to
issue rules and regulations for the effective enforcement of the provisions of the Tax
Code has been sustained by this Court in previous cases. 7
Even if we do not count the period from October 8, 1957 (the date when taxpayer
received notice of the denial of its request for reinvestigation) to December 31, 1957
(the deadline for the submission of the written waiver of the statute of limitations) in
reckoning the 30-day period within which the taxpayer may appeal to the Court of
Tax Appeals, said period had long lapsed when the Collector of Internal Revenue filed
the complaint in this case on September 2, 1958.
Taxpayer failure to appeal to the Court of Tax Appeals in due time made the
assessment in question final, executory and demandable. 8 And when the action was
instituted on September 2, 1958 to enforce the deficiency assessment in question, it
was already barred from disputing the correctness of the assessment or invoking any
defense that would reopen the question of his tax liability on merits. 9 Otherwise, the
period of thirty days for appeal to the Court of Tax Appeals would make little
sense. 10
In a proceeding like this the taxpayer's defenses are similar to those of the
defendant in a case for the enforcement of a judgment by judicial action under
Section 6 of Rule 39 of the Rules of Court. No inquiry can be made therein as to the
merits of the original case or the justness of the judgment relied upon, other than by

evidence of want of jurisdiction, of collusion between the parties, or of fraud in the


party offering the record with respect to the proceedings. 11 As held by this Court in
Insular Government vs.
Nico 12 the taxpayer may raise only the questions whether or not the Collector of
Internal Revenue had jurisdiction to do the particular act, and whether any fraud was
committed in the doing of the act. In that case, Doroteo Nico was fined by the
Collector of Internal Revenue for violation of subparagraphs (d), (e) and (g) of
Section 28 as well as Sections 36, 101 and 107 of Act 1189. Under Section 54 of the
same Act the taxpayer was given the right to appeal from the decision of the
Collector of Internal Revenue to the Court of First Instance within a period of ten days
from notice of imposition of the fine. Nico did not appeal, neither did he pay the fine.
Pursuant to Section 33 of the Act, the Collector of Internal Revenue filed an action in
the Court of First Instance to enforce his decision and collect the fine. The decision of
the Collector of Internal Revenue having become final, this Court, on appeal, allowed
no further inquiry into the merits of the same.
For the satisfaction of defendant, however, it may be worth stating that on its merits,
the assessment in question is correct. It is not controverted that, as appearing from
its 1952 income tax return Lim Tian Teng Sons & Co., Inc. employs the "accrual"
method of accounting. Following such accounting method the copra outturn in the
amount of P95,500.00 outstanding as of December 31, 1951, should have been
treated as accrued income for 1951, instead of as stock on hand on January 1, 1952.
Defendant took up the copra outturn in question as copra on hand in the beginning
inventory for 1952. Said beginning inventory, together with expenses, copra
purchased during the year and copra on hand as of December 31, 1952 were
deducted as "cost of goods sold" from the total gross sales for the purpose of
determining the net sales. Since the P95,500.00 copra outturn formed part of the
"cost of goods sold", it diminished the net sales by P95,500.00, thereby also
decreasing defendant's net taxable income by the same amount. This procedure of
treating the copra outturn in question is inconsistent with defendants accounting
method.
From the record, then, there is every indication that taxpayer's 1952 income tax
return is fraudulent, as alleged in paragraph (7) of the complaint in this case. Firstly,
taxpayer's beginning inventory for 1952 did not state the truth in considering the
copra outturn as copra on hand, for on December 31, 1951 such copra was not any
more in taxpayer's bodega. It was in transit to a foreign port. And the taxpayer no
longer owned the copra. As a matter of fact, it already received payment for the
same. Secondly, by observing regularly its own system of accounting, taxpayer had
no choice but to account the copra outturn as accrued income. This it did not do. For
such deviation, we see no other purpose than to lessen, if not obliterate as in fact it
did, its income tax liability per its return. The lower court therefore did not err in
imposing the 50% surcharge.

We now come to the appeal of the Government. It maintains that the lower court
erred in not imposing on defendant's tax liability a surcharge of 5% for late payment.
Subsection (c), Section 51 of the Tax Code states:
SEC. 51. Assessment and payment of income tax.
xxx

xxx

xxx

(c) Surcharge and interest in case of delinquency. - To any sum or sums due
and unpaid after the dates prescribed in subsections (b), (c) and (d) for the
payment of the same, there shall be added the sum of five per centum on the
amount of tax unpaid and interest at the rate of one per centum a month
upon said tax from the time the same became due . . . . (Emphasis supplied)
As may be gleaned from the above-quoted provision, the 5% surcharge is mandatory
and automatically due, once the tax is not paid on time. "Shall" is the word that law
uses a word normally imperative and a "language of demand". 13 Applicable herein is
what has been said of a similar provision the present Section 183 of the Tax Code
stating that:
If the percentage tax on any business is not paid within the time prescribed
above the amount of the tax shall be increased by twenty-five per centum, the
increment to be part of the tax. (Emphasis supplied)
Said this Court in Lim Co Chui vs. Posadas

14

This provision is mandatory. It provides a plan which works out automatically.


It confers no discretion on the Collector of Internal Revenue. That, official may
not disregard the law and substitute therefor his own personal judgment.
Finally, the Government questions the computation of the delinquency interest, due
on the deficiency tax, from October 8, 1957. It insists that payment of such interest
should commence from February 15, 1957. Such contention is well-founded.
Pursuant to Section 51(d), "the assessment made by the Collector of Internal
Revenue shall be paid ... immediately upon notification of the amount of such
assessment." Now, the income tax assessment notice gave defendant up to February
15, 1957 to pay the deficiency tax in question. No payment was made. Hence,
pursuant to Section 51 (e), quoted earlier, interest on the unpaid tax fell due starting
February 16, 1957 and continues to accrue until full payment of the tax.
Wherefore, the decision appealed from is modified. Lim Tian Teng Sons & Co., Inc. is
hereby ordered to pay the sum of P10,074.00 as deficiency income tax for 1952 plus
50% and 5% surcharges thereon for fraud and late payment, respectively, and 1%
monthly interest upon said tax of P10,074.00, computed from February 16, 1957
until the tax is fully paid. With costs against defendant-appellant. So ordered.

G.R. No. L-45277 August 5, 1985


AUGUSTO BASA, petitioner,
vs.
REPUBLIC OF THE PHILIPPINES, represented by the Solicitor General, and
Judge GUILLERMO F. VILLASOR, Branch XV, Court of First Instance of
Manila, respondent.
Angel R. Gonzales for petitioner.
The Solicitor General for respondents.

AQUINO, J.:
The issue in this case is whether the decision of the Court of First Instance of Manila
(not the Tax Court) in an income tax case is reviewable by the Appellate Court or by
this Court.
In a demand letter dated August 31, 1967, the Commissioner of Internal Revenue
assessed against Augusto Basa deficiency income taxes for 1957 to 1960 totalling
P16,353.12.*
As may be noted, the deficiencies were based on the taxpayer's failure to report in
full his capital gains on the sales of land. This omission or underdeclaration of
income justified the imposition of 50% surcharge.
The taxpayer did not contest the assessments in the Tax Court. The Commissioner's
letter-decision on the case was dated December 6, 1974. On the assumption that the
assessments had become final and incontestable, the Commissioner on September
3, 1975 sued the taxpayer in the Manila Court of First Instance for the collection of
said amount.
The trial court in a decision dated April 20, 1976 affirmed the assessments and
ordered Basa to pay P16,353.12 plus 5% surcharge and one percent monthly interest
from August 31, 1967 to August 31, 1970.
Instead of appealing to this Court directly under Republic Act No. 5440, in relation to
Rules 41 and 45 of the Rules of Court, since no factual issues are involved, Basa tried
to appeal to the Court of Appeals. He did not perfect his appeal within the
reglementary period. The trial court dismissed it in its order dated October 1, 1976.
On December 23, 1976 Basa filed the instant special civil action of certiorari wherein
he assailed the trial court's decision.

We hold that the petition is devoid of merit. The trial court acted within its
jurisdiction in rendering its decision and dismissing Basa's appeal. He should have
appealed to this Court. His failure to do so rendered the decision final and
executory. He has no cause of action for certiorari.
The decision is correct. If he wanted to contest the assessments, he should have
appealed to the Tax Court. Not having done so, he could not contest the same in the
Court of First Instance.
The issue of prescription raised by him is baseless. The assessments were predicated
on the fact that his income tax returns, if not fraudulent, were false because he
underdeclared his income. In such a case, the deficiency assessments may be made
within ten years after the discovery of the falsity or omission. The court action
should be instituted within five years after the assessment but this period is
suspended during the time that the Commission is prohibited from instituting a court
action.**
As explained in the Solicitor General's memorandum, Basa's requests for
reinvestigation tolled the prescriptive period of five years within which court action
may be brought (Commissioner of Internal Revenue vs. Capitol Subdivision, Inc., 119
Phil. 1051; Collector of Internal Revenue vs. Suyoc Consolidated Mining Company,
104 Phil. 819). Moreover, the issue of prescription should have been raised in the Tax
Court.
WHEREFORE, the trial court's judgment is affirmed. No costs.
SO ORDERED.
G.R. No. L-22356

July 21, 1967

REPUBLIC OF THE PHILIPPINES, plaintiff-appellant,


vs.
PEDRO B. PATANAO, defendant-appellee.
Office of the Solicitor General Arturo A. Alafriz, Solicitor A. B. Afurong and L. O. Gallang for plaintiff-appellant.
Tranquilino O. Calo, Jr. for defendant-appellee.
ANGELES, J.:
This is an appeal from an order of the Court of First Instance of Agusan in civil case
No. 925, dismissing plaintiff's complaint so far as concerns the collection of
deficiency income taxes for the years 1951, 1953 and 1954 and additional residence
taxes for 1951 and 1952, and requiring the defendant to file his answer with respect
to deficiency income tax for 1955 and residence taxes for 1953-1955.

In the complaint filed by the Republic of the Philippines, through the Solicitor
General, against Pedro B. Patanao, it is alleged that defendant was the holder of an
ordinary timber license with concession at Esperanza, Agusan, and as such was
engaged in the business of producing logs and lumber for sale during the years
1951-1955; that defendant failed to file income tax returns for 1953 and 1954, and
although he filed income tax returns for 1951, 1952 and 1955, the same were false
and fraudulent because he did not report substantial income earned by him from his
business; that in an examination conducted by the Bureau of Internal Revenue on
defendant's income and expenses for 1951-1955, it was ascertained that the sum of
P79,892.75, representing deficiency; income taxes and additional residence taxes for
the aforesaid years, is due from defendant; that on February 14, 1958, plaintiff,
through the Deputy Commissioner of Internal Revenue, sent a letter of demand with
enclosed income tax assessment to the defendant requiring him to pay the said
amount; that notwithstanding repeated demands the defendant refused, failed and
neglected to pay said taxes; and that the assessment for the payment of the taxes in
question has become final, executory and demandable, because it was not contested
before the Court of Tax Appeals in accordance with the provisions of section 11 of
Republic Act No. 1125.
Defendant moved to dismiss the complaint on two grounds, namely: (1) that the
action is barred by prior judgment, defendant having been acquitted in criminal
cases Nos. 2089 and 2090 of the same court, which were prosecutions for failure to
file income tax returns and for non-payment of income taxes; and (2) that the action
has prescribed.
After considering the motion to dismiss, the opposition thereto and the rejoinder to
the opposition, the lower court entered the order appealed from, holding that the
only cause of action left to the plaintiff in its complaint is the collection of the income
tax due for the taxable year 1955 and the residence tax (Class B) for 1953, 1954 and
1955. A motion to reconsider said order was denied, whereupon plaintiff interposed
the instant appeal, which was brought directly to this Court, the questions involved
being purely legal.
The conclusion of the trial court, that the present action is barred by prior judgment,
is anchored on the following rationale:
There is no question that the defendant herein has been accused in Criminal
Cases Nos. 2089 and 2090 of this Court for not filing his income tax returns
and for non-payment of income taxes for the years 1953 and 1954. In both
cases, he was acquitted. The rule in this jurisdiction is that the accused once
acquitted is exempt from both criminal and civil responsibility because when a
criminal action is instituted, civil action arising from the same offense is
impliedly instituted unless the offended party expressly waives the civil action
or reserves the right to file it separately. In the criminal cases abovementioned
wherein the defendant was completely exonerated, there was no waiver or
reservation to file a separate civil case so that the failure to obtain conviction

on a charge of non-payment of income taxes is fatal to any civil action to


collect the payment of said taxes.1wph1.t
Plaintiff-appellant assails the ruling as erroneous. Defendant-appellee on his part
urges that it should be maintained.
In applying the principle underlying the civil liability of an offender under the Penal
Code to a case involving the collection of taxes, the court a quo fell into error. The
two cases are circumscribed by factual premises which are diametrically opposed to
each either, and are founded on entirely different philosophies. Under the Penal Code
the civil liability is incurred by reason of the offender's criminal act. Stated
differently, the criminal liability gives birth to the civil obligation such that generally,
if one is not criminally liable under the Penal Code, he cannot become civilly liable
thereunder. The situation under the income tax law is the exact opposite. Civil
liability to pay taxes arises from the fact, for instance, that one has engaged himself
in business, and not because of any criminal act committed by him. The criminal
liability arises upon failure of the debtor to satisfy his civil obligation. The incongruity
of the factual premises and foundation principles of the two cases is one of the
reasons for not imposing civil indemnity on the criminal infractor of the income tax
law. Another reason, of course, is found in the fact that while section 73 of the
National Internal Revenue Code has provided the imposition of the penalty of
imprisonment or fine, or both, for refusal or neglect to pay income tax or to make a
return thereof, it failed to provide the collection of said tax in criminal proceedings.
The only civil remedies provided, for the collection of income tax, in Chapters I and II,
Title IX of the Code and section 316 thereof, are distraint of goods, chattels, etc. or
by judicial action, which remedies are generally exclusive in the absence of a
contrary intent from the legislator. (People vs. Arnault, G.R. No. L-4288, November
20, 1952; People vs. Tierra, G.R. Nos. L-17177-17180, December 28, 1964)
Considering that the Government cannot seek satisfaction of the taxpayer's civil
liability in a criminal proceeding under the tax law or, otherwise stated, since the
said civil liability is not deemed included in the criminal action, acquittal of the
taxpayer in the criminal proceeding does not necessarily entail exoneration from his
liability to pay the taxes. It is error to hold, as the lower court has held, that the
judgment in the criminal cases Nos. 2089 and 2090 bars the action in the present
case. The acquittal in the said criminal cases cannot operate to discharge defendant
appellee from the duty of paying the taxes which the law requires to be paid, since
that duty is imposed by statute prior to and independently of any attempts by the
taxpayer to evade payment. Said obligation is not a consequence of the felonious
acts charged in the criminal proceeding, nor is it a mere civil liability arising from
crime that could be wiped out by the judicial declaration of non-existence of the
criminal acts charged. (Castro vs. The Collector of Internal Revenue, G.R. No. L12174, April 20, 1962).
Regarding prescription of action, the lower court held that the cause of action on the
deficiency income tax and residence tax for 1951 is barred because appellee's
income tax return for 1951 was assessed by the Bureau of Internal Revenue only on

February 14, 1958, or beyond the five year period of limitation for assessment as
provided in section 331 of the National Internal Revenue Code. Appellant contends
that the applicable law is section 332 (a) of the same Code under which a proceeding
in court for the collection of the tax may be commenced without assessment at any
time within 10 years from the discovery of the falsity, fraud or omission.
The complaint filed on December 7, 1962, alleges that the fraud in the appellee's
income tax return for 1951, was discovered on February 14, 1958. By filing a motion
to dismiss, appellee hypothetically admitted this allegation as all the other
averments in the complaint were so admitted. Hence, section 332 (a) and not section
331 of the National Internal Revenue Code should determine whether or not the
cause of action of deficiency income tax and residence tax for 1951 has prescribed.
Applying the provision of section 332 (a), the appellant's action instituted in court on
December 7, 1962 has not prescribed.
Wherefore, the order appealed from is hereby set aside. Let the records of this case
be remanded to the court of origin for further proceedings. No pronouncement as to
costs.
[G.R. No. 128315. June 29, 1999]
COMMISSIONER OF INTERNAL REVENUE, petitioner, vs. PASCOR REALTY AND
DEVELOPMENT CORPORATION, ROGELIO A. DIO and VIRGINIA S.
DIO, respondents.
DECISION
PANGANIBAN, J.:
An assessment contains not only a computation of tax liabilities, but also a
demand for payment within a prescribed period. It also signals the time when
penalties and interests begin to accrue against the taxpayer. To enable the taxpayer
to determine his remedies thereon, due process requires that it must be served on
and received by the taxpayer. Accordingly, an affidavit, which was executed by
revenue officers stating the tax liabilities of a taxpayer and attached to a criminal
complaint for tax evasion, cannot be deemed an assessment that can be questioned
before the Court of Tax Appeals.
Statement of the Case
Before this Court is a Petition for Review on Certiorari under Rule 45 of the Rules
of Court praying for the nullification of the October 30, 1996 Decision [1] of the Court
of Appeals[2] in CA-GR SP No. 40853, which effectively affirmed the January 25, 1996
Resolution[3] of the Court of Tax Appeals [4] in CTA Case No. 5271. The CTA disposed
as follows:

WHEREFORE, finding [the herein petitioners] Motion to Dismiss as


UNMERITORIOUS, the same is hereby DENIED. [The CIR] is hereby given a period of
thirty (30) days from receipt hereof to file her answer.
Petitioner also seeks to nullify the February 13, 1997 Resolution [5] of the Court of
Appeals denying reconsideration.
The Facts
As found by the Court of Appeals, the undisputed facts of the case are as follows:
It appears that by virtue of Letter of Authority No. 001198, then BIR Commissioner
Jose U. Ong authorized Revenue Officers Thomas T. Que, Sonia T. Estorco and
Emmanuel M. Savellano to examine the books of accounts and other accounting
records of Pascor Realty and Development Corporation. (PRDC) for the years ending
1986, 1987 and 1988. The said examination resulted in a recommendation for the
issuance of an assessment in the amounts of P7,498,434.65 and P3,015,236.35 for
the years 1986 and 1987, respectively.
On March 1, 1995, the Commissioner of Internal Revenue filed a criminal complaint
before the Department of Justice against the PRDC, its President Rogelio A. Dio, and
its Treasurer Virginia S. Dio, alleging evasion of taxes in the total amount of
P10,513,671.00. Private respondents PRDC, et. al. filed an Urgent Request for
Reconsideration/Reinvestigation disputing the tax assessment and tax liability.
On March 23, 1995, private respondents received a subpoena from the DOJ in
connection with the criminal complaint filed by the Commissioner of Internal
Revenue (BIR) against them.
In a letter dated May 17, 1995, the CIR denied the urgent request for
reconsideration/reinvestigation of the private respondents on the ground that no
formal assessment has as yet been issued by the Commissioner.
Private respondents then elevated the Decision of the CIR dated May 17, 1995 to
the Court of Tax Appeals on a petition for review docketed as CTA Case No. 5271 on
July 21, 1995. On September 6, 1995, the CIR filed a Motion to Dismiss the petition
on the ground that the CTA has no jurisdiction over the subject matter of the
petition, as there was no formal assessment issued against the petitioners. The CTA
denied the said motion to dismiss in a Resolution dated January 25, 1996 and
ordered the CIR to file an answer within thirty (30) days from receipt of said
resolution. The CIR received the resolution on January 31, 1996 but did not file an
answer nor did she move to reconsider the resolution.
Instead, the CIR filed this petition on June 7, 1996, alleging as grounds that:

Respondent Court of Tax Appeals acted with grave abuse of discretion and without
jurisdiction in considering the affidavit/report of the revenue officer and the
indorsement of said report to the secretary of justice as assessment which may be
appealed to the Court of Tax Appeals;
Respondent Court of Tax Appeals acted with grave abuse of discretion in considering
the denial by petitioner of private respondents Motion for Reconsideration as [a]
final decision which may be appealed to the Court of Tax Appeals.
In denying the motion to dismiss filed by the CIR, the Court of Tax Appeals stated:
We agree with petitioners contentions, that the criminal complaint for tax evasion is
the assessment issued, and that the letter denial of May 17, 1995 is the decision
properly appealable to [u]s. Respondents ground of denial, therefore, that there
was no formal assessment issued, is untenable.
It is the Courts honest belief, that the criminal case for tax evasion is already an
assessment. The complaint, more particularly, the Joint Affidavit of Revenue
Examiners Lagmay and Savellano attached thereto, contains the details of the
assessment like the kind and amount of tax due, and the period covered.
Petitioners are right, in claiming that the provisions of Republic Act No. 1125,
relating to exclusive appellate jurisdiction of this Court, do not, make any mention of
formal assessment. The law merely states, that this Court has exclusive appellate
jurisdiction over decisions of the Commissioner of Internal Revenue on disputed
assessments, and other matters arising under the National Internal Revenue Code,
other law or part administered by the Bureau of Internal Revenue Code.
As far as this Court is concerned, the amount and kind of tax due, and the period
covered, are sufficient details needed for an assessment. These details are more
than complete, compared to the following definitions of the term as quoted
hereunder. Thus:
Assessment is laying a tax. Johnson City v. Clinchfield R. Co., 43 S.W. (2d) 386, 387,
163 Tenn. 332. (Words and Phrases, Permanent Edition, Vol. 4, p. 446)
The word assessment when used in connection with taxation, may have more than
one meaning. The ultimate purpose of an assessment to such a connection is to
ascertain the amount that each taxpayer is to pay. More commonly, the word
assessment means the official valuation of a taxpayers property for purpose of
taxation. State v. New York, N.H. and H.R. Co. 22 A. 765, 768, 60 Conn. 326, 325.
(Ibid. p. 445)
From the above, it can be gleaned that an assessment simply states how much tax
is due from a taxpayer. Thus, based on these definitions, the details of the tax as
given in the Joint Affidavit of respondents examiners, which was attached to the tax

evasion complaint, more than suffice to qualify as an assessment. Therefore, this


assessment having been disputed by petitioners, and there being a denial of their
letter disputing such assessment, this Court unquestionably acquired jurisdiction
over the instant petition for review.[6]
As earlier observed, the Court of Appeals sustained the CTA and dismissed the
petition.
Hence, this recourse to this Court.[7]
Ruling of the Court of Appeals
The Court of Appeals held that the tax court committed no grave abuse of
discretion in ruling that the Criminal Complaint for tax evasion filed by the
Commissioner of Internal Revenue with the Department of Justice constituted an
assessment of the tax due, and that the said assessment could be the subject of a
protest. By definition, an assessment is simply the statement of the details and the
amount of tax due from a taxpayer. Based on this definition, the details of the tax
contained in the BIR examiners Joint Affidavit, [8] which was attached to the criminal
Complaint, constituted an assessment. Since the assailed Order of the CTA was
merely interlocutory and devoid of grave abuse of discretion, a petition
for certiorari did not lie.
Issues
Petitioners submit for the consideration of this Court the following issues:
(1)
Whether or not the criminal complaint for tax evasion can be construed as
an assessment.
(2) Whether or not an assessment is necessary before criminal charges for tax
evasion may be instituted.
(3) Whether or not the CTA can take cognizance of the case in the absence of an
assessment.[9]
In the main, the Court will resolve whether the revenue officers Affidavit-Report,
which was attached to the criminal Complaint filed with the Department of Justice,
constituted an assessment that could be questioned before the Court of Tax Appeals.
The Courts Ruling
The petition is meritorious.
Main Issue: Assessment

Petitioner argues that the filing of the criminal complaint with the Department of
Justice cannot in any way be construed as a formal assessment of private
respondents tax liabilities. This position is based on Section 205 of the National
Internal Revenue Code[10] (NIRC), which provides that remedies for the collection of
deficient taxes may be by either civil or criminal action. Likewise, petitioner cites
Section 223(a) of the same Code, which states that in case of failure to file a return,
the tax may be assessed or a proceeding in court may be begun without
assessment.
Respondents, on the other hand, maintain that an assessment is not an action or
proceeding for the collection of taxes, but merely a notice that the amount stated
therein is due as tax and that the taxpayer is required to pay the same. Thus,
qualifying as an assessment was the BIR examiners Joint Affidavit, which contained
the details of the supposed taxes due from respondent for taxable years ending
1987 and 1988, and which was attached to the tax evasion Complaint filed with the
DOJ. Consequently, the denial by the BIR of private respondents request for
reinvestigation of the disputed assessment is properly appealable to the CTA.
We agree with petitioner. Neither the NIRC nor the revenue regulations
governing the protest of assessments [11] provide a specific definition or form of an
assessment. However, the NIRC defines the specific functions and effects of an
assessment. To consider the affidavit attached to the Complaint as a proper
assessment is to subvert the nature of an assessment and to set a bad precedent
that will prejudice innocent taxpayers.
True, as pointed out by the private respondents, an assessment informs the
taxpayer that he or she has tax liabilities. But not all documents coming from the
BIR containing a computation of the tax liability can be deemed assessments.
To start with, an assessment must be sent to and received by a taxpayer, and
must demand payment of the taxes described therein within a specific period. Thus,
the NIRC imposes a 25 percent penalty, in addition to the tax due, in case the
taxpayer fails to pay the deficiency tax within the time prescribed for its payment in
the notice of assessment. Likewise, an interest of 20 percent per annum, or such
higher rate as may be prescribed by rules and regulations, is to be collected from the
date prescribed for its payment until the full payment. [12]
The issuance of an assessment is vital in determining the period of limitation
regarding its proper issuance and the period within which to protest it. Section
203[13]of the NIRC provides that internal revenue taxes must be assessed within three
years from the last day within which to file the return. Section 222,[14] on the other
hand, specifies a period of ten years in case a fraudulent return with intent to evade
was submitted or in case of failure to file a return. Also, Section 228[15] of the same
law states that said assessment may be protested only within thirty days from
receipt thereof. Necessarily, the taxpayer must be certain that a specific document
constitutes an assessment. Otherwise, confusion would arise regarding the period

within which to make an assessment or to protest the same, or whether interest and
penalty may accrue thereon.
It should also be stressed that the said document is a notice duly sent to the
taxpayer. Indeed, an assessment is deemed made only when the collector of
internal revenue releases, mails or sends such notice to the taxpayer. [16]
In the present case, the revenue officers Affidavit merely contained a
computation of respondents tax liability. It did not state a demand or a period for
payment. Worse, it was addressed to the justice secretary, not to the taxpayers.
Respondents maintain that an assessment, in relation to taxation, is simply
understood to mean:
A notice to the effect that the amount therein stated is due as tax and a demand for
payment thereof.[17]
Fixes the liability of the taxpayer and ascertains the facts and furnishes the data for
the proper presentation of tax rolls. [18]
Even these definitions fail to advance private respondents case. That the BIR
examiners Joint Affidavit attached to the Criminal Complaint contained some details
of the tax liabilities of private respondents does not ipso facto make it an
assessment. The purpose of the Joint Affidavit was merely to support and
substantiate the Criminal Complaint for tax evasion. Clearly, it was not meant to be
a notice of the tax due and a demand to the private respondents for payment
thereof.
The fact that the Complaint itself was specifically directed and sent to the
Department of Justice and not to private respondents shows that the intent of the
commissioner was to file a criminal complaint for tax evasion, not to issue an
assessment. Although the revenue officers recommended the issuance of an
assessment, the commissioner opted instead to file a criminal case for tax
evasion. What private respondents received was a notice from the DOJ that a
criminal case for tax evasion had been filed against them, not a notice that the
Bureau of Internal Revenue had made an assessment.
In addition, what private respondents sent to the commissioner was a motion for
a reconsideration of the tax evasion charges filed, not of an assessment, as shown
thus:
This is to request for reconsideration of the tax evasion charges against my client,
PASCOR Realty and Development Corporation and for the same to be referred to the
Appellate Division in order to give my client the opportunity of a fair and objective
hearing[19]

Additional Issues: Assessment Not Necessary Before Filing of Criminal


Complaint
Private respondents maintain that the filing of a criminal complaint must be
preceded by an assessment. This is incorrect, because Section 222 of the NIRC
specifically states that in cases where a false or fraudulent return is submitted or in
cases of failure to file a return such as this case, proceedings in court may be
commenced without an assessment. Furthermore, Section 205 of the same Code
clearly mandates that the civil and criminal aspects of the case may be pursued
simultaneously. In Ungab v. Cusi,[20] petitioner therein sought the dismissal of the
criminal Complaints for being premature, since his protest to the CTA had not yet
been resolved. The Court held that such protests could not stop or suspend the
criminal action which was independent of the resolution of the protest in the
CTA. This was because the commissioner of internal revenue had, in such tax
evasion cases, discretion on whether to issue an assessment or to file a criminal
case against the taxpayer or to do both.
Private respondents insist that Section 222 should be read in relation to Section
255 of the NIRC,[21] which penalizes failure to file a return. They add that a tax
assessment should precede a criminal indictment. We disagree. To reiterate, said
Section 222 states that an assessment is not necessary before a criminal charge can
be filed. This is the general rule. Private respondents failed to show that they are
entitled to an exception. Moreover, the criminal charge need only be supported by
a prima facie showing of failure to file a required return. This fact need not be
proven by an assessment.
The issuance of an assessment must be distinguished from the filing of a
complaint. Before an assessment is issued, there is, by practice, a pre-assessment
notice sent to the taxpayer. The taxpayer is then given a chance to submit position
papers and documents to prove that the assessment is unwarranted. If the
commissioner is unsatisfied, an assessment signed by him or her is then sent to the
taxpayer informing the latter specifically and clearly that an assessment has been
made against him or her. In contrast, the criminal charge need not go through all
these. The criminal charge is filed directly with the DOJ. Thereafter, the taxpayer is
notified that a criminal case had been filed against him, not that the commissioner
has issued an assessment. It must be stressed that a criminal complaint is instituted
not to demand payment, but to penalize the taxpayer for violation of the Tax Code.
WHEREFORE, the petition is hereby GRANTED. The assailed Decision
is REVERSED and SET ASIDE. CTA Case No. 5271 is likewise DISMISSED. No costs.
SO ORDERED.
LUCAS G. ADAMSON, THERESE
JUNE D. ADAMSON, and SARA
S. DE LOS REYES, in their capacities

G.R. No. 120935

as President, Treasurer and Secretary


of Adamson Management Corporation,
Petitioners,
- versus -

COURT OF APPEALS and


LIWAYWAY VINZONS-CHATO,
in her capacity as Commissioner
of the Bureau of Internal Revenue,
Respondents.
x-- - - - - - - - - - - - - - - - - - - - - - - - x
COMMISSIONER OF
INTERNAL REVENUE,
Petitioner,

G.R. No. 124557

Present:
-versus-

COURT OF APPEALS, COURT


OF TAX APPEALS, ADAMSON
MANAGEMENT CORPORATION,
LUCAS G. ADAMSON, THERESE
JUNE D. ADAMSON, and SARA
S. DE LOS REYES,
Respondents.

PUNO, C.J., Chairperson,


CARPIO,
CORONA,
LEONARDO-DE CASTRO, and
BERSAMIN, JJ.

Promulgated:
May 21, 2009

x--------------------------------------------------x

DECISION

PUNO, C.J.:
Before the Court are the consolidated cases of G.R. No. 120935 and G.R.
No. 124557.
G.R. No. 120935 involves a petition for review on certiorari filed by
petitioners LUCAS G. ADAMSON, THERESE JUNE D. ADAMSON, and SARA S. DE

LOS REYES (private respondents), in their respective capacities as president,


treasurer and secretary of Adamson Management Corporation (AMC) against then
Commissioner of Internal Revenue Liwayway Vinzons-Chato (COMMISSIONER),
under Rule 45 of the Revised Rules of Court. They seek to review and reverse the
Decision promulgated onMarch 21, 1995 and Resolution issued on July 6, 1995 of
the Court of Appeals in CA-G.R. SP No. 35488 (Liwayway Vinzons-Chato, et al. v.
Hon. Judge Erna Falloran-Aliposa, et al.).
G.R. No. 124557 is a petition for review on certiorari filed by the
Commissioner, assailing the Decision dated March 29, 1996 of the Court of
Appeals in CA-G.R. SP No. 35520, titled Commissioner of Internal Revenue v.
Court of Tax Appeals, Adamson Management Corporation, Lucas G. Adamson,
Therese June D. Adamson and Sara S. de los Reyes. In the said Decision, the
Court of Appeals upheld the Resolution promulgated on September 19, 1994 by
the Court of Tax Appeals (CTA) in C.T.A. Case No. 5075 (Adamson Management
Corporation, Lucas G. Adamson, Therese Adamson and Sara de los Reyes v.
Commissioner of Internal Revenue).
The facts, as culled from the findings of the appellate court, follow:
On June 20, 1990, Lucas Adamson and AMC sold 131,897 common shares
of stock in Adamson and Adamson, Inc. (AAI) to APAC Holding Limited (APAC). The
shares were valued at P7,789,995.00.[1] On June 22, 1990, P159,363.21 was paid
as capital gains tax for the transaction.
On October 12, 1990, AMC sold to APAC Philippines, Inc. another 229,870
common shares of stock in AAI for P17,718,360.00. AMC paid the capital gains
tax ofP352,242.96.
On October 15, 1993, the Commissioner issued a Notice of Taxpayer to
AMC, Lucas G. Adamson, Therese June D. Adamson and Sara S. de los Reyes,
informing them of deficiencies on their payment of capital gains tax and Value
Added Tax (VAT). The notice contained a schedule for preliminary conference.

The events preceding G.R. No. 120935 are the following:


On October 22, 1993, the Commissioner filed with the Department of
Justice (DOJ) her Affidavit of Complaint [2] against AMC, Lucas G. Adamson, Therese
June D. Adamson and Sara S. de los Reyes for violation of Sections 45 (a) and (d)
[3]

, and 110[4], in relation to Section 100[5], as penalized under Section 255,[6] and

for violation of Section 253[7], in relation to Section 252 (b) and (d) of the National
Internal Revenue Code (NIRC).[8]
AMC, Lucas G. Adamson, Therese June D. Adamson and Sara S. de los
Reyes filed with the DOJ a motion to suspend proceedings on the ground of
prejudicial question, pendency of a civil case with the Supreme Court, and
pendency of their letter-request for re-investigation with the Commissioner. After
the preliminary investigation, State Prosecutor Alfredo P. Agcaoili found probable
cause. The Motion for Reconsideration against the findings of probable cause was
denied by the prosecutor.
On April 29, 1994, Lucas G. Adamson, Therese June D. Adamson and Sara
S. de los Reyes were charged before the Regional Trial Court (RTC) of Makati,
Branch 150 in Criminal Case Nos. 94-1842 to 94-1846.

They filed a Motion to

Dismiss or Suspend the Proceedings. They invoked the grounds that there was
yet no final assessment of their tax liability, and there were still pending relevant
Supreme Court and CTA cases. Initially, the trial court denied the motion. A
Motion for Reconsideration was however filed, this time assailing the trial courts
lack of jurisdiction over the nature of the subject cases. On August 8, 1994, the
trial court granted the Motion. It ruled that the complaints for tax evasion filed by
the Commissioner should be regarded as a decision of the Commissioner
regarding the tax liabilities of Lucas G. Adamson, Therese June D. Adamson and
Sara S. de los Reyes, and appealable to the CTA. It further held that the said
cases cannot proceed independently of the assessment case pending before the
CTA, which has jurisdiction to determine the civil and criminal tax liability of the
respondents therein.

On October 10, 1994, the Commissioner filed a Petition for Review with the
Court of Appeals assailing the trial courts dismissal of the criminal cases. She
averred that it was not a condition prerequisite that a formal assessment should
first be given to the private respondents before she may file the aforesaid
criminal complaints against them. She argued that the criminal complaints for
tax evasion may proceed independently from the assessment cases pending
before the CTA.
On March 21, 1995, the Court of Appeals reversed the trial courts decision
and reinstated the criminal complaints. The appellate court held that, in a
criminal prosecution for tax evasion, assessment of tax deficiency is not
required

because

the

offense

of

tax

evasion

is

complete

or

consummated when the offender has knowingly and willfully filed a


fraudulent return with intent to evade the tax.[9] It ruled that private
respondents filed false and fraudulent returns with intent to evade
taxes, and acting thereupon, petitioner filed an Affidavit of Complaint
with the Department of Justice, without an accompanying assessment of
the tax deficiency of private respondents, in order to commence criminal
action against the latter for tax evasion.[10]
Private respondents filed a Motion for Reconsideration, but the trial court
denied the motion on July 6, 1995. Thus, they filed the petition in G.R. No.
120935, raising the following issues:
1.

WHETHER OR NOT THE RESPONDENT HONORABLE COURT OF


APPEALS ERRED IN APPLYING THE DOCTRINE IN UNGAB V. CUSI (Nos. L41919-24, May 30, 1980, 97 SCRA 877) TO THE CASE AT BAR.

2.

WHETHER OR NOT AN ASSESSMENT IS REQUIRED UNDER THE


SECOND CATEGORY OF THE OFFENSE IN SECTION 253 OF THE NIRC.

3.

WHETHER OR NOT THERE WAS A VALID ASSESSMENT MADE BY


THE COMMISSIONER IN THE CASE AT BAR.

4.

WHETHER OR NOT THE FILING OF A CRIMINAL COMPLAINT


SERVES AS AN IMPLIED ASSESSMENT ON THE TAX LIABILITY OF THE
TAXPAYER.

5.

WHETHER OR NOT THE FILING OF THE CRIMINAL INFORMATION


FOR TAX EVASION IN THE TRIAL COURT IS PREMATURE BECAUSE THERE
IS YET NO BASIS FOR THE CRIMINAL CHARGE OF WILLFULL INTENT TO
EVADE THE PAYMENT OF A TAX.

6.

WHETHER OR NOT THE DOCTRINES LAID DOWN IN THE CASES


OF YABES V. FLOJO (No. L-46954, July 20, 1982, 115 SCRA
286) AND CIR V. UNION SHIPPING CORP. (G.R. No. 66160, May 21, 1990,
185 SCRA 547) ARE APPLICABLE TO THE CASE AT BAR.

7.

WHETHER
OR
NOT
THE
COURT
OF
TAX
APPEALS HAS JURISDICTION OVER THE DISPUTE ON WHAT CONSTITUTES
THE PROPER TAXES DUE FROM THE TAXPAYER.

In

parallel

circumstances,

the

following

events

preceded G.R.

No.

124557:
On December 1, 1993, AMC, Lucas G. Adamson, Therese June D. Adamson
and Sara S. de los Reyes filed a letter request for re-investigation with the
Commissioner of the Examiners Findings earlier issued by the Bureau of
Internal Revenue (BIR), which pointed out the tax deficiencies.
On March 15, 1994 before the Commissioner could act on their letterrequest, AMC, Lucas G. Adamson, Therese June D. Adamson and Sara S. de los
Reyes filed a Petition for Review with the CTA. They assailed the Commissioners
finding of tax evasion against them. The Commissioner moved to dismiss the
petition, on the ground that it was premature, as she had not yet issued a formal
assessment of the tax liability of therein petitioners. On September 19, 1994,
the CTA denied the Motion to Dismiss. It considered the criminal complaint filed
by the Commissioner with the DOJ as an implied formal assessment, and the filing
of the criminal informations with the RTC as a denial of petitioners protest
regarding the tax deficiency.

The Commissioner repaired to the Court of Appeals on the ground that


the CTA acted with grave abuse of discretion. She contended that, with regard to
the protest provided under Section 229 of the NIRC, there must first be a formal
assessment issued by the Commissioner, and it must be in accord with Section 6
of Revenue Regulation No. 12-85. She maintained that she had not yet issued a
formal assessment of tax liability, and the tax deficiency amounts mentioned in
her criminal complaint with the DOJ were given only to show the difference
between the tax returns filed and the audit findings of the revenue examiner.
The Court of Appeals sustained the CTAs denial of the Commissioners
Motion to Dismiss. Thus, the Commissioner filed the petition for review
under G.R. No. 124557, raising the following issues:
1.

WHETHER OR NOT THE INSTANT PETITION SHOULD BE


DISMISSED FOR FAILURE TO COMPLY WITH THE MANDATORY
REQUIREMENT OF A CERTIFICATION UNDER OATH AGAINST FORUM
SHOPPING;

2.

WHETHER OR NOT THE CRIMINAL CASE FOR TAX EVASION IN THE


CASE AT BAR CAN PROCEED WITHOUT AN ASSESSMENT;

3.

WHETHER OR NOT THE COMPLAINT FILED WITH THE


DEPARTMENT OF JUSTICE CAN BE CONSTRUED AS AN IMPLIED
ASSESSMENT; and

4.

WHETHER
OR
NOT
THE
COURT
OF
TAX
APPEALS HAS JURISDICTION TO ACT ON PRIVATE RESPONDENTS
PETITION FOR REVIEW FILED WITH THE SAID COURT.
The issues in G.R. No. 124557 and G.R. No. 120935 can be compressed

into three:
1.

WHETHER THE COMMISSIONER HAS ALREADY RENDERED


AN ASSESSMENT (FORMAL OR OTHERWISE) OF THE TAX
LIABILITY OF AMC, LUCAS G. ADAMSON, THERESE JUNE D.
ADAMSON AND SARA S. DE LOS REYES;

2.

WHETHER THERE IS BASIS FOR THE CRIMINAL CASES FOR


TAX EVASION TO PROCEED AGAINST AMC, LUCAS G. ADAMSON,
THERESE JUNE D. ADAMSON AND SARA S. DE LOS REYES; and

3.

WHETHER THE COURT OF TAX APPEALS HAS JURISDICTION


TO TAKE COGNIZANCE OF BOTH THE CIVIL AND THE CRIMINAL
ASPECTS OF THE TAX LIABILITY OF AMC, LUCAS G. ADAMSON,
THERESE JUNE D. ADAMSON AND SARA S. DE LOS REYES.

The case of CIR v. Pascor Realty, et al.[11] is relevant. In this case, then BIR
Commissioner Jose U. Ong authorized revenue officers to examine the books of
accounts and other accounting records of Pascor Realty and Development
Corporation (PRDC) for 1986, 1987 and 1988. This resulted in a recommendation for
the issuance of an assessment in the amounts of P7,498,434.65 and P3,015,236.35
for the years 1986 and 1987, respectively.

On March 1, 1995, the Commissioner filed a criminal complaint before the DOJ
against PRDC, its President Rogelio A. Dio, and its Treasurer Virginia S. Dio, alleging
evasion of taxes in the total amount of P10,513,671.00. Private respondents filed an
Urgent Request for Reconsideration/Reinvestigation disputing the tax assessment
and tax liability.

The Commissioner denied the urgent request for


reconsideration/reinvestigation because she had not yet issued a formal assessment.

Private respondents then elevated the Decision of the Commissioner to the


CTA on a petition for review. The Commissioner filed a Motion to Dismiss the petition
on the ground that the CTA has no jurisdiction over the subject matter of the
petition, as there was yet no formal assessment issued against the petitioners. The
CTA denied the said motion to dismiss and ordered the Commissioner to file an
answer within thirty (30) days. The Commissioner did not file an answer nor did she
move to reconsider the resolution. Instead, the Commissioner filed a petition for
review of the CTA decision with the Court of Appeals. The Court of Appeals upheld

the CTA order. However, this Court reversed the Court of Appeals decision and the
CTA order, and ordered the dismissal of the petition. We held:
An assessment contains not only a computation of tax liabilities, but also a
demand for payment within a prescribed period. It also signals the time when
penalties and interests begin to accrue against the taxpayer. To enable the
taxpayer to determine his remedies thereon, due process requires that it must
be served on and received by the taxpayer. Accordingly, an affidavit, which
was executed by revenue officers stating the tax liabilities of a taxpayer and
attached to a criminal complaint for tax evasion, cannot be deemed an
assessment that can be questioned before the Court of Tax Appeals.
Neither the NIRC nor the revenue regulations governing the protest of
assessments[12] provide
a
specific
definition
or
form
of
an
assessment. However, the NIRC defines the specific functions and effects of
an assessment. To consider the affidavit attached to the Complaint as a
proper assessment is to subvert the nature of an assessment and to set a bad
precedent that will prejudice innocent taxpayers.
True, as pointed out by the private respondents, an assessment informs
the taxpayer that he or she has tax liabilities. But not all documents coming
from the BIR containing a computation of the tax liability can be deemed
assessments.
To start with, an assessment must be sent to and received by a taxpayer,
and must demand payment of the taxes described therein within a specific
period. Thus, the NIRC imposes a 25 percent penalty, in addition to the tax
due, in case the taxpayer fails to pay the deficiency tax within the time
prescribed for its payment in the notice of assessment. Likewise, an interest
of 20 percent per annum, or such higher rate as may be prescribed by rules
and regulations, is to be collected from the date prescribed for its payment
until the full payment.[13]
The issuance of an assessment is vital in determining the period of
limitation regarding its proper issuance and the period within which to protest
it. Section 203[14] of the NIRC provides that internal revenue taxes must be
assessed within three years from the last day within which to file the
return. Section 222,[15] on the other hand, specifies a period of ten years in
case a fraudulent return with intent to evade was submitted or in case of
failure to file a return. Also, Section 228[16] of the same law states that said
assessment may be protested only within thirty days from receipt
thereof. Necessarily, the taxpayer must be certain that a specific document
constitutes an assessment. Otherwise, confusion would arise regarding the
period within which to make an assessment or to protest the same, or whether
interest and penalty may accrue thereon.

It should also be stressed that the said document is a notice duly sent to
the taxpayer. Indeed, an assessment is deemed made only when the collector
of internal revenue releases, mails or sends such notice to the taxpayer. [17]
In the present case, the revenue officers Affidavit merely contained a
computation of respondents tax liability. It did not state a demand or a period
for payment. Worse, it was addressed to the justice secretary, not to the
taxpayers.
Respondents maintain that an assessment, in relation to taxation, is
simply understood to mean:
A notice to the effect that the amount therein stated is due as
tax and a demand for payment thereof. [18]
Fixes the liability of the taxpayer and ascertains the facts and
furnishes the data for the proper presentation of tax rolls. [19]
Even these definitions fail to advance private respondents case. That the
BIR examiners Joint Affidavit attached to the Criminal Complaint contained
some details of the tax liabilities of private respondents does not ipso
facto make it an assessment. The purpose of the Joint Affidavit was merely to
support and substantiate the Criminal Complaint for tax evasion. Clearly, it
was not meant to be a notice of the tax due and a demand to the private
respondents for payment thereof.
The fact that the Complaint itself was specifically directed and sent to the
Department of Justice and not to private respondents shows that the intent of
the commissioner was to file a criminal complaint for tax evasion, not to issue
an assessment. Although the revenue officers recommended the issuance of
an assessment, the commissioner opted instead to file a criminal case
for tax evasion. What private respondents received was a notice from the DOJ
that a criminal case for tax evasion had been filed against them, not a notice
that the Bureau of Internal Revenue had made an assessment.
Private respondents maintain that the filing of a criminal complaint must
be preceded by an assessment. This is incorrect, because Section 222 of the
NIRC specifically states that in cases where a false or fraudulent return is
submitted or in cases of failure to file a return such as this case, proceedings
in court may be commenced without an assessment. Furthermore, Section
205 of the same Code clearly mandates that the civil and criminal aspects of
the case may be pursued simultaneously. In Ungab v. Cusi,[20] petitioner
therein sought the dismissal of the criminal Complaints for being premature,
since his protest to the CTA had not yet been resolved. The Court held that
such protests could not stop or suspend the criminal action which was
independent of the resolution of the protest in the CTA. This was because the
commissioner of internal revenue had, in such tax evasion cases, discretion on

whether to issue an assessment or to file a criminal case against the taxpayer


or to do both.
Private respondents insist that Section 222 should be read in relation to
Section 255 of the NIRC,[21] which penalizes failure to file a return. They add
that a tax assessment should precede a criminal indictment. We disagree. To
reiterate, said Section 222 states that an assessment is not necessary before
a criminal charge can be filed. This is the general rule. Private respondents
failed to show that they are entitled to an exception. Moreover, the criminal
charge need only be supported by a prima facie showing of failure to file a
required return. This fact need not be proven by an assessment.
The issuance of an assessment must be distinguished from the filing of a
complaint. Before an assessment is issued, there is, by practice, a preassessment notice sent to the taxpayer. The taxpayer is then given a chance
to submit position papers and documents to prove that the assessment is
unwarranted. If the commissioner is unsatisfied, an assessment signed by
him or her is then sent to the taxpayer informing the latter specifically and
clearly that an assessment has been made against him or her. In contrast, the
criminal charge need not go through all these. The criminal charge is filed
directly with the DOJ. Thereafter, the taxpayer is notified that a criminal case
had been filed against him, not that the commissioner has issued an
assessment. It must be stressed that a criminal complaint is instituted not to
demand payment, but to penalize the taxpayer for violation of the Tax Code.

In the cases at bar, the Commissioner denied that she issued a formal
assessment of the tax liability of AMC, Lucas G. Adamson, Therese June D.
Adamson and Sara S. de los Reyes. She admits though that she wrote the
recommendation letter[22] addressed to the Secretary of the DOJ recommending
the filing of criminal complaints against AMC and the aforecited persons for
fraudulent returns and tax evasion.
The first issue is whether the Commissioners recommendation letter can
be considered as a formal assessment of private respondents tax liability.
In the context in which it is used in the NIRC, an assessment is a written
notice and demand made by the BIR on the taxpayer for the settlement of a due
tax liability that is there definitely set and fixed. A written communication
containing a computation by a revenue officer of the tax liability of a taxpayer
and giving him an opportunity to contest or disprove the BIR examiners findings
is not an assessment since it is yet indefinite. [23]

We rule that the recommendation letter of the Commissioner cannot be


considered a formal assessment. Even a cursory perusal of the said letter would
reveal three key points:
1.

It was not addressed to the taxpayers.

2.

There was no demand made on the taxpayers to pay the tax liability, nor
a period for payment set therein.

3.

The letter was never mailed or sent to the taxpayers by the


Commissioner.
In fine, the said recommendation letter served merely as the prima

facie basis for filing criminal informations that the taxpayers had violated Section
45 (a) and (d), and 110, in relation to Section 100, as penalized under Section
255, and for violation of Section 253, in relation to Section 252 9(b) and (d) of
the Tax Code.[24]
The next issue is whether the filing of the criminal complaints against the
private respondents by the DOJ is premature for lack of a formal assessment.
Section 269 of the NIRC (now Section 222 of the Tax Reform Act of 1997)
provides:
Sec. 269. Exceptions as to period of limitation of assessment and
collection of taxes.-(a) In the case of a false or fraudulent return with intent to
evade tax or of failure to file a return, the tax may be assessed, or a
proceeding in court after the collection of such tax may be begun without
assessment, at any time within ten years after the discovery of the falsity,
fraud or omission: Provided, That in a fraud assessment which has become
final and executory, the fact of fraud shall be judicially taken cognizance of in
the civil or criminal action for collection thereof

The law is clear. When fraudulent tax returns are involved as in the cases
at bar, a proceeding in court after the collection of such tax may be
begun without assessment. Here, the private respondents had already filed

the capital gains tax return and the VAT returns, and paid the taxes they have
declared due therefrom. Upon investigation of the examiners of the BIR, there
was a preliminary finding of gross discrepancy in the computation of the capital
gains taxes due from the sale of two lots of AAI shares, first to APAC and then to
APAC Philippines, Limited. The examiners also found that the VAT had not been
paid for VAT-liable sale of services for the third and fourth quarters of
1990. Arguably, the gross disparity in the taxes due and the amounts actually
declared by the private respondents constitutes badges of fraud.
Thus, the applicability of Ungab v. Cusi[25] is evident to the cases at
bar. In this seminal case, this Court ruled that there was no need for precise
computation and formal assessment in order for criminal complaints to be filed
against him. It quoted Mertens Law of Federal Income Taxation, Vol. 10, Sec.
55A.05, p. 21, thus:
An assessment of a deficiency is not necessary to a criminal
prosecution for willful attempt to defeat and evade the income tax. A crime is
complete when the violator has knowingly and willfully filed a fraudulent
return, with intent to evade and defeat the tax. The perpetration of the crime
is grounded upon knowledge on the part of the taxpayer that he has made an
inaccurate return, and the governments failure to discover the error and
promptly to assess has no connections with the commission of the crime.
This hoary principle still underlies Section 269 and related provisions of the
present Tax Code.
We now go to the issue of whether the CTA has no jurisdiction to take
cognizance of both the criminal and civil cases here at bar.
Under Republic Act No. 1125 (An Act Creating the Court of Tax Appeals) as
amended, the rulings of the Commissioner are appealable to the CTA, thus:
SEC. 7. Jurisdiction. The Court of Tax Appeals shall exercise exclusive
appellate jurisdiction to review by appeal, as herein provided (1) Decisions of the Commissioner of Internal Revenue in cases
involving disputed assessments, refunds of internal revenue taxes, fees
or other charges, penalties imposed in relation thereto, or other matters

arising under the National Internal Revenue Code or other laws or part
of law administered by the Bureau of Internal Revenue;

Republic Act No. 8424, titled An Act Amending the National Internal Revenue
Code, As Amended, And For Other Purposes, later expanded the jurisdiction of the
Commissioner and, correspondingly, that of the CTA, thus:

SEC. 4. Power of the Commissioner to Interpret Tax Laws and to Decide


Tax Cases. The power to interpret the provisions of this Code and other tax
laws shall be under the exclusive and original jurisdiction of the
Commissioner, subject to review by the Secretary of Finance.
The power to decide disputed assessments, refunds of internal revenue
taxes, fees or other charges, penalties imposed in relation thereto, or other
matters arising under this Code or other laws or portions thereof administered
by the Bureau of Internal Revenue is vested in the Commissioner, subject to
the exclusive appellate jurisdiction of the Court of Tax Appeals.

The latest statute dealing with the jurisdiction of the CTA is Republic Act No.
9282.[26] It provides:
SEC. 7. Section 7 of the same Act is hereby amended to read as follows:

Sec. 7. Jurisdiction. The CTA shall exercise:


(a) Exclusive appellate jurisdiction to review by appeal, as herein
provided:
(1) Decisions of the Commissioner of Internal Revenue in cases
involving disputed assessments, refunds of internal revenue taxes, fees
or other charges, penalties in relation thereto, or other matters arising
under the National Internal Revenue or other laws administered by the
Bureau of Internal Revenue;
(2) Inaction by the Commissioner of Internal Revenue in cases
involving disputed assessments, refunds of internal revenue taxes, fees
or other charges, penalties in relation thereto, or other matters arising
under the National Internal Revenue Code or other laws administered by

the Bureau of Internal Revenue, where the National Internal Revenue


Code provides a specific period of action, in which case the inaction shall
be deemed a denial;
(3) Decisions, orders or resolutions of the Regional Trial Courts in
local tax cases originally decided or resolved by them in the exercise of
their original or appellate jurisdiction;
xxx
(b) Jurisdiction over cases involving criminal offenses as herein provided:
(1) Exclusive original jurisdiction over all criminal offenses arising
from violations of the National Internal Revenue Code or Tariff and
Customs Code and other laws administered by the Bureau of Internal
Revenue or the Bureau of Customs: Provided, however, That offenses or
felonies mentioned in this paragraph where the principal amount of taxes
and fees, exclusive of charges and penalties, claimed is less than One
million pesos (P1,000,000.00) or where there is no specified amount
claimed shall be tried by the regular courts and the jurisdiction of the
CTA shall be appellate. Any provision of law or the Rules of Court to the
contrary notwithstanding, the criminal action and the corresponding civil
action for the recovery of civil liability for taxes and penalties shall at all
times be simultaneously instituted with, and jointly determined in the
same proceeding by the CTA, the filing of the criminal action being
deemed to necessarily carry with it the filing of the civil action, and no
right to reserve the filling of such civil action separately from the criminal
action will be recognized.
(2) Exclusive appellate jurisdiction in criminal offenses:
(a) Over appeals from the judgments, resolutions or orders of
the Regional Trial Courts in tax cases originally decided by them,
in their respected territorial jurisdiction.
(b) Over petitions for review of the judgments, resolutions or
orders of the Regional Trial Courts in the exercise of their
appellate jurisdiction over tax cases originally decided by the
Metropolitan Trial Courts, Municipal Trial Courts and Municipal
Circuit Trial Courts in their respective jurisdiction.
(c) Jurisdiction over tax collection cases as herein provided:
(1) Exclusive original jurisdiction in tax collection cases
involving final and executory assessments for taxes, fees,
charges and penalties: Provided, however, That collection
cases where the principal amount of taxes and fees, exclusive
of charges and penalties, claimed is less than One million
pesos (P1,000,000.00) shall be tried by the proper Municipal
Trial Court, Metropolitan Trial Court and Regional Trial Court.
(2) Exclusive appellate jurisdiction in tax collection cases:

(a) Over appeals from the judgments, resolutions or


orders of the Regional Trial Courts in tax collection cases
originally decided by them, in their respective territorial
jurisdiction.
(b) Over petitions for review of the judgments,
resolutions or orders of the Regional Trial Courts in the
exercise of their appellate jurisdiction over tax collection
cases originally decided by the Metropolitan Trial Courts,
Municipal Trial Courts and Municipal Circuit Trial Courts, in
their respective jurisdiction.

These laws have expanded the jurisdiction of the CTA. However, they did
not change the jurisdiction of the CTA to entertain an appeal only from a final
decision or assessment of the Commissioner, or in cases where the Commissioner
has not acted within the period prescribed by the NIRC. In the cases at bar, the
Commissioner has not issued an assessment of the tax liability of private
respondents.
Finally, we hold that contrary to private respondents stance, the doctrines
laid down in CIR v. Union Shipping Co. and Yabes v. Flojo are not applicable
to the cases at bar. In these earlier cases, the Commissioner already rendered an
assessment of the tax liabilities of the delinquent taxpayers, for which reason the
Court ruled that the filing of the civil suit for collection of the taxes due was a
final denial of the taxpayers request for reconsideration of the tax assessment.
IN VIEW WHEREOF, premises considered, judgment is rendered:
1.

In G.R. No. 120935, AFFIRMING the CA decision dated March 21,


1995, which set aside the Regional Trial Courts Order dated August 8,
1994, and REINSTATING Criminal Case Nos. 94-1842 to 94-1846 for
further proceedings before the trial court; and

2.

In G.R. No. 124557, REVERSING and SETTING ASIDE the Decision


of the Court of Appeals dated March 29, 1996, and ORDERING the
dismissal of C.T.A. Case No. 5075.

No costs.

SO ORDERED.
[G.R. No. 104171. February 24, 1999]
COMMISSIONER OF INTERNAL REVENUE, petitioner, vs. B.F. GOODRICH
PHILS., INC. (now SIME DARBY INTERNATIONAL TIRE CO., INC.) and
THE COURT OF APPEALS, respondents.
DECISION
PANGANIBAN, J.:
Notwithstanding the expiration of the five-year prescriptive period, may the
Bureau of Internal Revenue (BIR) still assess a taxpayer even after the latter has
already paid the tax due, on the ground that the previous assessment was
insufficient or based on a false return?
The Case

This is the main question raised before us in this Petition for Review
on Certiorari assailing the Decision[1] dated February 14, 1992, promulgated by the
Court of Appeals[2] in CA-GR SP No. 25100. The assailed Decision reversed the Court
of Tax Appeals (CTA)[3] which upheld the BIR commissioners assessments made
beyond the five-year statute of limitations.
The Facts

The facts are undisputed.[4] Private Respondent BF Goodrich Phils., Inc. (now
Sime Darby International Tire Co. Inc.), was an American-owned and controlled
corporation previous to July 3, 1974. As a condition for approving the manufacture
by private respondent of tires and other rubber products, the Central Bank of the
Philippines required that it should develop a rubber plantation. In compliance with
this requirement, private respondent purchased from the Philippine government in
1961, under the Public Land Act and the Parity Amendment to the 1935 Constitution,
certain parcels of land located in Tumajubong, Basilan, and there developed a rubber
plantation.
More than a decade later, on August 2, 1973, the justice secretary rendered an
opinion stating that, upon the expiration of the Parity Amendment on July 3, 1974,
the ownership rights of Americans over public agricultural lands, including the right
to dispose or sell their real estate, would be lost. On the basis of this Opinion,
private respondent sold to Siltown Realty Philippines, Inc. on January 21, 1974, its
Basilan landholding for P500,000 payable in installments. In accord with the terms
of the sale, Siltown Realty Philippines, Inc. leased the said parcels of land to private
respondent for a period of 25 years, with an extension of another 25 years at the
latters option.

Based on the BIRs Letter of Authority No. 10115 dated April 14, 1975, the books
and accounts of private respondent were examined for the purpose of determining
its tax liability for taxable year 1974. The examination resulted in the April 23, 1975
assessment of private respondent for deficiency income tax in the amount
of P6,005.35, which it duly paid.
Subsequently the BIR also issued Letters of Authority Nos. 074420 RR and
074421 RR and Memorandum Authority Reference No. 749157 for the purpose of
examining Siltowns business, income and tax liabilities. On the basis of this
examination, the BIR commissioner issued against private respondent on October
10, 1980, an assessment for deficiency in donors tax in the amount of P1,020,850,
in relation to the previously mentioned sale of its Basilan landholdings to
Siltown. Apparently, the BIR deemed the consideration for the sale insufficient, and
the difference between the fair market value and the actual purchase price a taxable
donation.
In a letter dated November 24, 1980, private respondent contested this
assessment. On April 9, 1981, it received another assessment dated March 16,
1981, which increased to P1,092,949 the amount demanded for the alleged
deficiency donors tax, surcharge, interest and compromise penalty.
Private respondent appealed the correctness and the legality of these last two
assessments to the CTA. After trial in due course, the CTA rendered its Decision
dated March 29, 1991, the dispositive portion of which reads as follows:
WHEREFORE, the decision of the Commissioner of Internal Revenue assessing
petitioner deficiency gift tax is MODIFIED and petitioner is ordered to pay the amount
of P1,311,179.01 plus 10% surcharge and 20% annual interest from March 16, 1981
until fully paid provided that the maximum amount that may be collected as interest
on delinquency shall in no case exceed an amount corresponding to a period of three
years pursuant to Section 130(b) (1) and (c) of the 1977 Tax Code, as amended by
P.D. No. 1705, which took effect on August 1, 1980.
SO ORDERED.[5]
Undaunted, private respondent elevated the matter to the Court of Appeals,
which reversed the CTA, as follows:
What is involved here is not a first assessment; nor is it one within the 5-year period
stated in Section 331 above. Since what is involved in this case is a multiple
assessment beyond the five-year period, the assessment must be based on the
grounds provided in Section 337, and not on Section 15 of the 1974 Tax
Code. Section 337 utilizes the very specific terms fraud,
irregularity, and mistake. Falsity does not appear to be included in this
enumeration. Falsity suffices for an assessment, which is a first assessment made
within the five-year period. When it is a subsequent assessment made beyond the

five-year period, then, it may be validly justified only by fraud, irregularity and
mistake on the part of the taxpayer.[6]
Hence, this Petition for Review under Rule 45 of the Rules of Court. [7]
The Issues

Before us, petitioner raises the following issues:


I
Whether or not petitioners right to assess herein deficiency donors tax has indeed
prescribed as ruled by public respondent Court of Appeals
II
Whether or not the herein deficiency donors tax assessment for 1974 is valid and in
accordance with law
Prescription is the crucial issue in the resolution of this case.
The Courts Ruling

The petition has no merit.


Main Issue: Prescription

The petitioner contends that the Court of Appeals erred in reversing the CTA on
the issue of prescription, because its ruling was based on factual findings that should
have been left undisturbed on appeal, in the absence of any showing that it had
been tainted with gross error or grave abuse of discretion. [8] The Court is not
persuaded.
True, the factual findings of the CTA are generally not disturbed on appeal when
supported by substantial evidence and in the absence of gross error or grave abuse
of discretion. However, the CTAs application of the law to the facts of this
controversy is an altogether different matter, for it involves a legal question. There
is a question of law when the issue is the application of the law to a given set of
facts. On the other hand, a question of fact involves the truth or falsehood of
alleged facts.[9] In the present case, the Court of Appeals ruled not on the truth or
falsity of the facts found by the CTA, but on the latters application of the law on
prescription.
Section 331 of the National Internal Revenue Code provides:

SEC. 331. Period of limitation upon assessment and collection. Except as provided
in the succeeding section, internal-revenue taxes shall be assessed within five years
after the return was filed, and no proceeding in court without assessment for the
collection of such taxes shall be begun after expiration of such period. For the
purposes of this section, a return filed before the last day prescribed by law for the
filing thereof shall be considered as filed on such last day: Provided, That this
limitation shall not apply to cases already investigated prior to the approval of this
Code.
Applying this provision of law to the facts at hand, it is clear that the October 16,
1980 and the March 1981 assessments were issued by the BIR beyond the five-year
statute of limitations. The Court has thoroughly studied the records of this case and
found no basis to disregard the five-year period of prescription. As succinctly
pronounced by the Court of Appeals:
The subsequent assessment made by the respondent Commissioner on October 10,
1980, modified by that of March 16, 1981, violates the law. Involved in this petition
is the income of the petitioner for the year 1974, the returns for which were required
to be filed on or before April 15 of the succeeding year. The returns for the year
1974 were duly filed by the petitioner, and assessment of taxes due for such year -including that on the transfer of properties on June 21, 1974 -- was made on April 13,
1975 and acknowledged by Letter of Confirmation No. 101155 terminating the
examination on this subject. The subsequent assessment of October 10, 1980
modified, by that of March 16, 1981, was made beyond the period expressly set in
Section 331 of the National Intenal Revenue Code xxx. [10]
Petitioner relies on the CTA ruling, the salient portion of which reads:
Falsity is what we have here, and for that matter, we hasten to add that the second
assessment (March 16, 1981) of the Commissioner was well-advised having been
made in contemplation of his power under Section 15 of the 1974 Code (now Section
16, of NIRC) to assess the proper tax on the best evidence obtainable when there is
reason to believe that a report of a taxpayer is false, incomplete or
erroneous. More, when there is falsity with intent to evade tax as in this case, the
ordinary period of limitation upon assessment and collection does not apply so that
contrary to the averment of petitioner, the right to assess respondent has not
prescribed.
What is the considered falsity? The transfer through sales of the parcels of land in
Tumajubong, Lamitan, Basilan in favor of Siltown Realty for the sum of P500,000.00
only whereas said lands had been sworn to under Presidential Decree No. 76 (Dec. 6,
1972) as having a value of P2,683,467 (P2,475, 467 + P207,700) (see Declaration of
Real Property form, p. 28, and p. 15, no. 5, BIR Record).[11]
For the purpose of safeguarding taxpayers from any unreasonable examination,
investigation or assessment, our tax law provides a statute of limitations in the

collection of taxes. Thus, the law on prescription, being a remedial measure, should
be liberally construed in order to afford such protection. [12] As a corollary, the
exceptions to the law on prescription should perforce be strictly construed.
Section 15 of the NIRC, on the other hand, provides that [w]hen a report
required by law as a basis for the assessment of any national internal revenue tax
shall not be forthcoming within the time fixed by law or regulation, or when there is
reason to believe that any such report is false, incomplete, or erroneous, the
Commissioner of Internal Revenue shall assess the proper tax on the best evidence
obtainable. Clearly, Section 15 does not provide an exception to the statute of
limitations on the issuance of an assessment, by allowing the initial assessment to
be made on the basis of the best evidence available. Having made its initial
assessment in the manner prescribed, the commissioner could not have been
authorized to issue, beyond the five-year prescriptive period, the second and the
third assessments under consideration before us.
Nor is petitioners claim of falsity sufficient to take the questioned assessments
out of the ambit of the statute of limitations. The relevant part of then Section 332
of the NIRC, which enumerates the exceptions to the period of prescription, provides:
SEC. 332. Exceptions as to period of limitation of assessment and collection of
taxes. -- (a) In the case of a false or fraudulent return with intent to evade a tax or of
a failure to file a return, the tax may be assessed, or a proceeding in court for the
collection of such tax may be begun without assessment, at any time within ten
years after the discovery of the falsity, fraud, or omission: xxx.
Petitioner insists that private respondent committed falsity when it sold the
property for a price lesser than its declared fair market value. This fact alone did not
constitute a false return which contains wrong information due to mistake,
carelessness or ignorance.[13] It is possible that real property may be sold for less
than adequate consideration for a bona fide business purpose; in such event, the
sale remains an arms length transaction. In the present case, the private
respondent was compelled to sell the property even at a price less than its market
value, because it would have lost all ownership rights over it upon the expiration of
the parity amendment. In other words, private respondent was attempting to
minimize its losses. At the same time, it was able to lease the property for 25 years,
renewable for another 25. This can be regarded as another consideration on the
price.
Furthermore, the fact that private respondent sold its real property for a price
less than its declared fair market value did not by itself justify a finding of false
return. Indeed, private respondent declared the sale in its 1974 return submitted to
the BIR.[14] Within the five-year prescriptive period, the BIR could have issued the
questioned assessment, because the declared fair market value of said property was
of public record. This it did not do, however, during all those five years. Moreover,
the BIR failed to prove that respondent's 1974 return had been filed

fraudulently. Equally. significant was its failure to prove respondent's intent to evade
the payment of the correct amount of tax.
Ineludibly, the BIR failed to show that private respondent's 1974 return was filed
fraudulently with intent to evade the payment of the correct amount of tax.
[15]
Moreover, even though a donor's tax, which is defined as "a tax on the privilege
of transmitting one's property or property rights to another or others without
adequate and full valuable consideration," [16] is different from capital gains tax, a tax
on the gain from the sale of the taxpayer's property forming part of capital assets,
[17]
the tax return filed by private respondent to report its income for the year 1974
was sufficient compliance with the legal requirement to file a return. In other words,
the fact that the sale transaction may have partly resulted in a donation does not
change the fact that private respondent already reported its income for 1974 by
filing an income tax return.
Since the BIR failed to demonstrate clearly that private respondent had filed a
fraudulent return with the intent to evade tax, or that it had failed to file a return at
all, the period for assessments has obviously prescribed. Such instances of
negligence or oversight on the part of the BIR cannot prejudice taxpayers,
considering that the prescriptive period was precisely intended to give them peace
of mind.
Based on the foregoing, a discussion of the validity and legality of the assailed
assessments has become moot and unnecessary.
WHEREFORE, the Petition for Review is DENIED and the assailed Decision of the
Court of Appeals is AFFIRMED. No costs.
SO ORDERED.
G.R. No. L-22492

September 5, 1967

BASILAN ESTATES, INC., petitioner,


vs.
THE COMMISSIONER OF INTERNAL REVENUE and THE COURT OF TAX
APPEALS, respondents.
Felix A. Gulfin and Antonio S. Alano for petitioner.
Office of the Solicitor General for respondents.

BENGZON, J.P., J.:


A Philippine corporation engaged in the coconut industry, Basilan Estates, Inc., with
principal offices in Basilan City, filed on March 24, 1954 its income tax returns for
1953 and paid an income tax of P8,028. On February 26, 1959, the Commissioner of

Internal Revenue, per examiners' report of February 19, 1959, assessed Basilan
Estates, Inc., a deficiency income tax of P3,912 for 1953 and P86,876.85 as 25%
surtax on unreasonably accumulated profits as of 1953 pursuant to Section 25 of the
Tax Code. On non-payment of the assessed amount, a warrant of distraint and levy
was issued but the same was not executed because Basilan Estates, Inc. succeeded
in getting the Deputy Commissioner of Internal Revenue to order the Director of the
district in Zamboanga City to hold execution and maintain constructive embargo
instead. Because of its refusal to waive the period of prescription, the corporation's
request for reinvestigation was not given due course, and on December 2, 1960,
notice was served the corporation that the warrant of distraint and levy would be
executed.
On December 20, 1960, Basilan Estates, Inc. filed before the Court of Tax Appeals a
petition for review of the Commissioner's assessment, alleging prescription of the
period for assessment and collection; error in disallowing claimed depreciations,
travelling and miscellaneous expenses; and error in finding the existence of
unreasonably accumulated profits and the imposition of 25% surtax thereon. On
October 31, 1963, the Court of Tax Appeals found that there was no prescription and
affirmed the deficiency assessment in toto.
On February 21, 1964, the case was appealed to Us by the taxpayer, upon the
following issues:
1. Has the Commissioner's right to collect deficiency income tax prescribed?
2. Was the disallowance of items claimed as deductible proper?
3. Have there been unreasonably accumulated profits? If so, should the 25% surtax
be imposed on the balance of the entire surplus from 1947-1953, or only for 1953?
4. Is the petitioner exempt from the penalty tax under Republic Act 1823 amending
Section 25 of the Tax Code?
PRESCRIPTION
There is no dispute that the assessment of the deficiency tax was made on February
26, 1959; but the petitioner claims that it never received notice of such assessment
or if it did, it received the notice beyond the five-year prescriptive period. To show
prescription, the annotation on the notice (Exhibit 10, No. 52, ACR, p. 54-A of the BIR
records) "No accompanying letter 11/25/" is advanced as indicative of the fact that
receipt of the notice was after March 24, 1959, the last date of the five-year period
within which to assess deficiency tax, since the original returns were filed on March
24, 1954.
Although the evidence is not clear on this point, We cannot accept this interpretation
of the petitioner, considering the presence of circumstances that lead Us to presume
regularity in the performance of official functions. The notice of assessment shows
the assessment to have been made on February 26, 1959, well within the five-year
period. On the right side of the notice is also stamped "Feb. 26, 1959" denoting

the date of release, according to Bureau of Internal Revenue practice. The


Commissioner himself in his letter (Exh. H, p. 84 of BIR records) answering
petitioner's request to lift, the warrant of distraint and levy, asserts that notice had
been sent to petitioner. In the letter of the Regional Director forwarding the case to
the Chief of the Investigation Division which the latter received on March 10, 1959
(p. 71 of the BIR records), notice of assessment was said to have been sent to
petitioner. Subsequently, the Chief of the Investigation Division indorsed on March
18, 1959 (p. 24 of the BIR records) the case to the Chief of the Law Division. There it
was alleged that notice was already sent to petitioner on February 26, 1959. These
circumstances pointing to official performance of duty must necessarily prevail over
petitioner's contrary interpretation. Besides, even granting that notice had been
received by the petitioner late, as alleged, under Section 331 of the Tax Code
requiring five years within which to assess deficiency taxes, the assessment is
deemed made when notice to this effect is released, mailed or sent by the Collector
to the taxpayer and it is not required that the notice be received by the taxpayer
within the aforementioned five-year period.1
ASSESSMENT
The questioned assessment is as follows:
Net Income per return
Add:
Over-claimed depreciation
Mis. expenses disallowed
Officer's travelling expenses
disallowed

P40,142.9
0
P10,500.4
9
6,759.17
2,300.4019,560.06

Net Income per Investigation


20% tax on P59,702.96
Less: Tax already assessed
Deficiency income tax
Add: Additional tax of 25% on
P347,507.01
Tax Due & Collectible

The Commissioner disallowed:


Over-claimed depreciation
P10,500.49
Miscellaneous expenses
6,759.17
Officer's travelling expenses
2,300.40

P59,702.9
6
11,940.00
8,028.00
P3,912.00
86,876.75
P90,788.7
5
======
===

DEDUCTIONS
A. Depreciation. Basilan Estates, Inc. claimed deductions for the depreciation of its
assets up to 1949 on the basis of their acquisition cost. As of January 1, 1950 it
changed the depreciable value of said assets by increasing it to conform with the
increase in cost for their replacement. Accordingly, from 1950 to 1953 it deducted
from gross income the value of depreciation computed on the reappraised value.
In 1953, the year involved in this case, taxpayer claimed the following depreciation
deduction:
Reappraised assets
New assets consisting of hospital building
and equipment
Total depreciation

P47,342.5
3
3,910.45
P51,252.9
8

Upon investigation and examination of taxpayer's books and papers, the


Commissioner of Internal Revenue found that the reappraised assets depreciated in
1953 were the same ones upon which depreciation was claimed in 1952. And for the
year 1952, the Commissioner had already determined, with taxpayer's concurrence,
the depreciation allowable on said assets to be P36,842.04, computed on their
acquisition cost at rates fixed by the taxpayer. Hence, the Commissioner pegged the
deductible depreciation for 1953 on the same old assets at P36,842.04 and
disallowed the excess thereof in the amount of P10,500.49.
The question for resolution therefore is whether depreciation shall be determined on
the acquisition cost or on the reappraised value of the assets.
Depreciation is the gradual diminution in the useful value of tangible property
resulting from wear and tear and normal obsolescense. The term is also applied to
amortization of the value of intangible assets, the use of which in the trade or
business is definitely limited in duration. 2 Depreciation commences with the
acquisition of the property and its owner is not bound to see his property gradually
waste, without making provision out of earnings for its replacement. It is entitled to
see that from earnings the value of the property invested is kept unimpaired, so that
at the end of any given term of years, the original investment remains as it was in
the beginning. It is not only the right of a company to make such a provision, but it is
its duty to its bond and stockholders, and, in the case of a public service corporation,
at least, its plain duty to the public.3 Accordingly, the law permits the taxpayer to
recover gradually his capital investment in wasting assets free from income
tax.4 Precisely, Section 30 (f) (1) which states:
(1)In general. A reasonable allowance for deterioration of property arising
out of its use or employment in the business or trade, or out of its not being
used: Provided, That when the allowance authorized under this subsection

shall equal the capital invested by the taxpayer . . . no further allowance shall
be made. . . .
allows a deduction from gross income for depreciation but limits the recovery to the
capital invested in the asset being depreciated.
The income tax law does not authorize the depreciation of an asset beyond its
acquisition cost. Hence, a deduction over and above such cost cannot be claimed
and allowed. The reason is that deductions from gross income are privileges, 5 not
matters of right.6 They are not created by implication but upon clear expression in
the law.7
Moreover, the recovery, free of income tax, of an amount more than the invested
capital in an asset will transgress the underlying purpose of a depreciation
allowance. For then what the taxpayer would recover will be, not only the acquisition
cost, but also some profit. Recovery in due time thru depreciation of investment
made is the philosophy behind depreciation allowance; the idea of profit on the
investment made has never been the underlying reason for the allowance of a
deduction for depreciation.
Accordingly, the claim for depreciation beyond P36,842.04 or in the amount of
P10,500.49 has no justification in the law. The determination, therefore, of the
Commissioner of Internal Revenue disallowing said amount, affirmed by the Court of
Tax Appeals, is sustained.
B. Expenses. The next item involves disallowed expenses incurred in 1953, broken
as follows:
Miscellaneous expenses
Officer's travelling expenses
Total

P6,759.17
2,300.40
P9,059.57

These were disallowed on the ground that the nature of these expenses could not be
satisfactorily explained nor could the same be supported by appropriate papers.
Felix Gulfin, petitioner's accountant, explained the P6,759.17 was actual expenses
credited to the account of the president of the corporation incurred in the interest of
the corporation during the president's trip to Manila (pp. 33-34 of TSN of Dec. 5,
1962); he stated that the P2,300.40 was the president's travelling expenses to and
from Manila as to the vouchers and receipts of these, he said the same were made
but got burned during the Basilan fire on March 30, 1962 (p. 40 of same TSN).
Petitioner further argues that when it sent its records to Manila in February, 1959,
the papers in support of these miscellaneous and travelling expenses were not
included for the reason that by February 9, 1959, when the Bureau of Internal
Revenue decided to investigate, petitioner had no more obligation to keep the same
since five years had lapsed from the time these expenses were incurred (p. 41 of
same TSN). On this ground, the petitioner may be sustained, for under Section 337
of the Tax Code, receipts and papers supporting such expenses need be kept by the

taxpayer for a period of five years from the last entry. At the time of the
investigation, said five years had lapsed. Taxpayer's stand on this issue is therefore
sustained.
UNREASONABLY ACCUMULATED PROFITS
Section 25 of the Tax Code which imposes a surtax on profits unreasonably
accumulated, provides:
Sec. 25. Additional tax on corporations improperly accumulating profits or
surplus (a) Imposition of tax. If any corporation, except banks, insurance
companies, or personal holding companies, whether domestic or foreign, is
formed or availed of for the purpose of preventing the imposition of the tax
upon its shareholders or members or the shareholders or members of another
corporation, through the medium of permitting its gains and profits to
accumulate instead of being divided or distributed, there is levied and
assessed against such corporation, for each taxable year, a tax equal to
twenty-five per centum of the undistributed portion of its accumulated profits
or surplus which shall be in addition to the tax imposed by section twentyfour, and shall be computed, collected and paid in the same manner and
subject to the same provisions of law, including penalties, as that
tax.1awphl.nt
The Commissioner found that in violation of the abovequoted section, petitioner had
unreasonably accumulated profits as of 1953 in the amount of P347,507.01, based
on the following circumstances (Examiner's Report pp. 62-68 of BIR records):
1. Strong financial position of the petitioner as of December 31, 1953. Assets
were P388,617.00 while the liabilities amounted to only P61,117.31 or a ratio
of 6:1.
2. As of 1953, the corporation had considerable capital adequate to meet the
reasonable needs of the business amounting to P327,499.69 (assets less
liabilities).
3. The P200,000 reserved for electrification of drier and mechanization and
the P50,000 reserved for malaria control were reverted to its surplus in 1953.
4. Withdrawal by shareholders, of large sums of money as personal loans.
5. Investment of undistributed earnings in assets having no proximate
connection with the business as hospital building and equipment worth
P59,794.72.
6. In 1953, with an increase of surplus amounting to P677,232.01, the capital
stock was increased to P500,000 although there was no need for such
increase.

Petitioner tried to show that in considering the surplus, the examiner did not take
into account the possible expenses for cultivation, labor, fertilitation, drainage,
irrigation, repair, etc. (pp. 235-237 of TSN of Dec. 7, 1962). As aptly answered by the
examiner himself, however, they were already included as part of the working capital
(pp. 237-238 of TSN of Dec. 7, 1962).
In the unreasonable accumulation of P347,507.01 are included P200,000 for
electrification of driers and mechanization and P50,000 for malaria control which
were reserved way back in 1948 (p. 67 of the BIR records) but reverted to the
general fund only in 1953. If there were any plans for these amounts to be used in
further expansion through projects, it did not appear in the records as was properly
indicated in 1948 when such amounts were reserved. Thus, while in 1948 it was
already clear that the money was intended to go to future projects, in 1953 upon
reversion to the general fund, no such intention was shown. Such reversion therefore
gave occasion for the Government to consider the same for tax purposes. The
P250,000 reverted to the general fund was sought to be explained as later used
elsewhere: "part of it in the Hilano Industries, Inc. in building the factory site and
buildings to house technical men . . . part of it was spent in the facilities for the
waterworks system and for industrialization of the coconut industry" (p. 117 of TSN
of Dec. 6, 1962). This is not sufficient explanation. Persuasive jurisprudence on the
matter such as those in the United States from where our tax law was derived, 8 has
it that: "In order to determine whether profits were accumulated for the reasonable
needs of the business or to avoid the surtax upon shareholders, the controlling
intention of the taxpayer is that which is manifested at the time of the accumulation,
not subsequently declared intentions which are merely the products of afterthought."9 The reversion here was made because the reserved amount was not
enough for the projects intended, without any intent to channel the same to some
particular future projects in mind.
Petitioner argues that since it has P560,717.44 as its expenses for the year 1953, a
surplus of P347,507.01 is not unreasonably accumulated. As rightly contended by
the Government, there is no need to have such a large amount at the beginning of
the following year because during the year, current assets are converted into cash
and with the income realized from the business as the year goes, these expenses
may well be taken care of (pp. 238 of TSN of Dec. 7, 1962). Thus, it is erroneous to
say that the taxpayer is entitled to retain enough liquid net assets in amounts
approximately equal to current operating needs for the year to cover "cost of goods
sold and operating expenses" for "it excludes proper consideration of funds
generated by the collection of notes receivable as trade accounts during the course
of the year."10 In fact, just because the fatal accumulations are less than 70% of the
annual operating expenses of the year, it does not mean that the accumulations are
reasonable as a matter of law."11
Petitioner tried to show that investments were made with Basilan Coconut Producers
Cooperative Association and Basilan Hospital (pp. 103-105 of TSN of Dec. 6, 1962)
totalling P59,794.72 as of December 31, 1953. This shows all the more the
unreasonable accumulation. As of December 31, 1953 already P59,794.72 was spent
yet as of that date there was still a surplus of P347,507.01.

Petitioner questions why the examiner covered the period from 1948-1953 when the
taxable year on review was 1953. The surplus of P347,507.01 was taken by the
examiner from the balance sheet of petitioner for 1953. To check the figure arrived
at, the examiner traced the accumulation process from 1947 until 1953, and
petitioner's figure stood out to be correct. There was no error in the process applied,
for previous accumulations should be considered in determining unreasonable
accumulations for the year concerned. "In determining whether accumulations of
earnings or profits in a particular year are within the reasonable needs of a
corporation, it is neccessary to take into account prior accumulations, since
accumulations prior to the year involved may have been sufficient to cover the
business needs and additional accumulations during the year involved would not
reasonably be necessary."12
Another factor that stands out to show unreasonable accumulation is the fact that
large amounts were withdrawn by or advanced to the stockholders. For the year
1953 alone these totalled P197,229.26. Yet the surplus of P347,507.01 was left as of
December 31, 1953. We find unacceptable petitioner's explanation that these were
advances made in furtherance of the business purposes of the petitioner. As
correctly held by the Court of Tax Appeals, while certain expenses of the corporation
were credited against these amounts, the unspent balance was retained by the
stockholders without refunding them to petitioner at the end of each year. These
advances were in fact indirect loans to the stockholders indicating the unreasonable
accumulation of surplus beyond the needs of the business.
ALLEGED EXEMPTION
Petitioner wishes to avail of the exempting proviso in Sec. 25 of the Internal Revenue
Code as amended by R.A. 1823, approved June 22, 1957, whereby accumulated
profits or surplus if invested in any dollar-producing or dollar-earning industry or in
the purchase of bonds issued by the Central Bank, may not be subject to the 25%
surtax. We have but to point out that the unreasonable accumulation was in 1953.
The exemption was by virtue of Republic Act 1823 which amended Sec. 25 only on
June 22, 1957 more than three years after the period covered by the assessment.
In resume, Basilan Estates, Inc. is liable for the payment of deficiency income tax
and surtax for the year 1953 in the amount of P88,977.42, computed as follows:
Net Income per return
Add: Over-claimed
depreciation

P40,142.90

Net income per finding

P50,643.39

10,500.49

20% tax on P50,643.39


P10,128.67
Less: Tax already assessed
8,028.00
Deficiency income tax
Add: 25% surtax on
P347,507.01

P2,100.67
86,876.75

Total tax due and collectible

P88,977.42
=======
====

WHEREFORE, the judgment appealed from is modified to the extent that petitioner is
allowed its deductions for travelling and miscellaneous expenses, but affirmed
insofar as the petitioner is liable for P2,100.67 as deficiency income tax for 1953 and
P86,876.75 as 25% surtax on the unreasonably accumulated profit of P347,507.01.
No costs. So ordered.
COMMISSIONER OF INTERNAL

G.R. No. 166387

REVENUE,
Petitioner,
Present:

PUNO, C.J., Chairperson,


CARPIO,
-versus-

CORONA,
AZCUNA and
LEONARDO-DE CASTRO, JJ.

ENRON SUBIC POWER


CORPORATION,
Respondent.

Promulgated:

January 19, 2009


x - - - - - - - - - - - - - - - - - - - - - - - - - - - - - - - - - - - - - - - - - - - - - - - - - - -x

RESOLUTION
CORONA, J.:

In this petition for review on certiorari under Rule 45 of the Rules of Court,
petitioner Commissioner of Internal Revenue (CIR) assails the November 24, 2004
decision[1]of the Court of Appeals (CA) annulling the formal assessment notice issued
by the CIR against respondent Enron Subic Power Corporation (Enron) for failure to
state the legal and factual bases for such assessment.
Enron, a domestic corporation registered with the Subic Bay Metropolitan
Authority as a freeport enterprise, [2] filed its annual income tax return for the year
1996 on April 12, 1997. It indicated a net loss of P7,684,948. Subsequently, the
Bureau of Internal Revenue, through a preliminary five-day letter, [3] informed it of a
proposed assessment of an alleged P2,880,817.25 deficiency income tax.[4] Enron
disputed the proposed deficiency assessment in its first protest letter. [5]

On May 26, 1999, Enron received from the CIR a formal assessment
notice[6] requiring it to pay the alleged deficiency income tax of P2,880,817.25 for
the taxable year 1996. Enron protested this deficiency tax assessment. [7]

Due to the non-resolution of its protest within the 180-day period, Enron filed
a petition for review in the Court of Tax Appeals (CTA). It argued that the deficiency
tax assessment disregarded the provisions of Section 228 of the National Internal
Revenue Code (NIRC), as amended, [8] and Section 3.1.4 of Revenue Regulations (RR)
No. 12-99[9] by not providing the legal and factual bases of the assessment. Enron
likewise questioned the substantive validity of the assessment. [10]

In a decision dated September 12, 2001, the CTA granted Enrons petition and
ordered the cancellation of its deficiency tax assessment for the year 1996. The CTA

reasoned that the assessment notice sent to Enron failed to comply with the
requirements of a valid written notice under Section 228 of the NIRC and RR No. 1299. The CIRs motion for reconsideration of the CTA decision was denied in a
resolution dated November 12, 2001.

The CIR appealed the CTA decision to the CA but the CA affirmed it. The CA
held that the audit working papers did not substantially comply with Section 228 of
the NIRC and RR No. 12-99 because they failed to show the applicability of the cited
law to the facts of the assessment. The CIR filed a motion for reconsideration but
this was deemed abandoned when he filed a motion for extension to file a petition
for review in this Court.

The CIR now argues that respondent was informed of the legal and factual
bases of the deficiency assessment against it.

We adopt in toto the findings of fact of the CTA, as affirmed by the CA.
In Compagnie Financiere Sucres et Denrees v. CIR, [11] we held:

We reiterate the well-established doctrine that as a matter of practice and


principle, [we] will not set aside the conclusion reached by an agency, like the
CTA, especially if affirmed by the [CA]. By the very nature of its function, it has
dedicated itself to the study and consideration of tax problems and has
necessarily developed an expertise on the subject, unless there has been an
abuse or improvident exercise of authority on its part, which is not present
here.

The CIR errs in insisting that the notice of assessment in question complied
with the requirements of the NIRC and RR No. 12-99.

A notice of assessment is:

[A] declaration of deficiency taxes issued to a [t]axpayer who fails to respond


to a Pre-Assessment Notice (PAN) within the prescribed period of time, or
whose reply to the PAN was found to be without merit. The Notice of
Assessment shall inform the [t]axpayer of this fact, and that the report of
investigation submitted by the Revenue Officer conducting the audit shall be
given due course.

The formal letter of demand calling for payment of the taxpayers deficiency
tax or taxes shall state the fact, the law, rules and regulations or
jurisprudence on which the assessment is based, otherwise the
formal letter of demand and the notice of assessment shall be void.
(emphasis supplied)[12]

Section 228 of the NIRC provides that the taxpayer shall be informed in writing
of the law and the facts on which the assessment is made. Otherwise, the
assessment is void. To implement the provisions of Section 228 of the NIRC, RR No.
12-99 was enacted. Section 3.1.4 of the revenue regulation reads:

3.1.4. Formal Letter of Demand and Assessment Notice. The formal


letter of demand and assessment notice shall be issued by the Commissioner
or his duly authorized representative.The letter of demand calling for
payment of the taxpayers deficiency tax or taxes shall state the
facts, the law, rules and regulations, or jurisprudence on which the
assessment is based, otherwise, the formal letter of demand and
assessment notice shall be void. The same shall be sent to the taxpayer
only by registered mail or by personal delivery. xxx (emphasis supplied)

It is clear from the foregoing that a taxpayer must be informed in writing of


the legal and factual bases of the tax assessment made against him. The use of the
word shall in these legal provisions indicates the mandatory nature of the
requirements laid down therein. We note the CTAs findings:

In [this] case, [the CIR] merely issued a formal assessment and


indicated therein the supposed tax, surcharge, interest and compromise
penalty due thereon. The Revenue Officers of the [the CIR] in the issuance of
the Final Assessment Notice did not provide Enron with the written bases of
the law and facts on which the subject assessment is based. [The CIR] did not
bother to explain how it arrived at such an assessment. Moreso, he failed to
mention the specific provision of the Tax Code or rules and regulations which
were not complied with by Enron.[13]

Both the CTA and the CA concluded that the deficiency tax assessment merely
itemized the deductions disallowed and included these in the gross income. It also
imposed the preferential rate of 5% on some items categorized by Enron as costs.
The legal and factual bases were, however, not indicated.

The CIR insists that an examination of the facts shows that Enron was properly
apprised of its tax deficiency. During the pre-assessment stage, the CIR advised
Enrons representative of the tax deficiency, informed it of the proposed tax
deficiency assessment through a preliminary five-day letter and furnished Enron a
copy of the audit working paper[14] allegedly showing in detail the legal and factual

bases of the assessment. The CIR argues that these steps sufficed to inform Enron of
the laws and facts on which the deficiency tax assessment was based.

We disagree. The advice of tax deficiency, given by the CIR to an employee of


Enron, as well as the preliminary five-day letter, were not valid substitutes for the
mandatory notice in writing of the legal and factual bases of the assessment. These
steps were mere perfunctory discharges of the CIRs duties in correctly assessing a
taxpayer.[15] The requirement for issuing a preliminary or final notice, as the case
may be, informing a taxpayer of the existence of a deficiency tax assessment is
markedly different from the requirement of what such notice must contain. Just
because the CIR issued an advice, a preliminary letter during the pre-assessment
stage and a final notice, in the order required by law, does not necessarily mean that
Enron was informed of the law and facts on which the deficiency tax assessment was
made.

The law requires that the legal and factual bases of the assessment be stated
in the formal letter of demand and assessment notice. Thus, such cannot be
presumed. Otherwise, the express provisions of Article 228 of the NIRC and RR No.
12-99 would be rendered nugatory. The alleged factual bases in the advice,
preliminary letter and audit working papers did not suffice. There was no going
around the mandate of the law that the legal and factual bases of the assessment be
stated in writing in the formal letter of demand accompanying the assessment
notice.

We note that the old law merely required that the taxpayer be notified of the
assessment made by the CIR. This was changed in 1998 and the taxpayer must now

be informed not only of the law but also of the facts on which the assessment is
made.[16] Such amendment is in keeping with the constitutional principle that no
person shall be deprived of property without due process. [17] In view of the absence
of a fair opportunity for Enron to be informed of the legal and factual bases of the
assessment against it, the assessment in question was void. We reiterate our ruling
in Reyes v. Almanzor, et al.:[18]

Verily, taxes are the lifeblood of the Government and so should


be collected without unnecessary hindrance. However, such collection
should be made in accordance with law as any arbitrariness will negate
the very reason for the Government itself.
WHEREFORE, the petition is hereby DENIED. The November 24, 2004
decision of the Court of Appeals is AFFIRMED.

No costs.

SO ORDERED.
COMMISSIONER OF INTERNAL

G.R. No. 166387

REVENUE,
Petitioner,
Present:

PUNO, C.J., Chairperson,


CARPIO,
-versus-

CORONA,
AZCUNA and
LEONARDO-DE CASTRO, JJ.

ENRON SUBIC POWER


CORPORATION,
Respondent.

Promulgated:

January 19, 2009


x - - - - - - - - - - - - - - - - - - - - - - - - - - - - - - - - - - - - - - - - - - - - - - - - - - -x

RESOLUTION
CORONA, J.:

In this petition for review on certiorari under Rule 45 of the Rules of Court,
petitioner Commissioner of Internal Revenue (CIR) assails the November 24, 2004
decision[1]of the Court of Appeals (CA) annulling the formal assessment notice issued
by the CIR against respondent Enron Subic Power Corporation (Enron) for failure to
state the legal and factual bases for such assessment.
Enron, a domestic corporation registered with the Subic Bay Metropolitan
Authority as a freeport enterprise, [2] filed its annual income tax return for the year
1996 on April 12, 1997. It indicated a net loss of P7,684,948. Subsequently, the
Bureau of Internal Revenue, through a preliminary five-day letter, [3] informed it of a
proposed assessment of an alleged P2,880,817.25 deficiency income tax.[4] Enron
disputed the proposed deficiency assessment in its first protest letter. [5]

On May 26, 1999, Enron received from the CIR a formal assessment
notice[6] requiring it to pay the alleged deficiency income tax of P2,880,817.25 for
the taxable year 1996. Enron protested this deficiency tax assessment. [7]

Due to the non-resolution of its protest within the 180-day period, Enron filed
a petition for review in the Court of Tax Appeals (CTA). It argued that the deficiency
tax assessment disregarded the provisions of Section 228 of the National Internal
Revenue Code (NIRC), as amended, [8] and Section 3.1.4 of Revenue Regulations (RR)
No. 12-99[9] by not providing the legal and factual bases of the assessment. Enron
likewise questioned the substantive validity of the assessment. [10]

In a decision dated September 12, 2001, the CTA granted Enrons petition and
ordered the cancellation of its deficiency tax assessment for the year 1996. The CTA
reasoned that the assessment notice sent to Enron failed to comply with the
requirements of a valid written notice under Section 228 of the NIRC and RR No. 1299. The CIRs motion for reconsideration of the CTA decision was denied in a
resolution dated November 12, 2001.

The CIR appealed the CTA decision to the CA but the CA affirmed it. The CA
held that the audit working papers did not substantially comply with Section 228 of
the NIRC and RR No. 12-99 because they failed to show the applicability of the cited
law to the facts of the assessment. The CIR filed a motion for reconsideration but
this was deemed abandoned when he filed a motion for extension to file a petition
for review in this Court.

The CIR now argues that respondent was informed of the legal and factual
bases of the deficiency assessment against it.

We adopt in toto the findings of fact of the CTA, as affirmed by the CA.
In Compagnie Financiere Sucres et Denrees v. CIR, [11] we held:

We reiterate the well-established doctrine that as a matter of practice and


principle, [we] will not set aside the conclusion reached by an agency, like the
CTA, especially if affirmed by the [CA]. By the very nature of its function, it has
dedicated itself to the study and consideration of tax problems and has
necessarily developed an expertise on the subject, unless there has been an
abuse or improvident exercise of authority on its part, which is not present
here.

The CIR errs in insisting that the notice of assessment in question complied
with the requirements of the NIRC and RR No. 12-99.

A notice of assessment is:

[A] declaration of deficiency taxes issued to a [t]axpayer who fails to respond


to a Pre-Assessment Notice (PAN) within the prescribed period of time, or
whose reply to the PAN was found to be without merit. The Notice of
Assessment shall inform the [t]axpayer of this fact, and that the report of
investigation submitted by the Revenue Officer conducting the audit shall be
given due course.

The formal letter of demand calling for payment of the taxpayers deficiency
tax or taxes shall state the fact, the law, rules and regulations or
jurisprudence on which the assessment is based, otherwise the
formal letter of demand and the notice of assessment shall be void.
(emphasis supplied)[12]

Section 228 of the NIRC provides that the taxpayer shall be informed in writing
of the law and the facts on which the assessment is made. Otherwise, the
assessment is void. To implement the provisions of Section 228 of the NIRC, RR No.
12-99 was enacted. Section 3.1.4 of the revenue regulation reads:

3.1.4. Formal Letter of Demand and Assessment Notice. The formal


letter of demand and assessment notice shall be issued by the Commissioner
or his duly authorized representative.The letter of demand calling for
payment of the taxpayers deficiency tax or taxes shall state the
facts, the law, rules and regulations, or jurisprudence on which the
assessment is based, otherwise, the formal letter of demand and
assessment notice shall be void. The same shall be sent to the taxpayer
only by registered mail or by personal delivery. xxx (emphasis supplied)

It is clear from the foregoing that a taxpayer must be informed in writing of


the legal and factual bases of the tax assessment made against him. The use of the
word shall in these legal provisions indicates the mandatory nature of the
requirements laid down therein. We note the CTAs findings:

In [this] case, [the CIR] merely issued a formal assessment and


indicated therein the supposed tax, surcharge, interest and compromise
penalty due thereon. The Revenue Officers of the [the CIR] in the issuance of
the Final Assessment Notice did not provide Enron with the written bases of
the law and facts on which the subject assessment is based. [The CIR] did not
bother to explain how it arrived at such an assessment. Moreso, he failed to
mention the specific provision of the Tax Code or rules and regulations which
were not complied with by Enron.[13]

Both the CTA and the CA concluded that the deficiency tax assessment merely
itemized the deductions disallowed and included these in the gross income. It also
imposed the preferential rate of 5% on some items categorized by Enron as costs.
The legal and factual bases were, however, not indicated.

The CIR insists that an examination of the facts shows that Enron was properly
apprised of its tax deficiency. During the pre-assessment stage, the CIR advised
Enrons representative of the tax deficiency, informed it of the proposed tax
deficiency assessment through a preliminary five-day letter and furnished Enron a
copy of the audit working paper[14] allegedly showing in detail the legal and factual
bases of the assessment. The CIR argues that these steps sufficed to inform Enron of
the laws and facts on which the deficiency tax assessment was based.

We disagree. The advice of tax deficiency, given by the CIR to an employee of


Enron, as well as the preliminary five-day letter, were not valid substitutes for the
mandatory notice in writing of the legal and factual bases of the assessment. These
steps were mere perfunctory discharges of the CIRs duties in correctly assessing a
taxpayer.[15] The requirement for issuing a preliminary or final notice, as the case
may be, informing a taxpayer of the existence of a deficiency tax assessment is
markedly different from the requirement of what such notice must contain. Just
because the CIR issued an advice, a preliminary letter during the pre-assessment
stage and a final notice, in the order required by law, does not necessarily mean that
Enron was informed of the law and facts on which the deficiency tax assessment was
made.

The law requires that the legal and factual bases of the assessment be stated
in the formal letter of demand and assessment notice. Thus, such cannot be
presumed. Otherwise, the express provisions of Article 228 of the NIRC and RR No.
12-99 would be rendered nugatory. The alleged factual bases in the advice,
preliminary letter and audit working papers did not suffice. There was no going
around the mandate of the law that the legal and factual bases of the assessment be
stated in writing in the formal letter of demand accompanying the assessment
notice.

We note that the old law merely required that the taxpayer be notified of the
assessment made by the CIR. This was changed in 1998 and the taxpayer must now
be informed not only of the law but also of the facts on which the assessment is
made.[16] Such amendment is in keeping with the constitutional principle that no
person shall be deprived of property without due process. [17] In view of the absence
of a fair opportunity for Enron to be informed of the legal and factual bases of the
assessment against it, the assessment in question was void. We reiterate our ruling
in Reyes v. Almanzor, et al.:[18]

Verily, taxes are the lifeblood of the Government and so should


be collected without unnecessary hindrance. However, such collection
should be made in accordance with law as any arbitrariness will negate
the very reason for the Government itself.
WHEREFORE, the petition is hereby DENIED. The November 24, 2004
decision of the Court of Appeals is AFFIRMED.

No costs.

SO ORDERED.
COMMISSIONER OF INTERNAL
REVENUE,
Petitioner,

G.R. No. 167560


Present:
YNARES-SANTIAGO, J.,
Chairperson,
AUSTRIA-MARTINEZ,
CHICO-NAZARIO,
NACHURA, and
REYES, JJ.

- versus -

DOMINADOR MENGUITO,
Promulgated:
Respondent.
September 17, 2008
x----------------------------------------------------------x

DECISION

AUSTRIA-MARTINEZ, J.:

Before the Court is a Petition for Review on Certiorari under Rule 45 of the Rules of
Court, assailing the March 31, 2005 Decision [1] of the Court of Appeals (CA) which reversed
and set aside the Court of Tax Appeals (CTA) April 2, 2002 Decision[2] and October 10, 2002
Resolution[3] ordering Dominador Menguito (respondent) to pay the Commissioner of Internal
Revenue (petitioner) deficiency income and percentage taxes and delinquency interest.
Based on the Joint Stipulation of Facts and Admissions [4] of the parties, the CTA
summarized the factual and procedural antecedents of the case, the relevant portions of
which read:
Petitioner Dominador Menguito [herein respondent] is a Filipino citizen, of legal
age, married to Jeanne Menguito and is engaged in the restaurant and/or cafeteria
business. For the years 1991, 1992 and 1993, its principal place of business was at
Gloriamaris, CCP Complex, Pasay City and later transferred to Kalayaan Bar (Copper
Kettle
Cafeteria
Specialist
or
CKCS),
Departure
Area, Ninoy Aquino InternationalAirport, Pasay City. During the same years, he also
operated a branch at Club John Hay, Baguio City carrying the business name of
Copper Kettle Cafeteria Specialist (Joint Stipulation of Facts and Admissions, p. 133,
CTA records).
xxxx

Subsequently, BIR Baguio received information that Petitioner


[herein respondent] has undeclared income from Texas Instruments and
Club John Hay, prompting the BIR to conduct another investigation.
Through a letter dated July 28, 1997, Spouses Dominador Menguito and
Jeanne Menguito (Spouses Menguito) were informed by the Assessment
Division of the said office that they have underdeclared sales
totaling P48,721,555.96 (Exhibit 11, p. 83, BIR records). This was followed
by a Preliminary Ten (10) Day Letter dated August 11, 1997, informing
Petitioner [herein respondent] that in the investigation of his 1991, 1992
and 1993 income, business and withholding tax case, it was found out that
there is still due from him the total sum of P34,193,041.55 as deficiency
income and percentage tax.
On September 2, 1997, the assessment notices subject of the
instant petition were issued. These were protested by Ms. Jeanne
Menguito, through a letter dated September 28, 1997 (Exhibit 14, p. 112,
BIR Records), on the ground that the 40% deduction allowed on their
computed gross revenue, is unrealistic. Ms. Jeanne Menguito requested for
a period of thirty (30) days within which to coordinate with the BIR
regarding the contested assessment.
On October 10, 1997, BIR Baguio replied, informing the Spouses Menguito
that the source of assessment was not through the disallowance of claimed expenses
but on data received from Club John Hay and Texas Instruments Phils., Inc. Said letter
gave the spouses ten (10) days to present evidence (Exhibit 15, p. 110, BIR Records).
In an effort to clear an alleged confusion regarding Copper Kettle
Cafeteria Specialist (CKCS) being a sole proprietorship owned by the
Spouses, and Copper Kettle Catering Services, Inc. (CKCS, Inc.) being a
corporation with whom Texas Instruments and Club John Hay entered into a
contract, Petitioner [respondent] submitted to BIR Baguio a photocopy of
the SEC Registration of Copper Kettle Catering Services, Inc. on March 23,
1999 (pp. 134-141, BIR Records).
On April 12, 1999, BIR Baguio wrote a letter to Spouses Menguito,
informing the latter that a reinvestigation or reconsideration cannot be given due
course by the mere submission of an uncertified photocopy of the Certificate of
Incorporation. Thus, it avers that the amendment issued is still valid and enforceable.
On May 26, 1999, Petitioner [respondent] filed the present case, praying
for the cancellation and withdrawal of the deficiency income tax and percentage tax
assessments on account of prescription, whimsical factual findings, violation of
procedural due process on the issuance of assessment notices, erroneous address of
notices and multiple credit/ investigation by the Respondent [petitioner] of Petitioner's
[respondents] books of accounts and other related records for the same tax year.
Instead of filing an Answer, Respondent [herein petitioner] moved to
dismiss the instant petition on July 1, 1999, on the ground of lack of
jurisdiction. According to Respondent [petitioner], the assessment had long become

final and executory when Petitioner [respondent] failed to comply with the letter
dated October 10, 1997.
Petitioner opposed said motion on July 21, 1999, claiming that the final
decision on Petitioner's [respondents] protest is the April 12, 1999 letter of the
Baguio Regional Office; therefore, the filing of the action within thirty (30) days from
receipt of the said letter was seasonably filed. Moreover, Petitioner [respondent]
asserted that granting that the April 12, 1999 letter in question could not be
construed to mean as a denial or final decision of the protest, still Petitioner's
[respondents] appeal was timely filed since Respondent [petitioner] issued a Warrant
of Distraint and/or Levy against the Petitioner [respondent] on May 3, 1999, which
warrant constituted a final decision of the Respondent [petitioner] on the protest of
the taxpayer.
On September 3, 1999, this Court denied Respondent's [petitioners]
'Motion to Dismiss' for lack of merit.
Respondent [petitioner] filed his Answer on September 24, 1999, raising the
following Special and Affirmative Defenses:
xxxx
5.
Investigation disclosed that for taxable years 1991, 1992 and 1993,
petitioner [respondent] filed false or fraudulent income and percentage tax
returns with intent to evade tax by under declaring his sales.
6. The alleged duplication of investigation of petitioner [respondent] by the BIR
Regional Office in Baguio City and by the Revenue District Office in Pasay City
is justified by the finding of fraud on the part of the petitioner [respondent],
which is an exception to the provision in the Tax Code that the examination
and inspection of books and records shall be made only once in a taxable year
(Section 235, Tax Code). At any rate, petitioner [respondent], in a letter
dated July 18, 1994, waived his right to the consolidation of said investigation.
7.
The aforementioned falsity or fraud was discovered on August
5, 1997. The assessments were issued on September 2, 1997, or
within ten (10) years from the discovery of such falsity or fraud
(Section 223, Tax Code). Hence, the assessments have not
prescribed.
8.
Petitioner's [respondents] allegation that the assessments
were not properly addressed is rendered moot and academic by his
acknowledgment in his protest letter dated September 28, 1997 that
he received the assessments.
9.
Respondent [petitioner] complied with the provisions of
Revenue Regulations No. 12-85 by informing petitioner [respondent]
of the findings of the investigation in letters dated July 28,
1997 and August 11, 1997 prior to the issuance of the assessments.

10. Petitioner [respondent] did not allege in his administrative


protest that there was a duplication of investigation, that the
assessments have prescribed, that they were not properly
addressed, or that the provisions of Revenue Regulations No. 12-85
were not observed. Not having raised them in the administrative
level, petitioner [respondent] cannot raise the same for the first time
on appeal (Aguinaldo Industries Corp. vs. Commissioner of Internal
Revenue, 112 SCRA 136).
11. The assessments were issued in accordance with law and regulations.
12. All presumptions are in favor of the correctness of tax assessments (CIR
vs. Construction Resources of Asia, Inc., 145 SCRA 67), and the burden to prove
otherwise is upon petitioner [respondent].[5] (Emphasis supplied)
On April 2, 2002, the CTA rendered a Decision, the dispositive portion of which reads:
Accordingly, Petitioner [herein respondent] is ORDERED to PAY the
Respondent [herein petitioner] the amount of P11,333,233.94 and P2,573,655.82 as
deficiency income and percentage tax liabilities, respectively for taxable years 1991,
1992 and 1993 plus 20% delinquency interest from October 2, 1997 until full payment
thereof.
SO ORDERED.[6]
Respondent filed a motion for reconsideration but the CTA denied the same in its
Resolution of October 10, 2002.[7]
Through a Petition for Review[8] filed with the CA, respondent questioned the CTA
Decision and Resolution mainly on the ground that Copper Kettle Catering Services, Inc.
(CKCS, Inc.) was a separate and distinct entity from Copper Kettle Cafeteria Specialist (CKCS);
the sales and revenues of CKCS, Inc. could not be ascribed to CKCS; neither may the taxes
due from one, charged to the other; nor the notices to be served on the former, coursed
through the latter.[9] Respondent cited the Joint Stipulation in which petitioner acknowledged
that its (respondents) business was called Copper Kettle Cafeteria Specialist, not Copper
Kettle Catering Services, Inc.[10]
Based on the unrefuted[11] CTA summary, the CA rendered the Decision assailed
herein, the dispositive portion of which reads:
WHEREFORE, the instant petition is GRANTED. Reversing the assailed
Decision dated April 2, 2002 and Resolution dated October 10, 2002, the deficiency
income tax and percentage income tax assessments against petitioner in the

amounts of P11,333,233.94 and P2,573,655.82 for taxable years 1991, 1992 and
1993 plus the 20% delinquency interest thereon are annulled.
SO ORDERED.[12]
Petitioner filed a motion for reconsideration but the CA denied the same in
its October 10, 2002 Resolution.[13]
Hence, herein recourse to the Court for the reversal of the CA decision and resolution
on the following grounds:
I
The Court of Appeals erred in reversing the decision of the Court of Tax Appeals and in
holding that Copper Kettle Cafeteria Specialist owned by respondent and Copper
Kettle Catering Services, Inc. owned and managed by respondent's wife are not one
and the same.
II
The Court of Appeals erred in holding that respondent was denied due process for
failure of petitioner to validly serve respondent with the post-reporting and preassessment notices as required by law.
On the first issue, the CTA has ruled that CKCS, Inc. and CKCS are one and the
same corporation because [t]he contract between Texas Instruments and Copper Kettle was
signed by petitioners [respondents] wife, Jeanne Menguito as proprietress.[14]
However, the CA reversed the CTA on these grounds:
Respondents [herein petitioners] allegation that Copper Kettle Catering
Services, Inc. and Copper Kettle Cafeteria Specialists are not distinct entities and that
the under-declared sales/revenues of Copper Kettle Catering Services, Inc. pertain to
Copper Kettle Cafeteria Specialist are belied by the evidence on record. In the Joint
Stipulation of Facts submitted before the tax court, respondent [petitioner] admitted
that petitioners [herein respondents] business name is Copper Kettle Cafeteria
Specialist.
Also, the Certification of Club John Hay and Letter dated July 9, 1997 of Texas
Instruments both addressed to respondent indicate that these companies transacted
with Copper Kettle Catering Services, Inc., owned and managed by JEANNE G.
MENGUITO, NOT petitioner Dominador Menguito. The alleged under-declared sales
income subject of the present assessments were shown to have been earned
by Copper Kettle Catering Services, Inc. in its commercial transaction with Texas
Instruments and Camp John Hay; NOT by petitioners dealing with these companies.
In fact, there is nothing on record which shows that Texas Instruments and Camp John
Hay conducted business relations with Copper Kettle Cafeteria Specialist, owned by
herein petitioner Dominador Menguito. In the absence, therefore, of clear and

convincing evidence showing that Copper Kettle Cafeteria Specialist and Copper
Kettle Catering Services, Inc. are one and the same, respondent can NOT validly
impute alleged underdeclared sales income earned by Copper Kettle Catering
Services, Inc. as sales income of Copper Kettle Cafeteria Specialist.[15] (Emphasis
supplied)
Respondent is adamant that the CA is correct. Many times in the past, the BIR had
treated CKCS separately from CKCS, Inc.: from May 1994 to June 1995, the BIR sent audit
teams to examine the books of account and other accounting records of CKCS, and based on
said audits, respondent was held liable for deficiency taxes, all of which he had paid.
[16]

Moreover, the certifications[17] issued by Club John Hay and Texas Instruments identify the

concessionaire

operating

therein

as CKCS, Inc., owned and managed by his spouse Jeanne Menguito, and not
CKCS.[18]
Petitioner impugns the findings of the CA, claiming that these are contradicted by
evidence on record consisting of a reply to the September 2, 1997 assessment notice of BIR
Baguio which Jeanne Menguito wrote on September 28, 1997, to wit:
We are in receipt of the assessment notice you have sent us,
dated September 2, 1997. Having taken hold of the same only now following our
travel overseas, we were not able to respond immediately and manifest
our protest. Also, with the impending termination of our businesses at 19th Tee,
Club John Hay and at Texas Instruments, Loakan, Baguio City, we have
already started the transfer of our records and books in Baguio City to
Manila that we will need more time to review and sort the records that may have to
be presented relative to the assessment x x x.[19] (Emphasis supplied)
Petitioner insists that said reply confirms that the assessment notice is directed against the
businesses which she and her husband, respondent herein, own and operate at Club John
Hay and Texas Instruments, and establishes that she is protesting said notice not just for
herself but also for respondent.[20]
Moreover, petitioner argues that if it were true that CKCS, Inc. and CKCS are separate
and distinct entities, respondent could have easily produced the articles of incorporation of
CKCS, Inc.; instead, what respondent presented was merely a photocopy of the incorporation
articles.[21] Worse, petitioner adds, said document was not offered in evidence before the
CTA, but was presented only before the CA.[22]

Petitioner further insists that CKCS, Inc. and CKCS are merely employing the fiction of
their separate corporate existence to evade payment of proper taxes; that the CTA saw
through their ploy and rightly disregarded their corporate individuality, treating them instead
as one taxable entity with the same tax base and liability; [23] and that the CA should have
sustained the CTA.[24]
In effect, petitioner would have the Court resolve a purely factual issue [25] of whether
or not there is substantial evidence that CKCS, Inc. and CKCS are one and the same taxable
entity.
As a general rule, the Court does not venture into a trial of facts in proceedings under
Rule 45 of the Rules of Courts, for its only function is to review errors of law. [26] The Court
declines to inquire into errors in the factual assessment of the CA, for the latters findings are
conclusive, especially when these are synonymous to those of the CTA.[27] But when the CA
contradicts the factual findings of the CTA, the Court deems it necessary to determine
whether the CA was justified in doing so, for one basic rule in taxation is that the factual
findings of the CTA, when supported by substantial evidence, will not be disturbed on appeal
unless it is shown that the CTA committed gross error in its appreciation of facts.[28]
The Court finds that the CA gravely erred when it ignored the substantial evidence
on record and reversed the CTA.
In a number of cases, the Court has shredded the veil of corporate identity and ruled
that where a corporation is merely an adjunct, business conduit or alter ego of another
corporation or when they practice fraud on our internal revenue laws, [29] the fiction of their
separate and distinct corporate identities shall be disregarded, and both entities treated as
one taxable person, subject to assessment for the same taxable transaction.
The Court considers the presence of the following circumstances, to wit: when the
owner of one directs and controls the operations of the other, and the payments effected or
received by one are for the accounts due from or payable to the other; [30] or when the
properties or products of one are all sold to the other, which in turn immediately sells them to
the public,[31] as substantial evidence in support of the finding that the two are actually one
juridical taxable personality.

In the present case, overwhelming evidence supports the CTA in disregarding the
separate identity of CKCS, Inc. from CKCS and in treating them as one taxable entity.
First, in respondents Petition for Review before the CTA, he expressly admitted that he
is engaged in restaurant and/or cafeteria business and that [i]n 1991, 1992 and 1993,
he also operated a branch at Club John Hay, Baguio City with a business name of
Copper Kettle Cafeteria Specialist.[32] Respondent repeated such admission in the Joint
Stipulation.[33] And then in Exhibit 1[34] for petitioner, a July 18, 1994 letter sent by Jeanne
Menguito to BIR, Baguio City, she stated thus:
in connection with the investigation of Copper Kettle Cafeteria Specialist which
is located at 19th Tee Club John Hay, Baguio City under letter of authority nos.
0392897, 0392898, and 0392690 dated May 16, 1994, investigating myincome,
business, and withholding taxes for the years 1991, 1992, and 1993. [35] (Emphasis
supplied)
Jeanne Menguito signed the letter as proprietor of Copper Kettle Cafeteria Specialist.[36]
Related to Exhibit 1 is petitioner's Exhibit 14, which is another letter
dated September 28, 1997, in which Jeanne Menguito protested the September 2,
1997 assessment notices directed at Copper Kettle Cafeteria Specialist and referred to the
latter as our business at 19th Tee Club John Hay and at Texas Instruments.[37] Taken along
with the Joint Stipulation, Exhibits A through C and the August 3, 1993 Certification of
Camp John Hay, Exhibits 1 and 14, confirm that respondent, together with his spouse
Jeanne Menguito, own, operate and manage a branch of Copper Kettle Cafeteria Specialist,
also called Copper Kettle Catering Services at Camp John Hay.
Moreover, in Exhibits A to A-1,[38] Exhibits B to B-1[39] and Exhibits C to C1[40] which are lists of concessionaires that operated in Club John Hay in 1992, 1993 and
1991, respectively,[41] it appears that there is no outlet with the name Copper Kettle
Cafeteria Specialist as claimed by respondent. The name that appears in the lists is
19th TEE CAFETERIA (Copper Kettle, Inc.). However, in the light of the express admission of
respondent that in 1991, 1992 and 1993, he operated a branch called Copper Kettle
Cafeteria Specialist in Club John Hay, the entries in Exhibits A through C could only mean
that said branch refers to 19th Tee Cafeteria (Copper Kettle, Inc.). There is no evidence
presented by respondent that contradicts this conclusion.

In addition, the August 9, 1993 Certification issued by Club John Hay that COPPER
KETTLE CATERING SERVICES owned and managed by MS. JEANNE G. MENGUITO is a
concessionaire in John Hay since July 1991 up to the present and is operating the outlet 19TH
TEE CAFETERIA AND THE TEE BAR[42] convincingly establishes that respondent's branch
which he refers to as Copper Kettle Cafeteria Specialist at Club John Hay also appears in the
latter's records as Copper Kettle Catering Services with an outlet called 19 th Tee Cafeteria
and The Tee Bar.
Second, in Exhibit 8[43] and Exhibit E,[44] Texas Instruments identified the
concessionaire operating its canteen as Copper Kettle Catering Services, Inc. [45] and/or
COPPER KETTLE CAFETERIA SPECIALIST SVCS.[46] It being settled that respondent's
Copper Kettle Cafeteria Specialist is also known as Copper Kettle Catering Services, and
that respondent and Jeanne Menguito both own, manage and act as proprietors of the
business, Exhibit 8 and Exhibit E further establish that, through said business,
respondent also had taxable transactions with Texas Instruments.
In view of the foregoing facts and circumstances, the Articles of Incorporation of
CKCS, Inc. -- a certified true copy of which respondent attached only to his Reply filed with the
CA[47] -- cannot insulate it from scrutiny of its real identity in relation to CKCS. It is noted that
said Articles of Incorporation of CKCS, Inc. was issued in 1989, but documentary evidence
indicate that after said date, CKCS, Inc. has also assumed the name CKCS, and viceversa. The most concrete indication of this practice is the 1991 Quarterly Percentage Tax
Returns covering the business name/trade 19th Tee Camp John Hay. In said returns, the
taxpayer is identified as Copper Kettle Cafeteria Specialist[48] or CKCS, not CKCS, Inc. Yet, in
several documents already cited, the purported owner of 19th Tee Bar at Club John Hay is
CKCS, Inc.
All these pieces of evidence buttress the finding of the CTA that in 1991, 1992 and
1993, respondent, together with his spouse Jeanne Menguito, owned and operated outlets in
Club John Hay and Texas Instruments under the names Copper Kettle Cafeteria Specialist or
CKCS and Copper Kettle Catering Services or Copper Kettle Catering Services, Inc..
Turning now to the second issue.

In respondent's Petition for Review with the CTA, he questioned the validity of the
Assessment Notices,[49] all dated September 2, 1997, issued by BIR, Baguio City against him
on the following grounds:
1.

The assessment notices, based on income and percentage tax returns


filed for 1991, 1992 and 1993, were issued beyond the three-year prescriptive
period under Section 203 of the Tax Code;[50]

2.

The assessment notices were addressed to Copper Kettle Specialist,


Club John Hay, Baguio City, despite notice to petitioner that respondent's
principal place of business was at the CCP Complex, Pasay City.[51]

3.

The assessment notices were issued in violation of the requirement of


Revenue Regulations No. 12-85, dated November 27, 1985, that the taxpayer
be issued a post-reporting notice and pre-assessment notice before the
preliminary findings of deficiency may ripen into a formal assessment;[52] and

4.

The assessment notices did not give respondent a 15-day period to


reply to the findings of deficiency.[53]

The Court notes that nowhere in his Petition for Review did respondent deny that he
received the September 2, 1997 assessment notices. Instead, during the trial, respondent's
witness,

Ma.

Theresa Nalda (Nalda),

testified

that

she

informed

the

BIR, Baguio City that there was no Notice or letter, that we did not receive,
perhaps, because they were not addressed to Mr. Menguito's head office.[54]
The CTA correctly upheld the validity of the assessment notices. Citing Section 223 of
the Tax Code which provides that the prescriptive period for the issuance of assessment
notices based on fraud is 10 years, the CTA ruled that the assessment notices issued against
respondent on September 2, 1997 were timely because petitioner discovered the falsity in
respondent's tax returns for 1991, 1992 and 1993 only on February 19, 1997.
[55]

Moreover, in accordance with Section 2 of Revenue Regulation No. 12-85, which requires

that assessment notices be sent to the address indicated in the taxpayer's return, unless the
latter gives a notice of change of address, the assessment notices in the present case were
sent by petitioner to Camp John Hay, for this was the address respondent indicated in his tax
returns.[56] As to whether said assessment notices were actually received, the CTA correctly
held that since respondent did not testify that he did not receive said notices, it can be
presumed that the same were actually sent to and received by the latter. The Court agrees

with the CTA in considering as hearsay the testimony of Nalda that respondent did not
receive the notices, because Nalda was not competent to testify on the matter, as she was
employed by respondent only in June 1998, whereas the assessment notices were sent
on September 2, 1997.[57]
Anent compliance with the requirements of Revenue Regulation No. 12-85, the CTA
held:
BIR records show that on July 28, 1997, a letter was issued by
BIR Baguio to
Spouses Menguito,
informing
the
latter
of
their
supposed underdeclaration of sales totaling P48,721,555.96 and giving them 5 days
to communicate any objection to the results of the investigation (Exhibit 11, p. 83, BIR
Records). Records likewise reveal the issuance of a Preliminary Ten (10) Day Letter on
August 11, 1997, informing Petitioner [respondent herein] that the sum
of P34,193,041.55 is due from him as deficiency income and percentage tax (Exhibit
13, p. 173, BIR Records). Said letter gave the Petitioner [respondent herein] a period of
ten (10) days to submit his objection to the proposed assessment, either personally or
in writing, together with any evidence he may want to present.
xxxx
As to Petitioner's allegation that he was given only ten (10) days
to reply to the findings of deficiency instead of fifteen (15) days granted to a taxpayer
under Revenue Regulations No. 12-85, this Court believes that when Respondent
[petitioner herein] gave the Petitioner [respondent herein] on October 10, 1997 an
additional period of ten (10) days to present documentary evidence or a total of
twenty (20) days, there was compliance with Revenue Regulations No. 12-85 and the
latter was amply given opportunity to present his side x x x.[58]
The CTA further held that respondent was estopped from raising procedural issues
against the assessment notices, because these were not cited in the September 28,
1997 letter-protest which his spouse Jeanne Menguito filed with petitioner.[59]
On appeal by respondent,[60] the CA resolved the issue, thus:
Moreover, if the taxpayer denies ever having received an
assessment from the BIR, it is incumbent upon the latter to prove by
competent evidence that such notice was indeed received by the
addressee.Here, respondent [petitioner herein] merely alleged that it forwarded
the assessment notices to petitioner [respondent herein]. The respondent did not
show any proof of mailing, registry receipt or acknowledgment receipt signed by the
petitioner [respondent herein]. Since respondent [petitioner herein] has not
adduced sufficient evidence that petitioner [respondent herein] had in fact
received the pre-assessment notice and post-reporting notice required by
law, it cannot be assumed that petitioner [respondent herein] had been
served said notices.[61]

No other ground was cited by the CA for the reversal of the finding of the CTA on the issue.
The CA is gravely mistaken.
In their Petition for Review with the CTA, respondent expressly stated that
[s]ometime in

September

1997,

petitioner

[respondent

herein] received various

assessment notices, all dated 02 September 1997, issued by BIR-Baguio for alleged
deficiency income and percentage taxes for taxable years ending 31 December 1991, 1992
and 1993 x x x.[62] In their September 28, 1997 protest to the September 2, 1997
assessment notices, respondent, through his spouses Jeanne Menguito, acknowledged that
[they] are in receipt of the assessment notice you have sent us, dated September 2, 1997
x x x.[63]
Respondent is therefore estopped from denying actual receipt of the September 2,
1997 assessment notices, notwithstanding the denial of his witness Nalda.
As to the address indicated on the assessment notices, respondent cannot question
the same for it is the said address which appears in its percentage tax returns. [64] While
respondent claims that he had earlier notified petitioner of a change in his business address,
no evidence of such written notice was presented. Under Section 11 of Revenue Regulation
No. 12-85, respondent's failure to give written notice of change of address bound him to
whatever communications were sent to the address appearing in the tax returns for the
period involved in the investigation.[65]
Thus, what remain in question now are: whether petitioner issued and mailed a postreporting notice and a pre-assessment notice; and whether respondent actually received
them.
There is no doubt that petitioner failed to prove that it served on respondent a postreporting notice and a pre-assessment notice. Exhibit 11 [66] of petitioner is a mere
photocopy of a July 28, 1997 letter it sent to respondent, informing him of the initial outcome
of the investigation into his sales, and the release of a preliminary assessment upon
completion of the investigation, with notice for the latter to file any objection within five days
from receipt of the letter. Exhibit 13[67] of petitioner is also a mere photocopy of an August
11, 1997 Preliminary Ten (10) Day Letter to respondent, informing him that he had been

found to be liable for deficiency income and percentage tax and inviting him to submit a
written objection to the proposed assessment within 10 days from receipt of notice. But
nowhere on the face of said documents can be found evidence that these were sent to and
received by respondent. Nor is there separate evidence, such as a registry receipt of the
notices or a certification from the Bureau of Posts, that petitioner actually mailed said notices.
However, while the lack of a post-reporting notice and pre-assessment notice
is a deviation from the requirements under Section 1 [68] and Section 2[69] of Revenue
Regulation No. 12-85, the same cannot detract from the fact that formal assessments
were issued to and actually received by respondents in accordance with Section 228 of the
National Internal Revenue Code which was in effect at the time of assessment.
It should be emphasized that the stringent requirement that an assessment notice
be satisfactorily proven to have been issued and released or, if receipt thereof is denied, that
said assessment notice have been served on the taxpayer,[70] applies only to formal
assessments prescribed under Section 228 of the National Internal Revenue Code, but not to
post-reporting notices or pre-assessment notices. The issuance of a valid formal assessment
is a substantive prerequisite to tax collection,[71] for it contains not only a computation of tax
liabilities but also a demand for payment within a prescribed period, thereby signaling the
time when penalties and interests begin to accrue against the taxpayer and enabling the
latter to determine his remedies therefor. Due process requires that it must be served on
and received by the taxpayer.[72]
A post-reporting notice and pre-assessment notice do not bear the gravity of a
formal assessment notice. The post-reporting notice and pre-assessment notice merely hint
at the initial findings of the BIR against a taxpayer and invites the latter to an informal
conference or clarificatory meeting. Neither notice contains a declaration of the tax liability
of the taxpayer or a demand for payment thereof. Hence, the lack of such notices inflicts no
prejudice on the taxpayer for as long as the latter is properly served a formal assessment
notice. In the case of respondent, a formal assessment notice was received by him as
acknowledged in his Petition for Review and Joint Stipulation; and, on the basis thereof, he
filed a protest with the BIR, Baguio City and eventually a petition with the CTA.
WHEREFORE, the petition is GRANTED. The March 31, 2005 Decision of the Court
of Appeals is REVERSED and SET ASIDE and the April 2, 2002 Decision and October 10,
2002 Resolution of the Court of Tax Appeals are REINSTATED.

SO ORDERED.
COMMISSIONER OF INTERNAL
REVENUE,
Petitioner,

G.R. No. 172129


Present:

- versus MIRANT PAGBILAO CORPORATION


(Formerly SOUTHERN ENERGY
QUEZON, INC.),
Respondent.

QUISUMBING, J., Chairperson,


CARPIO MORALES,
TINGA,
VELASCO, JR., and
BRION, JJ.
Promulgated:
September 12, 2008

x-----------------------------------------------------------------------------------------x
DECISION
VELASCO, JR., J.:
Before us is a Petition for Review on Certiorari under Rule 45 assailing and
seeking to set aside the Decision[1] dated December 22, 2005 of the Court of Appeals
(CA) in CA-G.R. SP No. 78280 which modified the March 18, 2003 Decision [2] of the
Court of Tax Appeals (CTA) in CTA Case No. 6133 entitled Mirant Pagbilao
Corporation (Formerly Southern Energy Quezon, Inc.) v. Commissioner of Internal
Revenue and ordered the Bureau of Internal Revenue (BIR) to refund or issue a tax
credit certificate (TCC) in favor of respondent Mirant Pagbilao Corporation (MPC) in
the amount representing its unutilized input value added tax (VAT) for the second
quarter of 1998. Also assailed is the CAs Resolution [3] of March 31, 2006 denying
petitioners motion for reconsideration.
The Facts
MPC, formerly Southern Energy Quezon, Inc., and also formerly known as
Hopewell (Phil.) Corporation, is a domestic firm engaged in the generation of power
which it sells to the National Power Corporation (NPC). For the construction of the

electrical and mechanical equipment portion of its Pagbilao, Quezon plant, which
appears to have been undertaken from 1993 to 1996, MPC secured the services of
Mitsubishi Corporation (Mitsubishi) of Japan.
Under Section 13[4] of Republic Act No. (RA) 6395, the NPCs revised charter,
NPC is exempt from all taxes. In Maceda v. Macaraig,[5] the Court construed the
exemption as covering both direct and indirect taxes.
In the light of the NPCs tax exempt status, MPC, on the belief that its sale of
power generation services to NPC is, pursuant to Sec. 108(B)(3) of the Tax Code,
[6]

zero-rated for VAT purposes, filed on December 1, 1997 with Revenue District

Office (RDO) No. 60 in Lucena City an Application for Effective Zero Rating. The
application covered the construction and operation of its Pagbilao power station
under a Build, Operate, and Transfer scheme.
Not getting any response from the BIR district office, MPC refiled its
application in the form of a request for ruling with the VAT Review Committee at
the BIR national office on January 28, 1999. On May 13, 1999, the Commissioner of
Internal Revenue issued VAT Ruling No. 052-99, stating that the supply of electricity
by Hopewell Phil. to the NPC, shall be subject to the zero percent (0%) VAT, pursuant
to Section 108 (B) (3) of the National Internal Revenue Code of 1997.
It must be noted at this juncture that consistent with its belief to be zerorated, MPC opted not to pay the VAT component of the progress billings from
Mitsubishi for the period covering April 1993 to September 1996for the E & M
Equipment Erection Portion of MPCs contract with Mitsubishi. This prompted
Mitsubishi to advance the VAT component as this serves as its output VAT which is
essential for the determination of its VAT payment. Apparently, it was only on April
14, 1998 that MPC paid Mitsubishi the VAT component for the progress billings from
April 1993 to September 1996, and for which Mitsubishi issued Official Receipt (OR)
No. 0189 in the aggregate amount of PhP 135,993,570.
On August 25, 1998, MPC, while awaiting approval of its application
aforestated, filed its quarterly VAT return for the second quarter of 1998 where it
reflected an input VAT of PhP 148,003,047.62, which included PhP 135,993,570
supported by OR No. 0189. Pursuant to the procedure prescribed in Revenue

Regulations No. 7-95, MPC filed onDecember 20, 1999 an administrative claim for
refund of unutilized input VAT in the amount of PhP 148,003,047.62.
Since the BIR Commissioner failed to act on its claim for refund and obviously
to forestall the running of the two-year prescriptive period under Sec. 229 of the
National Internal Revenue Code (NIRC), MPC went to the CTA via a petition for
review, docketed as CTA Case No. 6133.
Answering the petition, the BIR Commissioner, citing Kumagai-Gumi Co. Ltd. v.
CIR,[7] asserted that MPCs claim for refund cannot be granted for this main reason:
MPCs sale of electricity to NPC is not zero-rated for its failure to secure an approved
application for zero-rating.
Before the CTA, among the issues stipulated by the parties for resolution were,
in gist, the following:
1. Whether or not [MPC] has unapplied or unutilized creditable input
VAT for the 2nd quarter of 1998 attributable to zero-rated sales to NPC which
are proper subject for refund pursuant to relevant provisions of the NIRC;
2. Whether the creditable input VAT of MPC for said period, if any, is
substantiated by documents; and
3. Whether the unutilized creditable input VAT for said quarter, if any,
was applied against any of the VAT output tax of MPC in the subsequent
quarter.

To provide support to the CTA in verifying and analyzing documents and


figures and entries contained therein, the Sycip Gorres & Velayo (SGV), an
independent auditing firm, was commissioned.
The Ruling of the CTA
On the basis of its affirmative resolution of the first issue, the CTA, by its
Decision dated March 18, 2003, granted MPCs claim for input VAT refund or credit,
but only for the amount of PhP 10,766,939.48. The fallo of the CTAs decision reads:
In view of all the foregoing, the instant petition is PARTIALLY
GRANTED. Accordingly, respondent is hereby ORDERED to REFUND or in the
alternative, ISSUE A TAX CREDIT CERTIFICATE in favor of the petitioner its

unutilized input VAT payments directly attributable to its effectively zero-rated


sales for the second quarter of 1998 in the reduced amount of
P10,766,939.48, computed as follows:
Claimed Input VAT

P148,003,047.62

Less: Disallowances
a.)

As summarized by SGV & Co. in its initial report (Exh. P)


I. Input Taxes on Purchases of Services:
1. Supported by documents
other than VAT Ors
P 10,629.46
2. Supported by photocopied VAT OR
879.09
II. Input Taxes on Purchases of Goods:
1. Supported by documents other than
VAT invoices
165,795.70
2. Supported by Invoices with TIN only
1,781.82
3. Supported by photocopied VAT
invoices
3,153.62
III. Input Taxes on Importation of Goods:
1. Supported by photocopied documents
[IEDs and/or Bureau of Customs
(BOC) Ors]
716,250.00
2.
Supported
by
computations
91,601.00
990,090.69

b.) Input taxes without supporting documents as


summarized in Annex A of SGV & Co.s
supplementary
report
(CTA
134)
252,447.45
c.)

records,

Claimed input taxes on purchases of services from


Mitsubishi
Corp.
for
being
substantiated
[8]
OR 135,996,570.00

Refundable Input

brokers

by

page

dubious

P10,766,939.48

SO ORDERED.[9]

Explaining the disallowance of over PhP 137 million claimed input VAT, the CTA
stated that most of MPCs purchases upon which it anchored its claims for refund or
tax credit have not been amply substantiated by pertinent documents, such as but
not limited to VAT ORs, invoices, and other supporting documents. Wrote the CTA:
We agree with the above SGV findings that out of the remaining taxes
of P136,246,017.45, the amount of P252,477.45 was not supported by any
document and should therefore be outrightly disallowed.

As to the claimed input tax of P135,993,570.00 (P136,246,017.45 less


P252,477.45 ) on purchases of services from Mitsubishi Corporation, Japan, the
same is found to be of doubtful veracity. While it is true that said amount is
substantiated by a VAT official receipt with Serial No. 0189 dated April 14,
1998 x x x, it must be observed, however, that said VAT allegedly paid
pertains to the services which were rendered for the period 1993 to 1996. x x
x
The Ruling of the CA
Aggrieved, MPC appealed the CTAs Decision to the CA via a petition for review
under Rule 43, docketed as CA-G.R. SP No. 78280. On December 22, 2005, the CA
rendered its assailed decision modifying that of the CTA decision by granting most of
MPCs claims for tax refund or credit. And in a Resolution of March 31, 2006, the CA
denied the BIR Commissioners motion for reconsideration. The decretal portion of
the CA decision reads:
WHEREFORE,
premises
considered,
the
instant
petition
is
GRANTED. The assailed Decision of the Court of Tax Appeals dated March 18,
2003 is hereby MODIFIED. Accordingly, respondent Commissioner of Internal
Revenue is ordered to refund or issue a tax credit certificate in favor of
petitioner Mirant Pagbilao Corporation its unutilized input VAT payments
directly attributable to its effectively zero-rated sales for the second quarter of
1998 in the total amount of P146,760,509.48.
SO ORDERED.[10]

The CA agreed with the CTA on MPCs entitlement to (1) a zero-rating for VAT
purposes for its sales and services to tax-exempt NPC; and (2) a refund or tax credit
for its unutilized input VAT for the second quarter of 1998. Their disagreement,
however, centered on the issue of proper documentation, particularly the evidentiary
value of OR No. 0189.
The CA upheld the disallowance of PhP 1,242,538.14 representing zero-rated
input VAT claims supported only by photocopies of VAT OR/Invoice, documents other
than VAT Invoice/OR, and mere brokers computations. But the CA allowed MPCs
refund claim of PhP 135,993,570 representing input VAT payments for purchases of
goods and/or services from Mitsubishi supported by OR No. 0189. The appellate
court ratiocinated that the CTA erred in disallowing said claim since the OR from
Mitsubishi was the best evidence for the payment of input VAT by MPC to Mitsubishi

as required under Sec. 110(A)(1)(b) of the NIRC. The CA ruled that the legal
requirement of a VAT Invoice/OR to substantiate creditable input VAT was complied
with through OR No. 0189 which must be viewed as conclusive proof of the payment
of input VAT. To the CA, OR No. 0189 represented an undisputable acknowledgment
and receipt by Mitsubishi of the input VAT payment of MPC.
The CA brushed aside the CTAs ruling and disquisition casting doubt on the
veracity and genuineness of the Mitsubishi-issued OR No. 0189. It reasoned that the
issuance date of the said receipt, April 14, 1998, must be taken conclusively to
represent the input VAT payments made by MPC to Mitsubishi as MPC had no real
control on the issuance of the OR. The CA held that the use of a different exchange
rate reflected in the OR is of no consequence as what the OR undeniably attests and
acknowledges was Mitsubishis receipt of MPCs input VAT payment.
The Issue
Hence, the instant petition on the sole issue of whether or not respondent
[MPC] is entitled to the refund of its input VAT payments made from 1993 to 1996
amounting to [PhP] 146,760,509.48.[11]
The Courts Ruling
As a preliminary matter, it should be stressed that the BIR Commissioner,
while making reference to the figure PhP 146,760,509.48, joins the CA and the CTA
on their disposition on the propriety of the refund of or the issuance of a TCC for the
amount of PhP 10,766,939.48. In fine, the BIR Commissioner trains his sight and
focuses his arguments on the core issue of whether or not MPC is entitled to a refund
for PhP 135,993,570 (PhP 146,760,509.48 - PhP 10,766,939.48 = PhP 135,993,570) it
allegedly

paid

as

creditable

input

VAT

for

services

and

goods

purchased

from Mitsubishi during the 1993 to 1996 stretch.

The divergent factual findings and rulings of the CTA and CA impel us to
evaluate the evidence adduced below, particularly the April 14, 1998 OR 0189 in the
amount of PhP 135,996,570 [for US$ 5,190,000 at US$1: PhP 26.203 rate of
exchange]. Verily, a claim for tax refund may be based on a statute granting tax

exemption,

or,

asCommissioner

of

Internal

Revenue

v.

Fortune

Tobacco

Corporation[12] would have it, the result of legislative grace. In such case, the claim
is to be construed strictissimi jurisagainst the taxpayer,[13] meaning that the claim
cannot be made to rest on vague inference. Where the rule of strict interpretation
against the taxpayer is applicable as the claim for refund partakes of the nature of
an exemption, the claimant must show that he clearly falls under the exempting
statute. On the other hand, a tax refund may be, as usually it is, predicated on tax
refund provisions allowing a refund of erroneous or excess payment of tax. The
return of what was erroneously paid is founded on the principle of solutio indebiti, a
basic postulate that no one should unjustly enrich himself at the expense of another.
The caveat against unjust enrichment covers the government. [14] And as decisional
law teaches, a claim for tax refund proper, as here, necessitates only the
preponderance-of-evidence threshold like in any ordinary civil case. [15]
We apply the foregoing elementary principles in our evaluation on whether OR
0189, in the backdrop of the factual antecedents surrounding its issuance,
sufficiently proves the alleged unutilized input VAT claimed by MPC.
The Court can review issues of fact where there are
divergent findings by the trial and appellate courts
As a matter of sound practice, the Court refrains from reviewing the factual
determinations of the CA or reevaluate the evidence upon which its decision is
founded. One exception to this rule is when the CA and the trial court diametrically
differ in their findings,[16] as here. In such a case, it is incumbent upon the Court to
review and determine if the CA might have overlooked, misunderstood, or
misinterpreted certain facts or circumstances of weight, which, if properly
considered, would justify a different conclusion. [17] In the instant case, the CTA, unlike
the CA, doubted the veracity of OR No. 0189 and did not appreciate the same to
support MPCs claim for tax refund or credit.
Petitioner BIR Commissioner, echoing the CTAs stand, argues against the
sufficiency of OR No. 0189 to prove unutilized input VAT payment by MPC. He states
in this regard that the BIR can require additional evidence to prove and ascertain
payment of creditable input VAT, or that the claim for refund or tax credit was filed
within the prescriptive period, or had not previously been refunded to the taxpayer.

To bolster his position on the dubious character of OR No. 0189, or its


insufficiency to prove input VAT payment by MPC, petitioner proffers the following
arguments:
(1) The input tax covered by OR No. 0189 pertains to purchases by MPC from
Mitsubishi covering the period from 1993 to 1996; however, MPCs claim for tax
refund or credit was filed on December 20, 1999, clearly way beyond the two-year
prescriptive period set in Sec. 112 of the NIRC;
(2) MPC failed to explain why OR No. 0189 was issued by Mitsubishi (Manila)
when the invoices which the VAT were originally billed came from the Mitsubishis
head office in Japan;
(3) The exchange rate used in OR No. 0189 was pegged at PhP 26.203: USD 1
or the exchange rate prevailing in 1993 to 1996, when, on April 14, 1998, the date
OR No. 0189 was issued, the exchange rate was already PhP 38.01 to a US dollar;
(4) OR No. 0189 does not show or include payment of accrued interest which
Mitsubishi was charging and demanded from MPC for having advanced a
considerable amount of VAT. The demand, per records, is embodied in the May 12,
1995 letter of Mitsubishi to MPC;
(5) MPC failed to present to the CTA its VAT returns for the second and third
quarters of 1995, when the bulk of the VAT payment covered by OR No. 0189
specifically PhP 109,329,135.17 of the total amount of PhP 135,993,570was billed
by Mitsubishi, when such return is necessary to ascertain that the total amount
covered by the receipt or a large portion thereof was not previously refunded or
credited; and
(6) No other documents proving said input VAT payment were presented
except OR No. 0189 which, considering the fact that OR No. 0188 was likewise issued
by Mitsubishi and presented before the CTA but admittedly for payments made by
MPC on progress billings covering service purchases from 1993 to 1996, does not
clearly show if such input VAT payment was also paid for the period 1993 to 1996
and would be beyond the two-year prescriptive period.

The petition is partly meritorious.


Belated payment by MPC of its obligation for creditable input VAT
As no less found by the CTA, citing the SGVs report, the payments covered by
OR No. 0189 were for goods and service purchases made by MPC through the
progress billings from Mitsubishi for the period covering April 1993 to September
1996for the E & M Equipment Erection Portion of MPCs contract with Mitsubishi.
[18]

It is likewise undisputed that said payments did not include payments for the

creditable input VAT of MPC. This fact is shown by the May 12, 1995 letter[19] from
Mitsubishi where, as earlier indicated, it apprised MPC of the advances Mitsubishi
made for the VAT payments, i.e., MPCs creditable input VAT, and for which it was
holding MPC accountable for interest therefor.
In net effect, MPC did not, for the VATable MPC-Mitsubishi 1993 to 1996
transactions adverted to, immediately pay the corresponding input VAT. OR No. 0189
issued on April 14, 1998 clearly reflects the belated payment of input VAT
corresponding to the payment of the progress billings from Mitsubishi for the period
covering April 7, 1993 to September 6, 1996. SGV found that OR No. 0189 in the
amount of PhP 135,993,570 (USD 5,190,000) was duly supported by bank statement
evidencing payment to Mitsubishi (Japan). [20] Undoubtedly, OR No. 0189 proves
payment by MPC of its creditable input VAT relative to its purchases from Mitsubishi.
OR No. 0189 by itself sufficiently proves payment of VAT
The CA, citing Sec. 110(A)(1)(B) of the NIRC, held that OR No. 0189
constituted sufficient proof of payment of creditable input VAT for the progress
billings from Mitsubishi for the period covering April 7, 1993 to September 6,
1996. Sec. 110(A)(1)(B) of the NIRC pertinently provides:
Section 110. Tax Credits.
A. Creditable Input Tax.
(1) Any input tax evidenced by a VAT invoice or official receipt issued in
accordance with Section 113 hereof on the following transactions shall be
creditable against the output tax:

(a) Purchase or importation of goods:


xxxx
(b) Purchase of services on which a value-added tax has been
actually paid. (Emphasis ours.)

Without necessarily saying that the BIR is precluded from requiring additional
evidence to prove that input tax had indeed paid or, in fine, that the taxpayer is
indeed entitled to a tax refund or credit for input VAT, we agree with the CAs above
disposition. As the Court distinctly notes, the law considers a duly-executed VAT
invoice or OR referred to in the above provision as sufficient evidence to support a
claim for input tax credit. And any doubt as to what OR No. 0189 was for or tended
to prove should reasonably be put to rest by the SGV report on which the CTA
notably placed much reliance. The SGV report stated that [OR] No. 0189 dated April
14, 1998 is for the payment of the VAT on the progress billings from Mitsubishi
Japan for the period April 7, 1993 to September 6, 1996 for the E & M Equipment
Erection Portion of the Companys contract with Mitsubishi Corporation (Japan). [21]
VAT presumably paid on April 14, 1998
While available records do not clearly indicate when MPC actually paid the
creditable input VAT amounting to PhP 135,993,570 (USD 5,190,000) for the
aforesaid 1993 to 1996 service purchases, the presumption is that payment was
made on the date appearing on OR No. 0189, i.e., April 14, 1998. In fact, said
creditable input VAT was reflected in MPCs VAT return for the second quarter of
1998.
The aforementioned May 12, 1995 letter from Mitsubishi to MPC provides
collaborating proof of the belated payment of the creditable input VAT angle. To
reiterate, Mitsubishi, via said letter, apprised MPC of the VAT component of the
service purchases MPC made and reminded MPC that Mitsubishi had advanced VAT
payments to which Mitsubishi was entitled and from which it was demanding interest
payment. Given the scenario depicted in said letter, it is understandable why
Mitsubishi, in its effort to recover the amount it advanced, used the PhP 26.203: USD
1 exchange formula in OR No. 0189 for USD 5,190,000.

No showing of interest payment not fatal to claim for refund


Contrary to petitioners posture, the matter of nonpayment by MPC of the
interests demanded by Mitsubishi is not an argument against the fact of payment by
MPC of its creditable input VAT or of the authenticity or genuineness of OR No. 0189;
for at the end of the day, the matter of interest payment was between Mitsubishi and
MPC and may very well be covered by another receipt. But the more important
consideration is the fact that MPC, as confirmed by the SGV, paid its obligation to
Mitsubishi, and the latter issued to MPC OR No. 0189, for the VAT component of its
1993 to 1996 service purchases.
The next question is, whether or not MPC is entitled to a refund or a TCC for
the alleged unutilized input VAT of PhP 135,993,570 covered by OR No. 0189 which
sufficiently proves payment of the input VAT.
We answer the query in the negative.
Claim for refund or tax credit filed out of time
The claim for refund or tax credit for the creditable input VAT payment made
by MPC embodied in OR No. 0189 was filed beyond the period provided by law for
such claim. Sec. 112(A) of the NIRC pertinently reads:
(A) Zero-rated or Effectively Zero-rated Sales. Any VAT-registered
person, whose sales are zero-rated or effectively zero-rated may, within two
(2) years after the close of the taxable quarter when the sales were
made, apply for the issuance of a tax credit certificate or refund of
creditable input tax due or paid attributable to such sales, except
transitional input tax, to the extent that such input tax has not been applied
against output tax: x x x. (Emphasis ours.)
The above proviso clearly provides in no uncertain terms that unutilized input
VAT payments not otherwise used for any internal revenue tax due the taxpayer
must be claimed within two years reckoned from the close of the taxable
quarter when the relevant sales were made pertaining to the input VAT
regardless of whether said tax was paid or not. As the CA aptly puts it, albeit
it

erroneously

applied

the

aforequoted

Sec.

112(A),

[P]rescriptive

period

commences from the close of the taxable quarter when the sales were made and not

from the time the input VAT was paid nor from the time the official receipt was
issued.[22] Thus, when a zero-rated VAT taxpayer pays its input VAT a year after the
pertinent transaction, said taxpayer only has a year to file a claim for refund or tax
credit of the unutilized creditable input VAT. The reckoning frame would always be
the end of the quarter when the pertinent sales or transaction was made, regardless
when the input VAT was paid. Be that as it may, and given that the last creditable
input VAT due for the period covering the progress billing of September 6, 1996 is
the third quarter of 1996 ending on September 30, 1996, any claim for unutilized
creditable input VAT refund or tax credit for said quarter prescribed two years after
September 30, 1996 or, to be precise, on September 30, 1998. Consequently, MPCs
claim for refund or tax credit filed on December 10, 1999 had already prescribed.
Reckoning for prescriptive period under
Secs. 204(C) and 229 of the NIRC inapplicable
To be sure, MPC cannot avail itself of the provisions of either Sec. 204(C) or
229 of the NIRC which, for the purpose of refund, prescribes a different starting point
for the two-year prescriptive limit for the filing of a claim therefor. Secs. 204(C) and
229 respectively provide:
Sec. 204. Authority of the Commissioner to Compromise, Abate and
Refund or Credit Taxes. The Commissioner may
xxxx
(c) Credit or refund taxes erroneously or illegally received or penalties
imposed without authority, refund the value of internal revenue stamps when
they are returned in good condition by the purchaser, and, in his discretion,
redeem or change unused stamps that have been rendered unfit for use and
refund their value upon proof of destruction. No credit or refund of taxes
or penalties shall be allowed unless the taxpayer files in writing with
the Commissioner a claim for credit or refund within two (2) years
after the payment of the tax or penalty: Provided, however, That a return
filed showing an overpayment shall be considered as a written claim for credit
or refund.
xxxx
Sec. 229. Recovery of Tax Erroneously or Illegally Collected. No suit
or proceeding shall be maintained in any court for the recovery of any national
internal revenue tax hereafter alleged to have been erroneously or illegally
assessed or collected, or of any penalty claimed to have been collected
without authority, of any sum alleged to have been excessively or in any
manner wrongfully collected without authority, or of any sum alleged to have

been excessively or in any manner wrongfully collected, until a claim for


refund or credit has been duly filed with the Commissioner; but such suit or
proceeding may be maintained, whether or not such tax, penalty, or sum has
been paid under protest or duress.
In any case, no such suit or proceeding shall be filed after
the expiration of two (2) years from the date of payment of the tax or
penalty regardless of any supervening cause that may arise after payment:
Provided, however, That the Commissioner may, even without a written claim
therefor, refund or credit any tax, where on the face of the return upon which
payment was made, such payment appears clearly to have been erroneously
paid. (Emphasis ours.)

Notably, the above provisions also set a two-year prescriptive period,


reckoned from date of payment of the tax or penalty, for the filing of a claim of
refund or tax credit. Notably too, both provisions apply only to instances of
erroneous payment or illegal collection of internal revenue taxes.
MPCs creditable input VAT not erroneously paid
For perspective, under Sec. 105 of the NIRC, creditable input VAT is an indirect
tax which can be shifted or passed on to the buyer, transferee, or lessee of the
goods, properties, or services of the taxpayer. The fact that the subsequent sale or
transaction involves a wholly-tax exempt client, resulting in a zero-rated or
effectively zero-rated transaction, does not, standing alone, deprive the taxpayer of
its right to a refund for any unutilized creditable input VAT, albeit the erroneous,
illegal, or wrongful payment angle does not enter the equation.
In Commissioner of Internal Revenue v. Seagate Technology (Philippines), the
Court explained the nature of the VAT and the entitlement to tax refund or credit of a
zero-rated taxpayer:
Viewed broadly, the VAT is a uniform tax x x x levied on every
importation of goods, whether or not in the course of trade or business, or
imposed on each sale, barter, exchange or lease of goods or properties or on
each rendition of services in the course of trade or business as they pass
along the production and distribution chain, the tax being limited only to the
value added to such goods, properties or services by the seller, transferor or
lessor. It is an indirect tax that may be shifted or passed on to the buyer,
transferee or lessee of the goods, properties or services. As such, it should be
understood not in the context of the person or entity that is primarily, directly
and legally liable for its payment, but in terms of its nature as a tax on
consumption. In either case, though, the same conclusion is arrived at.

The law that originally imposed the VAT in the country, as well as the
subsequent amendments of that law, has been drawn from the tax credit
method. Such method adopted the mechanics and self-enforcement features
of the VAT as first implemented and practiced in Europe x x x. Under the
present method that relies on invoices, an entity can credit against or subtract
from the VAT charged on its sales or outputs the VAT paid on its purchases,
inputs and imports.
If at the end of a taxable quarter the output taxes charged by a seller
are equal to the input taxes passed on by the suppliers, no payment is
required. It is when the output taxes exceed the input taxes that the excess
has to be paid. If, however, the input taxes exceed the output taxes, the
excess shall be carried over to the succeeding quarter or quarters. Should the
input taxes result from zero-rated or effectively zero-rated transactions or
from the acquisition of capital goods, any excess over the output taxes shall
instead be refunded to the taxpayer or credited against other internal revenue
taxes.
xxxx
Zero-rated transactions generally refer to the export sale of goods
and supply of services. The tax rate is set at zero. When applied to the tax
base, such rate obviously results in no tax chargeable against the
purchaser. The seller of such transactions charges no output tax, but
can claim a refund of or a tax credit certificate for the VAT previously
charged by suppliers.[23] (Emphasis added.)

Considering the foregoing discussion, it is clear that Sec. 112(A) of the NIRC,
providing a two-year prescriptive period reckoned from the close of the taxable
quarter when the relevant sales or transactions were made pertaining to the
creditable input VAT, applies to the instant case, and not to the other actions which
refer to erroneous payment of taxes.
As a final consideration, the Court wishes to remind the BIR and other tax
agencies of their duty to treat claims for refunds and tax credits with proper
attention and urgency. Had RDO No. 60 and, later, the BIR proper acted, instead of
sitting, on MPCs underlying application for effective zero rating, the matter of
addressing MPCs right, or lack of it, to tax credit or refund could have plausibly been
addressed at their level and perchance freed the taxpayer and the government from
the rigors of a tedious litigation.

The all too familiar complaint is that the government acts with dispatch when
it comes to tax collection, but pays little, if any, attention to tax claims for refund or
exemption. It is high time our tax collectors prove the cynics wrong.
WHEREFORE,

the

petition

is PARTLY

GRANTED. The

Decision

dated

December 22, 2005 and the Resolution dated March 31, 2006 of the CA in CA-G.R. SP
No. 78280 are AFFIRMED with the MODIFICATION that the claim of respondent
MPC for tax refund or credit to the extent of PhP 135,993,570, representing its input
VAT payments for service purchases from Mitsubishi Corporation of Japan for the
construction of a portion of its Pagbilao, Quezon power station, is DENIED on the
ground that the claim had prescribed. Accordingly, petitioner Commissioner of
Internal Revenue is ordered to refund or, in the alternative, issue a tax credit
certificate in favor of MPC, its unutilized input VAT payments directly attributable to
its effectively zero-rated sales for the second quarter in the total amount of PhP
10,766,939.48.
No pronouncement as to costs.
SO ORDERED.
TAMBUNTING PAWNSHOP, INC.,
Petitione
r,

- versus -

G.R. No.

179085

Present:
PUNO, C.J., Chairperson,
CARPIO MORALES,
LEONARDO-DE CASTRO,
BERSAMIN, and
VILLARAMA, JR., JJ.

COMMISSIONER OF INTERNAL
Promulgated:
REVENUE,
Responden January 21, 2010
t.
x------ ------------------------------------------ x
[1]
Petitioner protested the assessment.[2] As the protest merited no
response, it filed a Petition for Review [3] with the Court of Tax Appeals

(CTA) pursuant to Section 228 of the National Internal Revenue Code,


[4]

raising the following arguments:


A.

Pawnshops are not subject to Value Added


Tax pursuant to Section 108 of the National Internal
Revenue Code.[5]

B.

Petitioner properly withheld and remitted to


the respondent the correct amount of expanded
withholding tax for taxable year 1999.[6]

C.

Petitioner has already paid the assessed


amount of P14,398.38 [sic], representing deficiency
withholding tax on compensation, thus, assessment
on withholding on compensation must be cancelled.[7]

D.

Petitioners pawn tickets are not subject to


documentary stamp tax pursuant to existing laws
and jurisprudence.[8] (emphasis and underscoring in
the original)

The First Division of the CTA ruled that petitioner is liable for VAT and
documentary stamp tax but not for withholding tax on compensation and
expanded withholding tax.[9] Thus it disposed:
WHEREFORE, premises considered, the Petition for Review
is PARTIALLY GRANTED. Respondents assessments for deficiency
Expanded Withholding Tax and Withholding Tax on Compensation for
the taxable year 1999, in the amounts of Twenty One Thousand
Seven Hundred Twenty Three and 75/100 Pesos (P21,723.75)
and Sixty Seven Thousand Two Hundred One and 55/100
Pesos (P67,201.55), respectively, are hereby CANCELLED and SET
ASIDE. However, the assessments for deficiency Value-Added Tax and
Documentary Stamp Tax are hereby AFFIRMED.
Accordingly, petitioner is ORDERED TO PAY the respondent the
amount of Three Million Fifty Five Thousand Five Hundred Sixty Four
and 34/100 Pesos (P3,055,564.34) and Four Hundred Six Thousand
Ninety Two and 500/100 Pesos (P406,092.50) representing deficiency
Value-Added Tax and Documentary Stamp Tax, respectively, for the
taxable year 1999, plus 20% delinquency interest from February 18,
2003 up to the time such amount is fully paid pursuant to Section 249
(c) of the 1997 NIRC.
SO
supplied)

ORDERED.[10] (emphasis

in

the

original; underscoring

Petitioners Motion for Partial Reconsideration [11] having been denied,


[12]

it

filed

Petition

for

Review [13] before

the

CTA

En

Banc

which

dismissed[14] it as it did petitioners Motion for Reconsideration. [15]


Hence, the present Petition for Review on Certiorari. [16]
To petitioner, a pawnshop is not enumerated as one of those engaged
in sale or exchange of services [17] in Section 108 of the National Internal
Revenue Code.[18]

Citing Commissioner of Internal Revenue v. Michel J.

Lhuillier Pawnshops, Inc.,[19] it contends that the nature of the business of


pawnshops does not fall under service as defined under the Legal
Thesaurus of William C. Burton, viz:
accommodate, administer to, advance, afford, aid, assist, attend, be
of use, care for, come to the aid of, commodere, comply, confer a
benefit, contribute to, cooperate, deservire, discharge ones duty, do
a service, do ones bidding, fill an office, forward, furnish aid, furnish
assistance, give help, lend, aid, minister to, promote, render help,
servire, submit, succor, supply aid, take care of, tend, wait on, work
for.[20]

The petition is in part meritorious.


On the issue of whether pawnshops are liable to pay VAT, the Court,
in First Planters Pawnshop, Inc. v. Commissioner of Internal Revenue,
[21]

held:
In fine, prior to the [passage of the] EVAT Law [in 1994],
pawnshops were treated as lending investors subject to lending
investor's tax. Subsequently, with the Court's ruling in Lhuillier,
pawnshops were then treated as VAT-able enterprises under the
general classification of "sale or exchange of services" under Section
108 (A) of the Tax Code of 1997, as amended. R.A. No. 9238 [which
was passed in 2004] finally classified pawnshops as Other Non-bank
Financial Intermediaries.
The Court finds that pawnshops should have been treated as
non-bank financial intermediaries from the very beginning, subject to
the appropriate taxes provided by law, thus


Under the National Internal Revenue Code of
1977, pawnshops should have been levied the 5% percentage tax on
gross
receipts
imposed
on
bank
and
non-bank
financial
intermediaries under Section 119 (now Section 121 of the Tax Code of
1997);

With the imposition of the VAT under R.A. No. 7716 or


the EVAT Law, pawnshops should have been subjected to the 10%
VAT imposed on banks and non-bank financial intermediaries and
financial institutions under Section 102 of the Tax Code of 1977 (now
Section 108 of the Tax Code of 1997);

This was restated by R.A. No. 8241, 24 which amended


R.A. No. 7716, although the levy, collection and assessment of the
10% VAT on services rendered by banks, non-bank financial
intermediaries, finance companies, and other financial intermediaries
not performing quasi-banking functions, were made effective January
1, 1998;

R.A. No. 8424 or the Tax Reform Act of 1997 26 likewise


imposed a 10% VAT under Section 108 but the levy, collection and
assessment thereof were again deferred until December 31, 1999;

The levy, collection and assessment of the 10% VAT was


further deferred by R.A. No. 8761 until December 31, 2000, and by
R.A. No. 9010, until December 31, 2002;

With no further deferments given by law, the levy,


collection and assessment of the 10% VAT on banks, non-bank
financial intermediaries, finance companies, and other financial
intermediaries not performing quasi-banking functions were finally
made effective beginning January 1, 2003;

Finally, with the enactment of R.A. No. 9238 in 2004,


the services of banks, non-bank financial intermediaries, finance
companies, and other financial intermediaries not performing quasibanking functions were specifically exempted from VAT, 28 and the
0% to 5% percentage tax on gross receipts on other non-bank
financial intermediaries was reimposed under Section 122 of the Tax
Code of 1997.
At the time of the disputed assessment, that is, for the year
2000, pawnshops were not subject to 10% VAT under the general
provision on "sale or exchange of services" as defined under Section
108 (A) of the Tax Code of 1997, which states: "'sale or exchange of
services' means the performance of all kinds of services in the
Philippines
for
others
for
a
fee,
remuneration
or
consideration . . . ." Instead, due to the specific nature of its
business, pawnshops were then subject to 10% VAT under the
category of non-bank financial intermediaries[.]

Coming now to the issue at hand Since petitioner is a nonbank financial intermediary, it is subject to 10% VAT for the tax
years 1996 to 2002; however, with the levy, assessment and
collection of VAT from non-bank financial intermediaries being
specifically deferred by law, then petitioner is not liable for VAT
during these tax years. But with the full implementation of the VAT
system on non-bank financial intermediaries starting January 1, 2003,
petitioner is liable for 10% VAT for said tax year. And beginning
2004 up to the present, by virtue of R.A. No. 9238, petitioner is no
longer liable for VAT but it is subject to percentage tax on gross
receipts from 0% to 5%, as the case may be. (emphasis and
underscoring supplied)

In light of the foregoing ruling, since the imposition of VAT on


pawnshops, which are non-bank financial intermediaries, was deferred for
the tax years 1996 to 2002, petitioner is not liable for VAT for the tax year
1999.
In dodging liability for documentary stamp tax on its pawn tickets,
petitioner argues that such tickets are neither securities nor printed
evidence of indebtedness.[22] The argument fails.
Section 195 of the National Internal Revenue Code provides:
Section 195. On every mortgage or pledge of lands, estate or
property, real or personal, heritable or movable, whatsoever, where
the same shall be made as a security for the payment of any definite
and certain sum of money lent at the time or previously due and
owing or forborne to be paid, being payable, and on any conveyance
of land, estate, or property whatsoever, in trust or to be sold, or
otherwise converted into money which shall be and intended only as
security, either by express stipulation or otherwise, there shall be
collected a documentary stamp tax x x x. (underscoring supplied)

Construing this provision vis a vis pawn tickets, the Court held in Michel J.
Lhuillier Pawnshop, Inc. v. Commissioner of Internal Revenue:
x x x A D[ocumentary] S[tamp] T[ax] is an excise tax on
the exercise of a right or privilege to transfer obligations, rights or
properties incident thereto. x x x
xxxx

Pledge is among the privileges, the exercise of which is subject


to DST. A pledge may be defined as an accessory, real and unilateral
contract by virtue of which the debtor or a third person delivers to
the creditor or to a third person movable property as security for the
performance of the principal obligation, upon the fulfillment of which
the thing pledged, with all its accessions and accessories, shall be
returned to the debtor or to the third person. This is essentially the
business of pawnshops which are defined under Section 3 of
Presidential Decree No. 114, or the Pawnshop Regulation Act, as
persons or entities engaged in lending money on personal property
delivered as security for loans.
xxxx
Section 3 of the Pawnshop Regulation Act defines a pawn ticket
as follows:
Pawn ticket is the pawnbrokers receipt for a pawn. It
is neither a security nor a printed evidence of indebtedness.
True, the law does not consider said ticket as an evidence of
security or indebtedness. However, for purposes of taxation, the
same pawn ticket is proof of an exercise of a taxable privilege of
concluding a contract of pledge. There is therefore no basis in
petitioners assertion that a DST is literally a tax on a document and
that no tax may be imposed on a pawn ticket.[23] (emphasis and
underscoring supplied)

With respect to petitioners argument against liability for surcharges


and interest that it was in good faith in not paying documentary stamp
taxes, it having relied on the rulings of respondent CIR and the CTA that
pawn tickets are not subject to documentary stamp taxes[24] the Court
finds the same meritorious.
It is settled that good faith and honest belief that one is not subject
to tax on the basis of previous interpretations of government agencies
tasked to implement the tax law are sufficient justification to delete the
imposition of surcharges and interest.[25]
WHEREFORE, the petition is IN PART GRANTED. The May 24, 2007
Decision

of

the

Court

of

Tax

Appeals

is AFFIRMED with

the MODIFICATION that the assessment deficiency value-added taxes for

the taxable year 1999 and for surcharges and delinquency interest on
deficient Value-Added Tax and Documentary Income Tax are SET ASIDE.
SO ORDERED.
G.R. No. 175097
ALLIED BANKING CORPORATION, Petitioner,
vs.
COMMISSIONER OF INTERNAL REVENUE, Respondent.
DECISION
DEL CASTILLO, J.:
The key to effective communication is clarity.
The Commissioner of Internal Revenue (CIR) as well as his duly authorized
representative must indicate clearly and unequivocally to the taxpayer whether an
action constitutes a final determination on a disputed assessment. 1 Words must be
carefully chosen in order to avoid any confusion that could adversely affect the
rights and interest of the taxpayer.
Assailed in this Petition for Review on Certiorari2 under Section 12 of Republic Act
(RA) No. 9282,3 in relation to Rule 45 of the Rules of Court, are the August 23, 2006
Decision4 of the Court of Tax Appeals (CTA) and its October 17, 2006
Resolution5 denying petitioners Motion for Reconsideration.
Factual Antecedents
On April 30, 2004, the Bureau of Internal Revenue (BIR) issued a Preliminary
Assessment Notice (PAN) to petitioner Allied Banking Corporation for deficiency
Documentary Stamp Tax (DST) in the amount ofP12,050,595.60 and Gross Receipts
Tax (GRT) in the amount of P38,995,296.76 on industry issue for the taxable year
2001.6 Petitioner received the PAN on May 18, 2004 and filed a protest against it on
May 27, 2004.7
On July 16, 2004, the BIR wrote a Formal Letter of Demand with Assessment Notices
to petitioner, which partly reads as follows: 8
It is requested that the above deficiency tax be paid immediately upon receipt
hereof, inclusive of penalties incident to delinquency. This is our final decision based
on investigation. If you disagree, you may appeal the final decision within thirty (30)
days from receipt hereof, otherwise said deficiency tax assessment shall become
final, executory and demandable.

Petitioner received the Formal Letter of Demand with Assessment Notices on August
30, 2004.9
Proceedings before the CTA First Division
On September 29, 2004, petitioner filed a Petition for Review 10 with the CTA which
was raffled to its First Division and docketed as CTA Case No. 7062. 11
On December 7, 2004, respondent CIR filed his Answer. 12 On July 28, 2005, he filed a
Motion to Dismiss13 on the ground that petitioner failed to file an administrative
protest on the Formal Letter of Demand with Assessment Notices. Petitioner opposed
the Motion to Dismiss on August 18, 2005.14
On October 12, 2005, the First Division of the CTA rendered a Resolution 15 granting
respondents Motion to Dismiss. It ruled:
Clearly, it is neither the assessment nor the formal demand letter itself that is
appealable to this Court. It is the decision of the Commissioner of Internal Revenue
on the disputed assessment that can be appealed to this Court (Commissioner of
Internal Revenue vs. Villa, 22 SCRA 3). As correctly pointed out by respondent, a
disputed assessment is one wherein the taxpayer or his duly authorized
representative filed an administrative protest against the formal letter of demand
and assessment notice within thirty (30) days from date [of] receipt thereof. In this
case, petitioner failed to file an administrative protest on the formal letter of demand
with the corresponding assessment notices. Hence, the assessments did not become
disputed assessments as subject to the Courts review under Republic Act No. 9282.
(See also Republic v. Liam Tian Teng Sons & Co., Inc., 16 SCRA 584.)
WHEREFORE, the Motion to Dismiss is GRANTED. The Petition for Review is hereby
DISMISSED for lack of jurisdiction.
SO ORDERED.16
Aggrieved, petitioner moved for reconsideration but the motion was denied by the
First Division in its Resolution dated February 1, 2006. 17
Proceedings before the CTA En Banc
On February 22, 2006, petitioner appealed the dismissal to the CTA En Banc. 18 The
case was docketed as CTA EB No. 167.
Finding no reversible error in the Resolutions dated October 12, 2005 and February 1,
2006 of the CTA First Division, the CTA En Banc denied the Petition for Review19as
well as petitioners Motion for Reconsideration. 20

The CTA En Banc declared that it is absolutely necessary for the taxpayer to file an
administrative protest in order for the CTA to acquire jurisdiction. It emphasized that
an administrative protest is an integral part of the remedies given to a taxpayer in
challenging the legality or validity of an assessment. According to the CTA En
Banc, although there are exceptions to the doctrine of exhaustion of administrative
remedies, the instant case does not fall in any of the exceptions.
Issue
Hence, the present recourse, where petitioner raises the lone issue of whether the
Formal Letter of Demand dated July 16, 2004 can be construed as a final decision of
the CIR appealable to the CTA under RA 9282.
Our Ruling
The petition is meritorious.
Section 7 of RA 9282 expressly provides that the CTA exercises exclusive appellate
jurisdiction to review by appeal decisions of the CIR in cases involving disputed
assessments
The CTA, being a court of special jurisdiction, can take cognizance only of matters
that are clearly within its jurisdiction.21 Section 7 of RA 9282 provides:
Sec. 7. Jurisdiction. The CTA shall exercise:
(a) Exclusive appellate jurisdiction to review by appeal, as herein provided:
(1) Decisions of the Commissioner of Internal Revenue in cases involving
disputed assessments, refunds of internal revenue taxes, fees or other
charges, penalties in relation thereto, or other matters arising under the
National Internal Revenue Code or other laws administered by the Bureau of
Internal Revenue;
(2) Inaction by the Commissioner of Internal Revenue in cases involving
disputed assessments, refunds of internal revenue taxes, fees or other
charges, penalties in relation thereto, or other matters arising under the
National Internal Revenue Code or other laws administered by the Bureau of
Internal Revenue, where the National Internal Revenue Code provides a
specific period of action, in which case the inaction shall be deemed a denial;
(Emphasis supplied)
xxxx
The word "decisions" in the above quoted provision of RA 9282 has been interpreted
to mean the decisions of the CIR on the protest of the taxpayer against the

assessments.22 Corollary thereto, Section 228 of the National Internal Revenue Code
(NIRC) provides for the procedure for protesting an assessment. It states:
SECTION 228. Protesting of Assessment. When the Commissioner or his duly
authorized representative finds that proper taxes should be assessed, he shall first
notify the taxpayer of his findings: Provided, however, That a preassessment notice
shall not be required in the following cases:
(a) When the finding for any deficiency tax is the result of mathematical error
in the computation of the tax as appearing on the face of the return; or
(b) When a discrepancy has been determined between the tax withheld and
the amount actually remitted by the withholding agent; or
(c) When a taxpayer who opted to claim a refund or tax credit of excess
creditable withholding tax for a taxable period was determined to have carried
over and automatically applied the same amount claimed against the
estimated tax liabilities for the taxable quarter or quarters of the succeeding
taxable year; or
(d) When the excise tax due on excisable articles has not been paid; or
(e) When an article locally purchased or imported by an exempt person, such
as, but not limited to, vehicles, capital equipment, machineries and spare
parts, has been sold, traded or transferred to non-exempt persons.
The taxpayers shall be informed in writing of the law and the facts on which the
assessment is made; otherwise, the assessment shall be void.
Within a period to be prescribed by implementing rules and regulations, the taxpayer
shall be required to respond to said notice. If the taxpayer fails to respond, the
Commissioner or his duly authorized representative shall issue an assessment based
on his findings.
Such assessment may be protested administratively by filing a request for
reconsideration or reinvestigation within thirty (30) days from receipt of the
assessment in such form and manner as may be prescribed by implementing rules
and regulations. Within sixty (60) days from filing of the protest, all relevant
supporting documents shall have been submitted; otherwise, the assessment shall
become final.
If the protest is denied in whole or in part, or is not acted upon within one hundred
eighty (180) days from submission of documents, the taxpayer adversely affected by
the decision or inaction may appeal to the Court of Tax Appeals within thirty (30)
days from receipt of the said decision, or from the lapse of the one hundred eighty

(180)-day period; otherwise, the decision shall become final, executory and
demandable.
In the instant case, petitioner timely filed a protest after receiving the PAN. In
response thereto, the BIR issued a Formal Letter of Demand with Assessment
Notices. Pursuant to Section 228 of the NIRC, the proper recourse of petitioner was to
dispute the assessments by filing an administrative protest within 30 days from
receipt thereof. Petitioner, however, did not protest the final assessment notices.
Instead, it filed a Petition for Review with the CTA. Thus, if we strictly apply the rules,
the dismissal of the Petition for Review by the CTA was proper.
The case is an exception to the
rule on exhaustion of administrative remedies
However, a careful reading of the Formal Letter of Demand with Assessment Notices
leads us to agree with petitioner that the instant case is an exception to the rule on
exhaustion of administrative remedies, i.e., estoppel on the part of the
administrative agency concerned.
In the case of Vda. De Tan v. Veterans Backpay Commission, 23 the respondent
contended that before filing a petition with the court, petitioner should have first
exhausted all administrative remedies by appealing to the Office of the President.
However, we ruled that respondent was estopped from invoking the rule on
exhaustion of administrative remedies considering that in its Resolution, it said, "The
opinions promulgated by the Secretary of Justice are advisory in nature, which may
either be accepted or ignored by the office seeking the opinion, and any aggrieved
party has the court for recourse". The statement of the respondent in said case led
the petitioner to conclude that only a final judicial ruling in her favor would be
accepted by the Commission.
Similarly, in this case, we find the CIR estopped from claiming that the filing of the
Petition for Review was premature because petitioner failed to exhaust all
administrative remedies.
The Formal Letter of Demand with Assessment Notices reads:
Based on your letter-protest dated May 26, 2004, you alleged the following:
1. That the said assessment has already prescribed in accordance with the
provisions of Section 203 of the Tax Code.
2. That since the exemption of FCDUs from all taxes found in the Old Tax Code
has been deleted, the wording of Section 28(A)(7)(b) discloses that there are
no other taxes imposable upon FCDUs aside from the 10% Final Income Tax.

Contrary to your allegation, the assessments covering GRT and DST for taxable year
2001 has not prescribed for [sic] simply because no returns were filed, thus, the
three year prescriptive period has not lapsed.
With the implementation of the CTRP, the phrase "exempt from all taxes" was
deleted. Please refer to Section 27(D)(3) and 28(A)(7) of the new Tax Code.
Accordingly, you were assessed for deficiency gross receipts tax on onshore income
from foreign currency transactions in accordance with the rates provided under
Section 121 of the said Tax Code. Likewise, deficiency documentary stamp taxes was
[sic] also assessed on Loan Agreements, Bills Purchased, Certificate of Deposits and
related transactions pursuant to Sections 180 and 181 of NIRC, as amended.
The 25% surcharge and 20% interest have been imposed pursuant to the provision
of Section 248(A) and 249(b), respectively, of the National Internal Revenue Code, as
amended.
It is requested that the above deficiency tax be paid immediately upon receipt
hereof, inclusive of penalties incident to delinquency. This is our final decision based
on investigation. If you disagree, you may appeal this final decision within thirty (30)
days from receipt hereof, otherwise said deficiency tax assessment shall become
final, executory and demandable.24 (Emphasis supplied)
It appears from the foregoing demand letter that the CIR has already made a final
decision on the matter and that the remedy of petitioner is to appeal the final
decision within 30 days.
In Oceanic Wireless Network, Inc. v. Commissioner of Internal Revenue, 25 we
considered the language used and the tenor of the letter sent to the taxpayer as the
final decision of the CIR.
In this case, records show that petitioner disputed the PAN but not the Formal Letter
of Demand with Assessment Notices. Nevertheless, we cannot blame petitioner for
not filing a protest against the Formal Letter of Demand with Assessment Notices
since the language used and the tenor of the demand letter indicate that it is the
final decision of the respondent on the matter. We have time and again reminded the
CIR to indicate, in a clear and unequivocal language, whether his action on a
disputed assessment constitutes his final determination thereon in order for the
taxpayer concerned to determine when his or her right to appeal to the tax court
accrues.26 Viewed in the light of the foregoing, respondent is now estopped from
claiming that he did not intend the Formal Letter of Demand with Assessment
Notices to be a final decision.
Moreover, we cannot ignore the fact that in the Formal Letter of Demand with
Assessment Notices, respondent used the word "appeal" instead of "protest",
"reinvestigation", or "reconsideration". Although there was no direct reference for
petitioner to bring the matter directly to the CTA, it cannot be denied that the word

"appeal" under prevailing tax laws refers to the filing of a Petition for Review with the
CTA. As aptly pointed out by petitioner, under Section 228 of the NIRC, the terms
"protest", "reinvestigation" and "reconsideration" refer to the administrative
remedies a taxpayer may take before the CIR, while the term "appeal" refers to the
remedy available to the taxpayer before the CTA. Section 9 of RA 9282, amending
Section 11 of RA 1125,27 likewise uses the term "appeal" when referring to the action
a taxpayer must take when adversely affected by a decision, ruling, or inaction of
the CIR. As we see it then, petitioner in appealing the Formal Letter of Demand with
Assessment Notices to the CTA merely took the cue from respondent. Besides, any
doubt in the interpretation or use of the word "appeal" in the Formal Letter of
Demand with Assessment Notices should be resolved in favor of petitioner, and not
the respondent who caused the confusion.
To be clear, we are not disregarding the rules of procedure under Section 228 of the
NIRC, as implemented by Section 3 of BIR Revenue Regulations No. 12-99. 28 It is the
Formal Letter of Demand and Assessment Notice that must be administratively
protested or disputed within 30 days, and not the PAN. Neither are we deviating from
our pronouncement in St. Stephens Chinese Girls School v. Collector of Internal
Revenue,29 that the counting of the 30 days within which to institute an appeal in the
CTA commences from the date of receipt of the decision of the CIR on the disputed
assessment, not from the date the assessment was issued.1avvphi1
What we are saying in this particular case is that, the Formal Letter of Demand with
Assessment Notices which was not administratively protested by the petitioner can
be considered a final decision of the CIR appealable to the CTA because the words
used, specifically the words "final decision" and "appeal", taken together led
petitioner to believe that the Formal Letter of Demand with Assessment Notices was
in fact the final decision of the CIR on the letter-protest it filed and that the available
remedy was to appeal the same to the CTA.
We note, however, that during the pendency of the instant case, petitioner availed of
the provisions of Revenue Regulations No. 30-2002 and its implementing Revenue
Memorandum Order by submitting an offer of compromise for the settlement of the
GRT, DST and VAT for the period 1998-2003, as evidenced by a Certificate of
Availment dated November 21, 2007.30 Accordingly, there is no reason to reinstate
the Petition for Review in CTA Case No. 7062.
WHEREFORE, the petition is hereby GRANTED. The assailed August 23, 2006
Decision and the October 17, 2006 Resolution of the Court of Tax Appeals
are REVERSED and SET ASIDE. The Petition for Review in CTA Case No. 7062 is
hereby DISMISSED based solely on the Bureau of Internal Revenues acceptance of
petitioners offer of compromise for the settlement of the gross receipts tax,
documentary stamp tax and value added tax, for the years 1998-2003.
SO ORDERED.

G.R. No. 178087

May 5, 2010

COMMISSIONER OF INTERNAL REVENUE, Petitioner,


vs.
KUDOS METAL CORPORATION, Respondent.
DECISION
DEL CASTILLO, J.:
The prescriptive period on when to assess taxes benefits both the government and
the taxpayer.1 Exceptions extending the period to assess must, therefore, be strictly
construed.
This Petition for Review on Certiorari seeks to set aside the Decision2 dated March
30, 2007 of the Court of Tax Appeals (CTA) affirming the cancellation of the
assessment notices for having been issued beyond the prescriptive period and the
Resolution3 dated May 18, 2007 denying the motion for reconsideration.
Factual Antecedents
On April 15, 1999, respondent Kudos Metal Corporation filed its Annual Income Tax
Return (ITR) for the taxable year 1998.
Pursuant to a Letter of Authority dated September 7, 1999, the Bureau of Internal
Revenue (BIR) served upon respondent three Notices of Presentation of Records.
Respondent failed to comply with these notices, hence, the BIR issued a Subpeona
Duces Tecum dated September 21, 2006, receipt of which was acknowledged by
respondents President, Mr. Chan Ching Bio, in a letter dated October 20, 2000.
A review and audit of respondents records then ensued.
On December 10, 2001, Nelia Pasco (Pasco), respondents accountant, executed a
Waiver of the Defense of Prescription,4 which was notarized on January 22, 2002,
received by the BIR Enforcement Service on January 31, 2002 and by the BIR Tax
Fraud Division on February 4, 2002, and accepted by the Assistant Commissioner of
the Enforcement Service, Percival T. Salazar (Salazar).
This was followed by a second Waiver of Defense of Prescription 5 executed by Pasco
on February 18, 2003, notarized on February 19, 2003, received by the BIR Tax Fraud
Division on February 28, 2003 and accepted by Assistant Commissioner Salazar.
On August 25, 2003, the BIR issued a Preliminary Assessment Notice for the taxable
year 1998 against the respondent. This was followed by a Formal Letter of Demand
with Assessment Notices for taxable year 1998, dated September 26, 2003 which
was received by respondent on November 12, 2003.

Respondent challenged the assessments by filing its "Protest on Various Tax


Assessments" on December 3, 2003 and its "Legal Arguments and Documents in
Support of Protests against Various Assessments" on February 2, 2004.
On June 22, 2004, the BIR rendered a final Decision 6 on the matter, requesting the
immediate payment of the following tax liabilities:
Kind of Tax

Amount

Income Tax

P 9,693,897.85

VAT

13,962,460.90

EWT
Withholding Tax-Compensation
Penalties
Total

1,712,336.76
247,353.24
8,000.00
P25,624,048.76

Ruling of the Court of Tax Appeals, Second Division


Believing that the governments right to assess taxes had prescribed, respondent
filed on August 27, 2004 a Petition for Review 7 with the CTA. Petitioner in turn filed
his Answer.8
On April 11, 2005, respondent filed an "Urgent Motion for Preferential Resolution of
the Issue on Prescription."9
On October 4, 2005, the CTA Second Division issued a Resolution 10 canceling the
assessment notices issued against respondent for having been issued beyond the
prescriptive period. It found the first Waiver of the Statute of Limitations incomplete
and defective for failure to comply with the provisions of Revenue Memorandum
Order (RMO) No. 20-90. Thus:
First, the Assistant Commissioner is not the revenue official authorized to sign the
waiver, as the tax case involves more than P1,000,000.00. In this regard, only the
Commissioner is authorized to enter into agreement with the petitioner in extending
the period of assessment;
Secondly, the waiver failed to indicate the date of acceptance. Such date of
acceptance is necessary to determine whether the acceptance was made within the
prescriptive period;
Third, the fact of receipt by the taxpayer of his file copy was not indicated on the
original copy. The requirement to furnish the taxpayer with a copy of the waiver is
not only to give notice of the existence of the document but also of the acceptance
by the BIR and the perfection of the agreement.1avvphi1

The subject waiver is therefore incomplete and defective. As such, the three-year
prescriptive period was not tolled or extended and continued to run. x x x 11
Petitioner moved for reconsideration but the CTA Second Division denied the motion
in a Resolution12 dated April 18, 2006.
Ruling of the Court of Tax Appeals, En Banc
On appeal, the CTA En Banc affirmed the cancellation of the assessment notices.
Although it ruled that the Assistant Commissioner was authorized to sign the waiver
pursuant to Revenue Delegation Authority Order (RDAO) No. 05-01, it found that the
first waiver was still invalid based on the second and third grounds stated by the CTA
Second Division. Pertinent portions of the Decision read as follows:
While the Court En Banc agrees with the second and third grounds for invalidating
the first waiver, it finds that the Assistant Commissioner of the Enforcement Service
is authorized to sign the waiver pursuant to RDAO No. 05-01, which provides in part
as follows:
A. For National Office cases
Designated Revenue Official
1. Assistant Commissioner (ACIR), For tax fraud and policy Enforcement Service
cases
2. ACIR, Large Taxpayers Service For large taxpayers cases other than those cases
falling under Subsection B hereof
3. ACIR, Legal Service For cases pending verification and awaiting resolution of
certain legal issues prior to prescription and for issuance/compliance of Subpoena
Duces Tecum
4. ACIR, Assessment Service (AS) For cases which are pending in or subject to review
or approval by the ACIR, AS
Based on the foregoing, the Assistant Commissioner, Enforcement Service is
authorized to sign waivers in tax fraud cases. A perusal of the records reveals that
the investigation of the subject deficiency taxes in this case was conducted by the
National Investigation Division of the BIR, which was formerly named the Tax Fraud
Division. Thus, the subject assessment is a tax fraud case.
Nevertheless, the first waiver is still invalid based on the second and third grounds
stated by the Court in Division. Hence, it did not extend the prescriptive period to
assess.
Moreover, assuming arguendo that the first waiver is valid, the second waiver is
invalid for violating Section 222(b) of the 1997 Tax Code which mandates that the
period agreed upon in a waiver of the statute can still be extended by subsequent

written agreement, provided that it is executed prior to the expiration of the first
period agreed upon. As previously discussed, the exceptions to the law on
prescription must be strictly construed.
In the case at bar, the period agreed upon in the subject first waiver expired on
December 31, 2002. The second waiver in the instant case which was supposed to
extend the period to assess to December 31, 2003 was executed on February 18,
2003 and was notarized on February 19, 2003. Clearly, the second waiver was
executed after the expiration of the first period agreed upon. Consequently, the
same could not have tolled the 3-year prescriptive period to assess. 13
Petitioner sought reconsideration but the same was unavailing.
Issue
Hence, the present recourse where petitioner interposes that:
THE COURT OF TAX APPEALS EN BANC ERRED IN RULING THAT THE GOVERNMENTS
RIGHT TO ASSESS UNPAID TAXES OF RESPONDENT PRESCRIBED. 14
Petitioners Arguments
Petitioner argues that the governments right to assess taxes is not barred by
prescription as the two waivers executed by respondent, through its accountant,
effectively tolled or extended the period within which the assessment can be made.
In disputing the conclusion of the CTA that the waivers are invalid, petitioner claims
that respondent is estopped from adopting a position contrary to what it has
previously taken. Petitioner insists that by acquiescing to the audit during the period
specified in the waivers, respondent led the government to believe that the "delay"
in the process would not be utilized against it. Thus, respondent may no longer
repudiate the validity of the waivers and raise the issue of prescription.
Respondents Arguments
Respondent maintains that prescription had set in due to the invalidity of the waivers
executed by Pasco, who executed the same without any written authority from it, in
clear violation of RDAO No. 5-01. As to the doctrine of estoppel by acquiescence
relied upon by petitioner, respondent counters that the principle of equity comes into
play only when the law is doubtful, which is not present in the instant case.
Our Ruling
The petition is bereft of merit.
Section 20315 of the National Internal Revenue Code of 1997 (NIRC) mandates the
government to assess internal revenue taxes within three years from the last day
prescribed by law for the filing of the tax return or the actual date of filing of such
return, whichever comes later. Hence, an assessment notice issued after the three-

year prescriptive period is no longer valid and effective. Exceptions however are
provided under Section 22216 of the NIRC.
The waivers executed by respondents accountant did not extend the period within
which the assessment can be made
Petitioner does not deny that the assessment notices were issued beyond the threeyear prescriptive period, but claims that the period was extended by the two waivers
executed by respondents accountant.
We do not agree.
Section 222 (b) of the NIRC provides that the period to assess and collect taxes may
only be extended upon a written agreement between the CIR and the taxpayer
executed before the expiration of the three-year period. RMO 20-90 17 issued on April
4, 1990 and RDAO 05-0118 issued on August 2, 2001 lay down the procedure for the
proper execution of the waiver, to wit:
1. The waiver must be in the proper form prescribed by RMO 20-90. The
phrase "but not after ______ 19 ___", which indicates the expiry date of the
period agreed upon to assess/collect the tax after the regular three-year
period of prescription, should be filled up.
2. The waiver must be signed by the taxpayer himself or his duly authorized
representative. In the case of a corporation, the waiver must be signed by any
of its responsible officials. In case the authority is delegated by the taxpayer
to a representative, such delegation should be in writing and duly notarized.
3. The waiver should be duly notarized.
4. The CIR or the revenue official authorized by him must sign the waiver
indicating that the BIR has accepted and agreed to the waiver. The date of
such acceptance by the BIR should be indicated. However, before signing the
waiver, the CIR or the revenue official authorized by him must make sure that
the waiver is in the prescribed form, duly notarized, and executed by the
taxpayer or his duly authorized representative.
5. Both the date of execution by the taxpayer and date of acceptance by the
Bureau should be before the expiration of the period of prescription or before
the lapse of the period agreed upon in case a subsequent agreement is
executed.
6. The waiver must be executed in three copies, the original copy to be
attached to the docket of the case, the second copy for the taxpayer and the
third copy for the Office accepting the waiver. The fact of receipt by the
taxpayer of his/her file copy must be indicated in the original copy to show
that the taxpayer was notified of the acceptance of the BIR and the perfection
of the agreement.19

A perusal of the waivers executed by respondents accountant reveals the following


infirmities:
1. The waivers were executed without the notarized written authority of Pasco
to sign the waiver in behalf of respondent.
2. The waivers failed to indicate the date of acceptance.
3. The fact of receipt by the respondent of its file copy was not indicated in the
original copies of the waivers.
Due to the defects in the waivers, the period to assess or collect taxes was not
extended. Consequently, the assessments were issued by the BIR beyond the threeyear period and are void.
Estoppel does not apply in this case
We find no merit in petitioners claim that respondent is now estopped from claiming
prescription since by executing the waivers, it was the one which asked for additional
time to submit the required documents.
In Collector of Internal Revenue v. Suyoc Consolidated Mining Company, 20 the
doctrine of estoppel prevented the taxpayer from raising the defense of prescription
against the efforts of the government to collect the assessed tax. However, it must
be stressed that in the said case, estoppel was applied as an exception to the statute
of limitations on collection of taxes and not on the assessment of taxes, as the BIR
was able to make an assessment within the prescribed period. More important, there
was a finding that the taxpayer made several requests or positive acts to convince
the government to postpone the collection of taxes, viz:
It appears that the first assessment made against respondent based on its second
final return filed on November 28, 1946 was made on February 11, 1947. Upon
receipt of this assessment respondent requested for at least one year within which to
pay the amount assessed although it reserved its right to question the correctness of
the assessment before actual payment. Petitioner granted an extension of only three
months. When it failed to pay the tax within the period extended, petitioner sent
respondent a letter on November 28, 1950 demanding payment of the tax as
assessed, and upon receipt of the letter respondent asked for a reinvestigation and
reconsideration of the assessment. When this request was denied, respondent again
requested for a reconsideration on April 25, 1952, which was denied on May 6, 1953,
which denial was appealed to the Conference Staff. The appeal was heard by the
Conference Staff from September 2, 1953 to July 16, 1955, and as a result of these
various negotiations, the assessment was finally reduced on July 26, 1955. This is the
ruling which is now being questioned after a protracted negotiation on the ground
that the collection of the tax has already prescribed.
It is obvious from the foregoing that petitioner refrained from collecting the tax by
distraint or levy or by proceeding in court within the 5-year period from the filing of
the second amended final return due to the several requests of respondent for

extension to which petitioner yielded to give it every opportunity to prove its claim
regarding the correctness of the assessment. Because of such requests, several
reinvestigations were made and a hearing was even held by the Conference Staff
organized in the collection office to consider claims of such nature which, as the
record shows, lasted for several months. After inducing petitioner to delay collection
as he in fact did, it is most unfair for respondent to now take advantage of such
desistance to elude his deficiency income tax liability to the prejudice of the
Government invoking the technical ground of prescription.
While we may agree with the Court of Tax Appeals that a mere request for
reexamination or reinvestigation may not have the effect of suspending the running
of the period of limitation for in such case there is need of a written agreement to
extend the period between the Collector and the taxpayer, there are cases however
where a taxpayer may be prevented from setting up the defense of prescription even
if he has not previously waived it in writing as when by his repeated requests or
positive acts the Government has been, for good reasons, persuaded to postpone
collection to make him feel that the demand was not unreasonable or that no
harassment or injustice is meant by the Government. And when such situation
comes to pass there are authorities that hold, based on weighty reasons, that such
an attitude or behavior should not be countenanced if only to protect the interest of
the Government.
This case has no precedent in this jurisdiction for it is the first time that such has
risen, but there are several precedents that may be invoked in American
jurisprudence. As Mr. Justice Cardozo has said: "The applicable principle is
fundamental and unquestioned. He who prevents a thing from being done may not
avail himself of the nonperformance which he has himself occasioned, for the law
says to him in effect "this is your own act, and therefore you are not damnified." "(R.
H. Stearns Co. vs. U.S., 78 L. ed., 647). Or, as was aptly said, "The tax could have
been collected, but the government withheld action at the specific request of the
plaintiff. The plaintiff is now estopped and should not be permitted to raise the
defense of the Statute of Limitations." [Newport Co. vs. U.S., (DC-WIS), 34 F. Supp.
588].21
Conversely, in this case, the assessments were issued beyond the prescribed period.
Also, there is no showing that respondent made any request to persuade the BIR to
postpone the issuance of the assessments.
The doctrine of estoppel cannot be applied in this case as an exception to the statute
of limitations on the assessment of taxes considering that there is a detailed
procedure for the proper execution of the waiver, which the BIR must strictly follow.
As we have often said, the doctrine of estoppel is predicated on, and has its origin in,
equity which, broadly defined, is justice according to natural law and right. 22 As such,
the doctrine of estoppel cannot give validity to an act that is prohibited by law or one
that is against public policy.23 It should be resorted to solely as a means of
preventing injustice and should not be permitted to defeat the administration of the
law, or to accomplish a wrong or secure an undue advantage, or to extend beyond
them requirements of the transactions in which they originate. 24 Simply put, the
doctrine of estoppel must be sparingly applied.

Moreover, the BIR cannot hide behind the doctrine of estoppel to cover its failure to
comply with RMO 20-90 and RDAO 05-01, which the BIR itself issued. As stated
earlier, the BIR failed to verify whether a notarized written authority was given by
the respondent to its accountant, and to indicate the date of acceptance and the
receipt by the respondent of the waivers. Having caused the defects in the waivers,
the BIR must bear the consequence. It cannot shift the blame to the taxpayer. To
stress, a waiver of the statute of limitations, being a derogation of the taxpayers
right to security against prolonged and unscrupulous investigations, must be
carefully and strictly construed.25
As to the alleged delay of the respondent to furnish the BIR of the required
documents, this cannot be taken against respondent. Neither can the BIR use this as
an excuse for issuing the assessments beyond the three-year period because with or
without the required documents, the CIR has the power to make assessments based
on the best evidence obtainable.26
WHEREFORE, the petition is DENIED. The assailed Decision dated March 30, 2007
and Resolution dated May 18, 2007 of the Court of Tax Appeals are
hereby AFFIRMED.
SO ORDERED.
G.R. No. 166829

April 19 2010

TFS, INCORPORATED, Petitioner,


vs.
COMMISSIONER OF INTERNAL REVENUE, Respondent.
DECISION
DEL CASTILLO, J.:
Only in highly meritorious cases, as in the instant case, may the rules for perfecting
an appeal be brushed aside.
This Petition for Review on Certiorari under Rule 45 of the Rules of Court seeks to set
aside the November 18, 20041 Resolution of the Court of Tax Appeals (CTA) En Banc
in C.T.A. EB No. 29 which dismissed petitioners Petition for Review for having been
filed out of time. Also assailed is the January 24, 2005 2 Resolution denying the
motion for reconsideration.
Factual Antecedents
Petitioner TFS, Incorporated is a duly organized domestic corporation engaged in the
pawnshop business. On January 15, 2002, petitioner received a Preliminary
Assessment Notice (PAN)3 for deficiency value added tax (VAT), expanded
withholding tax (EWT), and compromise penalty in the amounts
of P11,764,108.74, P183,898.02 and P25,000.00, respectively, for the taxable year

1998. Insisting that there was no basis for the issuance of PAN, petitioner through a
letter4 dated January 28, 2002 requested the Bureau of Internal Revenue (BIR) to
withdraw and set aside the assessments.
In a letter-reply5 dated February 7, 2002, respondent Commissioner of Internal
Revenue (CIR) informed petitioner that a Final Assessment Notice (FAN) 6 was issued
on January 25, 2002, and that petitioner had until February 22, 2002 within which to
file a protest letter.
On February 20, 2002, petitioner protested the FAN in a letter 7 dated February 19,
2002.
There being no action taken by the CIR, petitioner filed a Petition for Review 8 with the
CTA on September 11, 2002, docketed as CTA Case No. 6535.
During trial, petitioner offered to compromise and to settle the assessment for
deficiency EWT with the BIR. Hence, on September 24, 2003, it filed a Manifestation
and Motion withdrawing its appeal on the deficiency EWT, leaving only the issue of
VAT on pawnshops to be threshed out. Since no opposition was made by the CIR to
the Motion, the same was granted by the CTA on November 4, 2003.
Ruling of the Court of the Tax Appeals
On April 29, 2004, the CTA rendered a Decision 9 upholding the assessment issued
against petitioner in the amount of P11,905,696.32, representing deficiency VAT for
the year 1998, inclusive of 25% surcharge and 20% deficiency interest, plus 20%
delinquency interest from February 25, 2002 until full payment, pursuant to Sections
248 and 249(B) of the National Internal Revenue Code of 1997 (NIRC). The CTA ruled
that pawnshops are subject to VAT under Section 108(A) of the NIRC as they are
engaged in the sale of services for a fee, remuneration or consideration. 10
Aggrieved, petitioner moved for reconsideration 11 but the motion was denied by the
CTA in its Resolution dated July 20, 2004,12 which was received by petitioner on July
30, 2004.
Ruling of the Court of Appeals
On August 16, 2004, petitioner filed before the Court of Appeals (CA) a Motion for
Extension of Time to File Petition for Review.13 On August 24, 2004, it filed a Petition
for Review14 but it was dismissed by the CA in its Resolution 15dated August 31, 2004,
for lack of jurisdiction in view of the enactment of Republic Act No. 9282 (RA 9282). 16
Ruling of the Court of Tax Appeals En Banc
Realizing its error, petitioner filed a Petition for Review 17 with the CTA En Banc on
September 16, 2004. The petition, however, was dismissed for having been filed out

of time per Resolution dated November 18, 2004. Petitioner filed a Motion for
Reconsideration but it was denied in a Resolution dated January 24, 2005.
Hence, this petition.
Issues
In its Memorandum,18 petitioner interposes the following issues:
WHETHER THE HONORABLE COURT OF TAX APPEALS EN BANC SHOULD HAVE
GIVEN DUE COURSE TO THE PETITION FOR REVIEW AND NOT STRICTLY
APPLIED THE TECHNICAL RULES OF PROCEDURE TO THE DETRIMENT OF
JUSTICE.
WHETHER OR NOT PETITIONER IS SUBJECT TO THE 10% VAT. 19
Petitioners Arguments
Petitioner admits that it failed to timely file its Petition for Review with the proper
court (CTA). However, it attributes the procedural lapse to the inadvertence or
honest oversight of its counsel, who believed that at the time the petition was filed
on August 24, 2004, the CA still had jurisdiction since the rules and regulations to
implement the newly enacted RA 9282 had not yet been issued and the membership
of the CTA En Banc was not complete. In view of these circumstances, petitioner
implores us to reverse the dismissal of its petition and consider the timely filing of its
petition with the CA, which previously exercised jurisdiction over appeals from
decisions/resolutions of the CTA, as substantial compliance with the then recently
enacted RA 9282.
Petitioner also insists that the substantive merit of its case outweighs the procedural
infirmity it committed. It claims that the deficiency VAT assessment issued by the BIR
has no legal basis because pawnshops are not subject to VAT as they are not
included in the enumeration of services under Section 108(A) of the NIRC.
Respondents Arguments
The CIR, on the other hand, maintains that since the petition was filed with the CTA
beyond the reglementary period, the Decision had already attained finality and can
no longer be opened for review. As to the issue of VAT on pawnshops, he opines that
petitioners liability is a matter of law; and in the absence of any provision providing
for a tax exemption, petitioners pawnshop business is subject to VAT.
Our Ruling
The petition is meritorious.

Jurisdiction to review decisions or resolutions issued by the Divisions of the CTA is no


longer with the CA but with the CTA En Banc. This rule is embodied in Section 11 of
RA 9282, which provides that:
SECTION 11. Section 18 of the same Act is hereby amended as follows:
SEC. 18. Appeal to the Court of Tax Appeals En Banc. No civil proceeding involving
matters arising under the National Internal Revenue Code, the Tariff and Customs
Code or the Local Government Code shall be maintained, except as herein provided,
until and unless an appeal has been previously filed with the CTA and disposed of in
accordance with the provisions of this Act.
A party adversely affected by a resolution of a Division of the CTA on a motion for
reconsideration or new trial, may file a petition for review with the CTA en banc.
(Emphasis supplied)
Procedural rules may be relaxed in the interest of substantial justice
It is settled that an appeal must be perfected within the reglementary period
provided by law; otherwise, the decision becomes final and executory. 20 However, as
in all cases, there are exceptions to the strict application of the rules for perfecting
an appeal.21
We are aware of our rulings in Mactan Cebu International Airport Authority v.
Mangubat22 and in Alfonso v. Sps. Andres,23 wherein we excused the late filing of the
notices of appeal because at the time the said notices of appeal were filed, the new
rules24 applicable therein had just been recently issued. We noted that judges and
lawyers need time to familiarize themselves with recent rules.
However, in Cuevas v. Bais Steel Corporation 25 we found that the relaxation of rules
was unwarranted because the delay incurred therein was inexcusable. The subject
SC Circular 39-98 therein took effect on September 1, 1998, but the petitioners
therein filed their petition for certiorari five months after the circular took effect.
In the instant case, RA 9282 took effect on April 23, 2004, while petitioner
filed its Petition for Review on Certiorari with the CA on August 24, 2004, or four
months after the effectivity of the law. By then, petitioners counsel should have
been aware of and familiar with the changes introduced by RA 9282. Thus, we find
petitioners argument on the newness of RA 9282 a bit of a stretch.
Petitioner likewise cannot validly claim that its erroneous filing of the petition with
the CA was justified by the absence of the CTA rules and regulations and the
incomplete membership of the CTA En Banc as these did not defer the
effectivity26 and implementation of RA 9282. In fact, under Section 2 of RA
9282,27 the presence of four justices already constitutes a quorum for En

Banc sessions and the affirmative votes of four members of the CTA En Banc are
sufficient to render judgment.28 Thus, to us, the petitioners excuse of "inadvertence
or honest oversight of counsel" deserves scant consideration.
However, we will overlook this procedural lapse in the interest of substantial justice.
Although a client is bound by the acts of his counsel, including the latters mistakes
and negligence, a departure from this rule is warranted where such mistake or
neglect would result in serious injustice to the client. 29 Procedural rules may thus be
relaxed for persuasive reasons to relieve a litigant of an injustice not commensurate
with his failure to comply with the prescribed procedure. 30 Such is the situation in
this case.
Imposition of VAT on pawnshops for the tax years 1996 to 2002 was deferred
Petitioner disputes the assessment made by the BIR for VAT deficiency in the amount
of P11,905,696.32 for taxable year 1998 on the ground that pawnshops are not
included in the coverage of VAT.
We agree.
In First Planters Pawnshop, Inc. v. Commissioner of Internal Revenue, 31 we ruled that:
x x x Since petitioner is a non-bank financial intermediary, it is subject to 10% VAT
for the tax years 1996 to 2002;however, with the levy, assessment and
collection of VAT from non-bank financial intermediaries being specifically
deferred by law, then petitioner is not liable for VAT during these tax
years. But with the full implementation of the VAT system on non-bank financial
intermediaries starting January 1, 2003, petitioner is liable for 10% VAT for said tax
year. And beginning 2004 up to the present, by virtue of R.A. No. 9238, petitioner is
no longer liable for VAT but it is subject to percentage tax on gross receipts from 0%
to 5%, as the case may be. (Emphasis in the original text)1avvphi1
Guided by the foregoing, petitioner is not liable for VAT for the year 1998.
Consequently, the VAT deficiency assessment issued by the BIR against petitioner
has no legal basis and must therefore be cancelled. In the same vein, the imposition
of surcharge and interest must be deleted. 32
In fine, although strict compliance with the rules for perfecting an appeal is
indispensable for the prevention of needless delays and for the orderly and
expeditious dispatch of judicial business, strong compelling reasons such as serving
the ends of justice and preventing a grave miscarriage may nevertheless warrant the
suspension of the rules.33 In the instant case, we are constrained to disregard
procedural rules because we cannot in conscience allow the government to collect
deficiency VAT from petitioner considering that the government has no right at all to
collect or to receive the same. Besides, dismissing this case on a mere technicality
would lead to the unjust enrichment of the government at the expense of petitioner,

which we cannot permit. Technicalities should never be used as a shield to


perpetrate or commit an injustice.
WHEREFORE, the Petition is GRANTED. The assailed November 18, 2004 Resolution
of the Court of Tax Appeals En Banc in C.T.A. EB No. 29 which dismissed petitioners
Petition for Review for having been filed out of time, and the January 24, 2005
Resolution which denied the motion for reconsideration, are
hereby REVERSED and SET ASIDE. The assessment for deficiency Value Added Tax
for the taxable year 1998, including surcharges, deficiency interest and delinquency
interest, are hereby CANCELLED and SET ASIDE.
SO ORDERED.
G.R. No. 179617

January 19, 2011

COMMISSIONER OF INTERNAL REVENUE, Petitioner,


vs.
ASIAN TRANSMISSION CORPORATION, Respondent.
DECISION
MENDOZA, J.:
This case is a petition for review on certiorari under Rule 45 of the Rules of Court
filed by petitioner Commissioner of Internal Revenue (CIR) seeking to reverse and set
aside the July 16, 2007 Decision1 of the Court of Tax Appeals En Banc (CTA-En Banc),
in C.T.A. EB No. 205 and its September 11, 2007 Resolution 2 denying its motion for
reconsideration.
Through the assailed issuances, the CTA-En Banc affirmed in toto the Decision 3 and
the Amended Decision4 of its First Division (CTA-First Division) in CTA Case No. 9282
ordering the CIR to refund or issue a tax credit certificate in favor of respondent
Asian Transmission Corporation (ATC) for unutilized creditable withholding taxes for
the taxable year 2001.
From the records, it appears that ATC is a domestic corporation engaged in the
manufacture of automotive parts. It filed its annual Income Tax Return (ITR) for the
year 20005 on April 10, 2001 where it declared a gross income ofP370,532,082.00, a
net loss of P279,926,225.00 and a minimum corporate income
tax (MCIT) of P7,410,642.00. The MCIT due was offset against the P38,301,198.00
existing tax credits and creditable taxes withheld of the ATC, thereby leaving an
excess tax credit or overpayment of P30,890,556.00, as shown below:
MCIT

P 7,410,642.00

Less: Tax Credits/Payments


a. Prior Year's Excess Credits

P23,250,734.00

b. Creditable Tax Withheld for First Three


Quarters

11,868,132.00

c. Creditable Tax Withheld for the Fourth Quarter

3,121,256.00

d. Foreign Tax Credits

61,076.00

38,301,198.00

Total Overpayment

P30,890,556.00

For the P30,890,556.00 overpayment, ATC opted "To be issued a Tax Credit
Certificate."
In its ITR for the year 2001,6 ATC declared a gross income of P322,839,802.00, a net
loss of P37,869,455.00, and MCIT of P6,456,796.00. After deducting its MCIT due
against its existing tax credits and creditable taxes, ATC was left with a total tax
credit of P51,760,312.00 detailed as follows:
MCIT

P 6,456,796.00

Less: Tax Credit/Payments


a.Prior Year's Excess Credits

P 30,890,556.00

b. Creditable Tax Withheld for First Three


Quarters

12,405,573.00

c. Creditable Tax Withheld for the Fourth


Quarter

14,920,979.00

Total Overpayment

58,217,108.00
P51,760,312.00

ATC, however, applied part of its unutilized creditable taxes for the year 2000
amounting to P7,639,822.00 to its MCIT due of P6,456,796.00 for the year 2001. Left
unapplied of its 2000 creditable taxes, therefore, was the amount of P1,183,026.00
as shown in the following computation:
Creditable Tax Withheld for the First Three Quarters of 2000
Creditable Tax Withheld for the Fourth Quarter of 2000
Foreign Tax Credits for 2000
Total
Less: 2000 MCIT

P 11,868,132.00
P 3,121,256.00
61,076.00
P 15,050,464.00
7,410,642.00

Unutilized 2000 Creditable Taxes Withheld

P 7,639,822.00

Less: 2001 MCIT

P 6,456,796.00

Remaining Unutilized 2000 Creditable Taxes Withheld

P 1,183,026.00

Again, ATC opted "To be issued a Tax Credit Certificate" for the excess income tax
payment.

On April 9, 2003, ATC filed with CIRs Large Taxpayers Service an administrative
claim7 for the issuance of tax credit certificate or cash refund in the amount
of P28,509,578.00, representing excess/unutilized creditable income taxes withheld
as of December 31, 2001, to wit:
Remaining Unutilized 2000 Creditable Taxes Withheld

P1,183,026.00

Unapplied 2001 Creditable Taxes Withheld:


a. Creditable Tax Withheld for the First Three
Quarters of 2001

P 12,405,573.0
0

b. Creditable Tax Withheld for the Fourth Quarter


of 2001

14,920,979.00 27,326,552.00

Total

P28,509,578.0
0

The next day, on April 10, 2003, ATC filed a petition for review 8 with the CTA without
waiting for an action from the CIR to avoid the prescriptive period under Section 229
of the Tax Code.
On July 30, 2003, both parties filed a Joint Stipulation of Facts and Issues with the
CTA-First Division, submitting the following issues for consideration of the tax
tribunal:
1. Whether petitioners claim for refund was filed within the two-year
prescriptive period as prescribed under Section 204 and 229 of the NIRC;
2. Whether the income upon which the creditable taxes withheld were
included and reported as income in the income tax returns of petitioner for
both years;
3. Whether the creditable taxes are duly substantiated by the necessary
statement issued by the withholding agent to the petitioner, showing the
amount paid and the amount of the tax withheld therefrom;
4. Whether petitioner incurred a net loss of P279,926,225.00
and P37,869,455.00 during the taxable years 2000 and 2001, respectively;
and
5. Whether petitioner is entitled to the refund and/or credit of the amount of
28,509,578.00 representing its excess/unutilized creditable income taxes as of
December 31, 2001.
After the CTA-First Division approved the Joint Stipulation of Facts and Issues, the
case was submitted for decision.9

On March 20, 2006, the CTA-First Division rendered its Decision partially granting
ATCs claim for refund on its unutilized creditable withholding taxes for the taxable
year 2001, viz:
WHEREFORE, the instant petition for review is hereby PARTIALLY GRANTED.
Respondent is ordered to ISSUE A TAX CREDIT CERTIFICATE in favor of petitioner in
the reduced amount of P24,325,856.58 representing the unutilized creditable
withholding taxes for the taxable year 2001.
The CTA-First Division found that, contrary to the contentions of the CIR, ATC was
able to establish the factual basis for its claim for refund or for the issuance of a tax
credit certificate, and that the same was filed within the period prescribed under
Section 229 of the Tax Code. Thus, it was written:
In the case of Citibank N.A. vs. Court of Appeals, the Supreme Court emphasized that
the burden of proving the factual basis of his claim for tax credit or refund is upon
the claimant. Thus, for a claim [for] tax credit or refund be granted, the taxpayer
must establish that:
(i) The claim for refund was filed within two years as prescribed in Sec. 230
(now 229) of the Tax Code;
(ii) The income upon which the taxes were withheld were included in the
return of the recipient; and
(iii) The fact of withholding is established by a copy of statement (BIR Form
1743-A) duly issued by the payer (withholding agent) to the payee showing
the amount paid and the amount of tax withheld therefrom.
Applying the above rule, the following are evident:
One, the petitioner complied with the first requirement. The claim for refund of
petitioner for the calendar years ended December 31, 2000 and December 31, 2001
were filed within the two-year prescriptive period reckoned from the date of payment
of the tax. The phrase "date of payment of tax" is construed to mean the dates of
the filing of the 2000 and 2001 annual income tax returns. Petitioner filed its 2000
and 2001 original annual income tax return on April 10, 2001 and April 15, 2002,
respectively. The administrative and judicial claims for refund were filed on April 9,
2003 and April 10, 2003, respectively. Both filings of claim for refund and Petition for
Review were made within the tw0-year prescriptive period.
Two, petitioner was able to establish its qualified compliance with requirement
numbers two and three. In the admitted 2000 and 2001 Certificates of Creditable
Withholding at Source, the following amounts of income payments and withholding
taxes were reflected
xxx

xxx

xxx

We have traced the income payments in the 2000 and 2001 income tax returns and
found out that petitioner declared the same. It should be noted though that the
substantiated 2000 and 2001 creditable taxes amounted only to P14,986,640.75
(instead of P15,050,464.00) and P24,325,856.58 (instead of P27,326,552.00)
respectively. Hence we recomputed the supported unapplied creditable taxes
withheld as of December 31, 2001, to wit:
Amount
2000 Supported Creditable Taxes Withheld
Less: 2000 MCIT
Unutilized 2000 Creditable Taxes Withheld
Less: 2001 MCIT

P14,986,640.75
7,410,642.00
P 7,575,998.75
6,456,796.00

Remaining Unutilized 2000 Creditable


Taxes Withheld

P 1,119.202.75

Add: 2001 Supported and Unapplied


Creditable Taxes Withheld

24,325,856.58

Supported Unapplied Creditable Taxes Withheld


as of December 31, 2001

P25,445,059.33

As to the losses declared by ATC for the years 2000 and 2001, the CTA-First Division
opined that ATC was not required to prove them. It explained:
Lastly, we do not agree with the respondent that petitioner is required to prove that
it incurred a net loss for the years 2000 and 2001. The implied allegation of
irregularity in the declared operational losses is a matter which must be proven by
competent evidence. And the burden of proof as to whether petitioner incurred net
losses from its operations rests on the respondent. This is the reason why
respondent is authorized by law to examine the books and accounting records to
ascertain the truthfulness of petitioners declaration in its income tax return. In the
absence of any showing that there is irregularity in claimed losses for 2000 and 2001
business operations and taking into account that income tax returns are prepared
under penalty of perjury, We consider the returns of petitioner to be accurate and
regular. 10
The CTA-First Division, however, noted that ATC could not be issued a tax credit
certificate for the remaining 2000 unutilized creditable taxes pursuant to Section 78
of the Tax Code, considering that ATC initially declared that it would opt "To be Issued
a Tax Credit Certificate" for its 2000 creditable taxes, but never really exercised this
option. Instead, it made use of the option to carry-over its excess income tax
payments, when it applied the same in reducing its 2001 MCIT.
Thus, the CTA-First Division ordered the CIR to issue a tax credit certificate in favor of
ATC in the reduced amount of P24,325,856.58 representing the unutilized creditable
withholding taxes for the taxable year 2001 based on its own computation, to wit:

Income
Payment

Tax Withheld

P 300,603,978.00
195,263.12
363,266,839.00
137,659.10
576,146,311.00
137,659.10
488,449,635.00
103,611.44

Withholding Agent

P 3,006,039.78 Mitsubishi Motors Phils. Corp.


1,952.63 Nidec-Shimpo Philippines Corp.
3,632,668.39 Mitsubishi Motors Phils. Corp.
1,376.59 Nidec-Shimpo Philippines Corp.
5,761,463.11 Mitsubishi Motors Phils. Corp.
1,376.59 Nidec-Shimpo Philippines Corp.
4,884,496.35 Mitsubishi Motors Phils. Corp.
2,072.23 Nidec-Shimpo Philippines Corp.

Exh.
S
T
U
V
W
X
Y
Z

44,663,912.73

6,702,586.91 MMC Sittipol Co. Ltd.

AA

22,212,158.06

331,824.00 MMC Sittipol Co. Ltd.

BB

P1,795,937,026.55 P24,325,856.58

Both parties sought reconsideration. On one hand, CIR insisted that ATC failed to
establish the net loss it incurred and the tax credits due it. 11 On the other hand, ATC
averred that the CTA-First Division erred in: a) crediting only the amount
of P331,824.00 as the amount withheld by MMC Sittipol Co. Ltd. instead of
the P3,831,824.00 it actually withheld from ATC; and b) in ordering the issuance of a
Tax Credit Certificate in the amount of P24,325,856.58.12
Finding merit only in the motion for reconsideration of ATC, the CTA-First Division
issued the Amended Decision13on August 4, 2006, disposing the case in the following
manner:
WHEREFORE, petitioners Motion is hereby GRANTED while respondents Motion is
hereby DENIED for lack of merit. Accordingly, respondent is ORDERED TO REFUND or
in the alternative, ISSUE A TAX CREDIT CERTIFICATE in favor of the petitioner the
amount of TWENTY SEVEN MILLION THREE HUNDRED TWENTY FIVE THOUSAND
EIGHT HUNDRED FIFTY SIX & 58/100 PESOS (P27,325,856.58) representing unutilized
creditable withholding taxes for taxable year 2001.
On appeal, the CTA-En Banc was convinced that ATC was able to provide sufficient
evidence to establish its claim for refund or issuance of a tax credit
certificate.14 Thus, it rendered its July 16, 2007 Decision, the decretal portion of
which states:

WHEREFORE, premises considered, the instant petition is hereby DENIED DUE


COURSE, and, accordingly, DISMISSED for lack of merit.
Hence this petition .
The CIR raises the sole issue of:
WHETHER OR NOT RESPONDENT IS ENTITLED TO REFUND IN THE AMOUNT
OFP27,325,856.58 REPRESENTING THE ALLEGED UNUTILIZED CREDITABLE
WITHHOLDING TAXES FOR THE TAXABLE YEAR 2001.15
The petition has no merit.
The CIR argues that while the certificates of withholding taxes and the annual
income tax returns for the years 2000 and 2001 submitted by ATC may prove the
inclusion of income payments which were the bases of the withholding taxes and the
fact of withholding, they are not sufficient to prove entitlement to the tax refund
requested. According to the CIR, since Section 2.58.3 (B) of Revenue Regulation
provides that "claims for refund or tax credit shall be given due course upon showing
that income payment has been declared as part of gross income and the fact of
withholding is established," the mere submission of the withholding tax statements
shall only mean that ATCs claim shall be given due course, i.e., heard or considered.
Accordingly, the CIR posits that ATC still has to show that it is entitled to the refund
requested by proving not only the income payments made but also the reported
losses.
It should be pointed out that the arguments raised by the CIR in support of its
position have already been thoroughly discussed both by the CTA-First Division and
the CTA-En Banc. Notwithstanding, the CIR comes to this Court insisting that the
same be once again reviewed. Oft-repeated is the rule that the Court will not lightly
set aside the conclusions reached by the CTA which, by the very nature of its
function of being dedicated exclusively to the resolution of tax problems, has
accordingly developed an expertise on the subject, unless there has been an abuse
or improvident exercise of authority.16 In Barcelon, Roxas Securities, Inc. (now known
as UBP Securities, Inc.) v. Commissioner of Internal Revenue, 17 this Court more
explicitly pronounced:
Jurisprudence has consistently shown that this Court accords the findings of fact by
the CTA with the highest respect. In Sea-Land Service Inc. v. Court of Appeals [G.R.
No. 122605, 30 April 2001, 357 SCRA 441, 445-446], this Court recognizes that the
Court of Tax Appeals, which by the very nature of its function is dedicated exclusively
to the consideration of tax problems, has necessarily developed an expertise on the
subject, and its conclusions will not be overturned unless there has been an abuse or
improvident exercise of authority. Such findings can only be disturbed on appeal if
they are not supported by substantial evidence or there is a showing of gross error or
abuse on the part of the Tax Court. In the absence of any clear and convincing proof
to the contrary, this Court must presume that the CTA rendered a decision which is
valid in every respect.

At any rate, the CIR is correct in stating that the taxpayer bears the burden of proof
to establish not only that a refund is justified under the law but also that the amount
that should be refunded is correct. In this case, however, the CTA-First Division and
the CTA-En Banc uniformly found that from the evidence submitted, ATC has
established its claim for refund or issuance of a tax credit certificate for unutilized
creditable withholding taxes for the taxable year 2001 in the amount
of P27,325,856.58. The Court finds no cogent reason to rule differently. As correctly
noted by the CTA-En Banc:
x x x proof of actual remittance by the respondent is not needed in order to prove
withholding and remittance of taxes to petitioner. Section 2.58.3 (B) of Revenue
Regulation No. 2-98 clearly provides that proof of remittance is the responsibility of
the withholding agent and not of the taxpayer-refund claimant. It should be borne in
mind by the petitioner that payors of withholding taxes are by themselves
constituted as withholding agents of the BIR. The taxes they withhold are held in
trust for the government. In the event that the withholding agents commit fraud
against the government by not remitting the taxes so withheld, such act should not
prejudice herein respondent who has been duly withheld taxes by the withholding
agents acting under government authority. Moreover, pursuant to Section 57 and 58
of the NIRC of 1997, as amended, the withholding of income tax and the remittance
thereof to the BIR is the responsibility of the payor and not the payee. Therefore,
respondent, x x x has no control over the remittance of the taxes withheld from its
income by the withholding agent or payor who is the agent of the petitioner. The
Certificates of Creditable Tax Withheld at Source issued by the withholding agents of
the government are prima facie proof of actual payment by herein respondent-payee
to the government itself through said agents. We stress that the pertinent provisions
of law and the established jurisprudence evidently demonstrate that there is no need
for the claimant, respondent in this case, to prove actual remittance by the
withholding agent (payor) to the BIR.
In this regard, We do not agree with petitioners allegation that respondent failed to
prove that creditable withholding taxes were duly supported by valid Certificates of
Creditable Tax Withheld at Source. As aptly ruled by the Court in Division, and We
reiterate, the evidence on record in which petitioner interposed no objection to its
admission and was subsequently admitted by the Court in Division, show that
respondent was able to substantiate its claim through the presentation of Exhibits "J"
to "P" and "R" to "Z", the Certificates of Creditable Tax Withheld At Source. The
documentary evidence presented were sufficient to establish that respondent was
withheld taxes and that there was an excess which remain unutilized and now
subject of refund.
With respect to the losses incurred by the ATC, it is true that the taxpayer bears the
burden to establish the losses, but it is quite clear from the evidence presented that
ATC has fulfilled its duty. Moreover, other than the bare assertion that ATC must
establish its losses, the CIR fails to point to any circumstance or evidence that would
cast doubt on ATCs sworn declaration that it incurred losses in 2000 and 2001.
Curiously, in its petition, the CIR further adds that ATC cannot claim a cash refund or
tax credit for the unutilized withholding tax for the year 2000 as this would be
violative of Section 76 of the Tax Code. This matter, however, was already acted

upon in favor of the CIR, when the CTA-First Division only partially granted ATCs
petition by disallowing its claim for cash refund or tax credit for the unutilized
withholding tax for the year 2000. This reiteration by the CIR of this argument
despite the fact that it has already been acted favorably by the tax court below, only
shows that the appeal has not been thoroughly studied.
WHEREFORE, the petition is DENIED.
SO ORDERED.
G.R. No. 180909

January 19, 2011

EXXONMOBIL PETROLEUM AND CHEMICAL HOLDINGS, INC. - PHILIPPINE


BRANCH, Petitioner,
vs.
COMMISSIONER OF INTERNAL REVENUE, Respondent.
DECISION
MENDOZA, J.:
This is a petition for review on certiorari under Rule 45 filed by petitioner Exxonmobil
Petroleum and Chemical Holdings, Inc. - Philippine Branch (Exxon) to set aside the
September 7, 2007 Decision1 of the Court of Tax Appeals En Banc (CTA-En Banc) in
CTA E.B. No. 204, and its November 27, 2007 Resolution 2 denying petitioners motion
for reconsideration.
THE FACTS
Petitioner Exxon is a foreign corporation duly organized and existing under the laws
of the State of Delaware, United States of America. 3 It is authorized to do business in
the Philippines through its Philippine Branch, with principal office address at the 17/F
The Orient Square, Emerald Avenue, Ortigas Center, Pasig City. 4
Exxon is engaged in the business of selling petroleum products to domestic and
international carriers.5 In pursuit of its business, Exxon purchased from Caltex
Philippines, Inc. (Caltex) and Petron Corporation (Petron) Jet A-1 fuel and other
petroleum products, the excise taxes on which were paid for and remitted by both
Caltex and Petron.6Said taxes, however, were passed on to Exxon which ultimately
shouldered the excise taxes on the fuel and petroleum products. 7
From November 2001 to June 2002, Exxon sold a total of 28,635,841 liters of Jet A-1
fuel to international carriers, free of excise taxes amounting to
Php105,093,536.47.8 On various dates, it filed administrative claims for refund with
the Bureau of Internal Revenue (BIR) amounting to Php105,093,536.47.9

On October 30, 2003, Exxon filed a petition for review with the CTA 10 claiming a
refund or tax credit in the amount of Php105,093,536.47, representing the amount of
excise taxes paid on Jet A-1 fuel and other petroleum products it sold to international
carriers from November 2001 to June 2002.11
Exxon and the Commissioner of Internal Revenue (CIR) filed their Joint Stipulation of
Facts and Issues on June 24, 2004, presenting a total of fourteen (14) issues for
resolution.12
During Exxons preparation of evidence, the CIR filed a motion dated January 28,
2005 to first resolve the issue of whether or not Exxon was the proper party to ask
for a refund.13 Exxon filed its opposition to the motion on March 15, 2005.
On July 27, 2005, the CTA First Division issued a resolution 14 sustaining the CIRs
position and dismissing Exxons claim for refund. Exxon filed a motion for
reconsideration, but this was denied on July 27, 2006. 15
Exxon filed a petition for review16 with the CTA En Banc assailing the July 27, 2005
Resolution of the CTA First Division which dismissed the petition for review, and the
July 27, 2006 Resolution17 which affirmed the said ruling.
RULING OF THE COURT OF TAX APPEALS EN BANC
In its Decision dated September 7, 2007, the CTA En Banc dismissed the petition for
review and affirmed the two resolutions of the First Division dated July 27, 2005 and
July 27, 2006. Exxon filed a motion for reconsideration, but it was denied on
November 27, 2007.
Citing Sections 130 (A)(2)18 and 204 (C) in relation to Section 135 (a)19 of the
National Internal Revenue Code of 1997 (NIRC), the CTA ruled that in consonance
with its ruling in several cases,20 only the taxpayer or the manufacturer of the
petroleum products sold has the legal personality to claim the refund of excise taxes
paid on petroleum products sold to international carriers. 21
The CTA stated that Section 130(A)(2) makes the manufacturer or producer of the
petroleum products directly liable for the payment of excise taxes. 22 Therefore, it
follows that the manufacturer or producer is the taxpayer. 23
This determination of the identity of the taxpayer designated by law is pivotal as the
NIRC provides that it is only the taxpayer who "has the legal personality to ask for a
refund in case of erroneous payment of taxes." 24
Further, the excise tax imposed on manufacturers upon the removal of petroleum
products by oil companies is an indirect tax, or a tax which is primarily paid by
persons who can shift the burden upon someone else. 25 The CTA cited the cases
of Philippine Acetylene Co., Inc. v. Commissioner of Internal Revenue,26 Contex

Corporation v. Commissioner of Internal Revenue,27 and Commissioner of Internal


Revenue v. Philippine Long Distance Telephone Company,28 and explained that with
indirect taxes, "although the burden of an indirect tax can be shifted or passed on to
the purchaser of the goods, the liability for the indirect tax remains with the
manufacturer."29Moreover, "the manufacturer has the option whether or not to shift
the burden of the tax to the purchaser. When shifted, the amount added by the
manufacturer becomes a part of the price, therefore, the purchaser does not really
pay the tax per se but only the price of the commodity."30
Going by such logic, the CTA concluded that a refund of erroneously paid or illegally
received tax can only be made in favor of the taxpayer, pursuant to Section 204(C)
of the NIRC.31 As categorically ruled in the Cebu Portland
Cement32 and Contex33 cases, in the case of indirect taxes, it is the manufacturer of
the goods who is entitled to claim any refund thereof. 34 Therefore, it follows that the
indirect taxes paid by the manufacturers or producers of the goods cannot be
refunded to the purchasers of the goods because the purchasers are not the
taxpayers.35
The CTA also emphasized that tax refunds are in the nature of tax exemptions and
are, thus, regarded as in derogation of sovereign authority and construed strictissimi
juris against the person or entity claiming the exemption. 36
Finally, the CTA disregarded Exxons argument that "in effectively holding that only
petroleum products purchased directly from the manufacturers or producers are
exempt from excise taxes, the First Division of [the CTA] sanctioned a universal
amendment of existing bilateral agreements which the Philippines have with other
countries, in violation of the basic principle of pacta sunt servanda."37 The CTA
explained that the findings of fact of the First Division (that when Exxon sold the Jet
A-1 fuel to international carriers, it did so free of tax) negated any violation of the
exemption from excise tax of the petroleum products sold to international carriers.
Second, the right of international carriers to invoke the exemption granted under
Section 135(a) of the NIRC was neither affected nor restricted in any way by the
ruling of the First Division. At the point of sale, the international carriers were free to
invoke the exemption from excise taxes of the petroleum products sold to them.
Lastly, the lawmaking body was presumed to have enacted a later law with the
knowledge of all other laws involving the same subject matter. 38
THE ISSUES
Petitioner now raises the following issues in its petition for review:
I.
WHETHER THE ASSAILED DECISION AND RESOLUTION ERRONEOUSLY
PROHIBITED PETITIONER, AS THE DISTRIBUTOR AND VENDOR OF
PETROLEUM PRODUCTS TO INTERNATIONAL CARRIERS REGISTERED IN

FOREIGN COUNTRIES WHICH HAVE EXISTING BILATERAL AGREEMENTS WITH


THE PHILIPPINES, FROM CLAIMING A REFUND OF THE EXCISE TAXES PAID
THEREON; AND
II.
WHETHER THE ASSAILED DECISIONS ERRED IN AFFIRMING THE DISMISSAL
OF PETITIONERS CLAIM FOR REFUND BASED ON RESPONDENTS "MOTION
TO RESOLVE FIRST THE ISSUE OF WHETHER OR NOT THE PETITIONER IS THE
PROPER PARTY THAT MAY ASK FOR A REFUND," SINCE SAID MOTION IS
ESSENTIALLY A MOTION TO DISMISS, WHICH SHOULD HAVE BEEN DENIED
OUTRIGHT BY THE COURT OF TAX APPEALS FOR HAVING BEEN FILED OUT OF
TIME.
RULING OF THE COURT
I. On respondents "motion to resolve first the issue of whether or not the
petitioner is the proper party that may ask for a refund."
For a logical resolution of the issues, the court will tackle first the issue of whether or
not the CTA erred in granting respondents Motion to Resolve First the Issue of
Whether or Not the Petitioner is the Proper Party that may Ask for a Refund. 39 In said
motion, the CIR prayed that the CTA First Division resolve ahead of the other
stipulated issues the sole issue of whether petitioner was the proper party to ask for
a refund.40
Exxon opines that the CIRs motion is essentially a motion to dismiss filed out of
time,41 as it was filed afterpetitioner began presenting evidence42 more than a year
after the filing of the Answer. 43 By praying that Exxon be declared as not the proper
party to ask for a refund, the CIR asked for the dismissal of the petition, as the grant
of the Motion to Resolve would bring trial to a close. 44
Moreover, Exxon states that the motion should have also complied with the threeday notice and ten-day hearing rules provided in Rule 15 of the Rules of
Court.45 Since the CIR failed to set its motion for any hearing before the filing of the
Answer, the motion should have been considered a mere scrap of paper. 46
Finally, citing Maruhom v. Commission on Elections and Dimaporo,47 Exxon argues
that a defendant who desires a preliminary hearing on special and affirmative
defenses must file a motion to that effect at the time of filing of his answer. 48
The CIR, on the other hand, counters that it did not file a motion to
dismiss.49 Instead, the grounds for dismissal of the case were pleaded as special and
affirmative defenses in its Answer filed on December 15, 2003. 50 Therefore, the issue
of "whether or not petitioner is the proper party to claim for a tax refund of the
excise taxes allegedly passed on by Caltex and Petron" was included as one of the

issues in the Joint Stipulation of Facts and Issues dated June 24, 2004 signed by
petitioner and respondent.51
The CIR now argues that nothing in the Rules requires the preliminary hearing to be
held before the filing of an Answer.52 However, a preliminary hearing cannot be held
before the filing of the Answer precisely because any ground raised as an affirmative
defense is pleaded in the Answer itself. 53
Further, the CIR contends that the case cited by petitioner, Maruhom v.
Comelec,54 does not apply here. In the said case, a motion to dismiss was filed after
the filing of the answer.55 And, the said motion to dismiss was found to be a frivolous
motion designed to prevent the early termination of the proceedings in the election
case therein.56 Here, the Motion to Resolve was filed not to delay the disposition of
the case, but rather, to expedite proceedings. 571avvphi1
Rule 16, Section 6 of the 1997 Rules of Civil Procedure provides:
SEC. 6. Pleading grounds as affirmative defenses. - If no motion to dismiss has
been filed, any of the grounds for dismissal provided for in this Rule may be pleaded
as an affirmative defense in the answer, and in the discretion of the court, a
preliminary hearing may be had thereon as if a motion to dismiss had been filed.
The dismissal of the complaint under this section shall be without prejudice to the
prosecution in the same or separate action of a counterclaim pleaded in the answer.
(Underscoring supplied.)
This case is a clear cut application of the above provision. The CIR did not file a
motion to dismiss. Thus, he pleaded the grounds for dismissal as affirmative
defenses in its Answer and thereafter prayed for the conduct of a preliminary hearing
to determine whether petitioner was the proper party to apply for the refund of
excise taxes paid.
The determination of this question was the keystone on which the entire case was
leaning. If Exxon was not the proper party to apply for the refund of excise taxes
paid, then it would be useless to proceed with the case. It would not make any sense
to proceed to try a case when petitioner had no standing to pursue it.
In the case of California and Hawaiian Sugar Company v. Pioneer Insurance and
Surety Corporation,58 the Court held that:
Considering that there was only one question, which may even be deemed to be the
very touchstone of the whole case, the trial court had no cogent reason to deny the
Motion for Preliminary Hearing. Indeed, it committed grave abuse of discretion when
it denied a preliminary hearing on a simple issue of fact that could have possibly
settled the entire case. Verily, where a preliminary hearing appears to suffice, there
is no reason to go on to trial. One reason why dockets of trial courts are clogged is

the unreasonable refusal to use a process or procedure, like a motion to dismiss,


which is designed to abbreviate the resolution of a case. 59 (Underscoring supplied.)
II. On whether petitioner, as the distributor and vendor of petroleum
products to international carriers registered in foreign countries which
have existing bilateral agreements with the Philippines, can claim a refund
of the excise taxes paid thereon
This brings us now to the substantive issue of whether Exxon, as the distributor and
vendor of petroleum products to international carriers registered in foreign countries
which have existing bilateral agreements with the Philippines, is the proper party to
claim a tax refund for the excise taxes paid by the manufacturers, Caltex and Petron,
and passed on to it as part of the purchase price.
Exxon argues that having paid the excise taxes on the petroleum products sold to
international carriers, it is a real party in interest consistent with the rules and
jurisprudence.60
It reasons out that the subject of the exemption is neither the seller nor the buyer of
the petroleum products, but the products themselves, so long as they are sold to
international carriers for use in international flight operations, or to exempt entities
covered by tax treaties, conventions and other international agreements for their use
or consumption, among other conditions.61
Thus, as the exemption granted under Section 135 attaches to the petroleum
products and not to the seller, the exemption will apply regardless of whether the
same were sold by its manufacturer or its distributor for two reasons. 62 First, Section
135 does not require that to be exempt from excise tax, the products should be sold
by the manufacturer or producer. 63 Second, the legislative intent was precisely to
make Section 135 independent from Sections 129 and 130 of the NIRC, 64 stemming
from the fact that unlike other products subject to excise tax, petroleum products of
this nature have become subject to preferential tax treatment by virtue of either
specific international agreements or simply of international reciprocity. 65
Respondent CIR, on the other hand, posits that Exxon is not the proper party to seek
a refund of excise taxes paid on the petroleum products. 66 In so arguing, the CIR
states that excise taxes are indirect taxes, the liability for payment of which falls on
one person, but the burden of payment may be shifted to another. 67 Here, the sellers
of the petroleum products or Jet A-1 fuel subject to excise tax are Petron and Caltex,
while Exxon was the buyer to whom the burden of paying excise tax was
shifted.68 While the impact or burden of taxation falls on Exxon, as the tax is shifted
to it as part of the purchase price, the persons statutorily liable to pay the tax are
Petron and Caltex.69As Exxon is not the taxpayer primarily liable to pay, and not
exempted from paying, excise tax, it is not the proper party to claim for the refund of
excise taxes paid.70

The excise tax, when passed on to the purchaser, becomes part of the purchase
price.
Excise taxes are imposed under Title VI of the NIRC. They apply to specific goods
manufactured or produced in the Philippines for domestic sale or consumption or for
any other disposition, and to those that are imported. 71 In effect, these taxes are
imposed when two conditions concur: first, that the articles subject to tax belong to
any of the categories of goods enumerated in Title VI of the NIRC; and second, that
said articles are for domestic sale or consumption, excluding those that are actually
exported.72
There are, however, certain exemptions to the coverage of excise taxes, such as
petroleum products sold to international carriers and exempt entities or agencies.
Section 135 of the NIRC provides:
SEC. 135. Petroleum Products Sold to International Carriers and Exempt
Entities or Agencies. - Petroleum products sold to the following are exempt from
excise tax:
(a) International carriers of Philippine or foreign registry on their use or
consumption outside the Philippines: Provided, That the petroleum products
sold to these international carriers shall be stored in a bonded storage tank
and may be disposed of only in accordance with the rules and regulations to
be prescribed by the Secretary of Finance, upon recommendation of the
Commissioner;
(b) Exempt entities or agencies covered by tax treaties, conventions and other
international agreements for their use of consumption: Provided, however,
That the country of said foreign international carrier or exempt entities or
agencies exempts from similar taxes petroleum products sold to Philippine
carriers, entities or agencies; and
(c) Entities which are by law exempt from direct and indirect taxes.
(Underscoring supplied.)
Thus, under Section 135, petroleum products sold to international carriers of foreign
registry on their use or consumption outside the Philippines are exempt from excise
tax, provided that the petroleum products sold to such international carriers shall be
stored in a bonded storage tank and may be disposed of only in accordance with the
rules and regulations to be prescribed by the Secretary of Finance, upon
recommendation of the Commissioner.73
The confusion here stems from the fact that excise taxes are of the nature of indirect
taxes, the liability for payment of which may fall on a person other than he who
actually bears the burden of the tax.

In Commissioner of Internal Revenue v. Philippine Long Distance Telephone


Company,74 the Court discussed the nature of indirect taxes as follows:
[I]ndirect taxes are those that are demanded, in the first instance, from, or are paid
by, one person to someone else. Stated elsewise, indirect taxes are taxes wherein
the liability for the payment of the tax falls on one person but the burden thereof can
be shifted or passed on to another person, such as when the tax is imposed upon
goods before reaching the consumer who ultimately pays for it. When the seller
passes on the tax to his buyer, he, in effect, shifts the tax burden, not the liability to
pay it, to the purchaser, as part of the goods sold or services rendered.
Accordingly, the party liable for the tax can shift the burden to another, as part of
the purchase price of the goods or services. Although the manufacturer/seller is the
one who is statutorily liable for the tax, it is the buyer who actually shoulders or
bears the burden of the tax, albeit not in the nature of a tax, but part of the purchase
price or the cost of the goods or services sold.
As petitioner is not the statutory taxpayer, it is not entitled to claim a refund of
excise taxes paid.
The question we are faced with now is, if the party statutorily liable for the tax is
different from the party who bears the burden of such tax, who is entitled to claim a
refund of the tax paid?
Sections 129 and 130 of the NIRC provide:
SEC. 129. Goods subject to Excise Taxes. - Excise taxes apply to goods
manufactured or produced in the Philippines for domestic sales or consumption or for
any other disposition and to things imported. The excise tax imposed herein shall be
in addition to the value-added tax imposed under Title IV.
For purposes of this Title, excise taxes herein imposed and based on weight or
volume capacity or any other physical unit of measurement shall be referred to as
'specific tax' and an excise tax herein imposed and based on selling price or other
specified value of the good shall be referred to as 'ad valorem tax.'
SEC. 130. Filing of Return and Payment of Excise Tax on Domestic Products.
(A) Persons Liable to File a Return, Filing of Return on Removal and
Payment of Tax. (1) Persons Liable to File a Return. - Every person liable to pay excise tax
imposed under this Title shall file a separate return for each place of production
setting forth, among others the description and quantity or volume of products to be
removed, the applicable tax base and the amount of tax due thereon: Provided,

however, That in the case of indigenous petroleum, natural gas or liquefied natural
gas, the excise tax shall be paid by the first buyer, purchaser or transferee for local
sale, barter or transfer, while the excise tax on exported products shall be paid by
the owner, lessee, concessionaire or operator of the mining claim.
Should domestic products be removed from the place of production without the
payment of the tax, the owner or person having possession thereof shall be liable for
the tax due thereon.
(2) Time for Filing of Return and Payment of the Tax. - Unless otherwise
specifically allowed, the return shall be filed and the excise tax paid by the
manufacturer or producer before removal of domestic products from place of
production: Provided, That the tax excise on locally manufactured petroleum
products and indigenous petroleum/levied under Sections 148 and 151(A)(4),
respectively, of this Title shall be paid within ten (10) days from the date of removal
of such products for the period from January 1, 1998 to June 30, 1998; within five (5)
days from the date of removal of such products for the period from July 1, 1998 to
December 31, 1998; and, before removal from the place of production of such
products from January 1, 1999 and thereafter: Provided, further, That the excise tax
on nonmetallic mineral or mineral products, or quarry resources shall be due and
payable upon removal of such products from the locality where mined or extracted,
but with respect to the excise tax on locally produced or extracted metallic mineral
or mineral products, the person liable shall file a return and pay the tax within fifteen
(15) days after the end of the calendar quarter when such products were removed
subject to such conditions as may be prescribed by rules and regulations to be
promulgated by the Secretary of Finance, upon recommendation of the
Commissioner. For this purpose, the taxpayer shall file a bond in an amount which
approximates the amount of excise tax due on the removals for the said quarter. The
foregoing rules notwithstanding, for imported mineral or mineral products, whether
metallic or nonmetallic, the excise tax due thereon shall be paid before their removal
from customs custody.
xxx
(Italics and underscoring supplied.)
As early as the 1960s, this Court has ruled that the proper party to question, or to
seek a refund of, an indirect tax, is the statutory taxpayer, or the person on whom
the tax is imposed by law and who paid the same, even if he shifts the burden
thereof to another.75
In Philippine Acetylene Co., Inc. v. Commissioner of Internal Revenue,76 the Court
held that the sales tax is imposed on the manufacturer or producer and not on the
purchaser, "except probably in a very remote and inconsequential
sense."77 Discussing the "passing on" of the sales tax to the purchaser, the Court

therein cited Justice Oliver Wendell Holmes opinion in Lashs Products v. United
States78 wherein he said:
"The phrase passed the tax on is inaccurate, as obviously the tax is laid and
remains on the manufacturer and on him alone. The purchaser does not really pay
the tax. He pays or may pay the seller more for the goods because of the sellers
obligation, but that is all. x x x The price is the sum total paid for the goods. The
amount added because of the tax is paid to get the goods and for nothing else.
Therefore it is part of the price x x x." 79
Proceeding from this discussion, the Court went on to state:
It may indeed be that the economic burden of the tax finally falls on the purchaser;
when it does the tax becomes a part of the price which the purchaser must pay. It
does not matter that an additional amount is billed as tax to the purchaser. x x x The
effect is still the same, namely, that the purchaser does not pay the tax. He pays or
may pay the seller more for the goods because of the sellers obligation, but that is
all and the amount added because of the tax is paid to get the goods and for nothing
else.
But the tax burden may not even be shifted to the purchaser at all. A decision to
absorb the burden of the tax is largely a matter of economics. Then it can no longer
be contended that a sales tax is a tax on the purchaser. 80
The above case was cited in the later case of Cebu Portland Cement Company v.
Collector (now Commissioner) of Internal Revenue,81 where the Court ruled that as
the sales tax is imposed upon the manufacturer or producer and not on the
purchaser, "it is petitioner and not its customers, who may ask for a refund of
whatever amount it is entitled for the percentage or sales taxes it paid before the
amendment of section 246 of the Tax Code." 82
The Philippine Acetylene case was also cited in the first Silkair (Singapore) Pte, Ltd.
v. Commissioner of Internal Revenue83 case, where the Court held that the proper
party to question, or to seek a refund of, an indirect tax is the statutory taxpayer, the
person on whom the tax is imposed by law and who paid the same even if he shifts
the burden thereof to another. 84
In the Silkair cases,85 petitioner Silkair (Singapore) Pte, Ltd. (Silkair), filed with the
BIR a written application for the refund of excise taxes it claimed to have paid on its
purchase of jet fuel from Petron. As the BIR did not act on the application, Silkair filed
a Petition for Review before the CTA.
In both cases, the CIR argued that the excise tax on petroleum products is the direct
liability of the manufacturer/producer, and when added to the cost of the goods sold
to the buyer, it is no longer a tax but part of the price which the buyer has to pay to
obtain the article.

In the first Silkair case, the Court ruled:


The proper party to question, or seek a refund of, an indirect tax is the
statutory taxpayer, the person on whom the tax is imposed by law and who
paid the same even if he shifts the burden thereof to another.Section 130 (A)
(2) of the NIRC provides that "[u]nless otherwise specifically allowed, the return shall
be filed and the excise tax paid by the manufacturer or producer before removal of
domestic products from place of production." Thus, Petron Corporation, not Silkair, is
the statutory taxpayer which is entitled to claim a refund based on Section 135 of
the NIRC of 1997 and Article 4(2) of the Air Transport Agreement between RP and
Singapore.
Even if Petron Corporation passed on to Silkair the burden of the tax, the
additional amount billed to Silkair for jet fuel is not a tax but part of the
price which Silkair had to pay as a purchaser. 86 (Emphasis and underscoring
supplied.)
Citing the above case, the second Silkair case was promulgated a few months after
the first, and stated:
The issue presented is not novel. In a similar case involving the same parties, this
Court has categorically ruled that "the proper party to question, or seek a refund of
an indirect tax is the statutory taxpayer, the person on whom the tax is imposed by
law and who paid the same even if he shifts the burden thereof to another." The
Court added that "even if Petron Corporation passed on to Silkair the burden of the
tax, the additional amount billed to Silkair for jet fuel is not a tax but part of the
price which Silkair had to pay as a purchaser." 87
The CTA En Banc, thus, held that:
The determination of who is the taxpayer plays a pivotal role in claims for refund
because the same law provides that it is only the taxpayer who has the legal
personality to ask for a refund in case of erroneous payment of taxes. Section 204
(C) of the 1997 NIRC, [provides] in part, as follows:
SEC. 204. Authority of the Commissioner to Compromise, Abate, and Refund or
Credit Taxes. The Commissioner may
xxx

xxx

xxx

(C) Credit or refund taxes erroneously or illegally received or penalties


imposed without authority, refund the value of internal revenue stamps when they
are returned in good condition by the purchaser, and, in his discretion, redeem or
change unused stamps that have been rendered unfit for use and refund their value
upon proof of destruction. No credit or refund of taxes or penalties shall be
allowed unless the taxpayer files in writing with the Commissioner a claim

for credit or refund within two (2) years after the payment of the tax or penalty:
Provided, however, That a return showing an overpayment shall be considered as a
written claim for credit or refund.
xxx

xxx

xxx

(Emphasis shown supplied by the CTA.) 88


Therefore, as Exxon is not the party statutorily liable for payment of excise taxes
under Section 130, in relation to Section 129 of the NIRC, it is not the proper party to
claim a refund of any taxes erroneously paid.
There is no unilateral amendment of existing bilateral agreements of the Philippines
with other countries.
Exxon also argues that in effectively holding that only petroleum products purchased
directly from the manufacturers or producers are exempt from excise taxes, the CTA
En Banc sanctioned a unilateral amendment of existing bilateral agreements which
the Philippines has with other countries, in violation of the basic international law
principle of pacta sunt servanda.89 The Court does not agree.
As correctly held by the CTA En Banc:
One final point, petitioners argument "that in effectively holding that only petroleum
products purchased directly from the manufacturers or producers are exempt from
excise taxes, the First Division of this Court sanctioned a unilateral amendment of
existing bilateral agreements which the Philippines have (sic) with other countries, in
violation of the basic international principle of "pacta sunt servanda" is misplaced.
First, the findings of fact of the First Division of this Court that "when petitioner sold
the Jet A-1 fuel to international carriers, it did so free of tax"negates any violation of
the exemption from excise tax of the petroleum products sold to international
carriers insofar as this case is concerned. Secondly, the right of international carriers
to invoke the exemption granted under Section 135 (a) of the 1997 NIRC has neither
been affected nor restricted in any way by the ruling of the First Division of this
Court. At the point of sale, the international carriers are free to invoke the exemption
from excise taxes of the petroleum products sold to them. Lastly, the law-making
body is presumed to have enacted a later law with the knowledge of all other laws
involving the same subject matter."90 (Underscoring supplied.)
WHEREFORE, the petition is DENIED.
SO ORDERED.
G.R. No. 190102

July 11, 2012

ACCENTURE, INC., Petitioner,


vs.
COMMISSIONER OF INTERNAL REVENUE, Respondent.
DECISION
SERENO, J.:
This is a Petition filed under Rule 45 of the 1997 Rules of Civil Procedure, praying for
the reversal of the Decision of the Court of Tax Appeals En Banc (CTA En Banc )
dated 22 September 2009 and its subsequent Resolution dated 23 October 2009. 1
Accenture, Inc. (Accenture) is a corporation engaged in the business of providing
management consulting, business strategies development, and selling and/or
licensing of software.2 It is duly registered with the Bureau of Internal Revenue (BIR)
as a Value Added Tax (VAT) taxpayer or enterprise in accordance with Section 236 of
the National Internal Revenue Code (Tax Code). 3
On 9 August 2002, Accenture filed its Monthly VAT Return for the period 1 July 2002
to 31 August 2002 (1st period). Its Quarterly VAT Return for the fourth quarter of
2002, which covers the 1st period, was filed on 17 September 2002; and an
Amended Quarterly VAT Return, on 21 June 2004. 4 The following are reflected in
Accentures VAT Return for the fourth quarter of 2002: 5
1wphi1
Purchases

Amount

Input VAT

Domestic Purchases- Capital Goods

P12,312,722.0
0

P1,231,272.20

Domestic Purchases- Goods other than capital


Goods

P64,789,507.9
0

P6,478,950.79

Domestic Purchases- Services

P16,455,868.1
0

P1,645,586.81

Total Input Tax

P9,355,809.80

Zero-rated Sales

P316,113,513.3
4

Total Sales

P335,640,544.7
4

Accenture filed its Monthly VAT Return for the month of September 2002 on 24
October 2002; and that for October 2002, on 12 November 2002. These returns were
amended on 9 January 2003. Accentures Quarterly VAT Return for the first quarter of
2003, which included the period 1 September 2002 to 30 November 2002 (2nd

period), was filed on 17 December 2002; and the Amended Quarterly VAT Return, on
18 June 2004. The latter contains the following information: 6
Purchases

Amount

Input VAT

Domestic Purchases- Capital Goods

P80,765,294.10

P8,076,529.41

Domestic Purchases- Goods other than capital


Goods

P132,820,541.7
P13,282,054.17
0

Domestic Purchases-Services

P63,238,758.00

P6,323,875.80

Total Input Tax

P27,682,459.3
8

Zero-rated Sales

P545,686,639.1
8

Total Sales

P572,880,982.6
8

The monthly and quarterly VAT returns of Accenture show that, notwithstanding its
application of the input VAT credits earned from its zero-rated transactions against
its output VAT liabilities, it still had excess or unutilized input VAT credits. These VAT
credits are in the amounts of P9,355,809.80 for the 1st period and P27,682,459.38
for the 2nd period, or a total of P37,038,269.18. 7
Out of the P37,038,269.18, only P35,178,844.21 pertained to the allocated input VAT
on Accentures "domestic purchases of taxable goods which cannot be directly
attributed to its zero-rated sale of services." 8 This allocated input VAT was broken
down to P8,811,301.66 for the 1st period and P26,367,542.55 for the 2nd period. 9
The excess input VAT was not applied to any output VAT that Accenture was liable for
in the same quarter when the amount was earnedor to any of the succeeding
quarters. Instead, it was carried forward to petitioners 2nd Quarterly VAT Return for
2003.10
Thus, on 1 July 2004, Accenture filed with the Department of Finance (DoF) an
administrative claim for the refund or the issuance of a Tax Credit Certificate (TCC).
The DoF did not act on the claim of Accenture. Hence, on 31 August 2004, the latter
filed a Petition for Review with the First Division of the Court of Tax Appeals
(Division), praying for the issuance of a TCC in its favor in the amount of
P35,178,844.21.
The Commissioner of Internal Revenue (CIR), in its Answer, 11 argued thus:
1. The sale by Accenture of goods and services to its clients are not zero-rated
transactions.

2. Claims for refund are construed strictly against the claimant, and Accenture
has failed to prove that it is entitled to a refund, because its claim has not
been fully substantiated or documented.
In a 13 November 2008 Decision,12 the Division denied the Petition of Accenture for
failing to prove that the latters sale of services to the alleged foreign clients
qualified for zero percent VAT.13
In resolving the sole issue of whether or not Accenture was entitled to a refund or an
issuance of a TCC in the amount of P35,178,844.21, 14 the Division ruled that
Accenture had failed to present evidence to prove that the foreign clients to which
the former rendered services did business outside the Philippines. 15 Ruling that
Accentures services would qualify for zero-rating under the 1997 National Internal
Revenue Code of the Philippines (Tax Code) only if the recipient of the services was
doing business outside of the Philippines, 16 the Division cited Commissioner of
Internal Revenue v. Burmeister and Wain Scandinavian Contractor Mindanao, Inc.
(Burmeister)17 as basis.
Accenture appealed the Divisions Decision through a Motion for Reconsideration
(MR).18 In its MR, it argued that the reliance of the Division on Burmeister was
misplaced19 for the following reasons:
1. The issue involved in Burmeister was the entitlement of the applicant to a
refund, given that the recipient of its service was doing business in the
Philippines; it was not an issue of failure of the applicant to present evidence
to prove the fact that the recipient of its services was a foreign corporation
doing business outside the Philippines. 20
2. Burmeister emphasized that, to qualify for zero-rating, the recipient of the
services should be doing business outside the Philippines, and Accenture had
successfully established that.21
3. Having been promulgated on 22 January 2007 or after Accenture filed its
Petition with the Division, Burmeister cannot be made to apply to this case. 22
Accenture also cited Commissioner of Internal Revenue v. American Express
(Amex)23 in support of its position. The MR was denied by the Division in its 12 March
2009 Resolution.24
Accenture appealed to the CTA En Banc. There it argued that prior to the amendment
introduced by Republic Act No. (R.A.) 9337, 25 there was no requirement that the
services must be rendered to a person engaged in business conducted outside the
Philippines to qualify for zero-rating. The CTA En Banc agreed that because the case
pertained to the third and the fourth quarters of taxable year 2002, the applicable
law was the 1997 Tax Code, and not R.A. 9337. 26 Still, it ruled that even though the
provision used in Burmeister was Section 102(b)(2) of the earlier 1977 Tax Code, the
pronouncement therein requiring recipients of services to be engaged in business
outside the Philippines to qualify for zero-rating was applicable to the case at bar,

because Section 108(B)(2) of the 1997 Tax Code was a mere reenactment of Section
102(b)(2) of the 1977 Tax Code.
The CTA En Banc concluded that Accenture failed to discharge the burden of proving
the latters allegation that its clients were foreign-based. 27
Resolute, Accenture filed a Petition for Review with the CTA En Banc, but the latter
affirmed the Divisions Decision and Resolution. 28 A subsequent MR was also denied
in a Resolution dated 23 October 2009.
Hence, the present Petition for Review29 under Rule 45.
In a Joint Stipulation of Facts and Issues, the parties and the Division have agreed to
submit the following issues for resolution:
1. Whether or not Petitioners sales of goods and services are zero-rated for
VAT purposes under Section 108(B)(2)(3) of the 1997 Tax Code.
2. Whether or not petitioners claim for refund/tax credit in the amount of
P35,178,884.21 represents unutilized input VAT paid on its domestic
purchases of goods and services for the period commencing from 1 July 2002
until 30 November 2002.
3. Whether or not Petitioner has carried over to the succeeding taxable
quarter(s) or year(s) the alleged unutilized input VAT paid on its domestic
purchases of goods and services for the period commencing from 1 July 2002
until 30 November 2002, and applied the same fully to its output VAT liability
for the said period.
4. Whether or not Petitioner is entitled to the refund of the amount of
P35,178,884.21, representing the unutilized input VAT on domestic purchases
of goods and services for the period commencing from 1 July 2002 until 30
November 2002, from its sales of services to various foreign clients.
5. Whether or not Petitioners claim for refund/tax credit in the amount of
P35,178,884.21, as alleged unutilized input VAT on domestic purchases of
goods and services for the period covering 1 July 2002 until 30 November
2002 are duly substantiated by proper documents. 30
For consideration in the present Petition are the following issues:
1. Should the recipient of the services be "doing business outside the
Philippines" for the transaction to be zero-rated under Section 108(B)(2) of the
1997 Tax Code?
2. Has Accenture successfully proven that its clients are entities doing
business outside the Philippines?

Recipient of services must be doing business outside the Philippines for the
transactions to qualify as zero-rated.
Accenture anchors its refund claim on Section 112(A) of the 1997 Tax Code, which
allows the refund of unutilized input VAT earned from zero-rated or effectively zerorated sales. The provision reads:
SEC. 112. Refunds or Tax Credits of Input Tax. (A) Zero-Rated or Effectively Zero-Rated Sales. - Any VAT-registered person, whose
sales are zero-rated or effectively zero-rated may, within two (2) years after the
close of the taxable quarter when the sales were made, apply for the issuance of a
tax credit certificate or refund of creditable input tax due or paid attributable to such
sales, except transitional input tax, to the extent that such input tax has not been
applied against output tax: Provided, however, That in the case of zero-rated sales
under Section 106(A)(2)(a)(1), (2) and (B) and Section 108 (B)(1) and (2), the
acceptable foreign currency exchange proceeds thereof had been duly accounted for
in accordance with the rules and regulations of the Bangko Sentral ng Pilipinas (BSP):
Provided, further, That where the taxpayer is engaged in zero-rated or effectively
zero-rated sale and also in taxable or exempt sale of goods of properties or services,
and the amount of creditable input tax due or paid cannot be directly and entirely
attributed to any one of the transactions, it shall be allocated proportionately on the
basis of the volume of sales. Section 108(B) referred to in the foregoing provision
was first seen when Presidential Decree No. (P.D.) 1994 31 amended Title IV of P.D.
1158,32 which is also known as the National Internal Revenue Code of 1977. Several
Decisions have referred to this as the 1986 Tax Code, even though it merely
amended Title IV of the 1977 Tax Code.
Two years thereafter, or on 1 January 1988, Executive Order No. (E.O.) 273 33 further
amended provisions of Title IV. E.O. 273 by transferring the old Title IV provisions to
Title VI and filling in the former title with new provisions that imposed a VAT.
The VAT system introduced in E.O. 273 was restructured through Republic Act No.
(R.A.) 7716.34 This law, which was approved on 5 May 1994, widened the tax base.
Section 3 thereof reads:
SECTION 3. Section 102 of the National Internal Revenue Code, as amended, is
hereby further amended to read as follows:
"SEC. 102. Value-added tax on sale of services and use or lease of properties. x x x
xxx

xxx

xxx

"(b) Transactions subject to zero-rate. The following services performed in the


Philippines by VAT-registered persons shall be subject to 0%:
"(1) Processing, manufacturing or repacking goods for other persons doing
business outside the Philippines which goods are subsequently exported,
where the services are paid for in acceptable foreign currency and accounted

for in accordance with the rules and regulations of the Bangko Sentral ng
Pilipinas (BSP).
"(2) Services other than those mentioned in the preceding sub-paragraph, the
consideration for which is paid for in acceptable foreign currency and
accounted for in accordance with the rules and regulations of the Bangko
Sentral ng Pilipinas (BSP)."
Essentially, Section 102(b) of the 1977 Tax Codeas amended by P.D. 1994, E.O.
273, and R.A. 7716provides that if the consideration for the services provided by a
VAT-registered person is in a foreign currency, then this transaction shall be
subjected to zero percent rate.
The 1997 Tax Code reproduced Section 102(b) of the 1977 Tax Code in its Section
108(B), to wit:
(B) Transactions Subject to Zero Percent (0%) Rate. - The following services
performed in the Philippines by VAT- registered persons shall be subject to zero
percent (0%) rate.
(1) Processing, manufacturing or repacking goods for other persons doing
business outside the Philippines which goods are subsequently exported,
where the services are paid for in acceptable foreign currency and accounted
for in accordance with the rules and regulations of the Bangko Sentral ng
Pilipinas (BSP);
(2) Services other than those mentioned in the preceding paragraph, the
consideration for which is paid for in acceptable foreign currency and
accounted for in accordance with the rules and regulations of the Bangko
Sentral ng Pilipinas (BSP); x x x.
On 1 November 2005, Section 6 of R.A. 9337, which amended the foregoing
provision, became effective. It reads:
SEC. 6. Section 108 of the same Code, as amended, is hereby further amended to
read as follows:
"SEC. 108. Value-added Tax on Sale of Services and Use or Lease of
Properties. (B) Transactions Subject to Zero Percent (0%) Rate. - The following services
performed in the Philippines by VAT-registered persons shall be subject to zero
percent (0%) rate:
(1) Processing, manufacturing or repacking goods for other persons doing
business outside the Philippines which goods are subsequently exported,
where the services are paid for in acceptable foreign currency and accounted

for in accordance with the rules and regulations of the Bangko Sentral ng
Pilipinas (BSP);
"(2) Services other than those mentioned in the preceding paragraph rendered
to a person engaged in business conducted outside the Philippines or to a
nonresident person not engaged in business who is outside the Philippines
when the services are performed, the consideration for which is paid for in
acceptable foreign currency and accounted for in accordance with the rules
and regulations of the Bangko Sentral ng Pilipinas (BSP); x x x." (Emphasis
supplied)
The meat of Accentures argument is that nowhere does Section 108(B) of the 1997
Tax Code state that services, to be zero-rated, should be rendered to clients doing
business outside the Philippines, the requirement introduced by R.A.
9337.35 Required by Section 108(B), prior to the amendment, is that the
consideration for the services rendered be in foreign currency and in accordance
with the rules of the Bangko Sentral ng Pilipinas (BSP). Since Accenture has complied
with all the conditions imposed in Section 108(B), it is entitled to the refund prayed
for.
In support of its claim, Accenture cites Amex, in which this Court supposedly ruled
that Section 108(B) reveals a clear intent on the part of the legislators not to impose
the condition of being "consumed abroad" in order for the services performed in the
Philippines to be zero-rated.36
The Division ruled that this Court, in Amex and Burmeister, did not declare that the
requirementthat the client must be doing business outside the Philippinescan be
disregarded, because this requirement is expressly provided in Article 108(2) of the
Tax Code.37
Accenture questions the Divisions application to this case of the pronouncements
made in Burmeister. According to petitioner, the provision applied to the present
case was Section 102(b) of the 1977 Tax Code, and not Section 108(B) of the 1997
Tax Code, which was the law effective when the subject transactions were entered
into and a refund was applied for.
In refuting Accentures theory, the CTA En Banc ruled that since Section 108(B) of
the 1997 Tax Code was a mere reproduction of Section 102(b) of the 1977 Tax Code,
this Courts interpretation of the latter may be used in interpreting the former, viz:
In the Burmeister case, the Supreme Court harmonized both Sections 102(b)(1) and
102(b)(2) of the 1977 Tax Code, as amended, pertaining to zero-rated transactions. A
parallel approach should be accorded to the renumbered provisions of Sections
108(B)(2) and 108(B)(1) of the 1997 NIRC. This means that Section 108(B)(2) must
be read in conjunction with Section 108(B)(1). Section 108(B)(2) requires as follows:
a) services other than processing, manufacturing or repacking rendered by VAT
registered persons in the Philippines; and b) the transaction paid for in acceptable
foreign currency duly accounted for in accordance with BSP rules and regulations.
The same provision made reference to Section 108(B)(1) further imposing the

requisite c) that the recipient of services must be performing business outside of


Philippines. Otherwise, if both the provider and recipient of service are doing
business in the Philippines, the sale transaction is subject to regular VAT as explained
in the Burmeister case x x x.
xxx

xxx

xxx

Clearly, the Supreme Courts pronouncements in the Burmeister case requiring that
the recipient of the services must be doing business outside the Philippines as
mandated by law govern the instant case.38
Assuming that the foregoing is true, Accenture still argues that the tax appeals
courts cannot be allowed to apply to Burmeister this Courts interpretation of Section
102(b) of the 1977 Tax Code, because the Petition of Accenture had already been
filed before the case was even promulgated on 22 January 2007, 39 to wit:
x x x. While the Burmeister case forms part of the legal system and assumes the
same authority as the statute itself, however, the same cannot be applied
retroactively against the Petitioner because to do so will be prejudicial to the latter. 40
The CTA en banc is of the opinion that Accenture cannot invoke the non-retroactivity
of the rulings of the Supreme Court, whose interpretation of the law is part of that
law as of the date of its enactment.41
We rule that the recipient of the service must be doing business outside the
Philippines for the transaction to qualify for zero-rating under Section 108(B) of the
Tax Code.
This Court upholds the position of the CTA en banc that, because Section 108(B) of
the 1997 Tax Code is a verbatim copy of Section 102(b) of the 1977 Tax Code, any
interpretation of the latter holds true for the former.
Moreover, even though Accentures Petition was filed before Burmeister was
promulgated, the pronouncements made in that case may be applied to the present
one without violating the rule against retroactive application. When this Court
decides a case, it does not pass a new law, but merely interprets a preexisting
one.42 When this Court interpreted Section 102(b) of the 1977 Tax Code in
Burmeister, this interpretation became part of the law from the moment it became
effective. It is elementary that the interpretation of a law by this Court constitutes
part of that law from the date it was originally passed, since this Court's construction
merely establishes the contemporaneous legislative intent that the interpreted law
carried into effect.43
Accenture questions the CTAs application of Burmeister, because the provision
interpreted therein was Section 102(b) of the 1977 Tax Code. In support of its
position that Section 108 of the 1997 Tax Code does not require that the services be
rendered to an entity doing business outside the Philippines, Accenture invokes this
Courts pronouncements in Amex. However, a reading of that case will readily reveal
that the provision applied was Section 102(b) of the 1977 Tax Code, and not Section

108 of the 1997 Tax Code. As previously mentioned, an interpretation of Section


102(b) of the 1977 Tax Code is an interpretation of Section 108 of the 1997 Tax
Code, the latter being a mere reproduction of the former.
This Court further finds that Accentures reliance on Amex is misplaced.
We ruled in Amex that Section 102 of the 1977 Tax Code does not require that the
services be consumed abroad to be zero-rated. However, nowhere in that case did
this Court discuss the necessary qualification of the recipient of the service, as this
matter was never put in question. In fact, the recipient of the service in Amex is a
nonresident foreign client.
The aforementioned case explains how the credit card system works. The issuance of
a credit card allows the holder thereof to obtain, on credit, goods and services from
certain establishments. As proof that this credit is extended by the establishment, a
credit card draft is issued. Thereafter, the company issuing the credit card will pay
for the purchases of the credit card holders by redeeming the drafts. The obligation
to collect from the card holders and to bear the lossin case they do not payrests
on the issuer of the credit card.
The service provided by respondent in Amex consisted of gathering the bills and
credit card drafts from establishments located in the Philippines and forwarding them
to its parent company's regional operating centers outside the country. It facilitated
in the Philippines the collection and payment of receivables belonging to its Hong
Kong-based foreign client.
The Court explained how the services rendered in Amex were considered to have
been performed and consumed in the Philippines, to wit:
Consumption is "the use of a thing in a way that thereby exhausts it." Applied to
services, the term means the performance or "successful completion of a contractual
duty, usually resulting in the performers release from any past or future liability x x
x." The services rendered by respondent are performed or successfully completed
upon its sending to its foreign client the drafts and bills it has gathered from service
establishments here. Its services, having been performed in the Philippines, are
therefore also consumed in the Philippines. 44
The effect of the place of consumption on the zero-rating of the transaction was not
the issue in Burmeister.1wphi1Instead, this Court addressed the squarely raised
issue of whether the recipient of services should be doing business outside the
Philippines for the transaction to qualify for zero-rating. We ruled that it should. Thus,
another essential condition for qualification for zero-rating under Section 102(b)(2) of
the 1977 Tax Code is that the recipient of the business be doing that business
outside the Philippines. In clarifying that there is no conflict between this
pronouncement and that laid down in Amex, we ruled thus:
x x x. As the Court held in Commissioner of Internal Revenue v. American Express
International, Inc. (Philippine Branch), the place of payment is immaterial, much less
is the place where the output of the service is ultimately used. An essential condition

for entitlement to 0% VAT under Section 102 (b) (1) and (2) is that the recipient of
the services is a person doing business outside the Philippines. In this case, the
recipient of the services is the Consortium, which is doing business not outside, but
within the Philippines because it has a 15-year contract to operate and maintain
NAPOCORs two 100-megawatt power barges in Mindanao. (Emphasis in the
original)45
In Amex we ruled that the place of performance and/or consumption of the service is
immaterial. In Burmeister, the Court found that, although the place of the
consumption of the service does not affect the entitlement of a transaction to zerorating, the place where the recipient conducts its business does.
Amex does not conflict with Burmeister. In fact, to fully understand how Section
102(b)(2) of the 1977 Tax Codeand consequently Section 108(B)(2) of the 1997 Tax
Codewas intended to operate, the two aforementioned cases should be taken
together. The zero-rating of the services performed by respondent in Amex was
affirmed by the Court, because although the services rendered were both performed
and consumed in the Philippines, the recipient of the service was still an entity doing
business outside the Philippines as required in Burmeister.
That the recipient of the service should be doing business outside the Philippines to
qualify for zero-rating is the only logical interpretation of Section 102(b)(2) of the
1977 Tax Code, as we explained in Burmeister:
This can only be the logical interpretation of Section 102 (b) (2). If the provider and
recipient of the "other services" are both doing business in the Philippines, the
payment of foreign currency is irrelevant. Otherwise, those subject to the regular VAT
under Section 102 (a) can avoid paying the VAT by simply stipulating payment in
foreign currency inwardly remitted by the recipient of services. To interpret Section
102 (b) (2) to apply to a payer-recipient of services doing business in the Philippines
is to make the payment of the regular VAT under Section 102 (a) dependent on the
generosity of the taxpayer. The provider of services can choose to pay the regular
VAT or avoid it by stipulating payment in foreign currency inwardly remitted by the
payer-recipient. Such interpretation removes Section 102 (a) as a tax measure in the
Tax Code, an interpretation this Court cannot sanction. A tax is a mandatory
exaction, not a voluntary contribution.
xxx

xxx

xxx

Further, when the provider and recipient of services are both doing business in the
Philippines, their transaction falls squarely under Section 102 (a) governing domestic
sale or exchange of services. Indeed, this is a purely local sale or exchange of
services subject to the regular VAT, unless of course the transaction falls under the
other provisions of Section 102 (b).
Thus, when Section 102 (b) (2) speaks of "services other than those mentioned in
the preceding subparagraph," the legislative intent is that only the services are
different between subparagraphs 1 and 2. The requirements for zero-rating,
including the essential condition that the recipient of services is doing business

outside the Philippines, remain the same under both subparagraphs. (Emphasis in
the original)46
Lastly, it is worth mentioning that prior to the promulgation of Burmeister, Congress
had already clarified the intent behind Sections 102(b)(2) of the 1977 Tax Code and
108(B)(2) of the 1997 Tax Code amending the earlier provision. R.A. 9337 added the
following phrase: "rendered to a person engaged in business conducted outside the
Philippines or to a nonresident person not engaged in business who is outside the
Philippines when the services are performed."
Accenture has failed to establish that the recipients of its services do business
outside the Philippines.
Accenture argues that based on the documentary evidence it presented, 47 it was able
to establish the following circumstances:
1. The records of the Securities and Exchange Commission (SEC) show that
Accentures clients have not established any branch office in which to do
business in the Philippines.
2. For these services, Accenture bills another corporation, Accenture
Participations B.V. (APB), which is likewise a foreign corporation with no
"presence in the Philippines."
3. Only those not doing business in the Philippines can be required under BSP
rules to pay in acceptable currency for their purchase of goods and services
from the Philippines. Thus, in a domestic transaction, where the provider and
recipient of services are both doing business in the Philippines, the BSP cannot
require any party to make payment in foreign currency. 48
Accenture claims that these documentary pieces of evidence are supported by the
Report of Emmanuel Mendoza, the Court-commissioned Independent Certified Public
Accountant. He ascertained that Accentures gross billings pertaining to zero-rated
sales were all supported by zero-rated Official Receipts and Billing Statements. These
documents show that these zero-rated sales were paid in foreign exchange currency
and duly accounted for in the rules and regulations of the BSP. 49
In the CTAs opinion, however, the documents presented by Accenture merely
substantiate the existence of the sales, receipt of foreign currency payments, and
inward remittance of the proceeds of these sales duly accounted for in accordance
with BSP rules. Petitioner presented no evidence whatsoever that these clients were
doing business outside the Philippines. 50
Accenture insists, however, that it was able to establish that it had rendered services
to foreign corporations doing business outside the Philippines, unlike in Burmeister,
which allegedly involved a foreign corporation doing business in the Philippines. 51
We deny Accentures Petition for a tax refund.

The evidence presented by Accenture may have established that its clients are
foreign.1wphi1 This fact does not automatically mean, however, that these clients
were doing business outside the Philippines. After all, the Tax Code itself has
provisions for a foreign corporation engaged in business within the Philippines and
vice versa, to wit:
SEC. 22. Definitions - When used in this Title:
xxx

xxx

xxx

(H) The term "resident foreign corporation" applies to a foreign corporation


engaged in trade or business within the Philippines.
(I) The term nonresident foreign corporation applies to a foreign corporation
not engaged in trade or business within the Philippines. (Emphasis in the
original)
Consequently, to come within the purview of Section 108(B)(2), it is not enough that
the recipient of the service be proven to be a foreign corporation; rather, it must be
specifically proven to be a nonresident foreign corporation.
There is no specific criterion as to what constitutes "doing" or "engaging in" or
"transacting" business. We ruled thus in Commissioner of Internal Revenue v. British
Overseas Airways Corporation:52
x x x. There is no specific criterion as to what constitutes "doing" or "engaging in" or
"transacting" business. Each case must be judged in the light of its peculiar
environmental circumstances. The term implies a continuity of commercial dealings
and arrangements, and contemplates, to that extent, the performance of acts or
works or the exercise of some of the functions normally incident to, and in
progressive prosecution of commercial gain or for the purpose and object of the
business organization. "In order that a foreign corporation may be regarded as doing
business within a State, there must be continuity of conduct and intention to
establish a continuous business, such as the appointment of a local agent, and not
one of a temporary character."53
A taxpayer claiming a tax credit or refund has the burden of proof to establish the
factual basis of that claim.1wphi1 Tax refunds, like tax exemptions, are construed
strictly against the taxpayer.54
Accenture failed to discharge this burden. It alleged and presented evidence to prove
only that its clients were foreign entities. However, as found by both the CTA Division
and the CTA En Banc, no evidence was presented by Accenture to prove the fact that
the foreign clients to whom petitioner rendered its services were clients doing
business outside the Philippines.
As ruled by the CTA En Banc, the Official Receipts, Intercompany Payment Requests,
Billing Statements, Memo Invoices-Receivable, Memo Invoices-Payable, and Bank
Statements presented by Accenture merely substantiated the existence of sales,

receipt of foreign currency payments, and inward remittance of the proceeds of such
sales duly accounted for in accordance with BSP rules, all of these were devoid of
any evidence that the clients were doing business outside of the Philippines. 55
WHEREFORE, the instant Petition is DENIED. The 22 September 2009 Decision and
the 23 October 2009 Resolution of the Court of Tax Appeals En Banc in C.T.A. EB No.
477, dismissing the Petition for the refund of the excess or unutilized input VAT
credits of Accenture, Inc., are AFFIRMED.
SO ORDERED.
G.R. No. 197151

October 22, 2012

SM LAND, INC. (Formerly Shoemart, Inc.) and WATSONS PERSONAL CARE


STORES, PHILS., INC.,Petitioners,
vs.
CITY OF MANILA, LIBERTY TOLEDO, in her official capacity as the City
Treasurer of Manila and JOSEPH SANTIAGO, in his official capacity as the
Chief of License Division of the City of Manila, Respondents.
DECISION
PERALTA, J.:
Before the Court is a petition for review on certiorari under Rule 45 of the Rules of
Court seeking to reverse and set aside the Decision 1 and Resolution2 of the Court of
Tax Appeals (CTA) En Banc, dated December 17, 2010 and May 27, 2011,
respectively, in CTA EB No. 548. The assailed Decision affirmed the July 3, 2009
Decision3 and September 30, 2009 Resolution4 of the CTA Second Division in CTA AC
No. 51, while the questioned Resolution denied herein petitioners' Motion for
Reconsideration.
The factual and procedural antecedents of the case are as follows:
On the strength of the provisions of Tax Ordinance Nos. 7988 and
8011, which amended Ordinance No. 7794, also known as the Revenue Code of
Manila, herein respondent City of Manila assessed herein petitioners, together with
their other sister companies, increased rates of business taxes for the year 2003 and
the first to third quarters of 2004.
Petitioners and their sister companies paid the additional taxes under protest.
Subsequently, petitioners and their sister companies claimed with herein respondent
City Treasurer of Manila a credit or refund of the increased business taxes which they
paid for the period abovementioned. However, the City Treasurer denied their claim.

Aggrieved, petitioners and their sister companies filed with the Regional Trial Court
(RTC) of Pasay City a Complaint for Refund and/or Issuance of Tax Credit of Taxes
Illegally Collected.5
On July 10, 2007, the RTC rendered a summary judgment in favor of herein
petitioners, disposing as follows:
WHEREFORE, this Court renders judgment in plaintiffs favor and directs the
defendants to grant a refund/tax credit:
(a) To Plaintff SM Mart, Inc.
i. The amount of P3,543,318.97 representing overpayment of increased local
business taxes under Sections 15, 16, 17, 18, and 19, under the rates imposed
by Ordinance Nos. 7988 and 8011, and
ii. The amount of P17,519,133.16 representing payment of the Section 21 tax;
(b) To Plaintiff SM Prime Holdings, Inc.
i. The amount of P667,377.21 representing overpayment of increased local
business taxes under Sections 15, 16, 17, 18, and 19, under the rates imposed
by Ordinance Nos. 7988 and 8011, and
ii. The amount of P6,711,068.38 representing payment of the Section 21 tax;
(c) To Plaintiff Shoemart, Inc.
i. The amount of P691,887.07 representing overpayment of increased local
business taxes under Section 17, under the rates imposed by Ordinance Nos.
7988 and 8011, and
ii. The amount of P2,954,520.24 representing payment of the Section 21 tax;
(d) To Plaintiff Star Appliances Center
i. The amount of P700,974.98 representing overpayment of increased local
business taxes under Section 17, under the rates imposed by Ordinance Nos.
7988 and 8011, and
ii. The amount of P3,459,812.76 representing payment of the Section 21 tax;
(e) To Plaintiff Supervalue, Inc.

i. The amount of P1,360,984.69 representing overpayment of increased local


business taxes under Sections 17 and 18, under the rates imposed by
Ordinance Nos. 7988 and 8011, and
ii. The amount of P2,774,859.82 representing payment of the Section 21 tax;
(f) To Plaintiff Ace Hardware Philippines, Inc.
i. The amount of P202,175.67 representing overpayment of increased local
business taxes under Section 17, under the rates imposed by Ordinance Nos.
7988 and 8011, and
ii. The amount of P988,347.16 representing payment of the Section 21 tax;
(g) To Plaintiff Watsons Personal Care Stores Philippines, Inc.
i. The amount of P214,667.73 representing overpayment of increased local
business taxes under Section 17, under the rates imposed by Ordinance Nos.
7988 and 8011, and
ii. The amount of P636,857.15 representing payment of the Section 21 tax;
(h) To Plaintiff Jollimart Phils., Corp.
i. The amount of P98,223.61 representing overpayment of increased local
business taxes under Section 17, under the rates imposed by Ordinance Nos.
7988 and 8011, and
ii. The amount of P296,178.13 representing payment of the Section 21 tax;
(i) To Plaintiff Surplus Marketing Corporation
i. The amount of P84,494.76 representing overpayment of increased local
business taxes under Section 17, under the rates imposed by Ordinance Nos.
7988 and 8011, and
ii. The amount of P399,942.81 representing payment of the Section 21 tax;
(j) To Plaintiff Signature Lines
i. The amount of P49,566.91 representing overpayment of increased local
business taxes under Section 17, under the rates imposed by Ordinance Nos.
7988 and 8011, and
ii. The amount of P222,565.79 representing payment of the Section 21 tax.

No Costs.
SO ORDERED.6
The RTC held that Tax Ordinance Nos. 7988 and 8011, which were the bases of the
City of Manila in imposing the assailed additional business taxes on petitioners and
their co-plaintiffs, had already been declared null and void by this Court in the case
of Coca-Cola Bottlers Philippines, Inc. v. City of Manila. 7 On this ground, the RTC ruled
that respondents cannot use the assailed Ordinances in imposing additional taxes on
petitioners and their co-plaintiffs.
Respondents moved for reconsideration, but the RTC denied it in its Order dated
December 14, 2007.
After the CTA granted their request for extension of time, herein respondents filed a
petition for review with the tax court.8 The case was raffled to the Second Division of
the said court.
On July 3, 2009, the CTA Second Division rendered its Decision, the dispositive
portion of which reads, thus:
WHEREFORE, premises considered, the instant Petition for Review is hereby
PARTIALLY GRANTED.1wphi1 The appealed Order dated July 10, 2007 and Order
dated December 14, 2007 of the Regional Trial Court of Pasay City, Branch 115, in
Civil Case No. 05-0051-CFM are hereby MODIFIED. Accordingly, with the exception of
Shoemart, Inc. and Watsons Personal Care Stores, Phils., petitioners are hereby
ORDERED to REFUND the rest of the respondents, their erroneously paid local
business taxes for taxable year 2003 and for the first to third quarters of taxable
year 2004 in the aggregate amount of THIRTY-NINE MILLION SEVENTY-EIGHT
THOUSAND NINE HUNDRED EIGHTY-EIGHT PESOS AND 81/100 (P39,078,988.81),
detailed as follows:9
The CTA Second Division sustained the ruling of the RTC that Ordinance Nos. 7988
and 8011 are null and void. Applying the doctrine of stare decisis, the CTA Second
Division held that the ruling in the Coca-Cola case cited by the RTC is applicable in
the present case as both cases involve substantially the same facts and issues. The
CTA Second Division, nonetheless, held that herein petitioners' claims for tax refund
should be denied because of their failure to comply with the provisions of the Rules
of Court requiring verification and submission of a certificate of non-forum shopping.
The CTA Second Division noted that petitioners failed to attach to the complaint filed
with the RTC their respective Secretary's Certificates authorizing their supposed
representative, a certain Atty. Rex Enrico V. Cruz III (Atty. Cruz), to file the said
complaint in their behalf. The CTA also observed that in the Verification and
Certification of Non-Forum Shopping attached to the complaint, petitioner SM Land,
Inc. was not included in the list of corporations represented by the person who
executed the said Verification and Certification.

Petitioners filed a Motion for Partial Reconsideration. 10 Attached to the said Motion
was the Verification and Certification executed by Atty. Cruz as the representative of
petitioner SM Land, Inc. Also attached were petitioners' Secretary's Certificates
authorizing Atty. Cruz as their representative. The CTA Second Division, however,
denied the Motion for Partial Reconsideration in its Resolution 11 dated September 30,
2009.
Aggrieved, petitioners filed a petition for review with the CTA En Banc, contending
that: (1) the CTA Second Division erred in holding that the 30-day period provided by
law within which to appeal decisions of the RTC to the CTA may be extended; and (2)
the CTA Second Division committed error in denying herein petitioners' claim for tax
refund on the ground that they violated the rules on verification and certification of
non-forum shopping.
On December 17, 2010, the CTA En Banc rendered its assailed Decision affirming in
toto the judgment of the CTA Second Division.
Petitioners' Motion for Reconsideration was subsequently denied by the CTA En Banc
in its Resolution dated May 27, 2011.
Hence, the present petition anchored on the following arguments:
A. SECTION 11, REPUBLIC ACT NO. 1125, AS AMENDED BY REPUBLIC ACT NO.
9282, CLEARLY DID NOT INTEND FOR THE THIRTY (30)-DAY PERIOD TO APPEAL
DECISIONS OF THE REGIONAL TRIAL COURT TO THE CTA TO BE EXTENDIBLE;
AND
B. ASSUMING HYPOTHETICALLY THAT THE CTA WAS CORRECT IN GRANTING
RESPONDENTS AN EXTENSION, THERE WERE STILL COMPELLING REASONS TO
JUSTIFY THE RELAXATION OF THE RULES REQUIRING VERIFICATION AND
CERTIFICATION OF NON-FORUM SHOPPING.12
The Court finds the petition meritorious. Nonetheless, the Court does not fully agree
with petitioners' contentions.
In the first argument raised, the Court is not persuaded by petitioners insistence
that the 30-day period to appeal decisions of the RTC to the CTA is non-extendible.
Petitioners cited cases decided by this Court wherein it was held that the 30-day
period within which to file an appeal with the CTA is jurisdictional and nonextendible. However, these rulings had been superseded by this Court's decision in
the case of City of Manila v. Coca-Cola Bottlers, Philippines, Inc., 13 as correctly cited
by the CTA En Banc. Suffice it to say that this Court's ruling in the said case is
instructive, to wit:
xxxx

The period to appeal the decision or ruling of the RTC to the CTA via a Petition for
Review is specifically governed by Section 11 of Republic Act No. 9282, and Section 3
(a), Rule 8 of the Revised Rules of the CTA.
Section 11 of Republic Act No. 9282 provides:
SEC. 11. Who May Appeal; Mode of Appeal; Effect of Appeal. Any party adversely
affected by a decision, ruling or inaction of the Commissioner of Internal Revenue,
the Commissioner of Customs, the Secretary of Finance, the Secretary of Trade and
Industry or the Secretary of Agriculture or the Central Board of Assessment Appeals
or the Regional Trial Courts may file an Appeal with the CTA within thirty (30) days
after the receipt of such decision or ruling or after the expiration of the period fixed
by law for action as referred to in Section 7(a)(2) herein.
Appeal shall be made by filing a petition for review under a procedure analogous to
that provided for under Rule 42 of the 1997 Rules of Civil Procedure with the CTA
within thirty (30) days from the receipt of the decision or ruling or in the case of
inaction as herein provided, from the expiration of the period fixed by law to act
thereon. x x x. (Emphasis supplied.)
Section 3(a), Rule 8 of the Revised Rules of the CTA states:
SEC 3. Who may appeal; period to file petition. (a) A party adversely affected by a
decision, ruling or the inaction of the Commissioner of Internal Revenue on disputed
assessments or claims for refund of internal revenue taxes, or by a decision or ruling
of the Commissioner of Customs, the Secretary of Finance, the Secretary of Trade
and Industry, the Secretary of Agriculture, or a Regional Trial Court in the exercise of
its original jurisdiction may appeal to the Court by petition for review filed within
thirty days after receipt of a copy of such decision or ruling, or expiration of the
period fixed by law for the Commissioner of Internal Revenue to act on the disputed
assessments. x x x. (Emphasis supplied.)
It is crystal clear from the afore-quoted provisions that to appeal an adverse decision
or ruling of the RTC to the CTA, the taxpayer must file a Petition for Review with the
CTA within 30 days from receipt of said adverse decision or ruling of the RTC.
It is also true that the same provisions are silent as to whether such 30-day period
can be extended or not. However, Section 11 of Republic Act No. 9282 does state
that the Petition for Review shall be filed with the CTA following the procedure
analogous to Rule 42 of the Revised
Rules of Civil Procedure. Section 1, Rule 42 of the Revised Rules of Civil Procedure
provides that the Petition for Review of an adverse judgment or final order of the RTC
must be filed with the Court of Appeals within: (1) the original 15-day period from
receipt of the judgment or final order to be appealed; (2) an extended period of 15
days from the lapse of the original period; and (3) only for the most compelling

reasons, another extended period not to exceed 15 days from the lapse of the first
extended period.
Following by analogy, Section 1, Rule 42 of the Revised Rules of Civil Procedure, the
30-day original period for filing a Petition for Review with the CTA under Section 11 of
Republic Act No. 9282, as implemented by Section 3 (a), Rule 8 of the Revised Rules
of the CTA, may be extended for a period of 15 days. No further extension shall be
allowed thereafter, except only for the most compelling reasons, in which case the
extended period shall not exceed 15 days.
x x x x14
Petitioners further contend that the Order of the CTA Second Division granting
petitioners' motion for extension to file their petition for review is invalid, because at
the time that the said motion was granted on March 4, 2008, this Court has not yet
promulgated its decision in the above-cited Coca-Cola case. It was only on August 4,
2009 that this Court issued its decision in the said case and, that petitioners reason
out that the same is inapplicable to the instant case as the ruling therein cannot be
applied retroactively. Petitioners argue that, aside from the Coca-Cola case, the CTA
Second Division had no clear statutory authority or jurisprudential basis in granting
petitioners' motion for extension to file their petition for review.
The Court does not agree.
At the time that the CTA Second Division granted petitioners' motion for extension to
file their petition for review, Republic Act 9282 15 (RA 9282), which amended certain
provisions of RA 1125,16 were already in effect,17 and it is clearly provided therein
that appeals from the RTC to the CTA shall follow a procedure analogous to that
provided for under Rule 42 of the Rules of Court. Rule 42 of the said Rules, in turn,
provides that the court may grant an extension of fifteen (15) days within which to
file the petition for review. Thus, independent of the Coca-Cola case, the CTA Second
Division had clear statutory authority in granting petitioners' motion for extension.
This Courts ruling in Coca-Cola is a mere clarification and affirmation of what is
provided for under the provisions of RA 1125, as amended by RA 9282.
Nonetheless, the Court agrees with petitioners' contention in its second argument
that there are compelling reasons in the present case which justify the relaxation of
the rules on verification and certification of non-forum shopping.
It must be kept in mind that while the requirement of the certification of non-forum
shopping is mandatory, nonetheless, the requirements must not be interpreted too
literally and, thus, defeat the objective of preventing the undesirable practice of
forum shopping.18
Time and again, this Court has held that rules of procedure are established to secure
substantial justice.19 Being instruments for the speedy and efficient administration of

justice, they must be used to achieve such end, not to derail it. 20 In particular, when
a strict and literal application of the rules on non-forum shopping and verification will
result in a patent denial of substantial justice, these may be liberally construed. 21
In the instant case, petitioner Watsons' procedural lapse was its belated submission
of a Secretary's Certificate authorizing Atty. Cruz as its representative. On the other
hand, petitioner SM Land, Inc.'s infraction was not only its late submission of its
Secretary's Certificate but also its failure to timely submit its verification and
certification of non-forum shopping.
In a number of cases, this Court has excused the belated filing of the required
verification and certification of non-forum shopping, citing that special circumstances
or compelling reasons make the strict application of the rule clearly unjustified. 22 This
Court ruled that substantial justice and the apparent merits of the substantive
aspect of the case are deemed special circumstances or compelling reasons to relax
the said rule.
In fact, this Court has held that even if there was complete non-compliance with the
rule on certification against forum shopping, the Court may still proceed to decide
the case on the merits, pursuant to its inherent power to suspend its own rules on
grounds, as stated above, of substantial justice and apparent merit of the case. 23
Thus, in Vda. de Formoso v. Philippine National Bank, 24 this Court reiterated, in
capsule form, the rule on non-compliance with the requirements on, or submission of
defective verification and certification of non-forum shopping, to wit:
1) A distinction must be made between non-compliance with the requirement
on or submission of defective verification, and non-compliance with the
requirement on or submission of defective certification against forum
shopping.
2) As to verification, non-compliance therewith or a defect therein does not
necessarily render the pleading fatally defective. The Court may order its
submission or correction or act on the pleading if the attending circumstances
are such that strict compliance with the Rule may be dispensed with in order
that the ends of justice may be served thereby.
3) Verification is deemed substantially complied with when one who has ample
knowledge to swear to the truth of the allegations in the complaint or petition
signs the verification, and when matters alleged in the petition have been
made in good faith or are true and correct.
4) As to certification against forum shopping, non-compliance therewith or a
defect therein, unlike in verification, is generally not curable by its subsequent
submission or correction thereof, unless there is a need to relax the Rule on

the ground of "substantial compliance" or presence of "special circumstances


or compelling reasons."
5) The certification against forum shopping must be signed by all the plaintiffs
or petitioners in a case; otherwise, those who did not sign will be dropped as
parties to the case. Under reasonable or justifiable circumstances, however, as
when all the plaintiffs or petitioners share a common interest and invoke a
common cause of action or defense, the signature of only one of them in the
certification against forum shopping substantially complies with the Rule.
6) Finally, the certification against forum shopping must be executed by the
party-pleader, not by his counsel. If, however, for reasonable or justifiable
reasons, the party-pleader is unable to sign, he must execute a Special Power
of Attorney designating his counsel of record to sign on his behalf. 25 (Emphasis
supplied)
In the present case, there is no dispute that Tax Ordinance Nos. 7988 and 8011 have
already been declared null and void by this Court as early as 2006 in the case of
Coca-Cola Bottlers Philippines, Inc. v. City of Manila. 26 The nullity of the said Tax
Ordinances is affirmed in the more recent case of City of Manila v. Coca-Cola
Bottlers, Philippines, Inc.,27 as cited above. Thus, to the mind of this Court, the
unquestioned nullity of the above assailed Tax Ordinances upon which petitioners
were previously taxed, makes petitioners' claim for tax refund clearly meritorious. In
fact, petitioners' sister companies, which were their co-plaintiffs in their Complaint
filed with the RTC, were granted tax refund in accordance with the judgments of the
trial court, the CTA Second Division and the CTA En Banc. On this basis, petitioners
meritorious claims are compelling reasons to relax the rule on verification and
certification of non-forum shopping.
In any case, it would bear to point out that petitioners and their co-plaintiffs in the
trial court filedtheir claim for tax refund as a collective group, because they share a
common interest and invoke a common cause of action. Hence, the signature of the
representative of the other co-plaintiffs may be considered as substantial compliance
with the rule on verification and certification of non-forum shopping, consistent with
this Court's pronouncement that when all the petitioners share a common interest
and invoke a common cause of action or defense, the signature of only one of them
in the certification against forum shopping substantially complies with the rules. 28
WHEREFORE, the instant petition is GRANTED. The Decision and Resolution of the
Court of Tax Appeals En Banc, dated December 17, 2010 and May 27, 2011,
respectively, in CTA EB No. 548, as well as the July 3, 2009 Decision and September
30, 2009 Resolution of the Court of Tax Appeals Second Division in CTA AC No. 51,
are REVERSED AND SET ASIDE and the Orders of the Regional Trial Court of Pasay
City, Branch 115, dated July 10, 2007 and December 14, 2007, are REINSTATED.
SO ORDERED.

S-ar putea să vă placă și